Naked Science Forum

On the Lighter Side => New Theories => Topic started by: mxplxxx on 22/01/2019 09:19:24

Title: How do we measure the energy of a photon?
Post by: mxplxxx on 22/01/2019 09:19:24
How do we measure the energy of a photon? In other words how do we validate the equation E=hf?
Title: Re: How do we measure the energy of a photon?
Post by: yor_on on 22/01/2019 15:04:54
You measure the frequency and get to a 'photon energy'. Planck's 'black body' is a experimental proof of light having a wave/particle duality that you can use for measuring a 'photons' energy, extended by Einsteins 'photoelectric effect' .  http://spiff.rit.edu/classes/phys314/lectures/photoe/photoe.html
Title: Re: How do we measure the energy of a photon?
Post by: chiralSPO on 22/01/2019 16:58:02
photoelectric effect is a good way to measure energy of single photons of ionizing radiation (UV and x-ray)

for visible and infrared light, you can use semiconductor devices (essentially photovoltaics) and measure the voltage produced by the frequencies.
Title: Re: How do we measure the energy of a photon?
Post by: evan_au on 22/01/2019 20:01:25
In the photoelectric effect, above a threshold frequency, an electron is ejected from the surface of a metal, with a kinetic energy that is proportional to the frequency of the incident light. This experiment must be conducted in a vacuum.

The slope of this line is h, Planck's constant.

By using a voltage to oppose the velocity of the emitted electron, you can get a direct measure of the electron's kinetic energy in units of electron-volts (eV).

See the graph at: https://en.wikipedia.org/wiki/Photoelectric_effect#Mathematical_description
Title: Re: How do we measure the energy of a photon?
Post by: mxplxxx on 22/01/2019 22:15:36
You measure the frequency and get to a 'photon energy'. Planck's 'black body' is a experimental proof of light having a wave/particle duality that you can use for measuring a 'photons' energy, extended by Einsteins 'photoelectric effect' .  http://spiff.rit.edu/classes/phys314/lectures/photoe/photoe.html
Thx. So, I am thinking we can't measure a photon's energy directly. So, how do we know that the E in E=hf is valid? Chicken and egg isn't it?
Title: Re: How do we measure the energy of a photon?
Post by: alancalverd on 22/01/2019 23:25:55
There's a very simple school experiment: measure the lowest voltage at which a light-emitting diode produces light. That tells you the energy E of those photons in electron volts. Now use a spectrometer to measure the wavelength λ of that light. E = hf = hc/λ by definition.


For a bit more sophistication you can generate x-rays with a known voltage on the x-ray tube anode, and use a crystal to diffract the x-rays. If the anode is,say, molybdenum, you will get a sudden increase in the x-ray beam intensity as the anode voltage exceeds 17,300V. This Kα radiation will produce a characteristic bright spot on the diffraction pattern, from which you can calculate the wavelength of a 17.3 keV photon.
Title: Re: How do we measure the energy of a photon?
Post by: mxplxxx on 23/01/2019 01:48:32
E = hf = hc/λ by definition.
Good to know, but it seems to me that none of these arguments involve directly measuring the energy E in joules of a single photon as per the E=hf equation. 
Title: Re: How do we measure the energy of a photon?
Post by: chiralSPO on 23/01/2019 02:46:14
it seems to me that none of these arguments involve directly measuring the energy E in joules of a single photon as per the E=hf equation. 

That is correct. There is no good way to directly measure the energy of a single photon.

But, if you go back and look at the answers that were given to you, you will see that we can transfer the energy from photons to electrons, and then measure the energy of the electron. We can do tests to make sure that each electron is only absorbing the energy from a single photon, and we can do tests to make sure that all of the energy from the photon goes to the electron. And we can do tests to measure the wavelength of the photons. Therefore, we can indirectly get very accurate measurements of the energy that a photon has as a function of wavelength.
Title: Re: How do we measure the energy of a photon?
Post by: mxplxxx on 23/01/2019 04:28:53
That is correct. There is no good way to directly measure the energy of a single photon.
Why is that?
Title: Re: How do we measure the energy of a photon?
Post by: mxplxxx on 23/01/2019 05:55:58
But, if you go back and look at the answers that were given to you, you will see that we can transfer the energy from photons to electrons, and then measure the energy of the electron. We can do tests to make sure that each electron is only absorbing the energy from a single photon, and we can do tests to make sure that all of the energy from the photon goes to the electron. And we can do tests to measure the wavelength of the photons. Therefore, we can indirectly get very accurate measurements of the energy that a photon has as a function of wavelength.
This is an extremely small set of energies and circumstances to generalize  E=hf from.
Title: Re: How do we measure the energy of a photon?
Post by: yor_on on 23/01/2019 07:34:28
You should look at it the other way around, which is a very nice title for a book btw :)
First you will need to check up how Planck got to the formula, after that how Einstein used it to further develop the idea of a 'light quanta'. When you've done that you need to check if the experimental values gained from that formula fits the descriptions made by Planck. If those work then the formula describes it correctly.

That's the way I did it.
Title: Re: How do we measure the energy of a photon?
Post by: yor_on on 23/01/2019 07:40:27
I think this one is a pretty good start
https://www.quora.com/In-laymans-terms-how-did-Max-Planck-come-up-with-the-Plancks-constant-and-what-problem-does-it-solve

What I find fascinating about Planck is his imagination. He had a very long sight, with that formula just being part of it. He would have made a excellent SF writer, in the best sense. https://www.nature.com/news/2007/071220/full/news.2007.389.html
Title: Re: How do we measure the energy of a photon?
Post by: alancalverd on 23/01/2019 09:04:48
Since all photons in monochromatic light have the same energy (by definition of "monochromatic") then any measurement of the wavelength of that light (including x-ray photons) is a measure of the energy of a single photon.

There are some interesting special cases such as the Mossbauer effect that filters photons to a very narrow range of energies, or photonuclear reactions that have a definable threshold, such as pair production which can only occur at or above 1.02 MeV.

We use electron volts conventionally because the number is more convenient than joules. 1eV = 1.6 x 10^-19 J - same thing, just differently expressed.

Since the double slit diffraction experiment works with a single photon, and the diffraction pattern depends on the photon wavelength, it is a direct measure of λ for a single photon. f = c/λ is a simple matter of definition so it is a measure of f. We measure E as I suggested earlier, as the threshold voltage required to emit light from a LED or  characteristic radiation from an x-ray anode. So what is missing?
Title: Re: How do we measure the energy of a photon?
Post by: evan_au on 23/01/2019 09:32:18
Quote from: alancalverd
Since the double slit diffraction experiment works with a single photon, and the diffraction pattern depends on the photon wavelength, it is a direct measure of λ for a single photon
I agree with Alan's answer to the original question.

I just have a quibble with the suggestion that the double slit experiment can directly measure the wavelength λ for a single photon.
- The probability that the photon will strike a particular place on the screen is certainly a function of wavelength
- But the probability is non-zero for a wide range of positions and a wide range of wavelengths
- So the fact that a single photon strikes a particular point on the screen cannot distinguish (say) λ from 1.5λ
- In particular, the maximum and minimum probability for λ is also a maximum and minimum for λ/2 (when the angle is small)

So I would say that the double-slit apparatus is a good way to determine the wavelength of many photons in monochromatic light. (end of quibble)
- The monochromatic nature of a light source can be checked by a number of methods, including:
- With a prism (producing a rainbow)
- With a diffraction grating
See: https://en.wikipedia.org/wiki/Diffraction_grating

The diffraction grating also allows direct calculation of the wavelength of a monochromatic light source, and is a way of producing monochromatic light, by selecting a single emission line from a gas-discharge lamp.
- You can also use highly monochromatic light sources, such as a distributed-feedback laser
See: https://en.wikipedia.org/wiki/Distributed_feedback_laser
Title: Re: How do we measure the energy of a photon?
Post by: yor_on on 23/01/2019 10:29:35
 Yes, a nice modern example that we use to prove a duality Alan, but it was introduced by Thomas Young in the beginning of 1800 to  argue for interference, aka a wave theory. I was thinking of it but to see how Planck got to his model I used the history. That first link is pretty good actually,  it has several nice links baked into it. It's interesting to read them and think about how we, ever so slowly, wandered  from Newtons 'corpuscles' to 'waves',  to then defining the modern definition which becomes a duality.  https://en.wikipedia.org/wiki/Young%27s_interference_experiment
Title: Re: How do we measure the energy of a photon?
Post by: alancalverd on 23/01/2019 11:51:00
Except that "duality" is not modern thinking!

Electromagnetic radiation can be modelled as particles or waves, but only human vanity (the antithesis of science) would pretend it is either or both. 

Anyway the statement that Ephoton= hf is either an implicit statement of duality (a property of a particle  equals a property of a wave), or blindingly obvious since h is defined as E/f or Eλ/c (deBroglie's original concept).

Apologies for the italics, but these words are often confused in the minds of nonscientists - particularly politicians and economists.
Title: Re: How do we measure the energy of a photon?
Post by: alancalverd on 23/01/2019 11:59:01
I just have a quibble with the suggestion that the double slit experiment can directly measure the wavelength λ for a single photon.- The probability that the photon will strike a particular place on the screen is certainly a function of wavelength- But the probability is non-zero for a wide range of positions and a wide range of wavelengths- So the fact that a single photon strikes a particular point on the screen cannot distinguish (say) λ from 1.5λ- In particular, the maximum and minimum probability for λ is also a maximum and minimum for λ/2 (when the angle is small)

If you use a LED to replicate Taylor's double-slit single-photon experiment, there are no harmonics or subharmonics  of f or λ present. You can even add a quarter-wave filter to narrow the bandwidth.

Now here's what occurred to me in my schooldays. If a single photon can interfere with itself, then the exit photon must appear in several places simultaneously to produce the observed pattern. So if we input a 1-attojoule photon (somewhere in the violet end of the visible spectrum, I think - too early for mental arithmetic!) we can detect 1 attojoule in several places simultaneously on the exit side. We have generated energy  from  nothing! This either destroys the entire concept of classical duality, or makes the Big Bang inevitable: you can generate an infinite amount of energy (hence mass) from a single photon.
Title: Re: How do we measure the energy of a photon?
Post by: mxplxxx on 23/01/2019 12:38:24
Except that "duality" is not modern thinking!
According to who?
Title: Re: How do we measure the energy of a photon?
Post by: alancalverd on 23/01/2019 13:58:26
Anyone who deals with quantum mechanics, radiation measurement, or simple logic.

To take one practical example, x-ray crystallographers have been exploiting the diffraction of electromagnetic radiation to study the structures of materials and molecules for over 100 years, but for the last 50 years or so we have started counting photons electronically (rather than using photographic film) to determine the intensity of the diffracted beams. The idea that a photon knows in advance what we want it to do at a particular point in the apparatus is absurd, but appropriate application of the wave and particle models allows us to capture the data (from "particle-like" events) and process it (through continuous fourier transform wave equations) it to reveal the spatial disposition of atoms in the crystal.

At very low photon fluxes we can detect individual interaction ("single photon") events. Now if a "single photon" consists of a truncated "chirp" burst of waves,  Fourier tells us that we should be able to detect harmonics of the fundamental frequency. associated with, say, a LED or similar single spectral line. We can't.
Title: Re: How do we measure the energy of a photon?
Post by: yor_on on 23/01/2019 15:18:25
Don't really know when the idea of a 'duality' was first coined Alan, but to me it's recent anyway :)
=
Sorry, missed that mxplxxx and you already discussed it
Title: Re: How do we measure the energy of a photon?
Post by: yor_on on 23/01/2019 15:26:10
And a very nice point there "  Fourier tells us that we should be able to detect harmonics of the fundamental frequency. associated with, say, a LED or similar single spectral line. We can't."

I know of people making what you state there a career Alan. And looking at a 'photon' as a wave function is a tricky part of physics, although making theoretical sense.
Title: Re: How do we measure the energy of a photon?
Post by: alancalverd on 23/01/2019 15:29:45
Indeed. But the wave function of a photon is very easy to use and understand.

And just to go back to the original question, you can't "validate" E = hf because it is the definition of h. You can't validate " a cow is a female bovine" because that is the definition of a cow. What you can do is to make lots of measurements that convince you that h is indeed a universal constant, and cows moo at one end, poo at the other, and make milk in the middle..
Title: Re: How do we measure the energy of a photon?
Post by: Bill S on 23/01/2019 18:14:44
Quote
....cows moo at one end, poo at the other, and make milk in the middle.

Surely, every physicist knows that a cow is a sphere, so, which end is which?. 
Title: Re: How do we measure the energy of a photon?
Post by: Bored chemist on 23/01/2019 19:41:41
Fourier tells us that we should be able to detect harmonics of the fundamental frequency. associated with, say, a LED or similar single spectral line. We can't.

Practically every "green laser" you see- and they aren't rare- is actually the second harmonic of an infrared laser beam.
Title: Re: How do we measure the energy of a photon?
Post by: Bored chemist on 23/01/2019 19:46:54
you can't "validate" E = hf because it is the definition of h.
Actually, you can. There are other ways of measuring h
https://en.wikipedia.org/wiki/Brownian_motion#Einstein's_theory
https://en.wikipedia.org/wiki/Noise_temperature

And other ways of measuring the energy- you can count the photons in a laser beam and then see how well the laser warms something up
https://en.wikipedia.org/wiki/Bolometer
And you can measure the wavelength with a ruler.
http://ipl.physics.harvard.edu/wp-uploads/2013/03/15c_s07_5.pdf

And e=hf is not the definition of h anyway.
The definition comes from Planck's work on black body radiation
https://en.wikipedia.org/wiki/Planck%27s_law#The_law

And, even is it was, then we could still validate it.

Imagine that I put forward the idea that the energy of a photon is proportional to the square of the wavelength (it's not , but that's the point).
I could do the same sort of experiments on energies and wavelengths and every time I tried to calculate my measured value for the "constant", it would change.
That observation would invalidate my theory.

Well, by contrast, the fact that every way you measure h gives the same answer does validate the idea that e =hf

Title: Re: How do we measure the energy of a photon?
Post by: alancalverd on 23/01/2019 23:14:30
Fourier tells us that we should be able to detect harmonics of the fundamental frequency. associated with, say, a LED or similar single spectral line. We can't.

Practically every "green laser" you see- and they aren't rare- is actually the second harmonic of an infrared laser beam.
A LED is not a laser. And the individual photons of a laser do not exhibit harmonics. 
Title: Re: How do we measure the energy of a photon?
Post by: alancalverd on 23/01/2019 23:39:25
Quote
In 1900, Max Planck empirically derived a formula for the observed spectrum by assuming that a hypothetical electrically charged oscillator in a cavity that contained black body radiation could only change its energy in a minimal increment, E, that was proportional to the frequency of its associated electromagnetic wave.

That is, by assuming E = hf as an axiom (i.e. defining a constant, h) Planck derived an equation that exactly describes the black body spectrum and thus, I admit, validated his axiom.

So the original questions, posed as a tautology, in fact weren't. The validation of E = hf had nothing to do with measuring the energy of an individual photon, but with describing the gross energy/wavelength curve for zillions of photons, that you can measure with a spectrobolometer. Not unreasonably, however, we find that individual photon energies do indeed fit exactly to Planck's assumption.
Title: Re: How do we measure the energy of a photon?
Post by: alancalverd on 23/01/2019 23:58:49
PS I like the cheek of the Harvard paper on "measuring the wavelength of light with a ruler". When you have worked your way through the geometrical optics theory, you get to the experimental setup where the critical element is a 1/64 inch ruler, and the everyday bit of kit you just happen to have in the kitchen drawer is a coherent monochromatic laser.

My next paper will be entitled "crossing the Irish Sea with a magnetised sewing needle". You float the needle carefully on the meniscus of a glass of water (it's a good trick - one method is to float it initially on a cigarette paper, which gradually sinks as it gets wet) then start up your airplane and just fly perpendicular to the needle.... Try this at home, with any steel needle.....
Title: Re: How do we measure the energy of a photon?
Post by: mxplxxx on 24/01/2019 01:48:12
[
Not unreasonably, however, we find that individual photon energies do indeed fit exactly to Planck's assumption.
A reference to an experiment that corroborates your statement would be nice. If I had a photon generator and a photon meter, the Visual Basic code below would do the trick.

const h = 6.62607004 × 10-34

for each photon in RandomSetOfPhotons
    E = PhotonMeter.MeasureEnergy(photon)
 
    If E = h * photon.frequency then
        MessageBox("photon " & photon.id & " is ok with E = " & E & " for frequency " & photon.frequency
    else
        MessageBox("photon " & photon.id & " not ok - E is " & E " for frequency " & photon.frequency & " - should be " h * photon.frequency
    end if

end for


Title: Re: How do we measure the energy of a photon?
Post by: mxplxxx on 24/01/2019 02:19:42
I have other, related, questions.

1. Can I generate a photon from anything else bar an electron?
2. For an electron to "leap" to another level, presumably it must have absorbed a photon of the necessary energy. Where does this photon come from?
Title: Re: How do we measure the energy of a photon?
Post by: chiralSPO on 24/01/2019 03:41:38
1. Yes, it is possible to generate photons without the involvement of electrons, but moving electrons (in atoms, or other sources of electric/magnetic fields) is one of the easiest ways.

Photons can be generated by accelerating charged particles (including electrons, or protons, or muons, or alpha particles etc.)

Photons (high energy) can be produced by the annihilation of particles with their antiparticles.

Photons (also typically high energy) can also be produced from nuclear reactions.

Electrons don't necessarily have to change energy levels--simply changing vibrational states in molecules or crystals can lead to emission (or absorbtion) of photons in the infrared region of the spectrum.

2. Electrons can "leap" to higher energy levels for many reasons, including colliding with other electrons or particles. Or high energy vibrations can be converted into excited electronic states with very little vibration. A good example of both of these phenomena is the blackbody radiation of hot objects. Thermal energy is essentially kinetic energy of vibrations and random motions. If there is enough thermal energy (if the sample is hot enough) then some of this kinetic energy is converted to light--hence hot objects glow, and the hotter they are, the more shorter wavelengths can be emitted (blue is hotter than yellow is hotter than red). Another example of collisions creating light is "triboluminescence"

There are also chemical reactions that produce light directly (like in fireflies and glowsticks).
Title: Re: How do we measure the energy of a photon?
Post by: chiralSPO on 24/01/2019 03:48:05
Of course, it is also possible for electrons to be promoted by light. This is how pigments appear to be colored, and how fluorescent dyes glow.
Title: Re: How do we measure the energy of a photon?
Post by: mxplxxx on 24/01/2019 03:49:30
Photons can be generated by accelerating charged particles (including electrons, or protons, or muons, or alpha particles etc.)
This is counter-intuitive. I would expect the charged particle to be absorbing photons.
Title: Re: How do we measure the energy of a photon?
Post by: mxplxxx on 24/01/2019 03:51:28
Of course, it is also possible for electrons to be promoted by light. This is how pigments appear to be colored, and how fluorescent dyes glow.
Presumably light of the exact wavelength needed to make the electron "leap". Is this a common event? How does it come about?
Title: Re: How do we measure the energy of a photon?
Post by: mxplxxx on 24/01/2019 04:20:05
Yet another question:) What is the wavelength relationship (if any) between an emitted photon and the emitting particle?
Title: Re: How do we measure the energy of a photon?
Post by: mxplxxx on 24/01/2019 06:49:49
Photons can be generated by accelerating charged particles (including electrons, or protons, or muons, or alpha particles etc.)
What of two electrons that are moving at the same speed and are then accelerated at the same rate (i.e. they are stationary with respect to one another), do they emit photons? It seems to me emitting of photons of an accelerating particle contradicts relativity.
Title: Re: How do we measure the energy of a photon?
Post by: Bored chemist on 24/01/2019 07:29:20
PS I like the cheek of the Harvard paper on "measuring the wavelength of light with a ruler". When you have worked your way through the geometrical optics theory, you get to the experimental setup where the critical element is a 1/64 inch ruler, and the everyday bit of kit you just happen to have in the kitchen drawer is a coherent monochromatic laser.

My next paper will be entitled "crossing the Irish Sea with a magnetised sewing needle". You float the needle carefully on the meniscus of a glass of water (it's a good trick - one method is to float it initially on a cigarette paper, which gradually sinks as it gets wet) then start up your airplane and just fly perpendicular to the needle.... Try this at home, with any steel needle.....
Yes, that's right.
This sort of everyday item you may have on a keyring, or in the kitchen drawer.
https://fetch.co.uk/petface-laser-chaser-cat-toy-317684011?gclid=EAIaIQobChMIko-SufGF4AIVFyjTCh2_UAliEAQYASABEgKj3_D_BwE&gclsrc=aw.ds
A LED is not a laser
Nobody said it was, but you said
a LED or similar single spectral line.
And LEDs don't give single spectral lines, but lasers (nearly) do.
Title: Re: How do we measure the energy of a photon?
Post by: evan_au on 24/01/2019 08:45:24
Quote from: mxplxxx
I would expect (accelerating) charged particles to be absorbing photons.
You can accelerate a charged particle by placing it in a static electric field (eg 1 million volts from a Van der Graaf generator).
From the frame of reference of the lab, the accelerating charged particle will radiate photons.

I am no expert in relativity, but I imagine that in the (accelerating) frame of reference of the charged particle, the electrodes producing the electric field are radiating photons(?) - some advice from someone who knows more than I would be appreciated!

In practice, to accelerate charged particles to really high energies (corresponding to billions of volts or more), scientists tend to use:
-  a circular track: which means the charged particles are continually accelerated around the circle, so they are continually radiating due to the acceleration towards the center
- oscillating electric fields, which generate photons of their own...
- This is one reason nobody is allowed in the LHC tunnel while the beam is operating (the other reason is that if a steering magnet failed, the beam could pierce the pipe - and anyone in the tunnel...)

See: https://en.wikipedia.org/wiki/Cyclotron_radiation

Quote
Presumably light of the exact wavelength needed to make the electron "leap".
An element in a diffuse gas or plasma has some clearly defined wavelengths which it will absorb or emit, over a very narrow range of wavelengths.

However, when the atoms are "in contact" in a liquid or solid, the Pauli Exclusion principle requires that the electrons take up slightly different energy levels, and this means that the spectrum covers a much broader. range of wavelengths, depending on which levels it jumps from and to.

Quote
What is the wavelength relationship (if any) between an emitted photon and the emitting particle?
An electron can be considered to have a certain wavelength, which becomes slightly smaller at higher energies (as in an electron microscope). But an electron in an atom can generate photons of many different frequencies, from microwave through visible to X-Rays.

A proton in an atomic nucleus can also be considered to have a certain wavelength, which is much shorter than the wavelength of an electron. The radiation produced by a proton changing energy levels tends to be in the gamma-ray range, ie much shorter than the wavelengths produced by electrons in atoms.

On the other hand, neutrons (no electric charge) have a similar wavelengths to protons (positive electric charge), but don't tend emit electromagnetic energy in the same way as protons.

So any relationship has a lot of caveats...
See: https://en.wikipedia.org/wiki/Matter_wave

Quote
It seems to me emitting of photons of an accelerating particle contradicts relativity.
Einstein's relativity is a more general form of Maxwell's equations, and incorporates Maxwell's equations.

Maxwell's equations describe how accelerating the electrons in the antenna of your smartphone radiates electromagnetic waves, allowing you to connect to WiFi.

Therefore, Einstein's relativity is WiFi-compatible!
Title: Re: How do we measure the energy of a photon?
Post by: alancalverd on 24/01/2019 09:06:34
Photons can be generated by accelerating charged particles (including electrons, or protons, or muons, or alpha particles etc.)
What of two electrons that are moving at the same speed and are then accelerated at the same rate (i.e. they are stationary with respect to one another), do they emit photons? It seems to me emitting of photons of an accelerating particle contradicts relativity.
As with Newtonian physics, relativity distinguishes between constant speed and acceleration.
Title: Re: How do we measure the energy of a photon?
Post by: alancalverd on 24/01/2019 09:23:26
Quote from: alancalverd on Yesterday at 13:58:26 a LED or similar single spectral line. And LEDs don't give single spectral lines, but lasers (nearly) do.
                           
                           

You should consider the whole paragraph, which concerns the "chirp" nature of a single photon.


I just find the "keyring laser" entertaining. Within living memory (mine) we progressed from neolithic oil lamps and candles, and postmen with bags of paper, through biplanes, incandescent lamps, gas mantles and telegrams, to interplanetary flight, hand-held instant worldwide communications, and kids' toys that invoke subtle applications of solid state quantum physics. How sad that politics, economics and religion have remained at their prehistoric level of sophistication. Or indeed that despite all this, some people still need convincing that E = hf, which does rather seem to be the basis of a lot of stuff we take for granted - like the screen you are looking at right now!.
Title: Re: How do we measure the energy of a photon?
Post by: evan_au on 24/01/2019 09:26:05
Quote from: bored chemist
LEDs don't give single spectral lines, but lasers (nearly) do.
To add to bored chemist's comment:
- Red LEDs and Red LASERs can be made from similar semiconductor materials
- But laser chips have a parallel mirrors on each end, so that most of the light bounces back and forth within this laser cavity; where an integer number of wavelengths fit in the cavity length, the oscillation is self-sustaining, while other wavelengths rapidly die out. This means that the spectrum from a laser chip is much more monochromatic than light from a LED.
- Since the cavity is much longer than the wavelength of the light, it is possible for there to be n, n+1, n-1 etc wavelengths of light in the length of the cavity. These are called different "modes" of the laser.
- As temperature changes, different modes are favored. So a monochromatic laser needs temperature controls.
- Under some conditions, the laser can randomly switch between adjacent modes - this is called "mode-hopping"
- Under other conditions, the laser can chaotically switch between many modes, producing a spectrum with several wavelength peaks
- For high-precision applications, an additional cavity is used to select just one of these modes

See examples here: https://www.newport.com/medias/sys_master/images/images/h4b/h48/8797049585694/AN08-Mode-Hopping-in-Semiconductor-Laser-Diodes.pdf
Title: Re: How do we measure the energy of a photon?
Post by: mxplxxx on 24/01/2019 10:07:02
Can anyone point me at a picture of Energy being integrated over time to give Planck's constant h? Preferably with different values of E and t. Frustratingly, I cannot find one on the internet.
Title: Re: How do we measure the energy of a photon?
Post by: mxplxxx on 24/01/2019 11:22:39
some people still need convincing that E = hf,
From my travels it would seem that this is mainly controversial when E is the energy of a photon. There appears to be little consensus amongst physicists regarding the nature of a photon. Iittle wonder then that laypeople like myself are totally confused and attempts at better theories are constantly being proposed. Here is one that does a fair job of marrying a photon and an EM wave, especially from a relativistic perspective.  http://vixra.org/pdf/1609.0359v1.pdf
Title: Re: How do we measure the energy of a photon?
Post by: alancalverd on 24/01/2019 13:03:11
E = hf is Planck's definition of h. How can that be controversial?

Regarding integration: Way back in the dawn of time I built a calorimeter to measure the energy deposited by photons emitted from a cobalt-60  source.  The integration time was around 100 seconds, and knowing the activity of the source and the solid angle covered by the calorimeter (plus a whole lot of corrections you can find in the textbooks), we could calculate the number of photons involved. It became the first UK national primary standard of absorbed dose for radiotherapy.

I don't think there is any doubt about the "nature" of a photon. It is what is emitted when a charged particle accelerates or changes state. The only confusion arises in the minds of people who confuse a mathematical model of a photon with an actual photon, because there are two useful models.

Back to the philosopher's favorite animal - the cow. There is no doubt as to the nature of a cow. Just like photons, you can see some of them. You can model a cow in terms of various ratios: grass input to milk output (a useful model of a live cow) or gross weight to edible pies (a useful model of a dead cow), but neither model tells you "what a cow is".

Title: Re: How do we measure the energy of a photon?
Post by: mxplxxx on 24/01/2019 20:09:40
As with Newtonian physics, relativity distinguishes between constant speed and acceleration.
Maybe the theory excludes acceleration because it invalidates the theory? Also, easy to point to time as the culprit when the theory doesn't work but not possible to explain away photon emission between two particles that think they are at rest even when seen to be accelerating by other particles.
Title: Re: How do we measure the energy of a photon?
Post by: mxplxxx on 24/01/2019 20:13:14
The only confusion arises in the minds of people who confuse a mathematical model of a photon with an actual photon, because there are two useful m
A perennial problem in physics!
Title: Re: How do we measure the energy of a photon?
Post by: Bored chemist on 24/01/2019 20:14:12
From my travels it would seem that this is mainly controversial when E is the energy of a photon.
It may seem that way to you.
There's still debate about the nature of the photon. (More about terminology than about how it behaves)
But none at all about the energy it carries.
Title: Re: How do we measure the energy of a photon?
Post by: mxplxxx on 24/01/2019 20:34:07
But none at all about the energy it carries.
There should always be  a question mark about E-hf for a photon until E can be measured directly:)
Title: Re: How do we measure the energy of a photon?
Post by: Bored chemist on 24/01/2019 21:00:18
But none at all about the energy it carries.
There should always be  a question mark about E-hf for a photon until E can be measured directly:)
There was a question.
And then it was measured. (the thread has given various examples of how this was done)
And now we have the answer.
So what are you banging on about?
Title: Re: How do we measure the energy of a photon?
Post by: alancalverd on 24/01/2019 23:50:07
As with Newtonian physics, relativity distinguishes between constant speed and acceleration.
Maybe the theory excludes acceleration because it invalidates the theory? Also, easy to point to time as the culprit when the theory doesn't work but not possible to explain away photon emission between two particles that think they are at rest even when seen to be accelerating by other particles.
Relativity does indeed deal with acceleration. Particles don't think. You would be well advised to learn some basic physics before pontificating on the subject.
Title: Re: How do we measure the energy of a photon?
Post by: mxplxxx on 25/01/2019 00:01:03
As with Newtonian physics, relativity distinguishes between constant speed and acceleration.
Maybe the theory excludes acceleration because it invalidates the theory? Also, easy to point to time as the culprit when the theory doesn't work but not possible to explain away photon emission between two particles that think they are at rest even when seen to be accelerating by other particles.
Relativity does indeed deal with acceleration. Particles don't think. You would be well advised to learn some basic physics before pontificating on the subject.
"Maybe" is hardly pontificating. It is inviting an opinion which you have given. Lots of people think differently. Maybe you could supply references.
Title: Re: How do we measure the energy of a photon?
Post by: alancalverd on 25/01/2019 00:02:40
you can't "validate" E = hf because it is the definition of h.
Actually, you can. There are other ways of measuring h
https://en.wikipedia.org/wiki/Brownian_motion#Einstein's_theory
https://en.wikipedia.org/wiki/Noise_temperature

I was so intrigued by your statement that I read both articles. Neither mentions Planck's constant. The symbol h appears in the first article, but means barometric height, not action.
Title: Re: How do we measure the energy of a photon?
Post by: esquire on 27/01/2019 19:23:29
photon are massless, they have no charge, they do have spin
yet photons have energy. this qualifies photons for not having a zero rest state, contradiction?
a charge references a particle state. a particle because of mass is energy.
a photon because of zero mass shouldn't qualify it as having energy.
charge and energy are two different things. without charge energy does not exist but charge can exist w/o energy.
 
Mass = energy, energy requires a charge. without a charge particle, energy cannot exist within that particle.  this is a contradiction for a photon, without a charge state, energy is null.

does a particle have a wave frequency? the uncertainty principle states either location or velocity can be determined, approximately. a particle traveling at the speed of light, can be located within a strictly short locational proximity. the particle moving at the SOL appears to be at both ends of the short locational boundary and all points in-between.  In this short locational boundary there has been no observance of a wave of any intensity or frequency. the observed particle is between two points and all points in-between. this doesn't resemble a wave with the characteristic undulating of frequency. the observance is flatlined.

so e=hf, mass = planck x frequency. with zero frequency, planck is a useless multiplier when it comes to the measurement of photonic energy . e=hf, most definitely has application in most arenas, just not with measuring the energy of a photon.
Title: Re: How do we measure the energy of a photon?
Post by: Bored chemist on 27/01/2019 20:13:07
you can't "validate" E = hf because it is the definition of h.
Actually, you can. There are other ways of measuring h
https://en.wikipedia.org/wiki/Brownian_motion#Einstein's_theory
https://en.wikipedia.org/wiki/Noise_temperature

I was so intrigued by your statement that I read both articles. Neither mentions Planck's constant. The symbol h appears in the first article, but means barometric height, not action.
Bother!
I got my constants muddled

Fortunately, there are still plenty of ways to measure h
https://en.wikipedia.org/wiki/Planck_constant#Determination

So, my point's still valid.
Title: Re: How do we measure the energy of a photon?
Post by: jeffreyH on 27/01/2019 21:35:18
The beginnings of quantum mechanics were very mundane. It was Planck's intention to solve a problem for experimenters. Read the story here https://www.google.com/amp/s/amp.theguardian.com/science/2000/dec/14/particlephysics
Title: Re: How do we measure the energy of a photon?
Post by: mxplxxx on 28/01/2019 02:46:14
That is correct. There is no good way to directly measure the energy of a single photon.
Why is that?
No answer to this, so I will have a guess. I would say that nothing that moves at the speed of light can be measured. According to relativity, time does not exist for such an object which would likely complicate any attempt to measure it. It seems to exist only in the present (which roars by at the speed of light!), with no past or future. Given that our experience of the present is dependent on photons, this makes sense.

Note also , the fact that time does not exist for a photon means also that it cannot "wave".  How then does it appear to exist as an EM wave? Imho, relativity needs to be updated to account for this situation. One possibility is that the wave exists in the future which makes sense given that the photon seems to exist initially as a possibility wave:)
Title: Re: How do we measure the energy of a photon?
Post by: jeffreyH on 28/01/2019 06:55:00
One last try at punching smoke. The ultraviolet catastrophe was solved by Planck introducing h. Otherwise the classical theory would still predict the energy approaching infinity at shorter wavelengths. https://en.m.wikipedia.org/wiki/Ultraviolet_catastrophe

If you still don't understand then there is no hope.
Title: Re: How do we measure the energy of a photon?
Post by: mxplxxx on 28/01/2019 07:33:53
One last try at punching smoke. The ultraviolet catastrophe was solved by Planck introducing h. Otherwise the classical theory would still predict the energy approaching infinity at shorter wavelengths. https://en.m.wikipedia.org/wiki/Ultraviolet_catastrophe

If you still don't understand then there is no hope.
What is this in reference to?
Title: Re: How do we measure the energy of a photon?
Post by: evan_au on 28/01/2019 09:19:29
Quote from: esquire
the uncertainty principle states either location or velocity can be determined, approximately.
There are several versions of the uncertainty principle.

But if you are only interested in the energy of the photon, and don't care too much about the position at which it arrived, or the time that it arrived, then you can measure the energy moderately accurately, even for (80% of) single photons.

Astronomers use CCD detectors that:
- Have a large fraction of the surface area available for detecting photons; the remainder is covered by wires, and some photons will be reflected rather than absorbed. Something like 80% of incoming photons can be detected.
- Are cooled to low temperatures, so there is a very low "dark current", and few spontaneous detections where there was no incoming photon (ie a "false alarm")
- Can be coupled to a large-area diffraction grating, which splits light into different wavelength/energy bands. There are several locations that a photon of a given energy could arrive, but if you put a CCD at all of them, you can estimate the energy regardless of where it arrives.

For this to work well, the energy of the incoming photons needs to be constrained within a reasonable range, otherwise a photon could be confused with one of a much higher or lower energy.
See: https://en.wikipedia.org/wiki/Charge-coupled_device#Use_in_astronomy
https://en.wikipedia.org/wiki/Diffraction_grating
 
Title: Re: How do we measure the energy of a photon?
Post by: alancalverd on 28/01/2019 10:13:00
Somewhere ikn the last 50 years, I forgot about the GeLi gamma spectrometers we used to use to calibrate x-ray sources. Wikipedia has a good explanatory article that I won't repeat here, but the key is that you can use a gamma emitter with a narrow spectrum to locate a single point on the resultant spectrum and thus measure the energy of any photon against your calibration point.

Getting down to single photon detection with GeLi is a bit of an art form but since we know photons are indistinguishable, if we find a narrow line on a spectrum we can say that it does at least represent the average energy of a few photons.

It's interesting that the questioner persists in telling us that something can't be done, in the face of the experience of those who have done it for a living.  Unshakeable faith in one's prejudice can be dangerous, but as a forlorn gesture of friendship I'll advise people not to play with fire or walk in front of moving buses.
Title: Re: How do we measure the energy of a photon?
Post by: mxplxxx on 28/01/2019 11:30:07
It's interesting that the questioner persists in telling us that something can't be done
Not at all. Where have I said this? I am interested in measuring the energy of a single photon - that is the question. Have/can you answer it? According to Chiralpso, "That is correct. There is no good way to directly measure the energy of a single photon." I wonder, is there any way to directly measure the energy of anything?

The answers I have been getting refer to measurig the energy of particular types of photons in particular sets of circumstances. Nothing like a generalised  PhotonMeter that I would have thought would have been dead easy to construct.

ps I would prefer to be referred to as mxplxxx rather than the very unfriendly "the questioner":)


 
Title: Re: How do we measure the energy of a photon?
Post by: chiralSPO on 28/01/2019 14:00:58
mxplxxx, I think you are misinterpreting my response. I said it is not easy to directly measure the energy of a photon, and that we make these measurements indirectly. But that doesn't mean that the measurement is any less valid (because we have rigorously established the validity of these indirect measurements).

When dealing with single particles, it is very rare that a direct measurement will be useful (think about what indirect and direct mean--how likely is it that machine the size of my head will be able to interact directly with a single elementary particle, and extract this information without having any working intermediate bits between the photon and the little LED readout that tells me what the energy is.)
Title: Re: How do we measure the energy of a photon?
Post by: chiralSPO on 28/01/2019 15:00:41
To clarify my point about direct vs indirect:

I think we can agree that when I talk to someone on the phone, that is a form of indirect communication: the sound waves of my voice are captured by a microphone, converted into electronic signals, which are then converted into electromagnetic radiation, before being received, turned back into electronic signals, which the speaker reconstitutes into my voice (this is a very simple version).

But what I urge people to consider is that talking face-to-face is also indirect. My vocal chords vibrate, bumping into air molecules, which bump into air molecules, which bump into air molecules, which bump into air molecules, which bump into air molecules, etc. etc. until those air molecules reach the ear of my listener. But wait, we're not done yet! The ear is really complex. Those moving molecules bump into their ear drum, causing vibrations that then emanate from the eardrum, and goes into a funny spiral-shaped organ, that is able to physically do a Fourier transform of the vibrational frequencies (it sorts them out by frequency due to how easily the sound waves are able to go around a bend), ultimately detecting those vibrations with little hair-like structures, that then encode the vibrations into neural signals, to be processed by the listeners ear.

My point is: when considering information transfer on a molecular scale or smaller, one must accept that there is not really any such thing as a direct measurement, other than the actual interaction between two particles (and there is no way for that kind of interaction to directly transfer the information from a single particle to any sort of macroscopic device.)
Title: Re: How do we measure the energy of a photon?
Post by: Colin2B on 28/01/2019 18:17:57
My point is: when considering information transfer on a molecular scale or smaller, one must accept that there is not really any such thing as a direct measurement, other than the actual interaction between two particles (and there is no way for that kind of interaction to directly transfer the information from a single particle to any sort of macroscopic device.)
This is an important point because many of the measurements we make are indirect.
Eg voltage is often measured by the movement of a coil inside a permanent magnet, moving a pointer. Even our weight is often the deflection of a spring.
Title: Re: How do we measure the energy of a photon?
Post by: alancalverd on 28/01/2019 18:23:15
The answers I have been getting refer to measurig the energy of particular types of photons in particular sets of circumstances. Nothing like a generalised  PhotonMeter that I would have thought would have been dead easy to construct.
ps I would prefer to be referred to as mxplxxx rather than the very unfriendly "the questioner":)

A bolometer will in principle directly measure the energy of any and all photons. As with rockets, the science is indeed dead easy but the engineering is extremely difficult. It took my team 15 man-years to measure the absolute energy of a radiotherapy photon beam to a reproducibility and traceability of 0.1%.

And it all depends on what you mean by "directly". Time is what comes out of the NIST primary standard atomic clock, but you wouldn't use it to decide when to stop boiling an egg. Length is defined in terms of time and the speed of light, but you wouldn't use radar to buy curtains. So we use different gadgets to measure the energy of different photons, and most easily, in a tiny weeny bit of the electromagnetic spectrum, we just use our eyes.

Interesting to note that an unpronounceable assembly of consonants can be considered a friendly form of address. But  Llanfair­pwllgwyngyll­gogery­chwyrn­drobwll­llan­tysilio­gogo­goch is probably the only word in any human language that  has three consecutive l's, so I'll assume our friendly questioner is Welsh. Iechyd da.
Title: Re: How do we measure the energy of a photon?
Post by: mxplxxx on 28/01/2019 22:42:30
Llanfair­pwllgwyngyll­gogery­chwyrn­drobwll­llan­tysilio­gogo­goch
Full meaning: “Saint Mary’s Church in the hollow of the white hazel near the rapid whirlpool and the Church of Saint Tysilio of the red cave”.
Title: Re: How do we measure the energy of a photon?
Post by: mxplxxx on 28/01/2019 22:44:45
Llanfair­pwllgwyngyll­gogery­chwyrn­drobwll­llan­tysilio­gogo­goch
Full meaning: “Saint Mary’s Church in the hollow of the white hazel near the rapid whirlpool and the Church of Saint Tysilio of the red cave”. Actually, I am 4th generation Australian.
Title: Re: How do we measure the energy of a photon?
Post by: mxplxxx on 28/01/2019 23:13:06
According to relativity, time does not exist for such an object
Actually (I think:)), the photon itself will experience the universe as if it were stationary. An observing object will experience the photon as travelling at the speed of light but also as ageless. Which is just great for (guess) the photon's purpose in life which is to transfer state information between fermions.
Title: Re: How do we measure the energy of a photon?
Post by: alancalverd on 29/01/2019 00:40:04
Iechyd da = g'day, sport, or cheers, mate. Much as I respect a culture whose origins predate mine by about 40,000 years, I can't accept that photons have a purpose in life. That's taking anthropomorphism beyond the ridiculous. When a photon goes walkabout, it has no idea of where it's going or why - it's just a teenage drongo rushing out of the house to get rid of some energy.

And an observer doesn't see a photon as ageless! Photons of considerable age will be redshifted according to Hubble's Law, so the energy of a photon compared with the expected energy of a freshly minted one from the same quantum process gives us an idea of how long it has been in the intergalactic outback.

Irrelevant perhaps, but the Celtic branch of my family held a wedding in LlanfairPG, presumably in the hope that the English would be too embarrassed to buy a train ticket, and thus save a bit on the bar bill. But we did.
Title: Re: How do we measure the energy of a photon?
Post by: mxplxxx on 29/01/2019 02:24:07
Much as I respect a culture whose origins predate mine by about 40,000 years,
My original ancestry is Irish. We have been in Australia since about 1850. Unfortunately, my adopted country was invaded by the British in 1788. The original inhabitants had been there for about 40000 years. They had a very advanced culture that embraced living in harmony with their environment. Each aboriginal child could speak 5 languages. Unfortunately the non-technological nature of the culture made it no match for the British army. An apology for past wrongs has recently been made in the Australian parliament. 
Title: Re: How do we measure the energy of a photon?
Post by: mxplxxx on 29/01/2019 02:34:17
And an observer doesn't see a photon as ageless! Photons of considerable age will be redshifted according to Hubble's Law, so the energy of a photon compared with the expected energy of a freshly minted one from the same quantum process gives us an idea of how long it has been in the intergalactic outback.
I am aware of this. It seems to contradict relativity. Actually it may not. An EM wave is really three interacting waves/particles) ... but I will discuss this more in  New Theories, as soon as I have worked out how they interact:).
Title: Re: How do we measure the energy of a photon?
Post by: mxplxxx on 29/01/2019 02:53:26
I can't accept that photons have a purpose in life.
Bosons, of which photons are a particular type, are to the universe what events are to a finite state machine. From that perspective, they have the purpose of changing the state of the current particle based on a change of state of another (usually) related particle. Actually, it is likely that the particle reacts to the photon rather than the photon changing the particle. Or it may be an interaction of the two.
Title: Re: How do we measure the energy of a photon?
Post by: alancalverd on 29/01/2019 09:44:13
This could go two ways, equally entertaining.

If the British (which at the time included the Irish) hadn't invaded Australia in 1788, I doubt that your family would have emigrated there in 1850, so whether you are grateful, apologetic, or both, is a matter way beyond the scope of a science chatroom. Anyway it's a fascinating place and I'm looking forward to visiting my grandchildren just as soon as  it cools down.

Relativity gives us all sorts of reasons for redshift, starting from the simplest Doppler effect.

As far as bosons are concerned, or anything else for that matter, it's important to note the ambiguity of "function". Colloquially, it certainly involves intended purpose, but scientifically it merely states the difference between before and after.

Title: Re: How do we measure the energy of a photon?
Post by: tanujagdale on 29/01/2019 09:58:50
Great information..
Title: Re: How do we measure the energy of a photon?
Post by: mxplxxx on 29/01/2019 10:26:19
Anyway it's a fascinating place and I'm looking forward to visiting my grandchildren just as soon as  it cools down
Your kids have good sense! Melbourne (for ages voted the world's most livable city) , where I live, enjoys warm summers, glorious springs, mild autumns and crisp winters. Today was just glorious at 30 degrees:). Some of the best research in the world happens in Melbourne. Esp. medical - e.g. the bionic ear that allows me to hear was invented here.

I will research red shifts and relativity. Thanks for the info.

Actually, it seems to me the red shift would be different for different observations of the same photon (if that were even possible). Which makes the properties of a photon something that is shared between an observer and the photon. But that is ridiculous isn't it!
Title: Re: How do we measure the energy of a photon?
Post by: yor_on on 29/01/2019 11:17:59
no, that's perfectly correct. the energy you measure a photon is dependent on what frame of reference you have, being 'at rest' with it or 'moving' relative it. To define a intrinsic energy you have to be 'at rest' with your experimental setup. Light can both be said to 'red shift' and 'blue shift' depending on ones frame of reference, measuring it. It's in principle similar to two cars colliding, depending on relative speed(s), but in the photons frame we better avoid talking about 'mass'.
Title: Re: How do we measure the energy of a photon?
Post by: mxplxxx on 29/01/2019 11:21:55
no, that's perfectly correct. the energy you measure a photon is dependent on what frame of reference you have, being 'at rest' with it or 'moving' relative it. To define a intrinsic energy you have to be 'at rest' with your experimental setup. Light can both be said to 'red shift' and 'blue shift' depending on ones frame of reference, measuring it. It's in principle similar to two cars colliding, depending on relative speed(s), but in the photons frame we better avoid talking about 'mass'.
Whoahhh!:)
Title: Re: How do we measure the energy of a photon?
Post by: yor_on on 29/01/2019 11:42:43
One thing though. The speed of light in a vacuum as defined in physics is always 'c', so the 'red' respectively 'blue shift' then can be seen as a equivalence to the 'speed' you would use when it comes to two cars colliding, although the 'red shift' is different in that you now may measure the photon to have a lower 'energy' than it will have being 'at rest' with it. It's somewhat like two cars moving in the same direction colliding but not really as it there will be mass and speeds that define the 'energy' and their respective mass won't go 'negative' if you see what I mean. With a photons red shift it can though if we use the energy' you would measure being at rest (experimental setup) as a intrinsic energy of it. That won't happen with those cars :)
Title: Re: How do we measure the energy of a photon?
Post by: Colin2B on 29/01/2019 14:12:49
Great information..
But not such a great attempt to spam us.
Your website and signature have been changed to ours and - at the speed of light - you are banned.
Title: Re: How do we measure the energy of a photon?
Post by: mxplxxx on 31/01/2019 08:23:32
no, that's perfectly correct. the energy you measure a photon is dependent on what frame of reference you have, being 'at rest' with it or 'moving' relative it. To define a intrinsic energy you have to be 'at rest' with your experimental setup. Light can both be said to 'red shift' and 'blue shift' depending on ones frame of reference, measuring it. It's in principle similar to two cars colliding, depending on relative speed(s), but in the photons frame we better avoid talking about 'mass'.
Whoahhh!:)
Actually, it seems that energy is relative, so measuring the energy of a photon can only be done relative to the energy of another particle. Like an electron! ... but what is the energy of electrons measured in relation to? Relativity a can of worms:) . See also https://www.thenakedscientists.com/forum/index.php?topic=75704.msg566749#msg566749
Title: Re: How do we measure the energy of a photon?
Post by: mxplxxx on 31/01/2019 08:48:51
no, that's perfectly correct. the energy you measure a photon is dependent on what frame of reference you have, being 'at rest' with it or 'moving' relative it. To define a intrinsic energy you have to be 'at rest' with your experimental setup. Light can both be said to 'red shift' and 'blue shift' depending on ones frame of reference, measuring it. It's in principle similar to two cars colliding, depending on relative speed(s), but in the photons frame we better avoid talking about 'mass'.
Maybe only fermions have changeable state? and maybe it changes depending on the state of an interacting boson/ (rather that the boson actually causing the state change). This would place reality firmly into the realm of a finite state machine.
Title: Re: How do we measure the energy of a photon?
Post by: alancalverd on 31/01/2019 09:10:42
Energy is measured in joules, which are related to the definitions of mass, length and time. As stated above, a bolometer measures the absolute energy of photons.
Title: Re: How do we measure the energy of a photon?
Post by: mxplxxx on 31/01/2019 09:45:25
And an observer doesn't see a photon as ageless! Photons of considerable age will be redshifted according to Hubble's Law, so the energy of a photon compared with the expected energy of a freshly minted one from the same quantum process gives us an idea of how long it has been in the intergalactic outback.
I am aware of this. It seems to contradict relativity. Actually it may not. An EM wave is really three interacting waves/particles) ... but I will discuss this more in  New Theories, as soon as I have worked out how they interact:).
Actually, ridiculous as it may seem, it may make much more sense to think of photons as having an absolute speed of zero, which means that particles run into them rather than vice versa and the EM waves defines the energy of the photon and the problem of a photon having infinite mass does not arise. Not to mention there is no issue with accelerating the photon to the speed of light and having to account for that energy.

Title: Re: How do we measure the energy of a photon?
Post by: alancalverd on 31/01/2019 09:54:55
The problem of photons having infinite mass does not arise because photons have zero mass. There is no such thing as absolute speed.

Right now I am being bombarded with photons from the sun, a light  bulb, and this screen. The idea that I am running in at least three directions at once is absurd - men can't multitask!
Title: Re: How do we measure the energy of a photon?
Post by: mxplxxx on 31/01/2019 10:32:27
Right now I am being bombarded with photons from the sun, a light  bulb, and this screen. The idea that I am running in at least three directions at once is absurd - men can't multitask!
Fair enough:) Unless some other mechanism is also involved, like gravity.
Title: Re: How do we measure the energy of a photon?
Post by: mxplxxx on 31/01/2019 10:55:56
The problem of photons having infinite mass does not arise because photons have zero mass
Not according to e=mc2
Title: Re: How do we measure the energy of a photon?
Post by: mxplxxx on 31/01/2019 11:01:49
There is no such thing as absolute speed.
Except for SOL?
Title: Re: How do we measure the energy of a photon?
Post by: alancalverd on 31/01/2019 12:16:18
E equals all sorts of things: mgh, ½mv2, QV, you name it. mc2 is the energy released by the conversion of mass into energy - you can't have your cake and eat it!

The speed of light is a (universal) limit. "Absolute speed" implies a measurement against absolute coordinates, but all experimental evidence shows that there is no absolute coordinate - c is constant in all directions, regardless of the movement of the measuring platform.
Title: Re: How do we measure the energy of a photon?
Post by: mxplxxx on 31/01/2019 13:37:49
The speed of light is a (universal) limit. "Absolute speed" implies a measurement against absolute coordinates, but all experimental evidence shows that there is no absolute coordinate - c is constant in all directions, regardless of the movement of the measuring platform.

According to http://www.einstein-online.info/elementary/specialRT/speed_of_light.html

The speed of light is the only speed that is, in this sense, independent of the observer and thus absolute. It also plays the lead role in all of special relativity. First of all, it defines the absolute speed limit for the transfer of energy, matter and information.
Title: Re: How do we measure the energy of a photon?
Post by: mxplxxx on 31/01/2019 13:41:11
E equals all sorts of things: mgh, ½mv2, QV, you name it. mc2 is the energy released by the conversion of mass into energy - you can't have your cake and eat it!
I am pretty certain that physics teaches that mass and energy are equivalent.
Title: Re: How do we measure the energy of a photon?
Post by: alancalverd on 31/01/2019 18:03:11
Only very bad physics.

The dimensions of energy are ML2T-2. The dimensions of mass are M. Obviously not equivalent.
Title: Re: How do we measure the energy of a photon?
Post by: yor_on on 06/02/2019 03:49:39
MP "  I would say that nothing that moves at the speed of light can be measured. "
Not really, you  measure photons daily by your senses. Seeing is one.

And thinking of seeing in form of photons is still a really big headache to me, as waves it becomes more understandable. Light has to be a duality.
=

the problem with defining it this way becomes how ones body interpret it? Does it 'choose' one way over the other? How can it 'know' that this should be waves? I can't see how 'photons' and 'waves' are the same, which puts me in a almost mystic position here as one now might presume that the way the body interpret is more than a passive receptor.

and Alan, I suspect MP was thinking of https://en.wikipedia.org/wiki/Mass%E2%80%93energy_equivalence
=

you might be able to argue that the eye can see both, but that the brain 'picks' waves as that interpretation makes the most sense to us, possibly?
Title: Re: How do we measure the energy of a photon?
Post by: mxplxxx on 06/02/2019 06:10:57
MP "  I would say that nothing that moves at the speed of light can be measured. "
Not really, you  measure photons daily by your senses. Seeing is one.

Not single photons it would seem http://math.ucr.edu/home/baez/physics/Quantum/see_a_photon.html.

In any case, photons we see are destroyed by the process which means? If we want to measure a photon, according to relativity, we need to be in the same frame of reference as the photon, but we can never attain the speed of light.
Title: Re: How do we measure the energy of a photon?
Post by: alancalverd on 06/02/2019 17:26:49
and Alan, I suspect MP was thinking of https://en.wikipedia.org/wiki/Mass%E2%80%93energy_equivalence

Wikipedia is very precise in its wording:

Quote
In physics, mass–energy equivalence states that anything having mass has an equivalent amount of energy and vice versa,

my emphasis. Note "has" , not "is". I have £10 in my pocket, I am not made of money!
Title: Re: How do we measure the energy of a photon?
Post by: alancalverd on 06/02/2019 17:31:39
Many things cannot be measured without changing them. How would you measure the kinetic energy of a bullet? You could measure its speed, then stop it and weigh it, but in doing so you have dissipated the energy you set out to measure. Likewise the energy of a photon.
Title: Re: How do we measure the energy of a photon?
Post by: syhprum on 06/02/2019 17:40:54
You don't have to stop and weigh it you simply measure its path in a known gravitational field the same way as we determine the mass of planets around remote stars
Title: Re: How do we measure the energy of a photon?
Post by: Bored chemist on 06/02/2019 19:07:10
You don't have to stop and weigh it you simply measure its path in a known gravitational field the same way as we determine the mass of planets around remote stars
It's actually easier to measure the mass of ions and electrons when they are moving.
https://en.wikipedia.org/wiki/Mass_spectrometry
Title: Re: How do we measure the energy of a photon?
Post by: mxplxxx on 07/02/2019 00:13:45
You don't have to stop and weigh it you simply measure its path in a known gravitational field the same way as we determine the mass of planets around remote stars
So, ditto for a photon?
Title: Re: How do we measure the energy of a photon?
Post by: chiralSPO on 07/02/2019 03:46:22
You don't have to stop and weigh it you simply measure its path in a known gravitational field the same way as we determine the mass of planets around remote stars
So, ditto for a photon?

Unfortunately, photons don't have mass or charge, so this type of approach doesn't work.
Title: Re: How do we measure the energy of a photon?
Post by: mxplxxx on 07/02/2019 03:57:14
Unfortunately, photons don't have mass or charge, so this type of approach doesn't work.
But light is curved by massive objects? I wonder what determines how much it is curved by? Like most things photonic, this curvature is little understood. https://physics.stackexchange.com/questions/107930/why-can-light-photons-bends-in-a-curve-through-space-without-mass
Title: Re: How do we measure the energy of a photon?
Post by: chiralSPO on 07/02/2019 04:46:54
Space is curved by massive objects. Light goes "straight" within that space. This is very different from the force of gravity on other massive objects.
Title: Re: How do we measure the energy of a photon?
Post by: mxplxxx on 07/02/2019 06:03:35
Space is curved by massive objects. Light goes "straight" within that space. This is very different from the force of gravity on other massive objects.
So massive objects attract each other as well as "falling" towards each other via space curvature. A double whammy?
Title: Re: How do we measure the energy of a photon?
Post by: mxplxxx on 07/02/2019 06:09:58
 
my emphasis. Note "has" , not "is". I have £10 in my pocket, I am not made of money!
But the $10 note IS made of money.
Title: Re: How do we measure the energy of a photon?
Post by: yor_on on 07/02/2019 08:00:42
MP "  I would say that nothing that moves at the speed of light can be measured. "
Not really, you  measure photons daily by your senses. Seeing is one.

Not single photons it would seem http://math.ucr.edu/home/baez/physics/Quantum/see_a_photon.html.


No, Baez is interpreting the statistics, but the experiment did prove that a human eye can see a 'photon', and if you think of a highly energetic 'photon' (gamma) it should even leave a mark.
Title: Re: How do we measure the energy of a photon?
Post by: yor_on on 07/02/2019 08:07:44
:)

Yes, wordings can be tricky. Still, energy densities seems a accepted term for it, if we now use language. At least it puts it as the energy of 'something' becomes a 'density', which can be interpreted as something defined through a limit, measurably containing a 'mass' of sorts, be it relativistic or not. Although I have such a grave problem seeing what should be defined as 'containing' that relativistic mass, as it isn't locally definable.
=

It would have been so much simpler if we by measuring a light in a 'black box scenario' could prove for example a 'relative speed'. Then we all could sit back and smile, as the universe started to make sense :)
Title: Re: How do we measure the energy of a photon?
Post by: yor_on on 07/02/2019 08:22:27
Depends on definitions. A geodesic isn't a 'force', in Einsteins terms it is more of the 'straightest path' something can take. Any deviation from said geodesic needs a acceleration, which then should cost 'energy'. But throw one kg in the air, and stand under it as it comes down, that you will define as a 'force' acting upon you, as you and Earth acts upon it.

But then you also have to remember that in Einsteins terms, Earth is constantly and uniformly 'accelerating' at one gravity approximately. So, standing on a accelerating object 'hitting' that kg? That gotta hurt.
=

Thinking of it we have three terms at least for what a motion is. Acceleration is the easiest one to define as it acts locally measured.

Then we have 'relative (uniform) motions'. There you have two types, one becomes a 'real motion' as defined relative something else, but on another tentacle it also can be defined as 'locally' still. This should be read that if you only have two objects on a collision course in a space, then you're free to define your and his 'relative motion' any way you like, him moving or you, or both. There exist no locally definable proof of you moving. Neither does it exist for the other object but as you see, you're still on a collision course.

And when you hit the energy released from that collision will be more than if both had been 'still'. That's also called a 'potential energy' of something happening. You can have a untold magnitude 'potential energies' simultaneously just by introducing more objects coming at you in that space, it all depends on your frame of reference.

although, if we introduce distance over time, you will be able to define who's going to hit first, and by that introduce only one 'outside' frame of reference existing creating your 'potential energy'.

Doing so you're still bound through observer dependencies though as with a higher 'relative speed' the universe around you must change both 'clock rates' as well as contract.

But if we now go back to the first postulate in where you locally has no way to define a relative motion at all, except relative another frame of reference? What then is your 'relative speed'? Relative what? A suns blueshift?
=

Actually, presuming you to be alone with just another object in a space, hurling against each other relatively, close to the speed of light. You're still unable to define who's 'moving'. So, you're free to define yourself in any way you like, from you speeding away to not 'moving' at all.

So, what happened to those 'clock rates'?
solve that one :)

==

If we look at it another way we can use NIST experiments with 'gravity' and elevations. There we find 'clock rates' to change relative ones wristwatch, by changing the elevation of those atomic clocks they use. That is a result of different 'accelerations' in Einsteinian terms, as 'gravity' then is a equivalence to a acceleration.

Against it we have that different relative motions (uniform motions) can be proven to exist, and will introduce time dilation's as well as LorentzFitzgerald contractions.

Now imagine the atomic clock resting on a shelf, showing you a time dilation relative your wristwatch. Then let the atomic clock fall of that shelf. As it does it's no longer in a 'acceleration', it's in a 'free fall', aka a 'geodesic', aka 'being still, aka being in a 'relative (uniform) motion, even though you find it to gravitationally accelerate. But that 'acceleration' you give the atomic clock is in Einsteins terms a result of Earth constantly uniformly accelerating at one gravity. So one could look at it as the clock passing different 'gravitaional potentials' in its 'free fall'.

Still, the clock is now in a 'relative motion' versus Earth, aka being 'still' in a geodesic. What happens to its 'clock rate'? Do you think it will change? Will it matter for the 'clock rate' measured in that geodesic how many 'gravities' it formerly accelerated at, resting on the shelf? Give that clock a really long 'free fall' and imagine that you have a way to see its dials.

No matter what 'accelerating gravity' it had, do you think the dials will present different readings in that geodesic (free fall) towards a floor?

Is gravity transformed away?
===

a third way is to use 'black box scenarios'. Doing so we encase the the atomic clock, you inside, placing ourselves at rest with it following its 'free fall'. Did your wrist watch ever change its 'clock rate' relative that atomic clock in our 'free fall' towards the floor?

Furthermore, did you notice it change on the shelf?
In what way did you then transform gravity away?
===

the one with defining yourself as being still will hold no matter how many objects you introduce, as well as defining some other frame of reference to be the one not moving.
Title: Re: How do we measure the energy of a photon?
Post by: mxplxxx on 08/02/2019 05:10:12
There exist no locally definable proof of you moving.
Except the fact that is blindingly obvious that you ARE moving:)
Title: Re: How do we measure the energy of a photon?
Post by: yor_on on 08/02/2019 07:36:22
Well MP, that's what I would call a 'global approach' to the puzzle. You do that one by introducing two objects and yourself in a space, at different speeds. Doing so you find that even though any motion is 'relative' different speeds do exist. Locally though there exist no proof of a difference between different relative speeds. That one you prove by using Einsteins famous 'black box' in where there is no way to differ between them, including testing by making 'repeatable experiments'.
Title: Re: How do we measure the energy of a photon?
Post by: mxplxxx on 08/02/2019 08:02:14
Locally though there exist no proof of a difference between different relative speeds. That one you prove by using Einsteins famous 'black box' in where there is no way to differ between them, including testing by making 'repeatable experiments'.
But the world is not a black box?
Title: Re: How do we measure the energy of a photon?
Post by: yor_on on 08/02/2019 08:41:43
No, not frames of reference interacting. But most of what you do and see is 'observer dependent', probably all. An observer dependency is always local, with the 'universe' being the place where you act, and get acted on, observer dependently.
Title: Re: How do we measure the energy of a photon?
Post by: Bored chemist on 08/02/2019 09:39:19
Except the fact that is blindingly obvious that you ARE moving:)
Not, for example, to an ant walking around through the hair on your head.
Title: Re: How do we measure the energy of a photon?
Post by: mxplxxx on 08/02/2019 10:36:11
Not, for example, to an ant walking around through the hair on your head.
The ant is aware it is moving (if ants can be aware and can process movement!)
Title: Re: How do we measure the energy of a photon?
Post by: Bored chemist on 08/02/2019 10:48:08
No, the ant thinks it's sitting still on your head. It may think that other things which it can see are moving. For example, it might see a bird fly past and think the bird is moving. But it's sure that you are stationary.
It's the same issue as the geocentric/ heliocentric view of the universe.
Title: Re: How do we measure the energy of a photon?
Post by: syhprum on 08/02/2019 11:09:07
It is indeed possible to measure the frequency of a photon with a ruler ,photons come in all shapes and sizes they are not all in the nanometre range.
In the early days of amateur radio one of the techniques for measuring the frequency was to measure the distance between null and peak voltages on an unterminated air spaced transmission line.   
Title: Re: How do we measure the energy of a photon?
Post by: mxplxxx on 08/02/2019 12:23:41
It is indeed possible to measure the frequency of a photon with a ruler ,photons come in all shapes and sizes they are not all in the nanometre range.
In the early days of amateur radio one of the techniques for measuring the frequency was to measure the distance between null and peak voltages on an unterminated air spaced transmission line.   
It is energy I am interested in measuring. I do not want to calculate energy, I want to measure it.
Title: Re: How do we measure the energy of a photon?
Post by: Bored chemist on 08/02/2019 12:32:01
I want to measure it.
Then I strongly urge you to look again at this post.
Somewhere ikn the last 50 years, I forgot about the GeLi gamma spectrometers we used to use to calibrate x-ray sources. Wikipedia has a good explanatory article that I won't repeat here, but the key is that you can use a gamma emitter with a narrow spectrum to locate a single point on the resultant spectrum and thus measure the energy of any photon against your calibration point.

Getting down to single photon detection with GeLi is a bit of an art form but since we know photons are indistinguishable, if we find a narrow line on a spectrum we can say that it does at least represent the average energy of a few photons.

It's interesting that the questioner persists in telling us that something can't be done, in the face of the experience of those who have done it for a living.  Unshakeable faith in one's prejudice can be dangerous, but as a forlorn gesture of friendship I'll advise people not to play with fire or walk in front of moving buses.

Title: Re: How do we measure the energy of a photon?
Post by: alancalverd on 08/02/2019 17:05:52
It is not only well understood (though often poorly explained) but completely demonstrated by experiment. Problem is that gravitational lensing is not dependent on photon energy.
Title: Re: How do we measure the energy of a photon?
Post by: esquire on 13/02/2019 14:42:41
each photon carries with it seperate waves of intensity from gamma to am. each of these seperate waves are subject to reflective elongation which in turn determines the strentgh of the intensity present in its 9 separate spectum waves of energy, momemtum, non-mass. also, the fact that its waves can be absorbed via different elements, presents another conundrum of being able to accurately measure the energy weight of a photon. so, there is an elongation issue that alters the wave's value. there is an absorbtion factor that alters it's value strength. there is also the speed of light momentum issue. that via the uncertainity principle can only provides an approviamation of single variable, which renders any determination scientifically invalid.
Title: Re: How do we measure the energy of a photon?
Post by: esquire on 13/02/2019 14:58:21
this image is of an attempt to weight a proton and difficulties this presents. the proton waves actually breaks apart into multiple waves in liquid hydrogen or helium. the attempt was to weight the bubble mass as the proton wave  cycled through the cylinder. the experiment fails because the proton wave, breaks into multiple wave of varying sized bubbles, never of the same size.  scientist remain stumped for an explanation. 

https:  //www.gettyimages.com/detail/news-photo/photograph-of-a-bev-proton-from-the-cosmotron-colliding-news-photo/929284356
Title: Re: How do we measure the energy of a photon?
Post by: Bored chemist on 13/02/2019 19:01:50
this image is of an attempt to weight a proton
No it isn't.
scientist remain stumped for an explanation.
No, they are not.
each photon carries with it seperate waves of intensity from gamma to am. each of these seperate waves are subject to reflective elongation which in turn determines the strentgh of the intensity present in its 9 separate spectum waves of energy, momemtum, non-mass. also, the fact that its waves can be absorbed via different elements, presents another conundrum of being able to accurately measure the energy weight of a photon. so, there is an elongation issue that alters the wave's value. there is an absorbtion factor that alters it's value strength. there is also the speed of light momentum issue. that via the uncertainity principle can only provides an approviamation of single variable, which renders any determination scientifically invalid.

Was that a quote from the script for Star Trek?
Title: Re: How do we measure the energy of a photon?
Post by: esquire on 13/02/2019 20:23:24
this image is of an attempt to weight a proton
No it isn't.

https://en.wikipedia.org/wiki/Bubble_chamber

The bubble chamber is similar to a cloud chamber, both in application and in basic principle. It is normally made by filling a large cylinder with a liquid heated to just below its boiling point. As particles enter the chamber, a piston suddenly decreases its pressure, and the liquid enters into a superheated, metastable phase. Charged particles create an ionization track, around which the liquid vaporizes, forming microscopic bubbles. Bubble density around a track is proportional to a particle's energy loss.

energy loss is a derivative function of the weight of the mass. is it not?

scientist remain stumped for an explanation.
No, they are not.


https://news.brown.edu/articles/2014/10/electron

although the following article provided by yor on, involves a electrons instead of protons, the experiment is similar and revolves around the measurement of electrons via their wave bubble displacement. displacement, a process for the measurement of weight. energy loss in both cases of protons and electrons being measured by their bubble wave displacement. 

as per the articles summary under "new experiments".
“We think this offers the best explanation for what we see in the experiments,” Maris said. We’ve got this body of data that goes back 40 years. The experiments are not wrong; they’ve been done by multiple people. We have a tradition called Occam’s razor, where we try to come up with the simplest explanation. This, so far as we can tell, is it.”

“No one is sure what actually constitutes a measurement. Perhaps physicists can agree that someone with a Ph.D. wearing a white coat sitting in the lab of a famous university can make measurements. But what about somebody who really isn’t sure what they are doing? Is consciousness required? We don’t really know.”


each photon carries with it separate waves of intensity from gamma to am. each of these separate waves are subject to reflective elongation which in turn determines the strength of the intensity present in its 9 separate spectrum waves of energy, momentum, non-mass. also, the fact that its waves can be absorbed via different elements, presents another conundrum of being able to accurately measure the energy weight of a photon. so, there is an elongation issue that alters the wave's value. there is an absorption factor that alters it's value strength. there is also the speed of light momentum issue. that via the uncertainty principle can only provides an approximation of a single variable, which renders any determination scientifically invalid.

Was that a quote from the script for Star Trek?

Unless you can dispute any of the facts cited above, please cite any alternate set of facts that you may use for your lack of ideas. I see many of your contentions have already been discounted. I truly hope such your pretentious is not contagious. lol

Title: Re: How do we measure the energy of a photon?
Post by: Bored chemist on 13/02/2019 20:47:29
energy loss is a derivative function of the weight of the mass. is it not?
Not really, no.
When people talk about the mass of a proton, they mean the rest mass.
And that's independent of energy.
Unless you can dispute any of the facts cited above,
No.
You made it up.
It is your job to prove that it is true.
It isn't anyone else's job to show it is false.

Over to you...
Title: Re: How do we measure the energy of a photon?
Post by: esquire on 13/02/2019 23:05:34
energy loss is a derivative function of the weight of the mass. is it not?

Not really, no.
When people talk about the mass of a proton, they mean the rest mass.
And that's independent of energy.

The mass of a proton is 1.67 x 10-27kg, the mass is addressed as a weight at rest. the particle charge is independent of energy and that is measured. however when the proton wave collapses into multiple bubbles, in random pattern sizes the measuring of the proton energy becomes problematic. the weighted bubble sizes can offer no clear method for determining the energy loss, hence an accurate approximation of the energy for a proton is indeterminate. the difficulty in determining the energy of the proton pales in comparison to that of a photon, which was my point. 

 
Unless you can dispute any of the facts cited above,
No.
You made it up.
It is your job to prove that it is true.
It isn't anyone else's job to show it is false.

Over to you...

still you didn't dispute any of the facts listed as to why the energy cannot be determined. did you? lol
Title: Re: How do we measure the energy of a photon?
Post by: esquire on 13/02/2019 23:08:23
energy loss is a derivative function of the weight of the mass. is it not?

Not really, no.
When people talk about the mass of a proton, they mean the rest mass.
And that's independent of energy.

The mass of a proton is 1.67 x 10-27kg, the mass is addressed as a weight at rest. the particle charge is independent of energy and that is measured. however when the proton wave collapses into multiple bubbles, in random pattern sizes the measuring of the proton energy becomes problematic. the weighted bubble sizes can offer no clear method for determining the energy loss, hence an accurate approximation of the energy for a proton is indeterminate. the difficulty in determining the energy of the proton pales in comparison to that of a photon, which was my point. 

 
Unless you can dispute any of the facts cited above,
No.
You made it up.
It is your job to prove that it is true.
It isn't anyone else's job to show it is false.

Over to you...

still you didn't dispute any of the facts listed as to why the energy of a photon cannot be determined. did you? you know why, because you can't.  lol

each photon carries with it separate waves of intensity from gamma to am. each of these separate waves are subject to reflective elongation which in turn determines the strength of the intensity present in its 9 separate spectrum waves of energy, momentum, non-mass. also, the fact that its waves can be absorbed via different elements, presents another conundrum of being able to accurately measure the energy weight of a photon. so, there is an elongation issue that alters the wave's value. there is an absorption factor that alters it's value strength. there is also the speed of light momentum issue. that via the uncertainty principle can only provides an approximation of a single variable, which renders any determination scientifically invalid.

Title: Re: How do we measure the energy of a photon?
Post by: alancalverd on 13/02/2019 23:49:41
Having spent many happy hours measuring the energy of photons by various means over many years, I find it surprising to be told that it can't be done.

But then anyone who thinks "waves of intensity" belongs in a scientific discussion has clearly been reading too many Mills & Boone bodice-rippers and not enough elementary physics texts.
Title: Re: How do we measure the energy of a photon?
Post by: esquire on 14/02/2019 00:06:18
Having spent many happy hours measuring the energy of photons by various means over many years, I find it surprising to be told that it can't be done.

But then anyone who thinks "waves of intensity" belongs in a scientific discussion has clearly been reading too many Mills & Boone bodice-rippers and not enough elementary physics texts.

purist - 1.
a person who insists on absolute adherence to traditional rules or structures, especially in language or style.
Title: Re: How do we measure the energy of a photon?
Post by: esquire on 14/02/2019 00:14:48
Having spent many happy hours measuring the energy of photons by various means over many years, I find it surprising to be told that it can't be done.

But then anyone who thinks "waves of intensity" belongs in a scientific discussion has clearly been reading too many Mills & Boone bodice-rippers and not enough elementary physics texts.

no doubt that some peoples purist attitudes are rather selective! ex:

It's a bit like colour. Some things, like the river thames,  are brown.
You can say that the Thames flows, but you can't say that brown flows.

no disassociation here, over flowery language, is there? that makes your comments questionable.
Title: Re: How do we measure the energy of a photon?
Post by: esquire on 14/02/2019 00:17:11
Having spent many happy hours measuring the energy of photons by various means over many years, I find it surprising to be told that it can't be done.

But then anyone who thinks "waves of intensity" belongs in a scientific discussion has clearly been reading too many Mills & Boone bodice-rippers and not enough elementary physics texts.

no doubt that some peoples purist attitudes are rather selective! ex:

It's a bit like colour. Some things, like the river thames,  are brown.
You can say that the Thames flows, but you can't say that brown flows.

no disassociation here, over flowery language, is there? that makes your comments questionable.

alan , please explain your selective approach at commentary.
Title: Re: How do we measure the energy of a photon?
Post by: esquire on 14/02/2019 00:24:26
Having spent many happy hours measuring the energy of photons by various means over many years, I find it surprising to be told that it can't be done.


and your measurements, do you feel that they will never be challenged in the future by better methods. if you cannot say that with 100% certainty, then I question whether the methods you employed or the results of those methods are valid. .   
Title: Re: How do we measure the energy of a photon?
Post by: esquire on 14/02/2019 00:27:02
and alan please don't arbitrarily ban me without offering me a chance to respond to your reply.
Title: Re: How do we measure the energy of a photon?
Post by: esquire on 14/02/2019 00:48:12
Since all photons in monochromatic light have the same energy (by definition of "monochromatic") then any measurement of the wavelength of that light (including x-ray photons) is a measure of the energy of a single photon.

https:   //www.quora.com/What-is-monochromatic-light
Monochromatic means "same color". Basically monochromatic is any light which is purely of a single color. In modern terms it is a light of same wavelength.

so, a photon measured in energy in a Gamma spectrum does not have the same energy as a photon measured in a AM frequency.  yet a photon is a photon. it's malleability is wave intensity as I asserted.   

Title: Re: How do we measure the energy of a photon?
Post by: chiralSPO on 14/02/2019 03:56:38
purist - 1.
a person who insists on absolute adherence to traditional rules or structures, especially in language or style.

Science requires the use of terminology with rigorous and specific definitions. In colloquial English, "forceful," "energetic," "intense," and "powerful" all can be used somewhat interchangeably. In science, each has a different and very specific meaning.

PS: esquire, please don't spam the threads with multiple posts. Also, most of your assertions are very wrong...
Title: Re: How do we measure the energy of a photon?
Post by: Bored chemist on 14/02/2019 07:34:22
still you didn't dispute any of the facts listed as to why the energy of a photon cannot be determined. did you? you know why, because you can't.  lol
It's true that I can't dispute facts.
However, you didn't post any.
Title: Re: How do we measure the energy of a photon?
Post by: alancalverd on 14/02/2019 16:37:30
so, a photon measured in energy in a Gamma spectrum does not have the same energy as a photon measured in a AM frequency.  yet a photon is a photon. it's malleability is wave intensity as I asserted.   

A pigmy shrew does not have the same mass as a whale. So what? A mammal is a mammal.

If "wave intensity" means anything, it might just refer to the root mean square of its amplitude, which isn't at all relevant to the energy of a photon.

It is important, in any discussion, to use a common language. The language of physics is very precise.  Merely assembling sciency words into what looks like a sentence with a question mark, won't improve your understanding: if you don't understand the question, you certainly won't understand the answer.

In response to the meaningful question, I have no doubt that there are ever more accurate measurements of everything that has ever been measured. That is why we are moving to redefinitions of all primary standards in terms of universal constants rather than lumps of metal, but it doesn't alter the fact that the energy of a photon is measurable in priniciple and adequately measurable in practice.

Those of a philosophical bent may indeed ask "what do you mean by measure?" but the straight answer is always "compare x with y where y is an agreed standard".
Title: Re: How do we measure the energy of a photon?
Post by: jeffreyH on 14/02/2019 18:45:28
@esquire Garbage in garbage out. You dump garbage in the forum, we can always bin it for you. You have been given various correct answers which you simply won't accept. The moderators here are all volunteers. They have better things to do than put your toys back in the pram. Please try harder to be a valued member. You never know you may learn enough to help others.
Title: Re: How do we measure the energy of a photon?
Post by: Bored chemist on 14/02/2019 18:48:37
and your measurements, do you feel that they will never be challenged in the future by better methods. if you cannot say that with 100% certainty, then I question whether the methods you employed or the results of those methods are valid. . 
Your point seems absurd.
A thousand years ago you would be lucky to find a clock that told the time reliably to the nearest hour.
By the 1920s we had clocks that kept time to about a second per year.
Today we have clocks that measure the time to tiny fractions of a second.

But, by your "logic", because we now measure time better, we were not previously measuring it at all.

Title: Re: How do we measure the energy of a photon?
Post by: mxplxxx on 19/02/2019 00:06:16
It seems to me that to measure the energy of a photon you have to be moving at the speed of light. According to relativity, a photon will perceive the universe as a mass of stationary bosons oscillating around fixed points according to their composite electromagnetic waves (interesting! we may live in two same-space interacting universes, one inhabited by bosons and one by fermions). So, somehow or other send a photon to measure another photon.
Title: Re: How do we measure the energy of a photon?
Post by: chiralSPO on 19/02/2019 01:12:05
It seems to me that to measure the energy of a photon you have to be moving at the speed of light.

This is a pretty unrealistic requirement. This thread is *full* of ways to measure the energy of a photon, and none of them has any requirement for velocity!

Actually, when moving at such speeds, measurements will tell you more about the angle of your trajectory with respect to the trajectory of the photon of interest, than anything intrinsic to the photon (and will be essentially worthless to anyone in a different frame of reference...)
Title: Re: How do we measure the energy of a photon?
Post by: mxplxxx on 19/02/2019 01:57:37
This thread is *full* of ways to measure the energy of a photon
Not true, unfortunately. Our chances of being able to measure the energy of a PARTICULAR photon seem to be  vanishingly slim. And even then, mostly it is the frequency that is measured, not the energy.
Title: Re: How do we measure the energy of a photon?
Post by: chiralSPO on 19/02/2019 02:29:37
mxplxxx, I would try to debate, but I see now that it is not worth my time, so I will leave you with this:

"There are none so blind as those who will not see."

I do hope that you will eventually open your mind to the possibility of experimentally determining facts, but I can not and will not force you to try to understand. Good day.
Title: Re: How do we measure the energy of a photon?
Post by: mxplxxx on 19/02/2019 03:39:40
mxplxxx, I would try to debate, but I see now that it is not worth my time, so I will leave you with this:

"There are none so blind as those who will not see."

I do hope that you will eventually open your mind to the possibility of experimentally determining facts, but I can not and will not force you to try to understand. Good day.
Pretty condescending! I repeat - this forum demonstrates, only too well,we cannot directly measure the energy of a photon and those methods that measure it indirectly do so for limited sets of photons.
Title: Re: How do we measure the energy of a photon?
Post by: Bored chemist on 19/02/2019 07:31:12
Pretty condescending! I
No, just accurate.

I repeat - this forum demonstrates, only too well,we cannot directly measure the energy of a photon and those methods that measure it indirectly do so for limited sets of photons.
You certainly do repeat that.
Many times.

It still isn't right.

People really have measured photon energies.
Title: Re: How do we measure the energy of a photon?
Post by: alancalverd on 19/02/2019 07:44:11
Perhaps mxplxxx would care to give us his definition of energy?
Title: Re: How do we measure the energy of a photon?
Post by: mxplxxx on 19/02/2019 08:56:45
Quote from: mxplxxx on Today at 03:39:40
Pretty condescending! I
No, just accurate.

Definitely condescending, as you often are, as per:

Dictionary result for condescending
/kɒndɪˈsɛndɪŋ/Submit
adjective
having or showing an attitude of patronizing superiority.
"she thought the teachers were arrogant and condescending"
synonyms:   patronizing, supercilious, superior, sn
Title: Re: How do we measure the energy of a photon?
Post by: mxplxxx on 19/02/2019 09:31:02
Perhaps mxplxxx would care to give us his definition of energy?
Don't like the standard "work" definition in physics mainly because it doesn't include that fact that energy is relative. For me a finite state machine-oriented definition is the go:

Energy is the ability of one object to change another object  based on the current states of the two objects.

It is interesting to note that because of relativity and the speed of light, a particular photon will have the same relationship and therefore the same energy with respect to all other particles in the universe. And vice versa, a particular particle will have the same relationship as all other particles in the universe to a particular photon. This is a very good reason why photons must travel at the speed of light. In other words events in the universe must appear the same to all particles in the universe.
Title: Re: How do we measure the energy of a photon?
Post by: alancalverd on 19/02/2019 19:09:20
Energy is the ability of one object to change another object  based on the current states of the two objects.

There is the nub of the problem. You might find a discussion more to your taste in a forum devoted to New Age thinking or even metaphysics, but you won't get much sympathy from scientists if you insist on speaking a wholly idiosyncratic language in which conventionally precise words have imprecise meanings..
Title: Re: How do we measure the energy of a photon?
Post by: Bored chemist on 19/02/2019 19:29:53
Quote from: mxplxxx on Today at 03:39:40
Pretty condescending! I
No, just accurate.

Definitely condescending, as you often are, as per:

Dictionary result for condescending
/kɒndɪˈsɛndɪŋ/Submit
adjective
having or showing an attitude of patronizing superiority.
"she thought the teachers were arrogant and condescending"
synonyms:   patronizing, supercilious, superior, sn
How do you distinguish that from simply being right?


Given that you seem to be trying to change the definitions of terms like energy while explaining that everybody except you is wrong, who is being arrogant and condescending?
Title: Re: How do we measure the energy of a photon?
Post by: mxplxxx on 19/02/2019 22:10:33
How do you distinguish that from simply being right?
By the effect on me. Maybe I am too touchy but I am a degree-qualified life coach so this possibility is remote.
Title: Re: How do we measure the energy of a photon?
Post by: mxplxxx on 19/02/2019 22:11:58
Given that you seem to be trying to change the definitions of terms like energy while explaining that everybody except you is wrong
Not true, I was asked for my definition of energy and where have I said that physics is wrong?
Title: Re: How do we measure the energy of a photon?
Post by: mxplxxx on 19/02/2019 22:13:55
You might find a discussion more to your taste in a forum devoted to New Age thinking or even metaphysics
Condescending! You asked me for my definition of energy. I gave it:).
Title: Re: How do we measure the energy of a photon?
Post by: alancalverd on 19/02/2019 23:18:56
Perceptive, not condescending. The use of "energy" to mean something other than the capacity to do work, is characteristic of New Age discussions and some branches of metaphysics, and has no place in a scientific forum.
Title: Re: How do we measure the energy of a photon?
Post by: PmbPhy on 20/02/2019 00:34:22
You might find a discussion more to your taste in a forum devoted to New Age thinking or even metaphysics
Condescending! You asked me for my definition of energy. I gave it:).
The definition is on my website at
http://www.newenglandphysics.org/physics_world/cm/what_is_energy.htm

Its what Feynman used in his Lectures.
Title: Re: How do we measure the energy of a photon?
Post by: yor_on on 20/02/2019 01:09:22
Reading your link Pete reminded me of what JP called it. 'A coin of exchange' I think it was.
Title: Re: How do we measure the energy of a photon?
Post by: mxplxxx on 20/02/2019 01:11:37
Perceptive, not condescending. The use of "energy" to mean something other than the capacity to do work, is characteristic of New Age discussions and some branches of metaphysics, and has no place in a scientific forum.
Maybe. But YOU asked for it!
Title: Re: How do we measure the energy of a photon?
Post by: mxplxxx on 20/02/2019 01:31:56
There is the nub of the problem. You might find a discussion more to your taste in a forum devoted to New Age thinking or even metaphysics, but you won't get much sympathy from scientists if you insist on speaking a wholly idiosyncratic language in which conventionally precise words have imprecise meanings..
What problem?

My definition I would have thought is a easily understood generalization (or abstraction if you like). It is not meant to be precise but there is nothing idiosyncratic about it that I can see (and 2 of my top 7 strengths are "explainer" and "simplifier"). A  concrete definition would , maybe, involve the momentum of two objects.

The standard physics definition of energy as "work", as I said, does not involve the relative nature of energy. It is also a local definition that relies on the gravity of earth which differs from point to point. Not to mention that when I google "work" I get it defined in terms of energy! 18th century stuff that needs updating.
Title: Re: How do we measure the energy of a photon?
Post by: mxplxxx on 20/02/2019 01:42:29
You might find a discussion more to your taste in a forum devoted to New Age thinking or even metaphysics
Condescending! You asked me for my definition of energy. I gave it:).
The definition is on my website at
http://www.newenglandphysics.org/physics_world/cm/what_is_energy.htm

Its what Feynman used in his Lectures.

Thx PmbPhy. You point out that defining something like energy is not easy. I couldn't agree more. But you don't give Feynman's definition of energy. In fact you say:

 Richard Feynman wrote [2]

It is important to realize that in physics today, we have no knowledge of what energy is. We do not have a picture that energy comes in little blobs of a definite amount. It is not that way. However, there are formulas for calculating some numerical quantity, and we add it all together it gives “28” -  always the same number. It is an abstract thing in that it does not tell us the mechanism or the reasons for the various formulas.
Title: Re: How do we measure the energy of a photon?
Post by: PmbPhy on 20/02/2019 05:31:28
Thx PmbPhy. You point out that defining something like energy is not easy. I couldn't agree more. But you don't give Feynman's definition of energy. In fact you say:

 Richard Feynman wrote [2]

It is important to realize that in physics today, we have no knowledge of what energy is. We do not have a picture that energy comes in little blobs of a definite amount. It is not that way. However, there are formulas for calculating some numerical quantity, and we add it all together it gives “28” -  always the same number. It is an abstract thing in that it does not tell us the mechanism or the reasons for the various formulas.
In fact my page doesn't define energy. It just describes the concept. I should have pointed that out. Feynman doesn't define energy either.
Title: Re: How do we measure the energy of a photon?
Post by: mxplxxx on 20/02/2019 06:02:14
So, it would seem that physics cannot precisely define what energy is. That would seem to make measuring the energy of a photon very difficult! Sort of like a Will of the Wisp:)
Title: Re: How do we measure the energy of a photon?
Post by: yor_on on 20/02/2019 09:44:50
Knowing what 'energy' is isn't the same thing as not being able to measure it mxplxxx?
Temperature is a measure of 'energy' for example.
Title: Re: How do we measure the energy of a photon?
Post by: mxplxxx on 20/02/2019 10:28:03
Temperature is a measure of 'energy' for example.
Temperature is a measure of Thermal energy of an object. And thermal energy is the internal energy of an atom. Both Googled.
You could go on forever and still not be sure what you are measuring in a photon.
Title: Re: How do we measure the energy of a photon?
Post by: alancalverd on 20/02/2019 12:22:23
And thermal energy is the internal energy of an atom.
No. It is the kinetic energy of all the atoms in a body.
Quote
You could go on forever and still not be sure what you are measuring in a photon.
On the contrary, the energy of every photon I have measured was its ability to do work, usually releasing electrons from a photocathode, heating a block of graphite, causing a current to flow in a circuit, ionising air, or initiating a chemical reaction.

Where I part company with philosophers is over their insistence on asking what something "is", because that can lead to a sterile reductio ad nauseam. In the case of energy or elephants, it is a lot more productive to define its characteristics. Energy eats shoots and leaves, elephants are conserved. I know because a friend works in elephant conservation - much more reliable than Google.
Title: Re: How do we measure the energy of a photon?
Post by: yor_on on 20/02/2019 12:31:54
Don't get stuck on words MXP
There is a lot we don't know.

But we can still measure the 'coin of exchange'.
Temperature is one.
Title: Re: How do we measure the energy of a photon?
Post by: mxplxxx on 21/02/2019 04:58:23
Don't get stuck on words MXP
There is a lot we don't know.

But we can still measure the 'coin of exchange'.
Temperature is one.
It seem to me what is being measured is the effects of something we call energy and we are not too sure exactly what energy is but the science of it is very useful anyway.

ps1 I think energy, like the speed of light, is absolute, not relative as I have previously stated (and not been challenged). If a particle absorbs a photon, it likely changes its momentum (and other?) relationship to ALL other particles. This makes sense given that a photon is not a relational entity. In fact, it seems to me that measuring the momentum of a particle before and after it has absorbed photon should give the  energy of the photon.

ps2 talking about the energy of a photon is meaningless anyway without the time to transfer the energy being involved. i.e. we should be measuring power, not energy.
Title: Re: How do we measure the energy of a photon?
Post by: alancalverd on 21/02/2019 07:28:33
ps2 talking about the energy of a photon is meaningless anyway with the time to transfer the energy being involved. i.e. we should be measuring power, not energy.
However you measure the energy of Co-60 gamma photons, you get the same result, though the initial interactions are very different. The constants (energy) tell us something about the source of the photons and are predictive of their other interactions. The power, though measurable (depending on how you define it) is not constant, explanatory or usefully predictive.
Title: Re: How do we measure the energy of a photon?
Post by: mxplxxx on 21/02/2019 09:15:24
ps2 talking about the energy of a photon is meaningless anyway with the time to transfer the energy being involved. i.e. we should be measuring power, not energy.
However you measure the energy of Co-60 gamma photons, you get the same result, though the initial interactions are very different. The constants (energy) tell us something about the source of the photons and are predictive of their other interactions. The power, though measurable (depending on how you define it) is not constant, explanatory or usefully predictive.
I guess with all photons "containing" a quantum of action, once we know the energy of a particular photon we know the power and vice versa. Let me try some maths:

E = hf
P = E/t
t = 1/f
P = Ef

QED?
Title: Re: How do we measure the energy of a photon?
Post by: alancalverd on 21/02/2019 09:29:51
The quantum of action is h, a universal constant. The energy of photons spans an enormous spectrum.

t≠1/f

Your handling of mathematical symbols is impeccable, but has nothing to do with photon physics.
Title: Re: How do we measure the energy of a photon?
Post by: mxplxxx on 21/02/2019 10:50:41
Your handling of mathematical symbols is impeccable, but has nothing to do with photon physics.
Can you tell me more? I would have thought it had everything to do with photon physics given e=hf is the basic equation of quantum physics and a photon "contains" one quantum of energy (h).

A better way of imagining how a quantum of action behaves, is to use the formula E=h/t applied to photons where t is the time of one oscillation of a photon. This formula is the equivalent of E=hf. As the time does down (i.e. the faster the oscillation/cycle or the higher the frequency), the energy goes up and vice versa and, of course, all the while h stays the same.  Of course where the frequency is increasing, the power is really taking off as energy is increasing and time is decreasing.
Title: Re: How do we measure the energy of a photon?
Post by: Bored chemist on 21/02/2019 19:37:40
we are not too sure exactly what energy is
Speak for yourself.
Title: Re: How do we measure the energy of a photon?
Post by: Bored chemist on 21/02/2019 19:41:56
E = hf
P = E/t
t = 1/f
P = Ef

QED?
No
Because you are using  "t" for two different things.
One is the time period over which energy is transferred, and the other is the time between successive peaks in the em field of the photon.

It's like muddling up the reciprocal of the frequency of a car radio  and time take to get from one town to the next.
Title: Re: How do we measure the energy of a photon?
Post by: mxplxxx on 21/02/2019 20:58:15
E = hf
P = E/t
t = 1/f
P = Ef

QED?
No
Because you are using  "t" for two different things.
One is the time period over which energy is transferred, and the other is the time between successive peaks in the em field of the photon.

It's like muddling up the reciprocal of the frequency of a car radio  and time take to get from one town to the next.
I am pretty sure, because of wave/particle duality, that a photon and an EM wave are nearly equivalent (see http://galileo.phys.virginia.edu/classes/531.cas8m.fall05/l26.pdf). Can you supply me a good reference to back up your statement that t in E=h/t has two different meanings? I certainly can't after a fairly long search.
Title: Re: How do we measure the energy of a photon?
Post by: mxplxxx on 21/02/2019 21:15:02
E = hf
P = E/t
t = 1/f
P = Ef

QED?
No
Because you are using  "t" for two different things.
One is the time period over which energy is transferred, and the other is the time between successive peaks in the em field of the photon.

It's like muddling up the reciprocal of the frequency of a car radio  and time take to get from one town to the next.
I am pretty sure, because of wave/particle duality, that a photon and an EM wave are nearly equivalent (see http://galileo.phys.virginia.edu/classes/531.cas8m.fall05/l26.pdf). Can you supply me a good reference to back up your statement that t in E=h/t has two different meanings? I certainly can't after a fairly long search.
Actually I did find something at https://www.quora.com/Does-it-take-any-time-for-photons-to-be-released-or-absorbed-or-does-it-happen-instantaneously:

However, the creation and annihilation of photons are random processes, whose statistics are governed by quantum mechanics. This means that the probability of creating or annihilating a photon is time-dependent. Typically, the frequency of the photon sets the timescale for the absorption/emission probability. So, for example, if a photon of wavelength λ= 500 nm (corresponding to green light) and frequency f=c/λ=6×1014 Hz encounters an atom with the appropriate resonance frequency, the maximum probability for absorption is 1 / (6×10−14 Hz) = 1.67×10−15 seconds after the interaction begins. So, the photon is most likely to be absorbed after 1.67 femtoseconds. But, the photon can really be absorbed at any instant before or after (with varying probability).

I am a great fan of the probability nature of a photon. The author of the reference also states that the absorption is instantaneous. So what you are saying makes a lot of sense. Pity physics makes this distinction so hard to come by:)
. What then does the time (t) variable in E=h/t mean?. How do we calculate the power of a photon?
Title: Re: How do we measure the energy of a photon?
Post by: mxplxxx on 21/02/2019 21:44:45
The author of the reference also states that the absorption is instantaneous.
But how can it be instantaneous? Is it possible the photon has attributes that are immediately transferred to the receiving particle, much like an event in a computer program immediately updates the status of an affected object. 
Title: Re: How do we measure the energy of a photon?
Post by: alancalverd on 21/02/2019 23:02:57
The quantum of action is not a quantum of energy. They are dimensionally distinct.

"Wave-particle duality" is outdated superstition. There are two mathematical models of a photon, neither of which "is" a photon.

We know exactly what energy is, because we (i.e. physicists) have defined it as a conserved scalar. Just as farmers have defined a cow as a quadruped that moos, poos and makes milk. Only philosophers pretend that we don't know what anything "is", which is why they are not put in charge of dangerous machines.

The problem with suggesting that a photon takes around 10-15 seconds to interact with an atom is that in that time it will have travelled about 3 x 10-7 meters, several thousand atom radii past its supposed target.
Title: Re: How do we measure the energy of a photon?
Post by: mxplxxx on 22/02/2019 01:26:43
The quantum of action is not a quantum of energy. They are dimensionally distinct.
E=hf. E is a quantum of energy, h is a quantum of action. E is "real", h is mathematical.
Title: Re: How do we measure the energy of a photon?
Post by: mxplxxx on 22/02/2019 01:33:15
The problem with suggesting that a photon takes around 10-15 seconds to interact with an atom is that in that time it will have travelled about 3 x 10-7 meters, several thousand atom radii past its supposed target.
Unless the photon is coming from the future (which is not out of the realms of possibility :) given the probability aspects of a photon) and the fact that the transverse waves of an EM wave are travelling faster than SOL.

If not what, then, is the mechanism for transferring the energy of a photon to a recipient particle? It would seem this must happen in a certain time to satisfy the power/conservation of energy aspects of the transfer. e.g. a gamma ray should transfer faster than a visible light ray.

The problem with a transfer of  energy in 0 seconds is that the energy will have traveled 0 meters in that time:) But,  totally unsatisfactorily, according to quantum physics that is what happens. Quantum physics is forcing us to speculate that our concept of reality is out of step with the actuality of reality. Is this really the case or do we need to advance our theories?
Title: Re: How do we measure the energy of a photon?
Post by: mxplxxx on 22/02/2019 01:43:31
We know exactly what energy is, because we (i.e. physicists) have defined it as a conserved scalar.
From https://www.quora.com/Why-is-energy-a-scalar-quantity

A “scalar” quantity is defined as one that does not change under a transformation. Making this meaningful requires specifying which quantity and which transformation: so for instance energy is a scalar under spatial rotations, or under spatial translations, but it is the time component of a four-vector under Lorentz boosts. Under the Poincare group composed of of all those operations, energy is not a scalar.

That means that facing another direction, or changing position, doesn’t change the measurement of some object’s energy; but changing your velocity does.
Title: Re: How do we measure the energy of a photon?
Post by: mxplxxx on 22/02/2019 03:31:05
There are two mathematical models of a photon, neither of which "is" a photon.
Pardon? Tell me more.
Title: Re: How do we measure the energy of a photon?
Post by: mxplxxx on 22/02/2019 03:35:11
The problem with suggesting that a photon takes around 10-15 seconds to interact with an atom is that in that time it will have travelled about 3 x 10-7 meters, several thousand atom radii past its supposed target.
But, presumably, the photon has been stopped in its tracks by the atom:) so no question of it travelling past the atom.
Title: Re: How do we measure the energy of a photon?
Post by: mxplxxx on 22/02/2019 04:10:04
"Wave-particle duality" is outdated superstition.
Can you give me a good reference for this? I do not see it personally. In my computer model of reality system objects travel through time, much like waves in physics. Each system object (e.g. a galaxy) consists of a central object  and peripheral system objects (e.g. in the case of a galaxy, a black hole and surrounding solar systems). The central object contains the state of the system and would be considered the particle in the wave/particle duality paradigm. This works fine for me and I cannot see why it would not also work for reality.

basically, System = structure, Wave = position, Particle = state
Title: Re: How do we measure the energy of a photon?
Post by: alancalverd on 22/02/2019 07:19:31
The problem with all computer models is GI(G+)O. Why +? because the response is not instantaneous or even related directly to reality, but is a function of the model and the system it runs on. Physics is different.
Title: Re: How do we measure the energy of a photon?
Post by: alancalverd on 22/02/2019 07:23:51
That means that facing another direction, or changing position, doesn’t change the measurement of some object’s energy; but changing your velocity does.
I think the light is dawning. The Pound-Rebka experiment is worth studying.
Title: Re: How do we measure the energy of a photon?
Post by: mxplxxx on 23/02/2019 07:32:11
It seems to me that to measure the energy of a photon you have to be moving at the speed of light. According to relativity, a photon will perceive the universe as a mass of stationary bosons oscillating around fixed points according to their composite electromagnetic waves (interesting! we may live in two same-space interacting universes, one inhabited by bosons and one by fermions). So, somehow or other send a photon to measure another photon.
It seems, given that a photon cannot detected/measured until it is observed, that photons (and most likely all bosons) exist as a constantly evolving probability wave (or waves?) until they are observed. This effectively means that a particular photon is duplicated in all places in the universe where is can possibly currently exist! And that all duplicates are destroyed when an observation is made of any one of them. Or that a photon NEVER actually exists in reality!!! Mind-boggling stuff, but so necessary for reality to work the way it does. It is also interesting to note that a photon can only be observed by one particle before being destroyed meaning there is probably a limit to the amount of intelligent life the universe can support, or that intelligence cannot happen until a certain mass of particles is reached.

It also means that we can probably interact with any photon in the universe that is observable from our current location with ultra-interesting possibilities - if only we knew how (maybe via quantum entanglement?)!
Title: Re: How do we measure the energy of a photon?
Post by: mxplxxx on 23/02/2019 10:06:12
A photon cannot be measured because it never actually exists in reality.
Title: Re: How do we measure the energy of a photon?
Post by: mxplxxx on 23/02/2019 12:06:16
A photon cannot be measured because it never actually exists in reality.
We theorize that particle p1 loses energy e and at the same time as this happens a probability wave  is generated. We do not know the wave function of this wave but we theorize that the wave evolves in time until a point in the wave is "observed" by particle p2. We do not know exactly what observing entails but we theorize that when the observation is done the wave "collapses" and particle p2 immediately absorbs energy e.

There is not photon in this description. No photon has ever been detected or directly measured.
Title: Re: How do we measure the energy of a photon?
Post by: Bored chemist on 23/02/2019 12:28:28
basically, System = structure, Wave = position, Particle = state
A photon cannot be measured because it never actually exists in reality.

And, when you can provide extraordinary evidence for those extraordinary claims, we can make progress.
Until you can back up those ideas you might as well stop posting here.
Title: Re: How do we measure the energy of a photon?
Post by: alancalverd on 23/02/2019 13:51:28
It would seem this must happen in a certain time to satisfy the power/conservation of energy aspects of the transfer.
power is not conserved.

I've often suggested that we should teach quantum physics much earlier in the syllabus so people don't make the mistake of thinking that it is just continuum classical physics on a small scale, but it is equally important that people understand a few of the known laws of classical physics - or at least speak the same language as scientists.
Title: Re: How do we measure the energy of a photon?
Post by: alancalverd on 23/02/2019 19:56:28
It seems, given that a photon cannot detected/measured until it is observed, that photons (and most likely all bosons) exist as a constantly evolving probability wave (or waves?) until they are observed. This effectively means that a particular photon is duplicated in all places in the universe where is can possibly currently exist!
This contradicts the inverse square law which is an everyday observation.
Title: Re: How do we measure the energy of a photon?
Post by: mxplxxx on 23/02/2019 23:18:21
power is not conserved.
What happens to power then? There seems to be much controversy about this in the physics community.

https://www.researchgate.net/post/Superposition_does_not_conserve_energy

But ideally, assume we have two wave sources , each give a wave with fixed power 0.5E^2, so the total power get out of the "combined" source should be twice of that value. However, Eans=2E, power form Eans is not the same as the power generated from the source. Problem remains.
Title: Re: How do we measure the energy of a photon?
Post by: mxplxxx on 23/02/2019 23:20:56
It seems, given that a photon cannot detected/measured until it is observed, that photons (and most likely all bosons) exist as a constantly evolving probability wave (or waves?) until they are observed. This effectively means that a particular photon is duplicated in all places in the universe where is can possibly currently exist!
This contradicts the inverse square law which is an everyday observation.
This is standard physics teaching as far as I am aware.
Title: Re: How do we measure the energy of a photon?
Post by: mxplxxx on 23/02/2019 23:27:26
basically, System = structure, Wave = position, Particle = state
Just an observation. Nothing extraordinary about it I would have thought. ps System is everything except elementary particles including the universe itself.
Title: Re: How do we measure the energy of a photon?
Post by: mxplxxx on 24/02/2019 06:42:43
power is not conserved.
Basically, according to E=h/t a certain amount of energy corresponds to a certain amount of time. But, as far as I can see,  we cannot change E, we can only move it about so E/t is conserved. No idea how a red etc. shifts occurs given E is a quantum of energy and can only be altered in quantized amounts (if at all).
Title: Re: How do we measure the energy of a photon?
Post by: mxplxxx on 24/02/2019 08:07:17
There are two mathematical models of a photon, neither of which "is" a photon.
Pardon? Tell me more.
Where can I find a description of these models? Where can I find  a description of the modern quantum physics (i.e. post wave/particle duality)? Is E=hf classical or quantum or both?
Title: Re: How do we measure the energy of a photon?
Post by: Bored chemist on 24/02/2019 09:26:48
basically, System = structure, Wave = position, Particle = state
Just an observation. Nothing extraordinary about it I would have thought. ps System is everything except elementary particles including the universe itself.
It is clearly nonsense.
A wave, necessarily has no single position.
Particles have many states.
And you haven't defined "structure" well enough to say what that bit means.

Title: Re: How do we measure the energy of a photon?
Post by: Bored chemist on 24/02/2019 09:29:31
Basically, according to E=h/t a certain amount of energy corresponds to a certain amount of time.

You are ignoring the fact that I already explained why you are wrong.

You are still stupidly using t for two different things.

Title: Re: How do we measure the energy of a photon?
Post by: Bored chemist on 24/02/2019 09:33:31
Can you supply me a good reference to back up your statement that t in E=h/t has two different meanings? I certainly can't after a fairly long search.
Learn to read.

I did not say that " t in E=h/t has two different meanings".
I said that you were using "t" for two different things.
And I explained what they are.


One is the time period over which energy is transferred, and the other is the time between successive peaks in the em field of the photon.
And you are still muddling them.
Title: Re: How do we measure the energy of a photon?
Post by: mxplxxx on 24/02/2019 10:36:54
You are still stupidly using t for two different things.
You are still stupidly using t for two different things.
If you can't be civil to seekers of wisdom I suggest you go elsewhere. Moderator, why do you allow this sort of  abuse.
Title: Re: How do we measure the energy of a photon?
Post by: mxplxxx on 24/02/2019 10:42:44
Quote from: Bored chemist on 21/02/2019 19:41:56
One is the time period over which energy is transferred, and the other is the time between successive peaks in the em field of the photon.
And you are still muddling them.
As explained with a reference in a recent post, energy is transferred in no time. The differences between classic physics and wave/particle quantum physics and modern quantum probability physics are not well explained by physicists who seem to either not know or are reluctant to divulge trade secrets or are incapable of lucid explanations! As a result I am certain there are vast numbers of people out there equally as clueless as me but maybe I am stupid in that I am attempting to get knowledge on a physics forum that seems to not be available.
Title: Re: How do we measure the energy of a photon?
Post by: Bored chemist on 24/02/2019 10:44:56
You are still stupidly using t for two different things.
You are still stupidly using t for two different things.
If you can't be civil to seekers of wisdom I suggest you go elsewhere. Moderator, why do you allow this sort of  abuse.

It is uncivil to continue to use arguments which you know are not valid.
Such an act is stupid.

If you don't want your stupid behaviour highlighted do not act stupidly.
Try acting in a civil manner instead.
Do not, for example, pretend that people who measure the energies of photons are either fools or liars.
Title: Re: How do we measure the energy of a photon?
Post by: Bored chemist on 24/02/2019 10:46:27
, energy is transferred in no time
Since it has to travel a finite distance, that would require an infinite velocity.
What carries the energy at this infinite velocity?
Are yo postulating some new superluminal particle- or have you just misunderstood things again?
Title: Re: How do we measure the energy of a photon?
Post by: mxplxxx on 24/02/2019 10:52:06
It has been interesting. Time to move on.
Title: Re: How do we measure the energy of a photon?
Post by: alancalverd on 24/02/2019 12:23:26
This is standard physics teaching as far as I am aware
Your awareness is either faulty or not suported by common sense.

Consider a point source of photons. If a particular photon is duplicated in all places in the universe where is can possibly currently exist, then placing a detector anywhere will attract all the photons. Don't look at the sun.
Title: Re: How do we measure the energy of a photon?
Post by: mxplxxx on 24/02/2019 21:39:23
This is standard physics teaching as far as I am aware
Your awareness is either faulty or not suported by common sense.

Consider a point source of photons. If a particular photon is duplicated in all places in the universe where is can possibly currently exist, then placing a detector anywhere will attract all the photons. Don't look at the sun.
It seems to me common sense in quantum mechanics is often dispensed with. For validation of my assertion, see  http://discovermagazine.com/2005/jun/cover
Title: Re: How do we measure the energy of a photon?
Post by: alancalverd on 24/02/2019 22:32:58
Eddington had a different view. He maintained that the student of physics should be come accustomed to having his common sense violated five times before breakfast, so that if he were to pass through the floor and reappear in the room below he would not consider it a miracle or magic, just a natural event of very small probability. 

And that is why macroscopic stuff seems to stay in one place. Whilst there is a finite probability of any of your atoms being somewhere else, the likelihood of all of you spontaeously materialising on the far side of the moon is of no practical consequence. IIRC a few buckballs have indeed been shown to tunnel through a chemically impermeable barrier.   
Title: Re: How do we measure the energy of a photon?
Post by: mxplxxx on 24/02/2019 23:07:20
the likelihood of all of you spontaeously materialising on the far side of the moon is of no practical consequence
But the likelihood of uncertainty causing a malfunction in one of our cells , I would imagine, is reasonably high. Which is good from evolution's point of view but not so good from a personal point of view. I would think one of our immune system's main functions is to make sure cells that don't conform to a type are repaired (or disposed of if badly damaged as e.g. in the case of cancer). But what of uncertainty and the type:) Maybe one reason for aging?
Title: Re: How do we measure the energy of a photon?
Post by: alancalverd on 25/02/2019 07:17:11
It's an interesting point. I've lost my lecture slide about the natural rate of DNA transcription errors but it runs into zillions per day. Most are harmless or ineffective but the adjoining "normal" cells have to somehow prevent the defective ones from multiplying. One oddity is the female breast, which may have to change its function several times at (relatively) short notice, so my hypothesis is that it is more tolerant than the rest of the body towards "deviant" behavior and may permit the growth of tumors that would be killed by the immune system if they occurred elsewhere.   

A complete digression, but an interesting one.
Title: Re: How do we measure the energy of a photon?
Post by: mxplxxx on 25/02/2019 08:22:43
It's an interesting point. I've lost my lecture slide about the natural rate of DNA transcription errors but it runs into zillions per day. Most are harmless or ineffective but the adjoining "normal" cells have to somehow prevent the defective ones from multiplying. One oddity is the female breast, which may have to change its function several times at (relatively) short notice, so my hypothesis is that it is more tolerant than the rest of the body towards "deviant" behavior and may permit the growth of tumors that would be killed by the immune system if they occurred elsewhere.   

A complete digression, but an interesting one.
Makes sense. There is also the fact that the breast may have to switch off at least part of the immune system to enable it to manufacture something that is meant for a different body.
Title: Re: How do we measure the energy of a photon?
Post by: Bored chemist on 25/02/2019 19:31:46
Breast tissue has to grow suddenly. To do so the cells needs to divide a lot faster than usual. They also need to grow quickly in response to a chemical signal.
That more or less primes them for being at greater risk of becoming cancerous.


There is also the fact that the breast may have to switch off at least part of the immune system to enable it to manufacture something that is meant for a different body.
That makes little sense.
Breasts make various things- proteins etc that are
(1) made for a different body, but also
(2) Pretty much identical to the ones that the breast's owner's mother's milk contained.

If someone had a reaction to the proteins they made for their baby, they would have had a reaction to the proteins their mother made for them; and they wouldn't have survived.
Title: Re: How do we measure the energy of a photon?
Post by: mxplxxx on 26/02/2019 21:39:25
The latest in photon measurement:) https://scienceblog.com/506371/entangling-photons-of-different-colors/?utm_source=feedburner&utm_medium=email&utm_campaign=Feed%3A+scienceblogrssfeed+%28ScienceBlog.com%29
Title: Re: How do we measure the energy of a photon?
Post by: yor_on on 27/02/2019 14:17:17
It sounds simple but it can't be MP, as far as I know it's really hard to entangle with precision.
=

We'll have to wait until those subs really can 'speak' and see if it works.
Although this seems to be carriers expected to run in fibers?
Ah, maybe not, anyway..

yeah, not what I was thinking of.
Title: Re: How do we measure the energy of a photon?
Post by: mxplxxx on 11/03/2019 22:26:49
Not too sure whether this experiment actually measure the energy of a photon, but it is interesting that a photon can be detected at all without destroying it.
https://www.sciencedaily.com/releases/2007/04/070402122514.htm
Title: Re: How do we measure the energy of a photon?
Post by: mxplxxx on 19/03/2019 07:40:26
The author of the reference also states that the absorption is instantaneous.
But how can it be instantaneous? Is it possible the photon has attributes that are immediately transferred to the receiving particle, much like an event in a computer program immediately updates the status of an affected object. 
Basically, energy takes time to transfer. How long depends on the power (Energy per second) of the object the energy is being transferred from. Planck's constant h has an aspect of power about it as it relates energy and time. There is no reason, I can see, to believe that a photon does not have power. How much power is given by P=E/t from E=h/t  which is the wave function of the photons's associated EM wave.

Title: Re: How do we measure the energy of a photon?
Post by: mxplxxx on 19/03/2019 08:15:32
Interestingly, it seem not all photons carry energy. https://phys.org/news/2015-03-photon-afterglow-transmit-transmitting-energy.html.

It would seem that some types of photons transfer state/energy from one object to another and other types of photons transfer information only about events in other objects. Yippee! I don't think computer science has woken up to the difference - but has now:).
Title: Re: How do we measure the energy of a photon?
Post by: mxplxxx on 24/03/2019 00:35:55
One last try at punching smoke. The ultraviolet catastrophe was solved by Planck introducing h. Otherwise the classical theory would still predict the energy approaching infinity at shorter wavelengths. https://en.m.wikipedia.org/wiki/Ultraviolet_catastrophe

If you still don't understand then there is no hope.
No hope for hordes of us then it would appear. I am far from alone. A heroic search for the meaning and derivation of h (Planck's constant) was in vain until I came across this post https://www.reddit.com/r/askscience/comments/3qxkvf/how_did_planck_calculate_his_constant/ (https://www.reddit.com/r/askscience/comments/3qxkvf/how_did_planck_calculate_his_constant/).

The youtube channel viascience has a very very goo series on quantum mechanics. I recommend it very highly. see here https://www.youtube.com/watch?v=SCUnoxJ5pho&index=2&list=PL193BC0532FE7B02C (https://www.youtube.com/watch?v=SCUnoxJ5pho&index=2&list=PL193BC0532FE7B02C) for the first video. It and the second explain exactly how Planck came up with quantisation, and in so doing his constant.

A simple explanation is this. Planck was studying light bulbs, and wanted to work out how to get the most light out of a bulb for the least power. To do this, he needed to model how energy moved about. He initially treated it as if energy moved continuously, rather than in little packets like quantum mechanics says.

He made the model in basically the same way you derive integration (area under a curve in calculus) from first principles. In calculus, to find the area under a continuous curve, we divide up the x axis into many small intervals of width h, then find h times the height of the curve at each interval on the x axis to get an area of a rectangle for each, then sum up the rectangles. We mathematically consider what would happen with smaller and smaller intervals (smaller h) to more and more closely approximate the area under the continuous curve, as opposed to the chunked up (quantised) approximate to the curve made out of rectangles. As you shrink h to zero, you approach the continuous case.

Planck modelled the movement of energy in a similar way - imagining that it moved about in small chunks of size h times the wavelength, and he mathematically imagined shrinking h to zero, to make the model represent energy moving continuously, not in chunks.

He applied this model to observed data, but it didn't match. What DID match, however, is if he neglected to shrink his value h to zero. The precise value you limit h to be in order to make it fit the observed data is what we now call Planck's constant - and it still has the symbol h.


It is my guess then that E in E=hf is only an approximation and the longer the wavelength, the more inaccurate E becomes.

While I am about it let me tentatively suggest based on the above that h is in fact Energy (E), not Action. h is actually the area of a rectangle with width t and height energy where energy is the energy being applied at an instant in time (so power maybe?), not total energy. Hooray, no more Action which is such a controversial unit in physics.
Title: Re: How do we measure the energy of a photon?
Post by: alancalverd on 24/03/2019 09:23:55
A very poor analogy, and not at all what Planck did. This is a fine example of what happens if you grab at a model without understanding its limitations (or, in this case, lack of limitation).

Quote
He assumed that a hypothetical electrically charged oscillator in a cavity that contained black body radiation could only change its energy in a minimal increment, E, that was proportional to the frequency of its associated electromagnetic wave.

but the size and shape of the hypothetical cavity in Planck's dervation is not quantised. Thus you can generate a continuous spectrum from a theoretical black body, even if a real source may have a few bits missing.

The dimensions  of h are a consequence of its definition. If E is energy [ML2T-2] and f is frequency [T-1], then h cannot be energy. The only interesting thing about h is its constancy, which is fairly easy to measure for single photon events from infrared to γ-ray frequencies and neatly predicts the observed black body spectrum down to long wave radiofrequencies.

Despite doing quantum physics for money for over 50 years, I've never encountered any "controversy" about the dimensions of h. Only priests, politicians and philosphers argue about the bloody obvious.
Title: Re: How do we measure the energy of a photon?
Post by: mxplxxx on 24/03/2019 09:51:06
Despite doing quantum physics for money for over 50 years, I've never encountered any "controversy" about the dimensions of h. Only priests, politicians and philosphers argue about the bloody obvious.
Not h, Action itself. It has undergone several metamorphoses and I am surprised these haven't affected h. h has units of energy times time. It is hard to see how such a bizarre thing can exist. It is more a concept than a physical unit. It is even harder to see how multiplying Action by frequency gives Energy (E). Why not just do the energy times time multiplication and use h = energy. Far less complicated and it doesn't affect the end result.

h as energy means that vastly more people can understand quantum physics. Few people, including I suspect, many physicists understand Action.
Title: Re: How do we measure the energy of a photon?
Post by: Bored chemist on 24/03/2019 10:02:46
While I am about it let me tentatively suggest based on the above that h is in fact Energy (E), not Action.
Why bother to suggest, even tentatively, something that is obviously wrong?
It is my guess then that E in E=hf is only an approximation
Your guess is wrong.
It is hard to see how such a thing can exist.
It's easy for me to see how it exists.
Perhaps this lack of understanding on your part is the root cause of the problem.
Title: Re: How do we measure the energy of a photon?
Post by: mxplxxx on 24/03/2019 23:21:22
In my travels I have come across Planck's Constant h being referred to as the "Quantum of Action" and is usually simultaneously trotted out as the basis for quantum theory. Give me a quantum of solace please:). As far as I can see, h is not a quantum of anything; it is a slice of an energy/time graph. The basis of quantum theory is the discovery that the energy of a photon/EM wave cycle can only interact as a whole (quantum) object
Title: Re: How do we measure the energy of a photon?
Post by: seeker3 on 26/03/2019 03:43:42
All these fifty years of conscious brooding have brought me no nearer to the answer to the question, 'What are light quanta?' Nowadays every Tom, D i c k and Harry thinks he knows it, but he is mistaken. -Albert Einstein

It is important to realize that in physics today, we have no knowledge of what energy is. -Richard P. Feynman

Photoelectric effect theory won Nobel, why the godfather of quantum theory made that quote?

Maybe there is NO photon particle?
Title: Re: How do we measure the energy of a photon?
Post by: seeker3 on 26/03/2019 03:57:12
Photoelectric effect theory won a Nobel Prize. But it is questionable.

If photon has momentum, why navy laser weapons have no kinetic impact?

Why light mill does not move in hard vacuum?

Why photons don't slow down and stop in water like bullets? How can photon accelerate from water to air? What is the mechanism?
Title: Re: How do we measure the energy of a photon?
Post by: Kryptid on 26/03/2019 04:19:47
If photon has momentum, why navy laser weapons have no kinetic impact?

They do.
Title: Re: How do we measure the energy of a photon?
Post by: seeker3 on 26/03/2019 05:32:26
Einstein was a true truth seeker.

His Nobel Prize is for photon/quanta theory not relativity.

Why scientific community don't take his word seriously? He was the godfather of quanta. Quantum anything is imaginary.
Title: Re: How do we measure the energy of a photon?
Post by: Kryptid on 26/03/2019 06:15:22
Why scientific community don't take his word seriously?

It's not like you take his word seriously either, given that you just got finished saying, "quantum anything is imaginary".
Title: Re: How do we measure the energy of a photon?
Post by: seeker3 on 26/03/2019 06:25:40
Did he said he knows not what are light quanta?

Why is that? Was he invented quanta? Was he won Prize for photon theory?

Did he said everyone thinks they know what are light quanta are mistaken?

I take his word, that why quantum anything is imaginary.
Title: Re: How do we measure the energy of a photon?
Post by: Kryptid on 26/03/2019 06:28:14
Did he said he knows not what are light quanta?

Why is that? Was he invented quanta? Was he won Prize for photon theory?

Did he said everyone thinks they know what are light quanta are mistaken?

I take his word, that why quantum anything is imaginary.

Saying that you don't know what something is is not the same as saying that it doesn't exist.
Title: Re: How do we measure the energy of a photon?
Post by: seeker3 on 26/03/2019 06:37:09
What is photon made of? How photon is emitted? What is the difference of red and blue photon?

How can photons pass through water without losing momentum and stop?

How can photons accelerate from glass into air?

Where are all the photons from all the stars shooting at us?
Title: Re: How do we measure the energy of a photon?
Post by: Kryptid on 26/03/2019 06:43:21
What is photon made of?

I answered that in another thread already.

How photon is emitted?

I answered that too.

What is the difference of red and blue photon?

Wavelength, energy and frequency (all three of these things are related for light).

How can photons pass through water without losing momentum and stop?

They don't work the same way as physical objects like bullets. It's at least in part due to their wave-like nature.

How can photons accelerate from glass into air?

Because air doesn't slow light down as much as glass does.

Where are all the photons from all the stars shooting at us?

They are either absorbed or reflected by the matter here on Earth.
Title: Re: How do we measure the energy of a photon?
Post by: seeker3 on 26/03/2019 06:50:20
What is energy? How photons carry energy? What frequency? What wave length? What mechanism? How photons vibrate? How photons wave? Why all photon have same speed? Why photons slow down in water?
Title: Re: How do we measure the energy of a photon?
Post by: seeker3 on 26/03/2019 06:56:53
Does photon carry electric field and magnetic field? Does photon carry EM wave? How exactly photon move or wave through space? what is exact mechanism?
Title: Re: How do we measure the energy of a photon?
Post by: Kryptid on 26/03/2019 07:02:39
Those are a lot of questions and I don't have time to answer them all tonight. This video, however, could be of help:


Part of the answer to your questions is that electromagnetic waves are composed of changing electric and magnetic fields at right angles to each other that create new changing electric and magnetic fields and so on and so on. That's what causes the wave to propagate through space. A photon is simply the smallest possible piece of that wave.

This may help as well:

Title: Re: How do we measure the energy of a photon?
Post by: seeker3 on 26/03/2019 07:08:26
If photon particle is real particle, to accelerate it to light speed takes energy, and a precise mechanism, and time. How come light accelerate from 0 to light speed instantly?

WHAT IS THE MECHANISM?



Title: Re: How do we measure the energy of a photon?
Post by: seeker3 on 26/03/2019 07:46:18
So the Sun shooting out photons at every direction at light speed?

Does Sun's gravity slow down photons that shooting away?

Why photons are not slowing down by time since gravity is constantly decelerating it?
Title: Re: How do we measure the energy of a photon?
Post by: mxplxxx on 26/03/2019 07:46:56
Does photon carry electric field and magnetic field? Does photon carry EM wave? How exactly photon move or wave through space? what is exact mechanism?
Not on topic.

You need to know physics at a fairly deep level to have any hope that your questions are answered. Especially you need to know QED. Even so, you are going to have questions that cannot be answered. The universe does not reveal its secrets easily. Check out https://www.researchgate.net/post/Is_Einsteins_photon_really_the_same_as_the_QED_photon for an example of the controversies raging in physics over the photon.
Title: Re: How do we measure the energy of a photon?
Post by: seeker3 on 26/03/2019 08:04:36
If gravity can attract light, slow down light, so very big mass can become black hole because light slow down to a stop?

If that's true, half black hole mass star light speed should be 1/2 light speed?

Small Sun's light speed faster?
Title: Re: How do we measure the energy of a photon?
Post by: seeker3 on 26/03/2019 08:55:18
Gravitational lensing might be caused by hot gas plasma on the surface of the Sun or gas clouds in deep space deflected light.



Title: Re: How do we measure the energy of a photon?
Post by: Kryptid on 26/03/2019 12:56:32
If photon particle is real particle, to accelerate it to light speed takes energy, and a precise mechanism, and time. How come light accelerate from 0 to light speed instantly?

WHAT IS THE MECHANISM?

Photons don't accelerate from zero speed to light speed. That implies that they start off at zero speed when they first come into existence. They don't. They are moving at the speed of light as soon as they are created. So there is no acceleration.

So the Sun shooting out photons at every direction at light speed?

Yes.

Does Sun's gravity slow down photons that shooting away?

No, but it does weaken them so that they have less energy.

Why photons are not slowing down by time since gravity is constantly decelerating it?

Gravity doesn't decelerate light.

If gravity can attract light, slow down light, so very big mass can become black hole because light slow down to a stop?

No, black holes redshift light out of existence.

If that's true, half black hole mass star light speed should be 1/2 light speed?

Small Sun's light speed faster?

No, the speed of light doesn't depend on a star's mass.

Gravitational lensing might be caused by hot gas plasma on the surface of the Sun or gas clouds in deep space deflected light.

It isn't. The amount of lensing matches the mathematical predictions of a gravitational cause.
Title: Re: How do we measure the energy of a photon?
Post by: chiralSPO on 26/03/2019 13:09:20
https://en.wikipedia.org/wiki/Sealioning
Title: Re: How do we measure the energy of a photon?
Post by: Colin2B on 26/03/2019 14:58:01
https://en.wikipedia.org/wiki/Sealioning
@seeker3
Posting multiple questions without waiting for an answer is also trolling, as is advertising new theories in this section of the forum.
Title: Re: How do we measure the energy of a photon?
Post by: seeker3 on 26/03/2019 16:02:48
If photon particle is real particle, to accelerate it to light speed takes energy, and a precise mechanism, and time. How come light accelerate from 0 to light speed instantly?

WHAT IS THE MECHANISM?

Photons don't accelerate from zero speed to light speed. That implies that they start off at zero speed when they first come into existence. They don't. They are moving at the speed of light as soon as they are created. So there is no acceleration.

So the Sun shooting out photons at every direction at light speed?

Yes.

Does Sun's gravity slow down photons that shooting away?

No, but it does weaken them so that they have less energy.

Why photons are not slowing down by time since gravity is constantly decelerating it?

Gravity doesn't decelerate light.

If gravity can attract light, slow down light, so very big mass can become black hole because light slow down to a stop?

No, black holes redshift light out of existence.

If that's true, half black hole mass star light speed should be 1/2 light speed?

Small Sun's light speed faster?

No, the speed of light doesn't depend on a star's mass.

Gravitational lensing might be caused by hot gas plasma on the surface of the Sun or gas clouds in deep space deflected light.

It isn't. The amount of lensing matches the mathematical predictions of a gravitational cause.

Word salad? What is the mechanism?
Title: Re: How do we measure the energy of a photon?
Post by: Kryptid on 26/03/2019 16:04:33
Word salad?

Nobody said anything about word salad.

What is the mechanism?

The mechanism for what? You've quote many different things at once.
Title: Re: How do we measure the energy of a photon?
Post by: seeker3 on 26/03/2019 16:15:10
Word salad?

Nobody said anything about word salad.

What is the mechanism?

The mechanism for what? You've quote many different things at once.
What is the mechanism of light? How electrons emit photon? How photons travel in space at light speed? How photons slow down in air and water? How photons accelerate from water into air? How photons carry energy? What is energy?
Title: Re: How do we measure the energy of a photon?
Post by: seeker3 on 26/03/2019 16:21:32
Quote from: Kryptid link=topic=75960.msg570947#msg570947 date=1553604
[/quote

It isn't. The amount of lensing matches the mathematical predictions of a gravitational cause.

What math prediction? What math equation?
Title: Re: How do we measure the energy of a photon?
Post by: seeker3 on 26/03/2019 16:32:29
If photon particle is real particle, to accelerate it to light speed takes energy, and a precise mechanism, and time. How come light accelerate from 0 to light speed instantly?

WHAT IS THE MECHANISM?

Photons don't accelerate from zero speed to light speed. That implies that they start off at zero speed when they first come into existence. They don't. They are moving at the speed of light as soon as they are created. So there is no acceleration.

Description is not explanation. What is the mechanism?

Quote from: seeker3 on Today at 07:46:18
So the Sun shooting out photons at every direction at light speed?

Yes.
What is the mechanism?
Quote from: seeker3 on Today at 07:46:18
Does Sun's gravity slow down photons that shooting away?

No, but it does weaken them so that they have less energy.
What is the mechanism?
Quote from: seeker3 on Today at 07:46:18
Why photons are not slowing down by time since gravity is constantly decelerating it?

Gravity doesn't decelerate light.
How black hole slow down light speed to zero?
Quote from: seeker3 on Today at 08:04:36
If gravity can attract light, slow down light, so very big mass can become black hole because light slow down to a stop?

No, black holes redshift light out of existence.
What is the mechanism?
Quote from: seeker3 on Today at 08:04:36
If that's true, half black hole mass star light speed should be 1/2 light speed?

Small Sun's light speed faster?

No, the speed of light doesn't depend on a star's mass.
If so, why black hole has no light?
Title: Re: How do we measure the energy of a photon?
Post by: Bored chemist on 26/03/2019 18:06:48
What is the mechanism of light?
Well, the classical version was sorted out by Maxwell in the 19th C.
The modern version requires an understanding of time dependent perturbation theory.
https://en.wikipedia.org/wiki/Perturbation_theory_(quantum_mechanics)
Did you not know that there is a well documented "mechanism"?

Once you have learned about them you can hope to tackle these.

How electrons emit photon? How photons travel in space at light speed? How photons slow down in air and water? How photons accelerate from water into air? How photons carry energy? What is energy?
Title: Re: How do we measure the energy of a photon?
Post by: Bored chemist on 26/03/2019 18:10:48
If photon particle is real particle, to accelerate it to light speed takes energy, and a precise mechanism, and time. How come light accelerate from 0 to light speed instantly?

That logical error is called the "no true Scotsman fallacy".

No, but it does weaken them so that they have less energy.
What is the mechanism?
General relativity

Title: Re: How do we measure the energy of a photon?
Post by: seeker3 on 26/03/2019 20:13:31
Einstein won Nobel for photoelectric effect, co-invented photon concept.

Why he still searching for the answer for 50 years? What are light quanta?

Did he find it?

Did you?

Title: Re: How do we measure the energy of a photon?
Post by: Kryptid on 26/03/2019 20:32:35
What is the mechanism of light? How electrons emit photon? How photons travel in space at light speed? How photons slow down in air and water? How photons accelerate from water into air? How photons carry energy? What is energy?

I already answered those questions. Just because you don't like the answers doesn't make them wrong.

What math prediction? What math equation?

The equation is at the beginning of this article: https://en.wikipedia.org/wiki/Gravitational_lensing_formalism

Description is not explanation. What is the mechanism?

There is no mechanism because what you are claiming to happen simply does not happen.

What is the mechanism?

Incandescence: https://en.wikipedia.org/wiki/Incandescence

What is the mechanism?

It's the same thing that causes a ball thrown into the air to lose energy: the gravitational force pulling on it causes it to lose energy.

How black hole slow down light speed to zero?

It doesn't.

What is the mechanism?

The same thing that I mentioned earlier about gravitational force pulling on objects to reduce their kinetic energy.

If so, why black hole has no light?

Because photons cannot get out of the event horizon. Space is literally warped in such a way that it cannot escape.
Title: Re: How do we measure the energy of a photon?
Post by: seeker3 on 26/03/2019 20:42:53
If photons cannot get out, does black holes radiate heat?

If so, how?

If not, will black holes temperature raise up by time?

 
Title: Re: How do we measure the energy of a photon?
Post by: Kryptid on 26/03/2019 20:47:42
If photons cannot get out, does black holes radiate heat?

If so, how?

If not, will black holes temperature raise up by time?

They can't radiate it directly, but they can lose energy indirectly via Hawking radiation:
Title: Re: How do we measure the energy of a photon?
Post by: esquire on 26/03/2019 22:15:42
gluons and photons share similiar characteristic, they both are massless, charge carriers with a 1 spin.
so, why not adopt gluon methods to measure the photon? create a photon singlet by combining/dividing two spectral
photon signatures into a single zero spin spectral signature, where one photon spectral signature is n^2 and other photon spectral signature iis N^2-1. next  divide that result, by the sq rt of 2.

this should give you the photons capacity as a charge carrier. it elimiunates spin as a variable.
Title: Re: How do we measure the energy of a photon?
Post by: alancalverd on 26/03/2019 23:30:23
It is even harder to see how multiplying Action by frequency gives Energy (E).
It doesn't. Or at least it didn't. Planck simply
Quote
assumed that a hypothetical electrically charged oscillator in a cavity that contained black body radiation could only change its energy in a minimal increment, E, that was proportional to the frequency of its associated electromagnetic wave
, so he wrote "assume E = hf......" from which he derived the shape of the black body spectrum, thus establishing both the principle of quantum electromagnetics and, by experiment, the fact that h is a universal constant.

Quote
Why not just do the energy times time multiplication and use h = energy. Far less complicated and it doesn't affect the end result.h as energy means that vastly more people can understand quantum physics.
Because stating that E = E  doesn't explain or predict anything. Which is fine if you are an aspiring politician, priest or philosopher, but physics is about explanation and prediction.

Quote
Few people, including I suspect, many physicists understand Action.
There's nothing to understand. However and whenever you measure E and f, it turns out that E is a constant times f, and that constant must have the dimensions of E/f,  joule-seconds. You could measure wavelength λ, and to nobody's surprise, E = hc/λ, same constant, same dimensions.

You cannot possibly "understand" the fact that my name is Alan, because it is a definition, not a derivation. It's just a definition of "that bloke" as distinct from "all the other blokes"  (though it's so common in the Midlands that in a previous occupation I was called "number five").   
Title: Re: How do we measure the energy of a photon?
Post by: mxplxxx on 27/03/2019 06:02:41
Because stating that E = E  doesn't explain or predict anything. Which is fine if you are an aspiring politician, priest or philosopher, but physics is about explanation and prediction.
Not E = E, E = ef where e is 6.62607004 × 10-34 joules, corresponding to the Action h. No diff to current formula except we  use h as energy, not action. Note current definition of h is joules times 1 second i.e. et joules = e. 

An Energy/Time graph using h = 6.6 and f = 3 follows. It indicates energy calculated using h is likely to be very inaccurate. Unless I have missed something (not beyond the realms of possibility). 

https://1drv.ms/u/s!AkkAzGDByUeBl9JPe3z3QpD5mHhgXQ (https://1drv.ms/u/s!AkkAzGDByUeBl9JPe3z3QpD5mHhgXQ)


Title: Re: How do we measure the energy of a photon?
Post by: mxplxxx on 27/03/2019 06:18:10
There's nothing to understand.
Sure there is. I would think it is always better to understand how an equation is derived. Particularly in this case, it seems, as we cannot directly measure the energy of a photon. Can you state irrevocably that all all photon energies are a constant times their frequency. If so, I would love to hear your reasons. E via E=hf, it would seem, is only accurate for high frequency photons.
Title: Re: How do we measure the energy of a photon?
Post by: Colin2B on 27/03/2019 14:27:57
E via E=hf, it would seem, is only accurate for high frequency photons.
That’s incorrect. I suspect you are misinterpreting the Rayleigh–Jeans law which approximates Planck’s law at low frequencies, but is incorrect.
Title: Re: How do we measure the energy of a photon?
Post by: alancalverd on 27/03/2019 20:47:59
Not E = E, E = ef where e is 6.62607004 × 10-34 joules, corresponding to the Action h
If h is in joules and E is in  joules, then f has no dimension, which is nonsense. The dimensions of action are joule.sec. It isn't the most familiar quantity for most people but the Dirac - Feynman Principle of Least Action underpins a great deal of physics, so it is a useful concept, as ias the universal notion that frequency is 1/time.
Title: Re: How do we measure the energy of a photon?
Post by: mxplxxx on 28/03/2019 01:10:51
E via E=hf, it would seem, is only accurate for high frequency photons.
That’s incorrect. I suspect you are misinterpreting the Rayleigh–Jeans law which approximates Planck’s law at low frequencies, but is incorrect.
The graph in my post https://www.thenakedscientists.com/forum/index.php?PHPSESSID=3hq7t3ivb5tfmkbmvs9harokc4&topic=75960.msg570982#msg570982 clearly shows h is inaccurate at a frequency of 3. Have I done something wrong when developing the graph?
Title: Re: How do we measure the energy of a photon?
Post by: mxplxxx on 28/03/2019 01:15:58
the universal notion that frequency is 1/time.
Providing time is the time of one cycle of whatever frequency is describing. Frequency is cycles per second, so it follows it is 1 (second)/time.
Title: Re: How do we measure the energy of a photon?
Post by: mxplxxx on 28/03/2019 01:24:02
If h is in joules and E is in  joules, then f has no dimension, which is nonsense.
Can you explain y our statement?
Title: Re: How do we measure the energy of a photon?
Post by: mxplxxx on 28/03/2019 04:16:39
E via E=hf, it would seem, is only accurate for high frequency photons.
That’s incorrect. I suspect you are misinterpreting the Rayleigh–Jeans law which approximates Planck’s law at low frequencies, but is incorrect.
How do you know it is incorrect? Has anyone ever measured the energy of an EM wave of frequency 3?
Title: Re: How do we measure the energy of a photon?
Post by: mxplxxx on 28/03/2019 04:48:07
E=hf is confusing because it is a shortened version of what is really happening in the calculation of E which takes place in two steps:

1. t = 1/f  where t is the time to complete one cycle of an EM wave and f is the cycles per second (frequency) of the EM wave
2. E = h/t where E is the energy of one cycle of an EM wave and h is Action (6.62607004 × 10-34 joules times seconds) which is the same for all EM waves irrespective of their frequency.
Title: Re: How do we measure the energy of a photon?
Post by: mxplxxx on 28/03/2019 05:50:32
One last try at punching smoke. The ultraviolet catastrophe was solved by Planck introducing h. Otherwise the classical theory would still predict the energy approaching infinity at shorter wavelengths. https://en.m.wikipedia.org/wiki/Ultraviolet_catastrophe

If you still don't understand then there is no hope.
No hope for hordes of us then it would appear. I am far from alone. A heroic search for the meaning and derivation of h (Planck's constant) was in vain until I came across this post https://www.reddit.com/r/askscience/comments/3qxkvf/how_did_planck_calculate_his_constant/ (https://www.reddit.com/r/askscience/comments/3qxkvf/how_did_planck_calculate_his_constant/).

The youtube channel viascience has a very very goo series on quantum mechanics. I recommend it very highly. see here https://www.youtube.com/watch?v=SCUnoxJ5pho&index=2&list=PL193BC0532FE7B02C (https://www.youtube.com/watch?v=SCUnoxJ5pho&index=2&list=PL193BC0532FE7B02C) for the first video. It and the second explain exactly how Planck came up with quantisation, and in so doing his constant.

A simple explanation is this. Planck was studying light bulbs, and wanted to work out how to get the most light out of a bulb for the least power. To do this, he needed to model how energy moved about. He initially treated it as if energy moved continuously, rather than in little packets like quantum mechanics says.

He made the model in basically the same way you derive integration (area under a curve in calculus) from first principles. In calculus, to find the area under a continuous curve, we divide up the x axis into many small intervals of width h, then find h times the height of the curve at each interval on the x axis to get an area of a rectangle for each, then sum up the rectangles. We mathematically consider what would happen with smaller and smaller intervals (smaller h) to more and more closely approximate the area under the continuous curve, as opposed to the chunked up (quantised) approximate to the curve made out of rectangles. As you shrink h to zero, you approach the continuous case.

Planck modelled the movement of energy in a similar way - imagining that it moved about in small chunks of size h times the wavelength, and he mathematically imagined shrinking h to zero, to make the model represent energy moving continuously, not in chunks.

He applied this model to observed data, but it didn't match. What DID match, however, is if he neglected to shrink his value h to zero. The precise value you limit h to be in order to make it fit the observed data is what we now call Planck's constant - and it still has the symbol h.


It is my guess then that E in E=hf is only an approximation and the longer the wavelength, the more inaccurate E becomes.

While I am about it let me tentatively suggest based on the above that h is in fact Energy (E), not Action. h is actually the area of a rectangle with width t and height energy where energy is the energy being applied at an instant in time (so power maybe?), not total energy. Hooray, no more Action which is such a controversial unit in physics.
I suspect this is garbage. He talks about Planck integrating Energy/time curves but getting an incorrect result when the slices of the curve were shrunk to 0. But getting a correct result when the slices were set to an area h. But there would have been many slices of different areas.

So my conclusion that h is a slice of an Energy/Time curve is incorrect. I apologise profusely for wasting members time on this and the subsequent posts wondering if Action could be eliminated from E=hf.
Title: Re: How do we measure the energy of a photon?
Post by: alancalverd on 28/03/2019 14:59:42
Has anyone ever measured the energy of an EM wave of frequency 3?
3 what?

If you think there is a break frequency below which E ≠ hf, please state its value, how you calculated it, and why the break occurs. I've certainly used the equation from microwaves through optical energies to x-rays with no apparent anomalies.
Title: Re: How do we measure the energy of a photon?
Post by: mxplxxx on 28/03/2019 19:12:17
Has anyone ever measured the energy of an EM wave of frequency 3?
3 what?

If you think there is a break frequency below which E ≠ hf, please state its value, how you calculated it, and why the break occurs. I've certainly used the equation from microwaves through optical energies to x-rays with no apparent anomalies.
Frequency in Hz. I thought, for a little while, that h in E=hf was related to Energy/Time integration but now realize that was likely incorrect. Physics makes it incredibly hard to get a handle on what h is all about. The quote below clarifies this. So sorry for inconvenience.

E=hf is confusing because it is a shortened version of what is really happening in the calculation of E which takes place in two steps:

1. t = 1/f  where t is the time to complete one cycle of an EM wave and f is the cycles per second (frequency) of the EM wave
2. E = h/t where E is the energy of one cycle of an EM wave and h is Action (6.62607004 × 10-34 joules times seconds) which is the same for all EM waves irrespective of their frequency.

Title: Re: How do we measure the energy of a photon?
Post by: alancalverd on 28/03/2019 19:47:22
Don't get hung up on the interpretation of h. It's just a constant. Suppose we count the number of pimples on teenagers' faces, and find a universal linear relationship between age and pimples. The we write N = px where N  is the number of pimples, x is the child's age, and p is an experimental constant whose value is N/x "pimples per year". p has no "interpretation" nor any deep philosophical meaning, but its value is very important.
Title: Re: How do we measure the energy of a photon?
Post by: mxplxxx on 28/03/2019 22:32:42
Don't get hung up on the interpretation of h. It's just a constant. Suppose we count the number of pimples on teenagers' faces, and find a universal linear relationship between age and pimples. The we write N = px where N  is the number of pimples, x is the child's age, and p is an experimental constant whose value is N/x "pimples per year". p has no "interpretation" nor any deep philosophical meaning, but its value is very important.
I think h is more than just a constant.

Say, for convenience, h = 9; then

9 (h) = 9 joules x 1 second (or frequency 1)
9 (h) = 18 joules x .5 second (or frequency 2)
9 (h) = 3 joules x 3 seconds (or frequency .3)
and so on.

i.e. the energy of all photons derives from the same Action h (which, incidentally,  makes Action a more fundamental value than Energy and also introduces the possibility that time is the only variable in the universe with time acting on a single value of h to produce all the variations in energy/power we experience in the universe!).

Surely this makes it more than just a constant?

My post below clarifies this.

E=hf is confusing because it is a shortened version of what is really happening in the calculation of E which takes place in two steps:

1. t = 1/f  where t is the time to complete one cycle of an EM wave and f is the cycles per second (frequency) of the EM wave
2. E = h/t where E is the energy of one cycle of an EM wave and h is Action (6.62607004 × 10-34 joules times seconds) which is the same for all EM waves irrespective of their frequency.



Title: Re: How do we measure the energy of a photon?
Post by: Bored chemist on 29/03/2019 18:57:02
It's good to see that you have spotted that planck's constant has teh same units as action.
But it's not meaningful.

I have a torque wrench which is calibrated in newton meres and energy is also measured in newton metres.
But that doesn't mean that energy is the same as torque.
Title: Re: How do we measure the energy of a photon?
Post by: mxplxxx on 29/03/2019 21:59:38
It's good to see that you have spotted that planck's constant has teh same units as action.
But it's not meaningful.

I have a torque wrench which is calibrated in newton meres and energy is also measured in newton metres.
But that doesn't mean that energy is the same as torque.
Energy is commonly measured in joules, and according to Wiki is sometimes measured in Newton Metres.

One newton metre is equal to the torque resulting from a force of one newton applied perpendicularly to the end of a moment arm that is one metre long. It is also used less commonly as a unit of work, or energy, in which case it is equivalent to the more common and standard SI unit of energy, the joule.

You don't seem to have grasped the underlying consequence of h, namely that the energy of all quanta in the universe (i.e the energy of everything in the universe) is derived from the same amount of action (6.62607004 × 10-34 joules times seconds).

As a corollary to this, the energy of all quanta in the universe is tied to a particular value of time. In a computer program Energy E would be best described as a read-only variable with value h/t. It is, IMHO, extremely like the universe does something similar i.e. a quantum of energy would "contain" only the amount of time used to create it (or maybe it is just pure time i.e. a quantum of time :)?). 

It seems to me that power, not energy, is what drives the universe. You need a time component in every interaction.
.
Title: Re: How do we measure the energy of a photon?
Post by: alancalverd on 29/03/2019 23:25:20
You don't seem to have grasped the underlying consequence of h, namely that the energy of all quanta in the universe (i.e the energy of everything in the universe) is derived from the same amount of action (6.62607004 × 10-34 joules times seconds).
No it isn't. The energy of a photon is hf. That is all that Planck said, and all than anyone who understands simple equations understands by that statement. And to nobody's surprise, since all photons are electromagnetic radiation, i.e. the same stuff, it's hardly surprising that h is a universal constant.

The energy of a tank of diesel fuel is E = dv where v is the volume of fuel and d is a fairly universal constant with the value 3.6 x 107 joules per liter. What is the "consequence" of d? Does it apply to all substances in the universe? 

£ = r$. It doesn't matter how many dollars you have in  your pocket, I can calculate exactly how many pounds you can spend in my shop, anywhere in the universe, because the exchange rate r (pounds per dollar) is a universal constant (at least for any given day!). But what, I hear you ask, is the philosophical meaning of "pounds per dollar"?  Surely we should simplify it by expressing R in pounds, which everyone understands.....and so ad nauseam.

Just to add to your confusion, if you tighten a bolt to n newton-meters, that is indeed a measure of the energy expended in altering the dimensions of the bolt. Most of it has gone into producing a permanent deformation, or heat, but some can in principle be recovered from the rotational elasticity of the bolt. A clock spring uses the same principle but is optimised to minimise heat and deformation losses so most of the mechanical potential energy can be recovered as useful kinetic energy.     
Title: Re: How do we measure the energy of a photon?
Post by: mxplxxx on 30/03/2019 00:00:23
You don't seem to have grasped the underlying consequence of h, namely that the energy of all quanta in the universe (i.e the energy of everything in the universe) is derived from the same amount of action (6.62607004 × 10-34 joules times seconds).
No it isn't. The energy of a photon is hf. That is all that Planck said, and all than anyone who understands simple equations understands by that statement. And to nobody's surprise, since all photons are electromagnetic radiation, i.e. the same stuff, it's hardly surprising that h is a universal constant.

The energy of a tank of diesel fuel is E = dv where v is the volume of fuel and d is a fairly universal constant with the value 3.6 x 107 joules per liter. What is the "consequence" of d? Does it apply to all substances in the universe? 

£ = r$. It doesn't matter how many dollars you have in  your pocket, I can calculate exactly how many pounds you can spend in my shop, anywhere in the universe, because the exchange rate r (pounds per dollar) is a universal constant (at least for any given day!). But what, I hear you ask, is the philosophical meaning of "pounds per dollar"?  Surely we should simplify it by expressing R in pounds, which everyone understands.....and so ad nauseam.

Just to add to your confusion, if you tighten a bolt to n newton-meters, that is indeed a measure of the energy expended in altering the dimensions of the bolt. Most of it has gone into producing a permanent deformation, or heat, but some can in principle be recovered from the rotational elasticity of the bolt. A clock spring uses the same principle but is optimised to minimise heat and deformation losses so most of the mechanical potential energy can be recovered as useful kinetic energy.     
Most equations in physics are best understood with a knowledge of the underlying theory. h is not just a constant, it is a constant of action with units joules times time. For the history of h, see https://www.quora.com/Why-is-Plancks-constant-called-quantum-of-action. If you don't think what I am saying makes sense, try criticizing what I am actually saying rather than supplying your alternative understanding.
Title: Re: How do we measure the energy of a photon?
Post by: alancalverd on 30/03/2019 00:29:36
The underlying theory is that the black body spectrum can be derived from Planck's hypothesis that
Quote
  a hypothetical electrically charged oscillator in a cavity that contained black body radiation could only change its energy in a minimal increment, E, that was proportional to the frequency of its associated electromagnetic wave.

The constant of proportionality obviously has to have the dimensions of energy x time. h is not a "quantum of action" - the phrase is meaningless - but a constant with the dimensions of action.
Title: Re: How do we measure the energy of a photon?
Post by: Bored chemist on 30/03/2019 02:25:53
It seems to me that power, not energy, is what drives the universe.
It does not seem that way to anyone else.
Title: Re: How do we measure the energy of a photon?
Post by: mxplxxx on 30/03/2019 03:49:59
The constant of proportionality obviously has to have the dimensions of energy x time. h is not a "quantum of action" - the phrase is meaningless - but a constant with the dimensions of action.
I am pretty sure all quanta "contain" the same constant amount of action - h. Hence h is , quite reasonably, often referred to as the Quantum of Action (see https://en.m.wikipedia.org/wiki/Planck_constant).
Title: Re: How do we measure the energy of a photon?
Post by: mxplxxx on 30/03/2019 03:52:36
Quote
  a hypothetical electrically charged oscillator in a cavity that contained black body radiation could only change its energy in a minimal increment, E, that was proportional to the frequency of its associated electromagnetic wave.
Mumbo jumbo. No h in this either.
Title: Re: How do we measure the energy of a photon?
Post by: alancalverd on 30/03/2019 08:52:17
The use of "contain" in inverted commas implies that you don't understand your own question, so I won't attempt to answer it. But you would be well advised to read the Wikipedia reference you gave: every word is significant, especially the one you left out.

Probably not a good idea to dismiss Planck's hypothesis as mumbo jumbo, since it explained the black body spectrum completely, predicted a whole lot of other experimental results, and nobody has produced a better one. A few sentences later, he gave the constant of proportionality the symbol h.
Title: Re: How do we measure the energy of a photon?
Post by: mxplxxx on 30/03/2019 09:53:09
The use of "contain" in inverted commas implies that you don't understand your own question
"contain" in inverted commas means nobody knows how a photon processes energy.
Title: Re: How do we measure the energy of a photon?
Post by: mxplxxx on 30/03/2019 09:55:47
But you would be well advised to read the Wikipedia reference you gave: every word is significant, especially the one you left out.
I left out electromagnetic because it is now well know that quanta are present in all particles, not only electromagnetic waves.  I repeat,  "Quantum of Action" is often used in physics to denote h (e.g. https://link.springer.com/article/10.1140/epjh/e2017-80041-1)
Title: Re: How do we measure the energy of a photon?
Post by: alancalverd on 30/03/2019 12:53:35
Adding drivel to, and subtracting essential words from, standard texts will not help you understand physics.

The difference between a teacher and an educationalist is that a teacher takes a subject he understands, and explains it in a way that his pupil can understand. An educationalist takes a subject he doesn't understand, and cloaks it in mystery and jargon that nobody can understand. Beware of becoming an educationalist in your own sphere of interest.

How sad that the Journal of European Physics still publishes articles on the Bohr atom, and is so slack in its editing. Please set your watch back 100 years. 
Title: Re: How do we measure the energy of a photon?
Post by: alancalverd on 30/03/2019 12:56:35
If h is in joules and E is in  joules, then f has no dimension, which is nonsense.
Can you explain y our statement?
I really think you should study dimensional analysis before confusing yourself any further. Ten minutes of humility will change your life irrevocably and for the better.
Title: Re: How do we measure the energy of a photon?
Post by: Bored chemist on 30/03/2019 14:34:43
Energy is commonly measured in joules, and according to Wiki is sometimes measured in Newton Metres.

One newton metre is equal to the torque resulting from a force of one newton applied perpendicularly to the end of a moment arm that is one metre long. It is also used less commonly as a unit of work, or energy, in which case it is equivalent to the more common and standard SI unit of energy, the joule


Yes, obviously. We all knew that. You wasted time  and bandwidth by posting it

Do you understand that torque and energy have the same units (as it happens, the unit is the kilogram metre per second per second)?

Do you understand that torque and energy  are not the same thing?

Do you understand that action and planck's constant have the same units?

Do you understand that action and planck's constant are not the same thing?
Title: Re: How do we measure the energy of a photon?
Post by: mxplxxx on 30/03/2019 21:47:41
i.e. the energy of all photons derives from the same Action h (which, incidentally,  makes Action a more fundamental value than Energy and also introduces the possibility that time is the only variable in the universe with time acting on a single value of h to produce all the variations in energy/power we experience in the universe!).
It is interesting that I cannot find an example of a photon transferring  energy to another particle. I can find plenty of examples where the momentum of a particle is changed in an interaction with a photon.  This is despite the fact that a photon has no mass and therefore no momentum (yes I know this is controversial). What if a photon transfers time (i.e. time as in E=h/t) to the particle it interacts with, thus increasing its velocity and hence its momentum? Is this what happens in practice?
Title: Re: How do we measure the energy of a photon?
Post by: Kryptid on 30/03/2019 22:08:27
yes I know this is controversial

It's not controversial. We know for a fact that electromagnetic radiation has momentum:

Title: Re: How do we measure the energy of a photon?
Post by: alancalverd on 31/03/2019 02:16:53
It is interesting that I cannot find an example of a photon transferring  energy to another particle.
Funny, that. I make most of my living through that phenomenon: the use of x-rays to ionise stuff. Other people use lower energy photons to promote chemical reactions (vision) or excite electrons (radio).

Quote
I can find plenty of examples where the momentum of a particle is changed in an interaction with a photon.  This is despite the fact that a photon has no mass and therefore no momentum (yes I know this is controversial)


Delete "therefore" and study dimensional analysis to resolve the controversy that exists, uniquely, in your mind.
Title: Re: How do we measure the energy of a photon?
Post by: Bored chemist on 31/03/2019 09:43:00
Bored Chemist and Alancalverd can I respectfully suggest that if you are not adding value to Naked Scientists that you desist from posting.
OK.
You first.
You are not adding any value to the website.
You are adding nonsense.
And that's why the likes so of Alan and I  keep correcting you.


Your current posts are way too negative and  often directed towards the person rather than the facts.

On the whole we are not being negative about the individual posting the words, but we are quite scathing of what is posted.
That's because it's wrong.

I always try to respect the person, stick with the facts
No, you do  not.
You say things like

I cannot find an example of a photon transferring  energy to another particle.
which is obviously nonsense.
That's essentially the only thing that photons do.

Alan pointed this out by explaining that many common events- such as vision, rely on photons transferring energy.
He might also have chosen the fact that the Sun's radiation warms the earth or that light makes plants grow.

include positive suggestions for change with the criticism.
Well, that's exactly what he did
Delete "therefore" and study dimensional analysis to resolve the controversy that exists, uniquely, in your mind.


So it's pretty clear that you don't know what you are talking about.
Title: Re: How do we measure the energy of a photon?
Post by: Bored chemist on 31/03/2019 10:19:21
I would think it is fairly obvious I know what photons do.
No.
Because you said this
I cannot find an example of a photon transferring  energy to another particle.
it is fairly obvious that you do not know what photons do.


I might be spouting nonsense but I expect to be told why, not just that my well thought out posts are nonsense.

We have repeatedly told you why your ideas make no sense.
For example
https://www.thenakedscientists.com/forum/index.php?topic=75960.msg566209#msg566209

Title: Re: How do we measure the energy of a photon?
Post by: mxplxxx on 31/03/2019 10:26:31
Not E = E, E = ef where e is 6.62607004 × 10-34 joules, corresponding to the Action h
Of course, I meant E=hf .
Title: Re: How do we measure the energy of a photon?
Post by: Bored chemist on 31/03/2019 11:10:26
Do you really think people would bother if I did not know heaps about the nature of a photon?
Roughly half the posts are from other posters
Perhaps people come here to look at what those people wrote, rather than you.
Perhaps they come to laugh at you.

You say you know about photons.
If that is true, why did you post this?

I cannot find an example of a photon transferring  energy to another particle.


Sorry to say, this is pretty much an example of what I was saying. It adds nothing to the topic, is totally negative and I feel you are attacking me.
LOL
Title: Re: How do we measure the energy of a photon?
Post by: Bored chemist on 31/03/2019 13:12:31
LOL
Get a life. Do not post answers to my posts in future. I cannot imagine why you are doing so if you are genuine about helping people on Naked Scientists.
Do you feel that quoting a blank section from me is helping people?
I think that it's helpful when people like Alan and me point out mistakes made by others.
And it's also helpful if people actually answer questions.

So, once again.

You say you know about photons.
If that is true, why did you post this?

I cannot find an example of a photon transferring  energy to another particle.
Also, re
There is little of this on the internet - in fact a search for "example of a photon interaction" yields no results.

Well I couldn't find any hits for "example of a football interaction" either.
But that's because it is a poorly constructed search string.
I would expect better from
a world class systems analyst
Title: Re: How do we measure the energy of a photon?
Post by: Kryptid on 01/04/2019 04:54:31
https://en.wikipedia.org/wiki/Compton_scattering
Title: Re: How do we measure the energy of a photon?
Post by: mxplxxx on 01/04/2019 05:13:11
https://en.wikipedia.org/wiki/Compton_scattering
Interesting, but I see no reference to measuring the energy of a photon there. BTW, the basic tenet of quantum physics is that photons interact with matter in whole units (quanta). The following quote from WIKI seems to indicate a photon can interact using part of its energy.

"Compton scattering, discovered by Arthur Holly Compton, is the scattering of a photon by a charged particle, usually an electron. It results in a decrease in energy (increase in wavelength) of the photon (which may be an X-ray or gamma ray photon)"

How would the photon's energy be measured? Guesswork I am thinking.
Title: Re: How do we measure the energy of a photon?
Post by: Kryptid on 01/04/2019 05:14:36
Interesting, but I see no reference to measuring the energy of a photon there. BTW, the basic tenet of quantum physics is that photons interact with matter in whole units (quanta).

It was just meant to be an example of a photon transferring energy to another particle.
Title: Re: How do we measure the energy of a photon?
Post by: mxplxxx on 01/04/2019 05:18:43
Interesting, but I see no reference to measuring the energy of a photon there. BTW, the basic tenet of quantum physics is that photons interact with matter in whole units (quanta).

It was just meant to be an example of a photon transferring energy to another particle.
OK, thx. A good example.
Title: Re: How do we measure the energy of a photon?
Post by: Bored chemist on 01/04/2019 19:31:08
This is one reason I am here on this forum, Unfortunately, you (not Alan) often do so in a disrespectful manner. You seem to take pleasure in attempting to belittle people. Your replies are often not helpful at all. There is little in the way of friendliness or humor about your posts.
I am disrespectful of people who continue to waste time cluttering up the site with stuff that is known to be wrong.

What have these people done to earn any respect?
It's clear what they have done to lose it.
Title: Re: How do we measure the energy of a photon?
Post by: Bored chemist on 01/04/2019 19:32:14
Guesswork I am thinking.
Well that's a silly thing to think.
This whole thread has been telling you how to measure  energy of photons and yet you think people just guess.

In addition it is not helpful, it is not humorous and it is disrespectful.

Don't complain that I'm rude when you do exactly the same things I do.
Title: Re: How do we measure the energy of a photon?
Post by: Bored chemist on 01/04/2019 19:41:04
I cannot for the life of me imagine why you think this phrase means I don't know about photons.
regarding this phrase
"I cannot find an example of a photon transferring  energy to another particle."

It shows that you don't know about photons because transferring energy is the characteristic thing that photons do.

It's like saying " I can't find any evidence that cows give milk" and then claiming to know all about cows.


Using Google isn't the whole world of knowledge.
It only works if you use it correctly.

If you google
photon energy transfer
you find lots of information on the transfer of energy to particles by photons.

So you are blaming me for your inability to use Google.
Well, as I pointed out,



Well I couldn't find any hits for "example of a football interaction" either.
But that's because it is a poorly constructed search string.
I would expect better from

"a world class systems analyst"
Title: Re: How do we measure the energy of a photon?
Post by: mxplxxx on 03/04/2019 07:53:54
Given that, according to relativity, time varies according to the frame of reference it is measured in, and E=h/t, it is possible that it is not possible to measure energy without also (just?) measuring time.

Or is it possible that relativity could be simplified?

The Scaled Up/Down Universe

The universe consists of systems within systems within systems .... Each system is, in many respects, a scaled down version of its parent. Scaled down in the sense that all systems inherit a common base "System" type  (e.g. all systems contain a central object) and a parent system is a larger version of a child system. Each system experiences itself as "normal" i.e.  if you suddenly became a solar system, you would not notice anything different about the universe apart from the fact that you were now a solar system:).

What if the photons in each scaled up/down system vary in speed? For example, photon going from a solar system to the galaxy system might gain speed and a photon going from a solar system to the earth system might lose speed, and the speed of all photons at a particular level of the system hierarchy would be the same. In these cases, the differences in distance in the scaled up/down system combine with the new speed to make the speed of light a "perceived" constant.

Makes a lot of sense, satisfies Occam's Razor and we don't have to use the complexities of relativity to adjust time in different frames of reference (which now become equivalent of systems).

Evidence for this scaling theory comes from the fact that the universe expanded faster than the speed of light after the big bang. It may also have to do with energy "shifts" (e.g. red shift) which seem to contradict quantum theory.

ps it is likely each system will have a boundary and somehow or other the boundary contains energy. A photon scaling down will lose energy to the boundary and a photon scaling up will gain energy from the boundary, thus satisfying the conservation of energy requirement.
Title: Re: How do we measure the energy of a photon?
Post by: mxplxxx on 04/04/2019 23:24:44
Backing up my previous post, it would seem from the latest experiments that the speed of light is not a constant? Where does this leave Relativity? Physics seems to be getting more and more flexible ... rapidly.

https://scienceblog.com/507094/ucf-researchers-develop-way-to-control-speed-of-light-send-it-backward/?utm_source=feedburner&utm_medium=email&utm_campaign=Feed%3A+scienceblogrssfeed+%28ScienceBlog.com%29 (https://scienceblog.com/507094/ucf-researchers-develop-way-to-control-speed-of-light-send-it-backward/?utm_source=feedburner&utm_medium=email&utm_campaign=Feed%3A+scienceblogrssfeed+%28ScienceBlog.com%29)
Title: Re: How do we measure the energy of a photon?
Post by: mxplxxx on 12/04/2019 23:36:20
Physics has done itself a disservice when it comes to measuring anything. By concentrating on single objects rather than the makeup of objects, it seems to me an opportunity to understand how reality works was missed.

In the case of a photon, it needs to be realised that the full state of a photon is influenced by the systems that make up the system that a photon currently belongs to. For example, a photon that has been emitted by an electron on earth is currently part of (at least) the earth system, the solar system, the galaxy, a galactic cluster and the universe itself. Its total momentum will be influenced by the momenta gained in each of these systems.

Changes in the momentum of any of these systems will affect the current momentum of the photon; possibly leading to the Heisenberg uncertainty of the current photon momentum and/or position?

Similarly, if the photon were not an elementary particle, its current momentum would be influenced by the makeup of its sub-systems. Basically, the state of any system is determined by the states of all systems that comprise the branch of the universe that it belongs to.

Actually, I just realised a photon is not affected by gravity so its momentum will not be changed by revolving systems . I was going to withdraw the post but think there is much in it that is useful.
Title: Re: How do we measure the energy of a photon?
Post by: Bored chemist on 12/04/2019 23:47:13
Backing up my previous post, it would seem from the latest experiments that the speed of light is not a constant? Where does this leave Relativity?
Relativity is just fine.
Thanks for asking.
What you seem to not understand is that the speed of light in a vacuum is constant. The speed of light in stuff is slower. In water the speed is about 3/4  of the speed in a  vacuum

Was there anything else you forgot to read/ didn't understand?
Title: Re: How do we measure the energy of a photon?
Post by: Bored chemist on 12/04/2019 23:50:21
Physics has done itself a disservice when it comes to measuring anything. By concentrating on single objects rather than the makeup of objects

In the very real sense that I can't measure the length of my garden without knowing what flowers are growing in it.
However, in a much more real sense, I can weigh myself each morning without knowing exactly what my chemical makeup is.
Had you not realised that?
Title: Re: How do we measure the energy of a photon?
Post by: mxplxxx on 16/04/2019 04:39:58
There seems to be two basic types of energy in this universe, Static and Mobile. Static energy would be energy of an object that doesn't change, such as its Rest Mass. Mobile energy is energy that can move from object to object, such as a photon.

An electron that moves from one orbit to a higher orbit will have absorbed a photon of a particular frequency. Similarly an electron that moves from one orbit to a lower orbit will have emitted a photon of a particular frequency. I would program an electron as an electron system with an electron state machine at its centre and zero subsystems (if the electron were an atom this would contain electron systems), The state machine would contain a substate for each photon absorbed with its frequency a a property of the substate.  The overall state of the state machine is the electron state and would contain the total energy of the electron (as per the total of the proton states).

Electron System
    Electron (Central state machine)
        Electron State
            Photon 1 State
            Photon 2 State
            etc.
    Subsystems
         ... no subsystems

Whoops. This cannot be correct. A photon is the equivalent of an event in a computer system. It cannot be an object (i.e. it cannot have a persistent state).

No, this is ok. A photon (a boson) is the past state of a fermion. The big difference between bosons and fermions is that bosons contain a state that is fixed when they are created and fermions can vary their state.  The electron needs to record this past state somewhere in order to randomly emit a photon of the required frequency.

In this way an electron travels backward in time!!!
Title: Re: How do we measure the energy of a photon?
Post by: Bored chemist on 16/04/2019 18:20:27
There seems to be two basic types of energy in this universe,
It seems that way to you.
That may be because you have  not thought about it properly.
If you had you would consider, for example, a uranium atom.
Static energy would be energy of an object that doesn't change, such as its Rest Mass.

Well, the rest mass of a uranium atom is known to about 8 significant figures.


Mobile energy is energy that can move from object to object, such as a photon.
Energy from uranium atoms is transferred to nuclear submarines which move.

. I would program an electron as an electron system
How you choose to model it is not going to affect how real electrons behave.
A photon (a boson) is the past state of a fermion.
That makes no sense.

bosons contain a state that is fixed when they are created
Helium atoms are bosons.
They have plenty of different states and can move from one state to another.
So your idea is wrong.

" The electron needs to record this past state somewhere in order to randomly emit a photon of the required frequency. "
Does a ball at the top of a hill need to "record" its potential energy to roll down it?
That's just misuse of the word.

In this way an electron travels backward in time!!!
Even without the previous errors, that's a non sequitur
Title: Re: How do we measure the energy of a photon?
Post by: mxplxxx on 16/04/2019 23:22:42
From https://en.wikipedia.org/wiki/Composite_boson.

"A composite boson is a bound state of fermions such that the combination gives a boson. Examples include Cooper pairs, mesons, superfluid helium, Bose–Einstein condensates, atomic bosons, and fermionic condensates. "

From https://www.definitions.net/definition/bound+state

Bound state

"In physics, a bound state describes a system where a particle is subject to a potential such that the particle has a tendency to remain localised in one or more regions of space. The potential may be either an external potential, or may be the result of the presence of another particle. In quantum mechanics, a bound state is a state in Hilbert space that corresponds to two or more particles whose interaction energy is less than the total energy of each separate particle, and therefore these particles cannot be separated unless energy is spent. The energy spectrum of a bound state is discrete, unlike the continuous spectrum of isolated particles. In general, a stable bound state is said to exist in a given potential of some dimension if stationary wavefunctions exist. The energies of these wavefunctions are negative."
Title: Re: How do we measure the energy of a photon?
Post by: mxplxxx on 16/04/2019 23:32:30
From https://en.wikipedia.org/wiki/Potential_energy

"Potential energy is associated with forces that act on a body in a way that the total work done by these forces on the body depends only on the initial and final positions of the body in space. These forces, that are called conservative forces, can be represented at every point in space by vectors expressed as gradients of a certain scalar function called potential."

It would seem that potential energy depends on forces outside the particle.
Title: Re: How do we measure the energy of a photon?
Post by: mxplxxx on 16/04/2019 23:42:11
The mechanism by which an electron emits a photon and falls to a lower orbit is not understood in physics.
Title: Re: How do we measure the energy of a photon?
Post by: mxplxxx on 16/04/2019 23:48:27
In relation to the topic, from http://www.slac.stanford.edu/econf/C990809/docs/butterworth.pdf

"1.2 How photon structure is measured

The quick answer to the question “How do you measure the structure of the
photon?” is unfortunately, “With great difficulty”. Experiments measuring the
photon structure, in general, use the almost on-shell photons accompanying e+
or e− beams. These photons are typically probed by some short distance process.
This may be deep inelastic scattering [1], high transverse energy (Et) jets [2, 3, 4,
5, 6] or particles, or heavy quark production [7, 8]."
Title: Re: How do we measure the energy of a photon?
Post by: mxplxxx on 17/04/2019 01:27:42
Static energy would be energy of an object that doesn't change, such as its Rest Mass

It was pointed out that energy is derived from Uranium rest mass. Fair enough. I change my statement to "Static energy would be energy of an object that changes little, such as its Rest Mass". It should be pointed out though that the energy gained per atom is ridiculously small:
 
From http://www.emc2-explained.info/Emc2/Decay.htm#.XLZuT-gzZ3g
 
"The amount of mass that was turned into energy during the α-decay was:0.000,000,000,000,000,000,000,000,000,007,600 kg. Needless to say, this wouldn't show up on any kitchen scales! However, this number has been experimentally verified"
Title: Re: How do we measure the energy of a photon?
Post by: Bored chemist on 17/04/2019 07:22:38
The mechanism by which an electron emits a photon and falls to a lower orbit is not understood in physics.
Not by you, perhaps, but it is understood.
It should be pointed out though that the energy gained per atom is ridiculously small:
The mass of an atom is also ridiculously small.

So, in percentage terms, the change in mass is quite big.
Title: Re: How do we measure the energy of a photon?
Post by: mxplxxx on 17/04/2019 07:36:37
So, in percentage terms, the change in mass is quite big.
From https://en.wikipedia.org/wiki/Nuclear_fission

"When a uranium nucleus fissions into two daughter nuclei fragments, about 0.1 percent of the mass of the uranium nucleus[7] appears as the fission energy of ~200 MeV"
Title: Re: How do we measure the energy of a photon?
Post by: mxplxxx on 17/04/2019 07:52:28
The mechanism by which an electron emits a photon and falls to a lower orbit is not understood in physics.
I repeat this assertion. I would be pleased to be convinced otherwise.
Title: Re: How do we measure the energy of a photon?
Post by: mxplxxx on 17/04/2019 08:20:12
So, in percentage terms, the change in mass is quite big.
From https://en.wikipedia.org/wiki/Nuclear_fission

"When a uranium nucleus fissions into two daughter nuclei fragments, about 0.1 percent of the mass of the uranium nucleus[7] appears as the fission energy of ~200 MeV"

FRom https://www.mpoweruk.com/nuclear_theory.htm

"From one atom of Uranium-235 the mass of Uranium actually converted into the 200 MeV of energy is 3.56 X 10 -28 Kg, an almost infinitesimal amount."
Title: Re: How do we measure the energy of a photon?
Post by: alancalverd on 17/04/2019 08:39:35
The mechanism by which an electron emits a photon and falls to a lower orbit is not understood in physics.
I repeat this assertion. I would be pleased to be convinced otherwise.
Because it isn't true. We abandoned "orbits" about 100 years ago because they never made sense.
Title: Re: How do we measure the energy of a photon?
Post by: mxplxxx on 17/04/2019 08:43:05
The mechanism by which an electron emits a photon and falls to a lower orbit is not understood in physics.
I repeat this assertion. I would be pleased to be convinced otherwise.
Because it isn't true. We abandoned "orbits" about 100 years ago because they never made sense.
And can you explain why they never made sense and what has replaced them?

https://www.shmoop.com/modern-physics/atom-model.html discusses orbits as being part of modern physics theory?
Title: Re: How do we measure the energy of a photon?
Post by: mxplxxx on 17/04/2019 08:53:55
Sorry to say, in 90% of cases on Naked Scientists where I have been told I am wrong, the naysayer never explains why. Please justify your criticisms, or don't post.
Title: Re: How do we measure the energy of a photon?
Post by: mxplxxx on 18/04/2019 04:26:26
The mechanism by which an electron emits a photon and falls to a lower orbit is not understood in physics.
I repeat this assertion. I would be pleased to be convinced otherwise.
From https://www.quora.com/What-is-the-difference-between-a-photon-and-an-electron

Miroslav Halza

Answered Jan 19, 2018 · Author has 70 answers and 23.1k answer views

The electron comprises mostly the quantum particles, which are photons when are free to travel.

A photon comes to exist when the quantum particle of a subatomic particle is free (from a ‘jail’) to travel. Photons travel in a transverse form. If you annul the speed of the photon— make its linear momentum equal to zero, then you must get a particle that circles or spins, and so creates a shape looked like a vortex, or a hourglass or a wheel. Or, when we add the linear momentum to the circling (move its centre of rotation) we get a sinusoid, as the combination of the rotational trajectory and the straight-line trajectory.

The main reservoir of photons is an electron. This is because the electron already has a speed that is near to the speed of light, which is constant in the constant gravitational field. Besides it, the electron spins. That is why the photon gets the needed speed already being a constituent of the electron.

When the quantum particle is free (the photon is emitted), the electron loses on its linear momentum. When the quantum particle lands on the electron then it increases the linear momentum of the electron. However, it is not all.

We know, when the photon had landed on the electron it kicked the electron farther from the nucleus. But soon, the electron returns back, so has been attracted by an additional force. Since the electron is bound to the nucleus by the Coulomb force due to negative charge of the electron, it means the landed photon has changed its (let’s say sinusoidal) shape to a shape that is for the electric effect. Thus, the photon has added its ‘charge’ to the electron. This new charge—increased charge has caused an additional attraction of the electron in accordance with the Coulomb law. The added force causes the electron to jump into a nearer orbit to the nucleus.

Hence, the dual effects of the photon took actions in time during the landing. The first is colliding (Newton’s Third Law) when the photon added its linear momentum to the electron, and then has come the second effect—creating additional charge to the electron. The additional charge decreases a radius (Coulomb’s law) but does not decrease the linear momentum of the electron. And here is a reason why the photon does not anchor firmly on the electron. A higher momentum for this orbit is solved so that a photon is again emitted. And so we have atomic absorption and emission spectra. We use them in analyses of chemical elements and substances, but also to measure time—the atomic clock.

The conclusions are: The photon can produce the electric charge as the electron does since both have the same origins for the electric energy. They differ in their numbers. The photon has the linear momentum related to the speed of light, c, but the electron has the linear momentum to a speed that is less than c. Here is the difference. Namely, the electron is affected by the gravitational force because has the rest mass, but the photon does not have a rest mass and therefore may move by the speed of light in any gravitational field. Hence, the electron comprises the quantum particles for the negative electric charge and the quantum particles for mass, but the photon (a big photon) comprises only the quantum particles for the negative electric charge.


Title: Re: How do we measure the energy of a photon?
Post by: mxplxxx on 18/04/2019 08:17:48
A photon molecule!. From https://phys.org/news/2013-09-scientists-never-before-seen.html

"Working with colleagues at the Harvard-MIT Center for Ultracold Atoms, a group led by Harvard Professor of Physics Mikhail Lukin and MIT Professor of Physics Vladan Vuletic have managed to coax photons into binding together to form molecules – a state of matter that, until recently, had been purely theoretical. The work is described in a September 25 paper in Nature."

This opens the intriguing possibility that an electron is just a type of photon that exists in time (i.e at sub-luminal speeds). No wonder the electron and the photon share such a cosy relationship. Could it also open the possibility of all bosons having a fermion counterpart and vice versa? Mass may be just a photon that exists in time.

In my computer programming I have introduced an event (a boson) that contains an instance of a regular class (a fermion). In this way I can react in a program based on the past state of an object. I have found that reality and computer systems parallel each other in many ways.
Title: Re: How do we measure the energy of a photon?
Post by: mxplxxx on 18/04/2019 08:41:35
A photon (a boson) is the past state of a fermion.
... or just information (as per optical fibre networks). The likely reason bosons travel at  the speed of light is they need to be timeless (i.e unchanging).
Title: Re: How do we measure the energy of a photon?
Post by: Bored chemist on 18/04/2019 11:57:28
The mechanism by which an electron emits a photon and falls to a lower orbit is not understood in physics.
I repeat this assertion. I would be pleased to be convinced otherwise.
It's not our fault that you don't understand it.
I have pointed you in the right direction
https://www.thenakedscientists.com/forum/index.php?topic=75960.msg570962#msg570962

Did you look into it?
Sorry to say, in 90% of cases on Naked Scientists where I have been told I am wrong, the naysayer never explains why.
Are you sure?
Could it  be that they explain it, but you don't listen?


And can you explain why they never made sense
An orbiting electron, looked at from the side is an oscillating dipole and would emit radiation continuously until the electron spiralled into the nucleus.
Atoms do not do that, so we know they don't have orbiting electrons.

Why don't you study science rather than coming here and cluttering the site with your nonsense?

So, in percentage terms, the change in mass is quite big.
From https://en.wikipedia.org/wiki/Nuclear_fission

"When a uranium nucleus fissions into two daughter nuclei fragments, about 0.1 percent of the mass of the uranium nucleus[7] appears as the fission energy of ~200 MeV"
So a definite change then- you can measure that on a good set of kitchen scales.
Yet you were saying it was constant.
Static energy would be energy of an object that doesn't change, such as its Rest Mass.

Title: Re: How do we measure the energy of a photon?
Post by: mxplxxx on 19/04/2019 01:31:25
So, in percentage terms, the change in mass is quite big.
From https://en.wikipedia.org/wiki/Nuclear_fission

"When a uranium nucleus fissions into two daughter nuclei fragments, about 0.1 percent of the mass of the uranium nucleus[7] appears as the fission energy of ~200 MeV"

FRom https://www.mpoweruk.com/nuclear_theory.htm

"From one atom of Uranium-235 the mass of Uranium actually converted into the 200 MeV of energy is 3.56 X 10 -28 Kg, an almost infinitesimal amount."
Actually, it would seem that the rest mass of uranium is what remains after fission.

From https://atlas.cern/glossary/mass-invariant-mass

Mass / Invariant mass

Particle physicists use the word "mass" to refer to the quantity (sometimes called "rest mass") which is proportional to the inertia of the particle when it is at rest. This is the "m" both in Newton's second law of motion, F=ma, and in Einstein's equation, E=mc2 (in which E must be interpreted as the energy of the particle at rest). When a particle decays and hence no longer exists, its mass before the decay can be calculated from the energies and momenta of the decay products. The inferred value of the mass is independent of the reference frame in which the energies and momenta are measured, so that that the mass called "invariant". The concept is frequently generalized, so that for any set of particles (e.g., two leptons emerging from a collision), one can apply the same formulas to obtain an "invariant mass" (also called the “effective mass”) of the set.
Title: Re: How do we measure the energy of a photon?
Post by: Bored chemist on 19/04/2019 10:26:26
Actually, it would seem that the rest mass of uranium is what remains after fission.
Which is the same as the mass before fission.
That's why it's invariant.
However, if you look at what people actually think of as mass (not the relativistic bit) then there's a change.
So your assertion remains wrong.

Why do you do this?
You had an idea
There seems to be two basic types of energy in this universe, Static and Mobile. Static energy would be energy of an object that doesn't change, such as its Rest Mass. Mobile energy is energy that can move from object to object, such as a photon.

and it turns out that idea is wrong.
Why do you still cling to it?
Title: Re: How do we measure the energy of a photon?
Post by: mxplxxx on 21/04/2019 11:07:29
According to relativity, time stops for a particle when it reaches the speed of light (i.e. the particle stops aging). This makes sense given that the present is unfolding at the speed of light. This stopping of time can only occur for Bosons given an infinite amount of energy is required for a Fermion to reach SOL.. A photon may not exist in spacetime:). Also, given a photon is a Boson, the fact that it is timeless would likely make it impossible to detect except when it reacts with an electron (and other particles?). In fact, its timeless nature may be what gives rise to the Heisenberg uncertainty.

Also, maybe it is possible that the photon is part of an EM wave but is hidden because it is timeless. Somehow an EM wave can transport a timeless particle to locations in a spacetime universe. Phew!

A bit whishy washy but worthy of inclusion in a discussion of measurement of a photon's energy.
Title: Re: How do we measure the energy of a photon?
Post by: Bored chemist on 23/04/2019 14:37:41
In fact, its timeless nature may be what gives rise to the Heisenberg uncertainty.
No
Because that's observed with other things apart from photons.

A bit whishy washy but worthy of inclusion in a discussion of measurement of a photon's energy.
Not unless it makes a testable prediction.
Title: Re: How do we measure the energy of a photon?
Post by: mxplxxx on 04/05/2019 22:47:14
The energy of a photon is hf. That is all that Planck said, and all than anyone who understands simple equations understands by that statement. And to nobody's surprise, since all photons are electromagnetic radiation, i.e. the same stuff, it's hardly surprising that h is a universal constant.
A photon is a particle. If is is also a wave it will have a wave function but since this has never been discovered the existence of a photon as a wave is unproven. Photons are not electromagnetic waves and so E=hf does not apply to them.

Also, from https://www.nobelprize.org/prizes/themes/the-dual-nature-of-light-as-reflected-in-the-nobel-archives

"Evidence for the particle nature of light

In physics textbooks two phenomena are usually quoted demonstrating the particle nature of light: 1) the photoelectric effect and 2) the Compton scattering of X-rays.

In some not so critical texts a third circumstance is erronously quoted, namely Planck‘s discovery of energy quanta, which he did in his analysis of heat radiation. The Nobel Committee honoured this monumental discovery by the Physics Prize in 1918, but did not make the mistake to give Planck credit for having discovered the particle nature of light."
Title: Re: How do we measure the energy of a photon?
Post by: Bored chemist on 05/05/2019 10:03:58
Photons are not electromagnetic waves and so E=hf does not apply to them.
Absolutely every experiment on the issue has shown that E=hf does apply to photons.
Why post something which is clearly wrong?
Title: Re: How do we measure the energy of a photon?
Post by: mxplxxx on 06/05/2019 11:32:46
No, this is ok. A photon (a boson) is the past state of a fermion. The big difference between bosons and fermions is that bosons contain a state that is fixed when they are created and fermions can vary their state.  The electron needs to record this past state somewhere in order to randomly emit a photon of the required frequency.

In this way an electron travels backward in time!!!
Which makes it possible that the universe is, in some way,  travelling forward and backward in time. i.e. things evolve and then revert to their original state. Possibly a reason for birth and death.

In fact, it would seem that the whole purpose of energy in the universe is to enable universal objects to function at higher and higher levels of state/evolution (until the energy is exhausted in some way and regression follows).
Title: Re: How do we measure the energy of a photon?
Post by: Bored chemist on 06/05/2019 11:47:43
In fact, it would seem that the whole purpose of energy in the universe is to enable universal objects to function at higher and higher levels of state/evolution
No.
It may seem that way to you, but that's probably wishful thinking.
The real fate of the universe is the so called "heat death".
On the whole, the universe is grinding to a halt.
Title: Re: How do we measure the energy of a photon?
Post by: mxplxxx on 06/05/2019 12:13:08
The universe consists of systems within systems within systems .... Each system is, in many respects, a scaled down version of its parent. Scaled down in the sense that all systems inherit a common base "System" type  (e.g. all systems contain a central object) and a parent system is a larger version of a child system. Each system experiences itself as "normal" i.e.  if you suddenly became a solar system, you would not notice anything different about the universe apart from the fact that you were now a solar system:).
It is possible that in a galactic photon exists that is a scaled up version of the photon that interacts with an electron. This galactic photon can interact with the earth so that the earth is boosted to Mar's orbit, much like a normal photon interacts with an electron to boost it to a higher orbit (maybe how dinosaurs became extinct:).

Maybe the solar wind is a galactic EM wave? ... Probably not,  but given moving charged particles generate EM fields, the existence of an EM wave as part of the solar wind is not beyond the bounds of possibility. Maybe I should have said " maybe the solar wind generates a galactic EM wave". See also https://agupubs.onlinelibrary.wiley.com/doi/pdf/10.1029/JA075i019p03735.
Title: Re: How do we measure the energy of a photon?
Post by: Bored chemist on 06/05/2019 13:19:55
Maybe the solar wind is a galactic EM wave?
No.
Why not learn some science?
Title: Re: How do we measure the energy of a photon?
Post by: mxplxxx on 09/05/2019 07:41:56
The universe consists of systems within systems within systems .... Each system is, in many respects, a scaled down version of its parent.
There will likely exist a constant that is the scale factor for all systems in the universe. Assuming this is 10 all dimensions in a particular system (including time) will be multiplied by 10 in its parent system. The child will observe the parent ageing at a factor of 10 less than it (the child) is ageing. But the parent system won't be aware of this. Because of scaling, the parent will measure things in its system exactly the same as the child system measures them. In fact, from their own perspectives, all systems behave identically to a single conceptual base system. This is effectively Relativity without the need to consider the speed of light limit.

Title: Re: How do we measure the energy of a photon?
Post by: mxplxxx on 20/05/2019 11:07:30
Actually the Universe itself is a system. And there may be other systems on top of it ad infinitum. This means that space and the Universe are separate entities with the likelihood that space is continuous. The fact that systems are digital in nature  indicates that there also may exist an aspect of space that is fully digital in nature. i.e it is filled with null-state points. This aspect may be conceptual only in nature and therefore tied to consciousness in some way. There is no movement in this universe. Each point takes on a state based on changes in other states over time. There will be a single universal algorithm that governs these state changes.

Systems and particles are just ways we mortals have invented to make these state changes easier to understand. Just as a computer language makes it easier to program a computer. They are in effect simulations of spacetime. The smaller a point in the simulation, the closer to reality is the simulation. Continuous space is merely infinitely small points.

Title: Re: How do we measure the energy of a photon?
Post by: Bored chemist on 20/05/2019 15:34:46
Assuming this is 10 all dimensions in a particular system (including time) will be multiplied by 10 in its parent system.
Why do you assume that the universe is determined by the number of fingers you have?
Title: Re: How do we measure the energy of a photon?
Post by: mxplxxx on 21/05/2019 02:03:45
The fact that systems are digital in nature  indicates that there also may exist an aspect of space that is fully digital in nature. i.e it is filled with null-state points. This aspect may be conceptual only in nature
Lets face it there are aspects of reality that don't make sense unless reality IS conceptual in nature. Such as "action at a distance" and gravity in general. Somehow or other, a graviton causes a particle to be "pulled" rather than "pushed". If I were to program this situation in a computer, I would do as as follows

if event  is photon then
    force.direction = forward
elseif event is graviton   
    force.direction = backward
end if

Does reality do something similar? I suspect so.
Title: Re: How do we measure the energy of a photon?
Post by: Bored chemist on 21/05/2019 19:28:45
What do you mean by this?
reality IS conceptual in nature.
Title: Re: How do we measure the energy of a photon?
Post by: mxplxxx on 22/05/2019 13:00:06
What do you mean by this?
reality IS conceptual in nature.

From Google

CONCEPT

noun
noun: concept; plural noun: concepts
an abstract idea.
"structuralism is a difficult concept"
synonyms:   idea, notion, conception, abstraction, conceptualization; More
PHILOSOPHY
an idea or mental image which corresponds to some distinct entity or class of entities, or to its essential features, or determines the application of a term (especially a predicate), and thus plays a part in the use of reason or language.


My take is that there is a "real" part to reality that is continuous/timeless in nature and a conceptual part that is digital/time-bound in nature that simulates aspects of the continuous part. It is very possible that we exist as part of something that is basically a computer.
Title: Re: How do we measure the energy of a photon?
Post by: mxplxxx on 22/05/2019 13:19:25
An atom is a system. The atom system contains a central Ion object and peripheral electron objects (which are likely to be systems themselves). The Ion object contains proton and neutron objects. It is likely that protons are the equivalent of reference objects in computer science. Each proton object references an electron object.   This means that Systems have two states. One that is basically an abstraction that is the combined states of their children (electrons in the case of an atom) and the other that is an abstraction of the total state of the system itself.   
Title: Re: How do we measure the energy of a photon?
Post by: Bored chemist on 22/05/2019 19:11:16
What do you mean by this?
reality IS conceptual in nature.

My take is that there is a "real" part to reality that is continuous/timeless in nature and a conceptual part that is digital/time-bound in nature that simulates aspects of the continuous part.
So, you don't mean anything that could actually be science.
Why post it here?
Title: Re: How do we measure the energy of a photon?
Post by: mxplxxx on 22/05/2019 23:46:22
What do you mean by this?
reality IS conceptual in nature.

My take is that there is a "real" part to reality that is continuous/timeless in nature and a conceptual part that is digital/time-bound in nature that simulates aspects of the continuous part.
So, you don't mean anything that could actually be science.
Why post it here?
From https://www.scientificamerican.com/article/is-string-theory-science

Is String Theory Science?
A debate between physicists and philosophers could redefine the scientific method and our understanding of the universe


Most of my theories are based on a comparison between physics science and computer science.

In any case, this is the New Theories section in which posts can be "on the lighter side".
Title: Re: How do we measure the energy of a photon?
Post by: mxplxxx on 23/05/2019 03:57:16
Assuming this is 10 all dimensions in a particular system (including time) will be multiplied by 10 in its parent system.
Why do you assume that the universe is determined by the number of fingers you have?
From Google

assume

VERB
suppose to be the case, without proof.
"topics which assume detailed knowledge of local events" · [more]
synonyms:
presume · suppose · take it · [more]

In other words, I am saying "For example, suppose this is 10, ..."
Title: Re: How do we measure the energy of a photon?
Post by: Bored chemist on 23/05/2019 19:50:10
Assuming this is 10 all dimensions in a particular system (including time) will be multiplied by 10 in its parent system.
Why do you assume that the universe is determined by the number of fingers you have?
From Google

assume

VERB
suppose to be the case, without proof.
"topics which assume detailed knowledge of local events" · [more]
synonyms:
presume · suppose · take it · [more]

In other words, I am saying "For example, suppose this is 10, ..."
Everyone knows what it means.
The question was why are you doing it?
Most of my theories ar
You have yet to post a competent  conjecture, never mind a theory.
https://en.wikipedia.org/wiki/Scientific_theory

Title: Re: How do we measure the energy of a photon?
Post by: Bored chemist on 23/05/2019 19:51:28
In any case, this is the New Theories section in which posts can be "on the lighter side".
Yes, but they can't be vacuous dross.
Title: Re: How do we measure the energy of a photon?
Post by: mxplxxx on 25/05/2019 01:48:57
Computers have been around now for over 50 years. But simulations as a means of describing/explaining physical systems are few and far between.

The reason for this could be that physics was initially described very successfully via mathematics. Ever since then, a gargantuan effort has been made to retain mathematics as the primary means of specifying physical systems. But mathematics is rigid in the extreme and horrendously difficult for lay people to understand. And the latest physical theories describe complex systems which mathematics has always had difficulty with.

The "how" of physical system has been lost in the pursuit of mathematical predictions. But mathematics is extremely limited in its ability to predict the results of  interactions of multiple systems.

Computer programming, on the other hand, is relatively easy to understand and can describe/simulate highly complex systems. Despite this, its uptake by the physics community appears to have been very limited to date . This may be  because computer programming is very general in nature and physicists have difficulty with general concepts. Or maybe it is an aversion to moving away from mathematics.

As my posts on this topic illustrate, a world of possibilities open up if computer science and physics are married. What appears to be dross to a physicist may well be the underlying gold for someone who has knowledge of computer science.
Title: Re: How do we measure the energy of a photon?
Post by: Bored chemist on 25/05/2019 01:59:37
But simulations as a means of describing/explaining physical systems are few and far between.
Not  in my world.

However, if it was true, it could indicate that "maths" is a better way of doing things than computer simulation .
(at this point, the grown ups are laughing but...)
So, maybe trying to model the real world as computer science is a silly idea...


Most of my theories are based on a comparison between physics science and computer science.
Title: Re: How do we measure the energy of a photon?
Post by: evan_au on 25/05/2019 09:18:56
Quote from: mplxxx
physics was initially described very successfully via mathematics
I have seen the section from Galileo's notebook where he determined that the velocity of a particle increases as time squared. He used an apparatus similar to the frets on a musical instrument fastened across an angled track, with a metal ball falling under gravity.

The commentators pointed to the section where he recorded his measurements.
- Galileo had crossed out some of the results and written in other numbers to make them fit the maths
- Because he realised that when the ball was moving slowly, the frets presented a significant obstruction to the ball, which wasn't a problem when the ball was moving quickly.

Pure physics might be describable by pure maths, but the complexities of the real world are far too complex for pure maths. That's why we have a field of "applied maths". Both areas of maths use computers extensively, as do pure and applied physicists.

Quote
(computer science) uptake by the physics community appears to have been very limited to date
Most of the very biggest supercomputers in the world are for physics problems, including predicting the weather, and designing atomic bombs. But let's look at some others:
- A simulation of the entire universe since the Big Bang, comparing the distribution of galaxies to what we see today. The Illustris project used 19 million CPU hours:

- A simulation of the last few hours of a star's life as it undergoes a supernova. The first successful simulation (published early 2019) took 6 months CPU time on a large supercomputer.

- The Square Kilometer Array, beginning construction in Australia and South Africa will have a supercompter more powerful than any currently on Earth.
Title: Re: How do we measure the energy of a photon?
Post by: mxplxxx on 25/05/2019 10:26:23
A simulation of the entire universe since the Big Bang, comparing the distribution of galaxies to what we see today.
I would love to see the code behind this. Or even a description of what the code does. Given we don't actually know what happened at the time of the Big Bang, I am skeptical that such a simulation could exist.

I would not class anything as  a simulation that does not simulate something. The use of mathematics is rarely a simulation of what has actually occurred. The fact is we don't actually know what has occurred in the evolution of most physics systems. In other words we can predict many results but we have no idea how they occurred. Therefore we cannot simulate them.

In any case I have said simulations in physics are few and far between and, even when available, the code behind them is rarely available. I suspect that very few physicists can program a reactive system or understand the workings of a hierarchical finite state machine which would be the minimum required computer science expertise required for a physics simulation.
Title: Re: How do we measure the energy of a photon?
Post by: mxplxxx on 25/05/2019 10:36:24
The first successful simulation (published early 2019) took 6 months CPU time on a large supercomputer.
Not sure what kind of a simulation this was, given we don't actually know what happens inside a super nova (admitted by scientist in the video who also admitted that gravity always won out in simulations attempted to date).
Title: Re: How do we measure the energy of a photon?
Post by: jeffreyH on 25/05/2019 10:54:38
A simulation of the entire universe since the Big Bang, comparing the distribution of galaxies to what we see today.
I would love to see the code behind this. Or even a description of what the code does. Given we don't actually know what happened at the time of the Big Bang, I am skeptical that such a simulation could exist.

I would not class anything as  a simulation that does not simulate something. The use of mathematics is rarely a simulation of what has actually occurred. The fact is we don't actually know what has occurred in the evolution of most physics systems. In other words we can predict many results but we have no idea how they occurred. Therefore we cannot simulate them.

In any case I have said simulations in physics are few and far between and, even when available, the code behind them is rarely available. I suspect that very few physicists can program a reactive system or understand the workings of a hierarchical finite state machine which would be the minimum required computer science expertise required for a physics simulation.

How dismissive and pompous of you. Well done! Of course only you could be smart enough for such achievements.

Scientists, the best ones, are often much more intelligent and inventive than you will ever give them credit for. They have to be. Try finding an exact solution to Einstein's field equations.

Try to start listening and get off your soap box.
Title: Re: How do we measure the energy of a photon?
Post by: jeffreyH on 25/05/2019 11:01:53
LDA #0
:LOOP
ADDA #1
CMPA INFINITY
JNE LOOP

That looks like mathematics to me.
Title: Re: How do we measure the energy of a photon?
Post by: Bored chemist on 25/05/2019 11:20:57
I'm going to get lynched for this but...
I suspect that very few physicists can program a reactive system or understand the workings of a hierarchical finite state machine which would be the minimum required computer science expertise required for a physics simulation.

Many physicists will also delegate the mundane work of repairing a faulty car engine to someone who can't do the physics, but is "good with their hands".
In much the same way, they will delegate writing code.
Did you know that the phrase "code monkeys" is fairly commonly used?
https://en.wikipedia.org/wiki/Code_monkey

The fact is that most scientists (at least those who, like me were teenagers in the 80s) are able to write code.
Some of us still do.

So here's the dissymmetry for you.
Physicists can (almost always)  programme.
Programmers  (almost always) can't understand physics.

You have demonstrated that you can't

Title: Re: How do we measure the energy of a photon?
Post by: mxplxxx on 25/05/2019 13:29:15
How dismissive and pompous of you. Well done! Of course only you could be smart enough for such achievements.
And your statement is not dismissive and pompous + sarcastic:) ? I think you may be confusing impact and intent?

I try and state facts and keep emotion out of my posts. I have an applied psychology degree from the Australia College of Applied Psychology (can post the certificate if necessary). I have no need to be dismissive and pompous. I would really like you guys to criticize what I say rather than what sort of a person you think I am. Use this as a rule to live by and life suddenly becomes much easier:).

If I am pressing a button you would rather not have pressed, maybe there is a lesson involved for you (or maybe I am just plain wrong, in which case tell me what you think is wrong and why and give me an alternative if one exists).
Title: Re: How do we measure the energy of a photon?
Post by: mxplxxx on 25/05/2019 13:53:03
Physicists can (almost always)  programme.
Programmers  (almost always) can't understand physics.
What is your basis for these statements?

 I am a full stack software developer. I design all aspects of an App as well as doing the programming, solo. The difference between me and a developer who cuts code is enormous. I have university maths to HD standard.  I am amongst the elite of my profession. A physicist who desires his theory to be simulated would explain the theory to me and I would do the rest. Unless, of course, the physicist is also a pretty smart software developer. 

Many computer programs are amongst the most complex entities in existence. Creating them requires someone with considerable simplification, improving, explaining and creating skills. Is this typical of physicists? It certainly is of top software developers.
Title: Re: How do we measure the energy of a photon?
Post by: mxplxxx on 25/05/2019 14:54:38
In much the same way, they will delegate writing code.
Did you know that the phrase "code monkeys" is fairly commonly used?
https://en.wikipedia.org/wiki/Code_monkey
From Urban Dictionary:

code monkey

One who copies all code from other sources and prays that their code compiles. Normally used in a negative manner when used by more experienced programmers.


If you are trusting code monkeys to write your programs, you will be skating on very thin ice:). For something like a physics simulation, either do it yourself if you are good enough or assign it to a rock-star Software Developer with a physics bent (like me:)).

Title: Re: How do we measure the energy of a photon?
Post by: Bored chemist on 25/05/2019 15:09:48
Physicists can (almost always)  programme.
Programmers  (almost always) can't understand physics.
What is your basis for these statements?


Well, let's start with the second.
You are a perfect example.
A programmer who doesn't understand physics (if you did, you wouldn't have started this thread).
And, while, strictly, I'm a chemist, rather than a physicist, I do get paid to write code.

So, based on  a rather small sample size, I'm right.
For something like a physics simulation, either do it yourself if you are good enough or assign it to a rock-star Software Developer with a physics bent (like me:)).

No.
If I wanted someone to code physics for me I'd certainly not choose someone who, like you, doesn't even understand how we measure photon energies- even after being repeatedly  told how we do it.

You make the monkey look good.
Title: Re: How do we measure the energy of a photon?
Post by: mxplxxx on 25/05/2019 22:27:13
LDA #0
:LOOP
ADDA #1
CMPA INFINITY
JNE LOOP

That looks like mathematics to me.
It looks like a snippet of Assembler code containing some maths to me.  What is your point?
Title: Re: How do we measure the energy of a photon?
Post by: mxplxxx on 25/05/2019 23:22:23
Getting back to the topic, I am going to postulate an explanation of how a photon behaves like an EM wave and the process involved in transferring the photon's energy to another particle.

Most (possibly all) objects in the universe are systems. A system consists of a central (centre of gravity) object surrounded by peripheral system objects. So we have a solar system containing a central Sun object surrounded by peripheral planetary system objects such as the Earth system. There exist very similar bosonic and fermionic systems. A bosonic system has a central object of that can assume a single state only. A fermionic system has a central object that can assume multiple states. A bosonic system is basically related to events whilst a fermionic system is related  to data.
The system/central object relationship may correspond to the wave/particle duality relationship in physics.

It would also seem that inner and outer shells are present. Inner shells are part of the central object. They contain systems that relate to the state of the central object only. Outer shells surround the central object and contain peripheral systems. The beauty of a shell is that it can, literally, confine systems that exist as waves or as particles when measured (and thus Schrodinger's equation applies?).

A photon system is a bosonic system. It consists of a central photon object containing a single shell which in turn contains a quantum of energy. The central photon object is surrounded by a single shell containing interacting ELECTRO amd MAGNETIC waves. These  would be observed as E and M particles (my name) if they could be observed.The E and M particles rotate in circles with the centre of the circle at the centre of the photon system. The E particle rotates in the direction of travel of the photon system, and the M particle rotates at right angles (i.e. transversely) to the E particle 

When travelling at the speed of light this photon system behaves like an EM wave. The circular rotation of the E and M waves become sinusoidal E and M waves. The wave length of the EM wave is the diameter of the outer shell.

When interacting with a fermionic system, the photon system is absorbed by the fermionic system. The time taken for this absorption is λ/c where λ is the wave length of the photon system and c is SOL. The energy of the central photon is transferred to the fermionic system and the E and M particles annihilate each other. Note that this energy will likely vary according to changes in the diameter of the outer shell of the photon system (according to E=hc/λ). This may well explain red etc. "shifts", which are postulated in current physics despite, apparently, their breaking the conservation of energy.

Because of the quantum nature of reality the energy transfer process cannot be interrupted. All or nothing of the photon energy must be transferred.

What a shell is made of is unknown. But it may form due to spacetime being modified by the presence of massive particles (see https://einstein.stanford.edu/SPACETIME/spacetime2.html).


Title: Re: How do we measure the energy of a photon?
Post by: evan_au on 25/05/2019 23:59:44
Quote from: mplxxx
I would love to see the code behind this. Or even a description of what the code does. Given we don't actually know what happened at the time of the Big Bang, I am skeptical that such a simulation could exist.
The references at the bottom of the Wikipedia article will give you more details than you can handle.

Although the physics of gravity is fairly simple (Newton got it almost right), one of the challenges in a simulation like this is that particles which are far apart can use large (simulated) time steps, while those approaching closely need very small time steps. That makes it hard to run on parallel computers, since all the computers have to share information with each other in simulated time.

Despite the existence of compilers that can spread a single piece of code across thousands of CPUs, the need for parallelism does somewhat obscure the code.

See: https://en.wikipedia.org/wiki/Illustris_project

Quote
Not sure what kind of a simulation this was, given we don't actually know what happens inside a super nova
It was the first successful simulation of a supernova, with a full 3D simulation of the star on a large supercomputer.
Previous attempts had used a 1D or 2D simulation of a star, modeling composition, temperature and pressure with depth, as well as the nuclear effects expected under these conditions. They could only get enough computer time to simulate about 1 second of the star's last moments. As the video said, all these ended with a black hole, not a supernova.

That's very not surprising, as astronomers estimate that there should be around 4-8 times as many supernova in our galaxy as we actually see; one theory is that the star might get swallowed by a black hole at its center before it can explode as a supernova.

This supernova simulation needed to include the 3-dimensional nature of the star and magnetic/conductivity effects. They also simulated a whole 3 minutes of the stars last moments. Even then, they needed to tweak some parameters to get the simulated star to explode. A 3D model of this nature takes an enormous computer.
See: https://www.space.com/30133-supernova-3d-star-explosion-simulation.html

...and it looks like it was published in 2015 - I thought I heard about it much more recently than that!
Title: Re: How do we measure the energy of a photon?
Post by: mxplxxx on 26/05/2019 03:06:12
The references at the bottom of the Wikipedia article will give you more details than you can handle.
Except the one I really need, which is a look at the code! As well, much of an attempt to explain the workings of the simulation is incomprehensible, even to experienced physicists I suspect - let alone a lay person. Both these facts make me highly suspicious that the simulation is a furphy.

From Wikipedia.

A furphy is Australian slang for an erroneous or improbable story that is claimed to be factual. Furphies are supposedly 'heard' from reputable sources, sometimes secondhand or thirdhand, and widely believed until discounted.

Ditto to the SuperNova simulation which was apparently fudged anyway due to their code not producing a SuperNova.
Title: Re: How do we measure the energy of a photon?
Post by: mxplxxx on 26/05/2019 03:44:14
That makes it hard to run on parallel computers, since all the computers have to share information with each other in simulated time.
Impossible not hard, I would have thought. A quantum computer may be the shot, but these appear to be a long way off if at all possible. I doubt very much whether real simulations are being run. A real simulation will start with particles and model how aggregations of them evolve over time. But, hey, isn't this sort of simulation how the universe itself operates? i.e. a processor per particle.We probably need to start at some level of abstraction that arises from the a theory of the way particle interactions occur at  a basic level. But I have not come across such a theory.
Title: Re: How do we measure the energy of a photon?
Post by: jeffreyH on 26/05/2019 05:35:17
I tell you what I believe is a 'Furfy', a self professed 'rock star' coder with an over inflated ego trying to smear the reputations of professional scientists. Much like any other grifter.
Title: Re: How do we measure the energy of a photon?
Post by: mxplxxx on 26/05/2019 06:40:21
I tell you what I believe is a 'Furfy', a self professed 'rock star' coder with an over inflated ego trying to smear the reputations of professional scientists. Much like any other grifter.
This is incorrect and probably libelous.
Title: Re: How do we measure the energy of a photon?
Post by: evan_au on 26/05/2019 07:13:59
Quote from: mplxxx
A furphy is Australian slang for an erroneous or improbable story
A Furphy (Trademark) was a real thing in Australia - a water tank used for agriculture, firefighting, and providing water to troops in World War 1.
 [ Invalid Attachment ]
 [ Invalid Attachment ]
Unfortunately the chain covers a crucial part of the story, but during WW1, the driver taking the water cart around to the troops was the carrier of the latest rumours*. This is the origin of "Furphy" in Australian slang. Since you are in Australia, you can see a real Furphy at the Museum of Fire in Penrith (and other museums around the country).

But for the purpose of this discussion, I can assure you that:
- Many physicists know how to write computer software
- Even more have competent programmers they can call on for particular tasks
- Software is extensively used in all branches of science
- When I went to university, we had to learn how to program in FORTRAN (FORmula TRANslation) in first year. At that time, FORTRAN had been the main language for scientific programming for many years
- Later language options at the time included C, LISP, APL, COBOL, etc.

A joke doing the rounds when I was at university:
Question: "What style of programming language will be used for scientific programming in the year 2000?".
Answer: "I have no idea what style of programming it will be - but I do know what it will be called - FORTRAN!"
...and the joke turned out to be true - large scientific simulations are still written in FORTRAN, but with far more structure than earlier versions, and with options to assist running on large parallel computers.
So if you want to understand physics simulations, you had better go back to the future, and learn FORTRAN!

Listen, for example, to the discussion of software used for simulating nuclear fusion experiments (a 100+ dimensional optimisation problem). Much of this podcast is about physics simulations, but listen for 10 minutes from:
https://omegataupodcast.net/312-the-wendelstein-7-x-fusion-experiment/#t=1:40:00

So the only furphy here is the suggestion that physicists don't use simulations.

*Incidentally, "Scuttlebutt" has a similar origin, but in a naval context.
Title: Re: How do we measure the energy of a photon?
Post by: mxplxxx on 26/05/2019 07:41:39
So the only furphy here is the suggestion that physicists don't use simulations.
Where did I say that?  What I am saying is that physics simulations are hard to find. Even harder to find are ones that come with lucid descriptions of the software. And of the few I found, none supplied code.

The reason Fortran is used for modelling physics phenomena is that it it the best language for implementing mathematical formula. The reason I say modelling rather than simulating is that most simulations I am aware of are based on UML StateCharts which is a kind of software framework especially designed to implement software simulations. I am an expert in these. Such simulations are based on the "how" of physics rather than mathematical predictions.  (see http://ce.u-sys.org/Veranstaltungen/Simulation%20and%20Modeling%201/%C3%9Cbung%2002/SM_E2_RTUMLAnyLogic.pdf)

Thx for the information on furphies. President Trump's false news is in effect furphies. It is wise these days to question anything that comes without evidence of its validity. There is grant money to be made in convincing furphies. How about the Nature article that talks about particles travelling at 40 time the speed of light with no evidence of the validity of the statement. If these guys aren't angling for grant money, I'll be very surprised.

You say "Many physicists know how to write computer software" Fair enuff. I wonder how many of these know how to program simulations using UML Statecharts?

Actually I have my own simulation framework that I hope to foist on the computer profession soon. It is called 3dAbstractions and is based on Hierarchical Finite State Machines theory. 3d as in D for Display, D for Data and D for Datastore. I am a great fan of the Holographic Universe theory (see https://www.southampton.ac.uk/news/2017/01/holographic-universe.page). I have applied the principles of the theory to my software. Data is equivalent to 2d data at the edge of the universe. Display is a 3d projection of the Data onto
a  computer screen. Datastore is permanent memory (which  would exists as 4d spacetime in Einstein's universe).
Title: Re: How do we measure the energy of a photon?
Post by: jeffreyH on 26/05/2019 08:22:40
And I quote.
If you are trusting code monkeys to write your programs, you will be skating on very thin ice:). For something like a physics simulation, either do it yourself if you are good enough or assign it to a rock-star Software Developer with a physics bent (like me:)).
End quote.

That looks like the statement of a self professed rock star coder. It also looks like the words of someone with an over inflated ego.
Title: Re: How do we measure the energy of a photon?
Post by: mxplxxx on 26/05/2019 08:41:13
That looks like the statement of a self professed rock star coder. It also looks like the words of someone with an over inflated ego.
IYHO. Your post is inflammatory and not based on fact. I mention I am a developer not a coder.  And I am at the top of my profession. Why should I not make this known and rejoice in the fact? I really applaud people who take pride in their achievements. I urge my coaching clients to develop HUGGs (Huge Unbelievably Great Goals) rather than develop false modesty which you seem to be advocating. Plus I have a smiley at the end of " assign it to a rock-star Software Developer with a physics bent (like me:))." which means I do not take the fact too seriously.

And calling me a grifter which has criminal connotations is totally uncalled for.

I am aware as a life coach that blame is rarely all one way. I encourage my clients during difficult conversations to map the contribution system:). I am a typical Aries person who tends to be very fortnight in their opinions. But I always mean well.
Title: Re: How do we measure the energy of a photon?
Post by: jeffreyH on 26/05/2019 09:12:08
You, who really don't have a good grasp on physics, would like to promote yourself as knowledgeable enough to be able to take on paid work developing simulation software.

Isn't that seeking to obtain employment under false pretences? You can argue all you want that you do have the knowledge, but your posting history tells a different story.
Title: Re: How do we measure the energy of a photon?
Post by: mxplxxx on 26/05/2019 09:16:44
ou, who really don't have a good grasp on physics, would like to promote yourself as knowledgeable enough to be able to take on paid work developing simulation software.
You are once again bordering on libel. You have no evidence for these inflammatory and untrue statements. I retired 10 years ago and have little need for money.

I have had a fascination with physics since my schooldays. I have a library that is chock a block full of physics books. Physics is far from an exact science. The fact that so many people view my posts would seem to indicate that my opinions are valued. Why should people value your opinions over mine just because you think they should?
Title: Re: How do we measure the energy of a photon?
Post by: jeffreyH on 26/05/2019 09:18:14
BTW Take note: Advertising and self promotion are against the forum rules.
Title: Re: How do we measure the energy of a photon?
Post by: mxplxxx on 26/05/2019 09:31:09
BTW Take note: Advertising and self promotion are against the forum rules.
Yet you continue to promote yourself as an expert in your field. And Advertising, what advertising? Come on give up on your inflammatory and vindictive posts.
Title: Re: How do we measure the energy of a photon?
Post by: jeffreyH on 26/05/2019 09:42:40
And I quote.
Actually I have my own simulation framework that I hope to foist on the computer profession soon. It is called 3dAbstractions and is based on Hierarchical Finite State Machines theory. 3d as in D for Display, D for Data and D for Datastore.
End quote.

That states quite plainly the name of your product. Maybe that is why you signed up, to get data to use in testing. That is a generous interpretation.
Title: Re: How do we measure the energy of a photon?
Post by: jeffreyH on 26/05/2019 10:08:46
One more quote
I have had a fascination with physics since my schooldays. I have a library that is chock a block full of physics books. Physics is far from an exact science. The fact that so many people view my posts would seem to indicate that my opinions are valued. Why should people value your opinions over mine just because you think they should?
End quote.

Firstly, if you have read so extensively on physics then why do you ask the questions you do in the way you do?

Secondly, my opinions on science should be taken as just that. There are professionals here who are a fantastic learning resource. It is a unique learning opportunity for anyone. The site podcasts are another excellent source of learning. That assumes that members are here to learn. That is not always the case. Some have ulterior motives.

Lastly, controversy is no indication of value. A lot of people slow down to rubber neck an RTA. That doesn't make the accident valuable. It just makes it a lot sadder.
Title: Re: How do we measure the energy of a photon?
Post by: mxplxxx on 26/05/2019 10:13:59
And I quote.
Actually I have my own simulation framework that I hope to foist on the computer profession soon. It is called 3dAbstractions and is based on Hierarchical Finite State Machines theory. 3d as in D for Display, D for Data and D for Datastore.
End quote.

That states quite plainly the name of your product. Maybe that is why you signed up, to get data to use in testing. That is a generous interpretation.
From Google.

foist
verb
impose an unwelcome or unnecessary person or thing on.

I said "foist the product on the physics community". I said nothing about selling it (which would likely be impossible if it needed foisting to bring it to people's attention:)). It is personal software. It is a hobby. You are consistently reading additional meaning into my posts where it does not exist.
Title: Re: How do we measure the energy of a photon?
Post by: mxplxxx on 26/05/2019 10:24:24
Firstly, if you have read so extensively on physics then why do you ask the questions you do in the way you do
I only ask questions that I need help on. I have no idea what you mean when you say "in the way you do". I rarely ask questions anyway in New Theories. Anyway, time to move on; I have wasted much time defending myself and it is seemingly a never-ending process. I seem to have many disturbing opinions on physics and maybe this is a good thing for physics.
Title: Re: How do we measure the energy of a photon?
Post by: Bored chemist on 26/05/2019 10:36:15
This is incorrect and probably libelous.
Said Mr
Impossible not hard,
Both these facts make me highly suspicious that the simulation is a furphy.

While we are at it, how do you square this circle?
And I am at the top of my profession.
I retired 10 years ago ...
Title: Re: How do we measure the energy of a photon?
Post by: jeffreyH on 26/05/2019 10:38:12
Start quote.
I would love to see the code behind this. Or even a description of what the code does. Given we don't actually know what happened at the time of the Big Bang, I am skeptical that such a simulation could exist.
End quote.

Maybe you would like some tips on how to code a simulation. Best interpretation.

Maybe you would like to plagiarise their code.
Title: Re: How do we measure the energy of a photon?
Post by: jeffreyH on 26/05/2019 10:50:22
You said your software is a hobby. Well post it to github, post a link to the repository, and we can review it to see if it is as good as you say.
Title: Re: How do we measure the energy of a photon?
Post by: evan_au on 26/05/2019 12:04:54
Quote from: mplxxx
most simulations I am aware of are based on UML StateCharts which is a kind of software framework especially designed to implement software simulations
One of the languages I studied at uni was called SIMULA, a simulation language.
- SIMULA is a Discrete Event Simulation language which can model objects and events. It has similar ideas to the UML link that you posted
- And much of my workplace software experience has been in event-driven and message-passing Real-Time software which can be represented in RT-UML

But I think that you may have missed the object of your sentence
- My impression is that UML is intended to "simulate software". Software can be modelled as a discrete event system (and many obscure bugs happen when those discrete events are no longer atomic!)
- However, you are interpreting UML as "software to simulate physics", which is not the case
- A lot of physics is represented as differential equations which needs different tools than discrete event simulation
- Other types of physics uses matrix operations, which are not a natural data type in UML the way they are in FORTRAN (although overloading C++ operators could come close)

While there may be physics systems which could benefit from a UML-style tool, I think it would be a fairly narrow set.
I suggest that UML should be reserved for it's strength: simulating software systems.

See: https://en.wikipedia.org/wiki/Discrete-event_simulation
https://en.wikipedia.org/wiki/Continuous_simulation
Title: Re: How do we measure the energy of a photon?
Post by: mxplxxx on 26/05/2019 12:28:47
- A lot of physics is represented as differential equations which needs different tools than discrete event simulation
- Other types of physics uses matrix operations, which are not a natural data type in UML the way they are in FORTRAN (although overloading C++ operators could come close)
This is my whole point. Differential equations and matrix arithmetic were originally used to describe concepts in physical systems because computers were not available at the time. Why they have continued on being used now that good simulation software like UML exists is a bit of a mystery. A computer simulation deals naturally in concepts and murders a mathematical model for ease of understanding and has far more capabilities than available with mathematics alone.

Title: Re: How do we measure the energy of a photon?
Post by: jeffreyH on 26/05/2019 12:33:27
Not willing to post your code then. I wonder why that is. You were eager to have access to someone else's code though.
Title: Re: How do we measure the energy of a photon?
Post by: mxplxxx on 26/05/2019 12:34:01
- My impression is that UML is intended to "simulate software". Software can be modelled as a discrete event system (and many obscure bugs happen when those discrete events are no longer atomic!)
UML StateCharts can simulate anything that is object-oriented (see https://www.embedded.com/design/uml/4219602/UML-Statecharts )

See also http://scs.org/wp-content/uploads/2018/06/van-mierlo-_-edited.pdf
Title: Re: How do we measure the energy of a photon?
Post by: mxplxxx on 26/05/2019 12:49:34
Not willing to post your code then. I wonder why that is.
More jumping to conclusions. More aggression. More sarcasm. Who do you think you are? You don't "wonder why that is", you know deep down in your hard, unyielding heart, it is because it doesn't exist. Actually, I don't give even a tiny rats ass what you think (Friends 1994):).
Title: Re: How do we measure the energy of a photon?
Post by: jeffreyH on 26/05/2019 13:02:47
- My impression is that UML is intended to "simulate software". Software can be modelled as a discrete event system (and many obscure bugs happen when those discrete events are no longer atomic!)
UML StateCharts can simulate anything that is object-oriented (see https://www.embedded.com/design/uml/4219602/UML-Statecharts )

So it is for embedded control systems then. The example given was traffic lights.
Title: Re: How do we measure the energy of a photon?
Post by: jeffreyH on 26/05/2019 13:06:35
So you are saying your software doesn't exist then. Pedalling vapourware?
Title: Re: How do we measure the energy of a photon?
Post by: Bored chemist on 26/05/2019 13:08:12
you know deep down in your hard, unyielding heart, it is because it doesn't exist.
That's almost certainly libelous.

Here's a question for you,
Imagine I got a copy of UML and unpicked how it works (as if I was reverse engineering it).
Do you think that, somewhere in the depths, I'd fine a routine that was doing matrix algebra or solving differential equations?

I'm certain I would.

So, what you are saying is "I don't use matrices- because my computer does that for me".

Why they have continued on being used now that good simulation software like UML exists is a bit of a mystery.
Because (in all conceivable probability) UML uses them anyway.
Title: Re: How do we measure the energy of a photon?
Post by: jeffreyH on 26/05/2019 13:15:41
I think our friend needs to use software because he is unable to comprehend the concepts enough to program the maths by hand.
Title: Re: How do we measure the energy of a photon?
Post by: mxplxxx on 26/05/2019 13:19:04
- My impression is that UML is intended to "simulate software". Software can be modelled as a discrete event system (and many obscure bugs happen when those discrete events are no longer atomic!)
UML StateCharts can simulate anything that is object-oriented (see https://www.embedded.com/design/uml/4219602/UML-Statecharts )

So it is for embedded control systems then. The example given was traffic lights.
They are used for many types of systems including simulations.
Title: Re: How do we measure the energy of a photon?
Post by: Bored chemist on 26/05/2019 13:20:53
I think our friend needs to use software because he is unable to comprehend the concepts enough to program the maths by hand.
That's' not a problem.
There are plenty of people who drive cars but couldn't hope to design the engine for one.

The problem is when they claim that the designer is lying, and there is no engine.
Title: Re: How do we measure the energy of a photon?
Post by: Bored chemist on 26/05/2019 13:24:13
- My impression is that UML is intended to "simulate software". Software can be modelled as a discrete event system (and many obscure bugs happen when those discrete events are no longer atomic!)
UML StateCharts can simulate anything that is object-oriented (see https://www.embedded.com/design/uml/4219602/UML-Statecharts )

So it is for embedded control systems then. The example given was traffic lights.
They are used for many types of systems including simulations.
That's nice to know.
But not a great shock.
Most computer programing systems(be they languages or s/w packages) are Turing complete.
You can pretty much do anything with any of them.
Title: Re: How do we measure the energy of a photon?
Post by: jeffreyH on 26/05/2019 13:35:50
They might even be used to measure the energy of a photon someday.
Title: Re: How do we measure the energy of a photon?
Post by: evan_au on 26/05/2019 23:51:46
Quote from: mplxxx
most simulations I am aware of are based on UML StateCharts
When a man has only a hammer, all the world looks like a nail...
      ....and when he meets something which is definitely not a nail, it looks like an impossible task.
      ....like simulating the entire life of the universe, or the last few minutes before a supernova

Quote
Differential equations and matrix arithmetic were originally used to describe concepts in physical systems because computers were not available at the time
For an object-oriented view of the universe you could simulate every atom (or every star-sized mass), and it would have an "event" when that atom (or star-sized mass) bumps into another one
- But that requires a computer with as many memory words as the universe has atoms (or stars - still not practical)
- And ignores the fact that atoms interact via gravity continuously, not just when they bump into each other
- That is why continuous-time models (like differential equations) are still in use for simulating the motions of matter under gravity (like the universe) or in fluids (like the weather, an airplane or an exploding star)
- And even with the most powerful computers available, turbulence in these systems (and chaotic behavior in general) causes immense problems, since the behavior on the smallest scales impacts the behavior on the largest scales.

Quote from: bored chemist
Because (in all conceivable probability) UML uses (matrices) anyway.
Actually, the core data structure of discrete event simulation software is a scheduling queue.

This queue is often hidden from the programmer, but every time a future event is triggered (perhaps after a random delay), a future event is posted on the queue. The simulation environment takes the closest approaching event off the queue to process events in sequence.
Title: Re: How do we measure the energy of a photon?
Post by: mxplxxx on 27/05/2019 14:37:02
But that requires a computer with as many memory words as the universe has atoms (or stars - still not practical)
Like I have mentioned before, this fact implies that the universe IS a simulation of something much bigger and much more complicated!

Simulation is a computer are rarely done on base data. A computer is real good at processing abstractions. The less abstract the abstraction the  closer the simulation is to reality.

The only way to simulate the universe is via abstractions. It just so happens this is possible because the universe is a abstraction hierarchy. I have previous described the universal system object which can be used to simulate universal system such as a solar system (or an Atom!). An abstraction connects to black boxes. It processes events from its parent and its children but needs no knowledge of how the parent or children work. To simulate the universe you only need to know the events its galactic clusters are sending the universe system object and what events to send the the galactic clusters in response to changes in the universe system object .Easy peesy lemon squeezy.

A solar system object will consist of a central sun object and peripheral planetary systems. The central sun object has state variables relating to the overall state of the object, a parent system that is the solar system object and child shell systems that correspond to the sun's core, photosphere etc.. The solar system object's parent will be a galaxy system. The galaxy system is more abstract than the solar system (or the solar system is more concrete than the galaxy system).

Each of these systems/objects  can be dealt with totally abstractly or can contain more concrete child objects all the way down to quarks or up to the level of the universe itself. The more this system of objects is abstract, the less accurate the final simulation and vice versa for concreteness. But the point of this is that simulations of part or all of the universe are possible with current computers. My 3dAbstractions software can deal with these sorts of simulation with ease. Hopeful it will be on open source soon.

This obviously is an extremely precied version of the technology required to run simulations on a computer and I would be very surprised if it gave you more than an inkling of the power of abstractions. 

BTW, quarks seem to be the equivalent of computer digits (0/1). They may be the only part of reality that can have state.
Title: Re: How do we measure the energy of a photon?
Post by: mxplxxx on 27/05/2019 14:41:24
- And ignores the fact that atoms interact via gravity continuously, not just when they bump into each other
I am investigating gravity as a revolving sequence of photons between the central object of a system and its peripheral (child) systems. i.e. how everything is PUSHED towards the centre of gravity of the system. A photon reaching the central object has nowhere to go but back where it came from. And therefore? ... well that is what I am considering at present. Can a photon change the position of an object without breaking the conservation of energy law?

Just recently? in physics, Newtonian gravity suddenly seems to be affecting the whole universe at the same time. Makes sense given the chaos that would like result form gravity of millions of years ago affecting the present. But seems to defy current physics re the graviton. But there is a lot of this sort of thing happening in physics today:) 

On the other hand it is very likely that the universe is a continuous entity and we exist in a simulation of part of it. That being the case, changes in the universe because of gravity can be reflected simultaneously over the total universe. BTW if the universe is a continuous entity then abstractions like a galaxy, solar system etc. probably don't actually exist. A quark exists with a single motion. All of the structures in the universe will be the result of quarks motions in relationship to other quarks. Is gravity involved in this? Possibly but not necessarily if Einstein's theory that matter warps spacetime is correct.

Theoretically, it may be that we can simulate the universe using only the motion of quarks and how aggregations of quarks warp spacetime.

Title: Re: How do we measure the energy of a photon?
Post by: mxplxxx on 27/05/2019 15:07:55
Can't give you examples of my code. But here is a picture of my personal (not for sale) gambling software as used today.

MOD EDIT: Link removed. Please do not post any sort of advertising.
Title: Re: How do we measure the energy of a photon?
Post by: Bored chemist on 27/05/2019 15:22:12
Can't give you examples of my code.
Why not?
Title: Re: How do we measure the energy of a photon?
Post by: evan_au on 27/05/2019 22:30:57
Quote from: mxplxxx
this fact (the complexity of the universe) implies that the universe IS a simulation of something much bigger and much more complicated!
No, they were just trying to simulate our universe, and it still takes a lot of time on the largest supercomputers.
They were not trying to simulate anything bigger.

Quote
The only way to simulate the universe is via abstractions.
I agree. Usual ways of abstracting in astronomy are:
- Model a small part of the universe (but still big enough to be "typical")
- Don't model individual atoms, but model galaxy-mass "blobs"
- Model the Dark Matter separately from the visible matter (with both interacting via gravity)
- Don't model individual stars, but use an "average" behavior of a galaxy

And the conclusion was that statistically, it is similar to the universe we see.

An abstraction where you don't model the individual cards in a deck, but just whole decks would fail in a game of chance!

Quote
I would be very surprised if it gave you more than an inkling of the power of abstractions.
I am aware of the usefulness of abstraction for software developers.

But when it comes to turbulent events (like a supernova), where tiny details are tightly coupled to the high-level outcome, abstraction fails. For these tightly-coupled problems, you need a numerical solution - with today's technology, the optimum answer seems to be a continuous-time simulation in FORTRAN, running on a large supercomputer.

Once the astronomical community understands what fraction of stars go supernova at certain stages vs those that turn into black holes (and publish it in a peer-reviewed journal), then other astronomers and cosmologists can use that encapsulated knowledge in predicting the life course of a whole galaxy. This is a level where there is loose coupling between stars and their galaxy and abstraction works.

Quote
The central sun object has state variables relating to the overall state of the object
And if you want to simulate the state of this sun object, I suggest that you read up on continuous-time numerical methods.
See, for example: https://en.wikipedia.org/wiki/Runge%E2%80%93Kutta_methods
Title: Re: How do we measure the energy of a photon?
Post by: mxplxxx on 28/05/2019 01:43:41
An abstraction where you don't model the individual cards in a deck, but just whole decks would fail in a game of chance!
So, so True. My gambling (racing) software is based on historical averages of odds/dividends over 4 meetings, which is approximately the same on any particular day. Unfortunately, in an individual betting pool (usually a set of 4 meetings) spikes often occur. There will be a run of favourites losing (or winning which is almost as bad) which breaks the bank. To date, no amount of smoothing can handle this situation.
Title: Re: How do we measure the energy of a photon?
Post by: mxplxxx on 28/05/2019 01:51:14
I agree. Usual ways of abstracting in astronomy are:
- Model a small part of the universe (but still big enough to be "typical")
- Don't model individual atoms, but model galaxy-mass "blobs"
- Model the Dark Matter separately from the visible matter (with both interacting via gravity)
- Don't model individual stars, but use an "average" behavior of a galaxy

And the conclusion was that statistically, it is similar to the universe we see.
Not surprised. Software that simulates the universe will be helped by the fact that all systems in the universe seem to derive (inherit in computer science) from the same underlying system object as I have explained in previous posts. Also the knowledge that all systems in the universe are scaled-down versions of their parent can be used in the simulation. Additionally, parent systems are abstractions of their children. A galaxy system is an abstraction of the solar systems that comprise it.
Title: Re: How do we measure the energy of a photon?
Post by: mxplxxx on 28/05/2019 01:58:40
But when it comes to turbulent events (like a supernova), where tiny details are tightly coupled to the high-level outcome, abstraction fails. For these tightly-coupled problems, you need a numerical solution - with today's technology, the optimum answer seems to be a continuous-time simulation in FORTRAN, running on a large supercomputer.
Related to Chaos theory? Actually I am not sure a supercomputer is required for heavy computational problems other than the fact that they are quick. You can simulate any system an event at a time as happens in computers without "core" processors. It is quite possible the universe is running an event at a time but doing it extremely quickly.
Title: Re: How do we measure the energy of a photon?
Post by: Bored chemist on 28/05/2019 08:35:32
Also the knowledge that all objects in the universe are scaled-down versions of their parent can be used
Of what is an electron the "scaled down" version?
(Or, if you prefer, what do you get when you scale up an electron?)
If there's no clear answer to that, then your central idea is wrong.

To date, no amount of smoothing can handle this situation.
And yet, the bookkeepers all drive expensive cars.
Actually I am not sure a supercomputer is required for heavy computational problems other than the fact that they are quick.
Well, duh!
Yes the advantage of fast  computers is that they are fast.
That's very important if you want to get an answer before the grant runs out..
So, in practical terms, yes, they are required.
Title: Re: How do we measure the energy of a photon?
Post by: evan_au on 28/05/2019 11:24:13
Quote from: mxplxxx
Actually I am not sure a supercomputer is required for heavy computational problems other than the fact that they are quick.
For tasks like weather forecasting, there is actually a very specific speed requirement: To be useful, you have to produce a 1 day weather forecast in less than 24 hours.
- To be really useful, you need to produce around ten 7-day forecasts in under 24 hours, with different starting points.  This then allows you to provide a probabilistic weather forecast, eg "60% chance of rain"
- This requires the latest, most powerful supercomputers.

Another task that requires supercomputer power from perhaps 5 years ago is for autonomous vehicles:
- These have a strict real-time requirement - you must process images, extract the details, correlate different sensors and react - before you hit a pedestrian, a car or a pole.
- It has additional constraints on physical size, power consumption, operation in temperature extremes and limited cooling.
Title: Re: How do we measure the energy of a photon?
Post by: Bored chemist on 28/05/2019 11:30:11
Actually I am not sure a supercomputer is required for heavy computational problems other than the fact that they are quick.
Come to think of it, isn't that a bit like saying "I don't think food is required for eating- except that it needs to be edible"?
Title: Re: How do we measure the energy of a photon?
Post by: mxplxxx on 28/05/2019 12:16:19
Quote from: mxplxxx
Actually I am not sure a supercomputer is required for heavy computational problems other than the fact that they are quick.
For tasks like weather forecasting, there is actually a very specific speed requirement: To be useful, you have to produce a 1 day weather forecast in less than 24 hours.
- To be really useful, you need to produce around ten 7-day forecasts in under 24 hours, with different starting points.  This then allows you to provide a probabilistic weather forecast, eg "60% chance of rain"
- This requires the latest, most powerful supercomputers.

Another task that requires supercomputer power from perhaps 5 years ago is for autonomous vehicles:
- These have a strict real-time requirement - you must process images, extract the details, correlate different sensors and react - before you hit a pedestrian, a car or a pole.
- It has additional constraints on physical size, power consumption, operation in temperature extremes and limited cooling.
Yep!! Real-time system processing requires a computer quick enough to process whatever load is put on the system in question when running under time constraints.
Title: Re: How do we measure the energy of a photon?
Post by: mxplxxx on 29/05/2019 01:44:22
Yep!! Real-time system processing requires a computer quick enough to process whatever load is put on the system in question when running under time constraints.
But I am not sure a simulation of a super nova is a real-time system. In this case the extra processing power of a super computer may just be to speed up a computationally intensive process.

The ideal for real-time processing would be a processor per object, each object processes one event at a time, and objects to be totally independent of each other. This seems to be what reality does (the quantum nature of reality ensures that one event at a time is processed).

Real-time systems need to handle the situation when not enough time is available. Our brain is  a real-time system that has to do this. Putting you hand on a hot item on the stove gives you an immediate withdrawal reaction from your hand because your brain is not quick enough to process the event.
Title: Re: How do we measure the energy of a photon?
Post by: Bored chemist on 29/05/2019 19:21:59
In this case the extra processing power of a super computer may just be to speed up a computationally intensive process.
Come to think of it, isn't that a bit like saying "I don't think food is required for eating- except that it needs to be edible"?
Title: Re: How do we measure the energy of a photon?
Post by: mxplxxx on 31/05/2019 02:55:25
To date, no amount of smoothing can handle this situation.
And yet, the bookkeepers all drive expensive cars.
They set the odds. And they often lose in the short term but win over the long term - much like a casino  The tote on the other had takes a 25% cut of outlays before paying a dividend. THEY can't lose.
Title: Re: How do we measure the energy of a photon?
Post by: mxplxxx on 31/05/2019 03:12:48
Of what is an electron the "scaled down" version?
(Or, if you prefer, what do you get when you scale up an electron?)
If there's no clear answer to that, then your central idea is wrong.
Or I have made a mistake.

When I am talking about scaling up/down, I am referring to systems. All central objects occur as part of a system. A sun is the central object of a solar system. An electron is the central object of an electron system. The electron system is the wave and the electron is the particle. The purpose of a system is to enable the central object to move and also (probably) to provide the necessary structure for the central object to interact with the central object of another system. An electron system scales up to an atom system (which is a central ion with electron subsystems) or (same deal) an Atom System scales down to an electron system.  Being an elementary particle, an Electron system has no subsystems (unless a. it contains particles that are too small to be detected (as per the photon systems E and M particles) or b. the electron subsystems are electron neutrino systems).

In the case of an Atom system, the central ion object also contains proton and neutron subsystems.

It is likely the electron system is one of the bottom systems (along with other elementary particles) in the universe system hierarchy.

Central objects in my software contain the internal state of the system they belong to. The external state of the system is given via the sum of the states of its subsystems. The central object can also contain an abstraction hierarchy where the state of the object is contained in a top-level "core" object and other objects radiate out from the core becoming more and more concrete until they reach the surface of the hierarchy where they are as concrete as they can be. It is possible this abstract-to-concrete transformation is present in the universe and is driven by gravity.  Think of the Earth with a very dense simple core and a very light but complex atmosphere.

ps there are Shells involved in all this, but for clarity I have omitted them.
 
Title: Re: How do we measure the energy of a photon?
Post by: Bored chemist on 31/05/2019 07:17:14
Or your understanding of my theory is incorrect.
I can only base my understanding  on what you said which was.
all objects in the universe are scaled-down versions of their parent
An electron is an object.
What parent is it a scaled down version of?


Title: Re: How do we measure the energy of a photon?
Post by: mxplxxx on 31/05/2019 08:47:42
An electron is an object.
What parent is it a scaled down version of?

It is a mistake. It should read "All systems in the universe are scaled down versions of their parents".
Title: Re: How do we measure the energy of a photon?
Post by: mxplxxx on 12/06/2019 06:09:44
ps there are Shells involved in all this, but for clarity I have omitted them
A "shell" is a sphere with the "core" object of a system at its centre that encloses a subsystem of a system. For example the solar system has an Earth shell that encloses the Earth system. Its centre is the core of the sun.

Actually, it is two adjoining shells that enclose a subsystem. For example, in the solar system, the Earth Shell in conjunction with the Venus shell encloses the Earth subsystem. This may have the effect of stopping or limiting the number of positions a system can occupy in a quantum universe.

Shells can have subshells where the systems involved are small enough to fit in the subshells.

The outermost shell of of a system encloses the system itself.

In my software, a shell is a category. For example a collection is a category and all objects in a collection are enclosed by the collection category. A similar situation seems to exist for shells.

Of course, a central object in a system also has shells that are subShells of the central object shell. And these subShells will also have subShells to an unknown (?) depth. So the Earth central object is surrounded by a Shell. The first subShell in this surrounding shell is the atmosphere. The atmosphere is composed of four layers (or subshells), troposphere, stratosphere, mesosphere and thermosphere and so on.

It is very likely an atomic Ion (the central object of a  Atom system) has a similar makeup. In this case, each shell in the ion would likely correspond to an electron shell. The first ion shell would correspond to the outermost electron shell, the second to the outermost minus 1 electron shell and so on. Each ion shell would likely contain one of more proton systems, equal to the number of electrons in the corresponding electron shell. No idea where neutrons appear in this scheme of things but surmise they will be there to balance out mass in some way.

This makeup of atomic ion shells is likely to extend to all systems in the universe. The solar system central object (the sun) is likely to have shells that correspond to the planets. The fact that these shells  are not visible (or are they?) may be due to the intense gravity of these objects which has pulled apart previously existing systems.

In this scheme of things, a photon ion (i.e a photon system central object) would have a single shell (maybe two) containing two particles corresponding to the M and E particles discussed previously but with opposite charges.

It would seem that the central objects of systems can be regarded as positive and systems themselves as negative. Given positive attracts negative, could this be the mechanism for gravity?
Title: Re: How do we measure the energy of a photon?
Post by: Bored chemist on 12/06/2019 07:23:58
As far as I can tell, all you have said is that any thing has some sort of border between "thing" and "not thing".
That's true, but uselessly obvious.
Title: Re: How do we measure the energy of a photon?
Post by: mxplxxx on 26/06/2019 00:10:02
It would seem likely that the energy of a single photon can be stored and retrieved. https://scienceblog.com/508559/new-approach-aims-to-advance-quantum-computers/?utm_source=feedburner&utm_medium=email&utm_campaign=Feed%3A+scienceblogrssfeed+%28ScienceBlog.com%29

- sounds a bit like a photon system (https://www.thenakedscientists.com/forum/index.php?topic=75960.msg575784;topicseen#msg575784) where the subsystems are  also photon systems (i.e. confined photons).
Title: Re: How do we measure the energy of a photon?
Post by: mxplxxx on 27/06/2019 04:47:03
Events in computer science are usually tied to some action. Physics, it would seem, treats photons mainly as random events. What if this were not the case? It makes a lot of sense given that the universe is so ordered. Using the "System" described previously in this post it could be theorized that:

All systems have  parents. Parents react to child photons/events. Child systems react in specific ways to events sent to them by a parent. Reactions, possibly to photons of specific energies, are specifically programmed.  This is essentially how a hierarchical state machine works. Photon reactions going up the hierarchy are "pull" ("react to") events whilst photon reactions going down the hierarchy are "push" ("do something") events.

A photon only exists in the system that it was emitted (created) in. A solar system photon cannot be found in a galaxy system for example. 

Other types of photon/event structures are possible that handle sibling-to-sibling system reactions.

It seems to me that trying to study photons (in fact, any particle!) as stand-alone objects is fraught with difficulty (as quantum physics seems to be telling us).

BTW, in computer science the object type is the basis for all other types in a system Why not in physics?
Title: Re: How do we measure the energy of a photon?
Post by: mxplxxx on 02/07/2019 04:23:11
In software development, rather than refer to objects directly they are often referred to via a reference. This is the case for Microsoft languages C# and Visual Basic.NET. What this achieves is being able to update the same object instantaneously in multiple places in the program.

It seem to me that a similar scheme in reality would make a lot of sense. Particularly so in the "spooky action at a distance" phenomenon of entanglement where changing one entangled particle results in the changes being replicated instantaneously in another entangled object. Something to ponder.

A proton, for example, may be a reference object to an electron system.

It is also interesting to speculate that, because I am aware and have a mind, that the universe has this capability also. If this capability is used (likely), then the universe can imagine galactic clusters and reference them from any point in its mind. We may be just systems referenced in a universal mind. Maybe this is why galactic clusters seen from earth resemble a brain.
Title: Re: How do we measure the energy of a photon?
Post by: Bored chemist on 02/07/2019 19:28:21
You seem to think that all this is a simplification.
You can talk of "an electron system".
Why not just talk about "an electron"?
Title: Re: How do we measure the energy of a photon?
Post by: mxplxxx on 03/07/2019 02:07:56
You seem to think that all this is a simplification.
A computer is basically combinations of 0/1 digits. These combinations can be used to simulate a universe (which makes it very possible that a universe is simpler than we currently understand) Complexity comes in when language is used to manipulate 0/1 combinations (to make the combinations more understandable).
Title: Re: How do we measure the energy of a photon?
Post by: mxplxxx on 03/07/2019 02:11:40
You can talk of "an electron system".
Why not just talk about "an electron"?
An electron system is basically the wave part of an electron wave/particle duality. The electron itself is the central object of this system and is only found when "observed" (i.e when the system interacts with the "observer").

Actually, things are a bit more complicated than this. The central object of a system is a type of system in its own right and is basically a hierarchical finite state machine (HFSM) for the object. e.g. the central object of a solar system is the sun which is composed of a single top-level state (possibly a black hole) and vast numbers of sub-states. This is awfully difficult to describe unless you have a good knowledge of HFSM's. I may attempt to do so in a later post.

One way to differentiate between a system and a central object system is that the former is an open system and the latter is a closed system.

So it is very possible that an electron comes with software that controls its state and how it reacts with other objects. This is how it would be programmed in a simulation of the universe.
Title: Re: How do we measure the energy of a photon?
Post by: Bored chemist on 03/07/2019 18:47:07
So it is very possible that an electron comes with software that controls its state and how it reacts with other objects. This is how it would be programmed in a simulation of the universe.
"so it is very possible that an electron comes with software that controls its state..."
It's not impossible, but there's no reason to believe it.
It's like saying electrons are always accompanied by imps who tell the electron how to behave.
Also "very possible"  is poor grammar. Things are possible or they are not. It's like being very pregnant.
https://www.quickanddirtytips.com/education/grammar/modifying-absolutes

"This is how it would be programmed in a simulation of the universe."
Well, that might be how you would program it. There are clearly alternatives.
Most simulations of electrons that I have seen, draw little coloured balls.

But that's not really important to science because we are interested in how real electrons work, rather than in how simulated ones behave.
Title: Re: How do we measure the energy of a photon?
Post by: mxplxxx on 04/07/2019 00:12:51
but there's no reason to believe it
Nor is than any reason to disbelieve it. The mechanism for photon interactions with any other object in physics is not known as far as I am aware.

I would have thought that the fact that a software mechanism for photon interactions is available and no other mechanism has yet been discovered would have given credence to the software mechanism theory. Software is, at its most basic, just an arrangement of electricity flowing through logic gates.
Title: Re: How do we measure the energy of a photon?
Post by: mxplxxx on 04/07/2019 00:16:30
Well, that might be how you would program it.
True. As to simulation of the universe alternatives, I have yet to see one that elucidates me as to the underlying programming. Lets face it, we really don't know how the universe works and if we don't how can we simulate it?
Title: Re: How do we measure the energy of a photon?
Post by: mxplxxx on 04/07/2019 00:34:03
Also "very possible"  is poor grammar.
But "very likely" is not? No mention of "very possible" in your reference. A hellish oversight.
Title: Re: How do we measure the energy of a photon?
Post by: Bored chemist on 04/07/2019 19:31:10
Nor is than any reason to disbelieve it.
Actually, there is.
https://en.wikipedia.org/wiki/Occam%27s_razor

It is far simpler to have a universe than a Universe and the thing that simulates the universe.
(no matter how weird the universe may be).
Title: Re: How do we measure the energy of a photon?
Post by: Bored chemist on 04/07/2019 19:33:21
Software is, at its most basic, just an arrangement of electricity flowing through logic gates.
And the forces that make the electrons move are carried by virtual photons.
So you use a staggeringly huge number of photons to simulate each photon.
How does that help?
You can only build the computer because you already know what photons do.
Title: Re: How do we measure the energy of a photon?
Post by: mxplxxx on 05/07/2019 04:21:44
Software is, at its most basic, just an arrangement of electricity flowing through logic gates.
And the forces that make the electrons move are carried by virtual photons.
So you use a staggeringly huge number of photons to simulate each photon.
How does that help?
You can only build the computer because you already know what photons do.

I am not about simulating the universe.  I am postulating ways that the universe might work. In any case, a simulation is normally an abstraction (model) of the way something works. Given that the universe is an an abstraction hierarchy, it is very likely it is a simulation of something much larger and more complicated (maybe it is a custom-designed as a teaching tool:).). Sooner or later we will design and build our own universe that we can pop into for a fun adventure or just to learn stuff.

Although we refer to the Universe as a single thing, it is a actually a system we are talking about. The Universe system, like all universe-based systems will very likely contain a central system whose centre, in turn, will very likely be a black hole where everything that currently exists will end up.  This central system is THE Universe.

As for Occam's razor, it has to do with energy. The universe has evolved (or is designed) to be as energy-efficient as possible in its evolution. This favors the natural selection of things that work simply because they tend to use the least energy. This does not mean that everything is simple. Human beings are obviously hugely complex entities but maybe we are as simple as we can be:).

My concept of the universe being built on a base/inherited system type is a pretty simple one I would have thought. Designing things to work as simply as they can is a characteristic of a good software developer. It is one of my fortes.
Title: Re: How do we measure the energy of a photon?
Post by: Bored chemist on 05/07/2019 19:01:40
Given that the universe is an an abstraction hierarchy,
That's not "given"
Title: Re: How do we measure the energy of a photon?
Post by: evan_au on 06/07/2019 01:55:50
Quote from: mplxxx
A photon only exists in the system that it was emitted (created) in. A solar system photon cannot be found in a galaxy system for example.
About 7 hours after a photon is emitted from the surface of the Sun, it has passed the farthest point on Pluto's orbit, and is well out of the Solar System.

About a year after a photon is emitted from the surface of the Sun, it has passed the Kuiper Belt and the Oort cloud, and is in interstellar space. It is effectively part of the galaxy, and no longer part of the Sun.

About 100,000 years after a photon is emitted from the surface of the Sun, it has passed the outer edges of the Galaxy, and is in intergalactic space.

So I think you need to expand your hierarchy to include the fact that the Sun is part of the Milky Way galaxy, which is in turn part of the Universe.

Quote
This favors the natural selection of things that work simply because they tend to use the least energy.
"Natural selection" seems to imply living things.
But living things use far more energy than random collections of the same atoms.

Quote
A computer is basically combinations of 0/1 digits. These combinations can be used to simulate a universe
Unfortunately, the real universe has more states than just 0/1. Real particles are often in states that are best described with complex numbers, which can represent states between 0 & 1.

It is possible to simulate groups of particles with these entangled states, but it takes a powerful computer. A Google researcher described how, as they increased the number of entangled states, they moved from simulating their system on a laptop computer, to a desktop computer, to a server, and a warehouse of servers. This was over a period of months.

It is argued that a system with more than 50 entangled states cannot be simulated on any conceivable binary computer, even if it was as large as the universe.

So I think that to simulate the universe, you will need a quantum computer.
Title: Re: How do we measure the energy of a photon?
Post by: mxplxxx on 06/07/2019 04:04:47
Real particles are often in states that are best described with complex numbers, which can represent states between 0 & 1.
Complex numbers can be represented in a binary number system; see https://www.springerprofessional.de/en/complex-binary-number-system/4635020. As far as I am aware, a binary number system can represent all the states in the universe. This, as I have previously mentioned, points strongly to the existence of a particle that has only two states (preon theory states this; see the Rishon Model https://en.wikipedia.org/wiki/Rishon_model).
Title: Re: How do we measure the energy of a photon?
Post by: mxplxxx on 06/07/2019 05:46:31
So I think you need to expand your hierarchy to include the fact that the Sun is part of the Milky Way galaxy, which is in turn part of the Universe.
This is new theories:). What I am postulating is that photons behave much like events in a software program. They travel up and down or sideways across an abstraction hierarchy. Those travelling up the hierarchy inform upper level systems of a change of state of a child object. Those travelling down the hierarchy request child systems to take some action (very much like a neural network). A system that processes a photon does so by taking some action based on the photon's state or by forwarding the photon to another system. This is how a photon can travel 30 billion light years across the universe without interacting with another particle (an extremely likely event IMHO especially as the photon will likely be zigzagging all over the place due to quantum probability) and being destroyed in the process. In fact it is likely destroyed at some stage but its clone carries on travelling in the same direction with the same wavelength.

The systems I am postulating also have boundaries that a photon cannot traverse without being cloned. Evidence for these boundaries can be found here:

https://www.google.com/search?q=edge+of+solar+system&rlz=1C1CHBD_en-GBAU826AU826&tbm=isch&source=iu&ictx=1&fir=zjzI-rUCpCkW7M%253A%252Cy4wpYUqBmNQxuM%252C_&vet=1&usg=AI4_-kSbnNOkJkK9rdQuXHp0FWQuvvBAaQ&sa=X&ved=2ahUKEwiT8IyKup_jAhWTjeYKHVR8DQ0Q9QEwAHoECAIQAw#imgrc=zjzI-rUCpCkW7M:
Title: Re: How do we measure the energy of a photon?
Post by: mxplxxx on 06/07/2019 06:00:46
So I think that to simulate the universe, you will need a quantum computer.
Pity noone can explain the workings of a quantum computer to me. Definitely a furphy candidate:).
Title: Re: How do we measure the energy of a photon?
Post by: mxplxxx on 06/07/2019 06:11:27
"Natural selection" seems to imply living things.
I see all things, not just living things as being candidates for natural selection. I would think a lot of the universe exists because it is better suited to its purpose than past systems that were less well suited.
Title: Re: How do we measure the energy of a photon?
Post by: mxplxxx on 06/07/2019 06:14:49
It is argued that a system with more than 50 entangled states cannot be simulated on any conceivable binary computer, even if it was as large as the universe.
Which is why maths is probably not a great way to describe entanglement. A computer system should be able to handle entanglement via generalizations and abstractions and a finite state machine.  See the Dynamic model of the Object Modeling Technique (https://en.wikipedia.org/wiki/Object-modeling_technique). This technique shows how state explosions can be managed (it requires a huge amount of expertise unfortunately - such is the price of hard-to-find knowledge)
Title: Re: How do we measure the energy of a photon?
Post by: mxplxxx on 06/07/2019 10:47:42
That's not "given"
How do you know?
Title: Re: How do we measure the energy of a photon?
Post by: Bored chemist on 06/07/2019 11:46:28
That's not "given"
How do you know?
You have things the wrong way round.
You should justify strange assertions like "Given that the universe is an an abstraction hierarchy,"

A solar system photon cannot be found in a galaxy system for example. 
That is at odds with the usual definition of the word "in".
Because the solar system is inside a galaxy, anything in the solar system is in a galaxy.


The universe has evolved (or is designed) to be as energy-efficient as possible in its evolution.
That's not a helpful use of the word "evolved"
Also the definition of "efficiency" doesn't apply to the universe.
Title: Re: How do we measure the energy of a photon?
Post by: Bored chemist on 06/07/2019 11:47:49
I would think a lot of the universe exists because it is better suited to its purpose than past systems that were less well suited.
Who made the choice about what is "better"?
Title: Re: How do we measure the energy of a photon?
Post by: mxplxxx on 06/07/2019 12:01:21
You should justify strange assertions like "Given that the universe is an an abstraction hierarchy,"
Have been doing so for quite a while. Have you not been paying attention?:). You, on the other hand, do not back up your statement "that's not a given". Please do so.
Title: Re: How do we measure the energy of a photon?
Post by: mxplxxx on 06/07/2019 12:03:40
Because the solar system is inside a galaxy, anything in the solar system is in a galaxy.
Better said as "a solar system photon cannot be found outside the solar system".
Title: Re: How do we measure the energy of a photon?
Post by: mxplxxx on 06/07/2019 12:04:51
That's not a helpful use of the word "evolved"
Also the definition of "efficiency" doesn't apply to the universe.
IYHO. Maybe you would like to expound?
Title: Re: How do we measure the energy of a photon?
Post by: mxplxxx on 06/07/2019 12:06:31
I would think a lot of the universe exists because it is better suited to its purpose than past systems that were less well suited.
Who made the choice about what is "better"?
Who?? Natural selection did!
Title: Re: How do we measure the energy of a photon?
Post by: Bored chemist on 06/07/2019 13:13:42
I would think a lot of the universe exists because it is better suited to its purpose than past systems that were less well suited.
Who made the choice about what is "better"?
Who?? Natural selection did!
That's just silly.
How did natural selection do away with the universe where things were different?
Title: Re: How do we measure the energy of a photon?
Post by: Bored chemist on 06/07/2019 13:15:44
That's not a helpful use of the word "evolved"
Also the definition of "efficiency" doesn't apply to the universe.
IYHO. Maybe you would like to expound?
For evolution to happen the "failed" versions of the thing have to die out and fail to produce progeny.
That just doesn't happen with the universe.

As has been pointed out
"Natural selection" seems to imply living things.
Title: Re: How do we measure the energy of a photon?
Post by: mxplxxx on 07/07/2019 00:21:14
How did natural selection do away with the universe where things were different?
As natural selection always does. Better solutions replace ones that don't work as well.

Very simply, at some stage in the past, a molecule became capable of reproducing itself. It did so successfully until, by accident, one of its progeny became capable of doing so better, whereupon the progeny molecule started to out-reproduce its parents. A few billions years and billions of accidents later, here are we.

The universe seems to have evolved (or been designed) so that these "accidents" are part of its capabilities (very likely via quantum probability).
Title: Re: How do we measure the energy of a photon?
Post by: Bored chemist on 07/07/2019 09:16:15
Better solutions replace ones that don't work as well.
But the universe is still here.
It wasn't replaced.


Very simply, at some stage in the past, a molecule became capable of reproducing itself. It did so successfully until, by accident, one of its progeny became capable of doing so better, whereupon the progeny molecule started to out-reproduce its parents. A few billions years and billions of accidents later, here are we.
Yes, we all know about that but, as you say, it depends on having progeny.

The universe doesn't have children.
It does not reproduce.
There was one universe in the beginning and there will be one universe in the end.

So the criteria for evolution can not possibly be met by the Universe.
Title: Re: How do we measure the energy of a photon?
Post by: jeffreyH on 07/07/2019 19:52:16
@mxplxxx You can't stand to be proved wrong, can you? If you wish to boost your own ego there are better places to post, I'm sure.

To argue support for more and more outlandish ideas to justify your views won't give you the adoration that you appear to crave.

You could, if you tried hard enough, swallow your misplaced pride and become a positive presence. Take the risk. It is worth it.
Title: Re: How do we measure the energy of a photon?
Post by: mxplxxx on 07/07/2019 23:30:20
You can't stand to be proved wrong, can you? If you wish to boost your own ego there are better places to post, I'm sure.
Can anybody stand to be proved wrong? I post here because, thankfully, your colleagues allow me to. It is fun and I think what I have to say is entertaining at the least, and possibly useful at best. I think it is good for physics to entertain different ideas, outlandish as they may seem. Dare I say that  Einstein's ideas were/are outlandish?:).

ps I would think boosting ones ego is a good thing if it is done to a well-functioning ego (mine is:)).
Title: Re: How do we measure the energy of a photon?
Post by: mxplxxx on 07/07/2019 23:46:29
The universe doesn't have children.
You know this because?

A computer program contains classes from which are derived objects. Multiple objects are derived from single classes. It seems to me that the same thing applies in physics. For example, all photons are the same except for their current states. Where is the photon class in physics? We don't know but very possibly it is part of a sub-structure in the universe. Plato proposed such a sub-structure I think in his theory of forms (https://en.wikipedia.org/wiki/Theory_of_forms). Or maybe classes exist as dark matter?

Who is to say that there is not a Universe class? I think it is more likely than not, in which case it could be subject to natural selection along with all other cosmological classes.


Title: Re: How do we measure the energy of a photon?
Post by: Bored chemist on 08/07/2019 07:25:22
You know this because?
Because the definition of the "Universe" is "everything".
Any children of the universe would be part of the universe.
Title: Re: How do we measure the energy of a photon?
Post by: Bored chemist on 08/07/2019 07:26:21
I think what I have to say is entertaining at the least, and possibly useful at best.
You missed it being annoying and misleading.
Title: Re: How do we measure the energy of a photon?
Post by: mxplxxx on 08/07/2019 09:20:29
Because the definition of the "Universe" is "everything".
Unus ,duo, tribus; Latin for one, two three. You have a winner:). Cool (or frigus in Latin). I will in future refer to the Universe as the Multiverse. But wait, doesn't the multiverse a contain more than one universe? Maybe our universe will be a victim of natural selection in the multiverse and we will cease to exist.
Title: Re: How do we measure the energy of a photon?
Post by: mxplxxx on 08/07/2019 09:43:36
You missed it being annoying and misleading.
No I didn't. This is your interpretation and you being typically anti-social. I suspect the bulk of the numerous viewers of the topic think differently. Who keeps viewing something that is annoying them?
Title: Re: How do we measure the energy of a photon?
Post by: Bored chemist on 08/07/2019 18:17:43
Who keeps viewing something that is annoying them?
People who are so annoyed by your errors that they want to correct them.
You have only been thanked twice, and neither of those was in this thread.
Title: Re: How do we measure the energy of a photon?
Post by: mxplxxx on 09/07/2019 00:03:38
People who are so annoyed by your errors that they want to correct them.
Or people who have a pathological need to bring down others.
Title: Re: How do we measure the energy of a photon?
Post by: Bored chemist on 09/07/2019 07:21:43
People who are so annoyed by your errors that they want to correct them.
Or people who have a pathological need to bring down others.
This is a science forum.
People can only bring you down when you post stuff that's wrong.
Title: Re: How do we measure the energy of a photon?
Post by: mxplxxx on 15/07/2019 11:37:10
The base System object in my 3dAbstractions software is inherited by all higher-level system objects. For example the Race.System object inherits the System object. All of the states  and structures and functions of the System object are available to objects that inherit it. The System object is also declared "Must Inherit" meaning it cannot exist on its own.

It is quite possible that reality also works in this way. In other words, the System object is essentially a particle that we can never detect because it is a "Must Inherit" type and therefore cannot exist on its own. Given that the System object also contains a Central Object that is "Must Inherit" it is possible that the Central Object is the smallest particle in the universe (very likely a Preon).

Along the same lines, it is possible that a Higgs particle is a type of System object that is in turn in inherited by all massive particles. In other words, all massive particles are a type of Higgs particle.
 
Title: Re: How do we measure the energy of a photon?
Post by: Bored chemist on 15/07/2019 14:02:49
It is quite possible that reality also works in this way.
Possible, but unlikely.
It is remarkable conceited to think that the whole universe works in the same way that some bunch of apes chose to use computers.
In other words, all massive particles are a type of Higgs particle.
That's either untrue or unhelpful, depending on whether or not you allow random redefinitions of words.
Title: Re: How do we measure the energy of a photon?
Post by: mxplxxx on 15/07/2019 23:35:49
"Are Humans Smarter Than Chimps? Think Again" (https://www.sciencefriday.com/articles/are-humans-smarter-than-chimps-think-again/). See also https://www.sciencedaily.com/releases/2006/08/060801231359.htm and   https://en.wikipedia.org/wiki/Speciesism.
Title: Re: How do we measure the energy of a photon?
Post by: mxplxxx on 16/07/2019 05:43:42
In my 3dAbstractions software, an App system is a special type of System. It is what runs a Microsoft App. In fact the 3d in 3dAbstractions comes from the structure of the App system.

The App system has subsystems Display, Data and Datastore, hence 3xD. Each subsystem has a dimension associated with it; Display -  3D, Data - 2D and Datastore - 4D. This has correspondent subsystems in Reality.

Display, Space
Data, Data
Datastore, SpaceTime

It is interesting to note that the universe can be simulated in a computer via 2 dimensional data. This is reflected in the Holographic Principle (https://en.wikipedia.org/wiki/Holographic_principle)

"The holographic principle is a tenet of string theories and a supposed property of quantum gravity that states that the description of a volume of space can be thought of as encoded on a lower-dimensional boundary to the region"

We may experience Reality as a 3D projection of the 2D data located (possibly) in a black hole.
Title: Re: How do we measure the energy of a photon?
Post by: Bored chemist on 16/07/2019 09:00:21
We may experience Reality as a 3D projection of the 2D data located (possibly) in a black hole.
No; we can't. Because stuff doesn't come out of black holes.

But we may be experiencing reality as the 3D projection of an 11 dimensional universe.
Title: Re: How do we measure the energy of a photon?
Post by: Bored chemist on 16/07/2019 09:01:40
"Are Humans Smarter Than Chimps? Think Again" (https://www.sciencefriday.com/articles/are-humans-smarter-than-chimps-think-again/). See also https://www.sciencedaily.com/releases/2006/08/060801231359.htm and   https://en.wikipedia.org/wiki/Speciesism.
Thanks for that.
However, what I said was apes.

You still have to explain why you think the Universe  copies from a computer language you have learned about.
Title: Re: How do we measure the energy of a photon?
Post by: mxplxxx on 16/07/2019 10:51:35
Ape, including Humans (98% Chimpanzee DNA):). https://en.wikipedia.org/wiki/Ape. Ape is a genus not a type of animal. Gorilla, a type of Ape much nicer than many humans.
Title: Re: How do we measure the energy of a photon?
Post by: Bored chemist on 16/07/2019 10:56:47
Ape, including Humans (98% Chimpanzee DNA):). https://en.wikipedia.org/wiki/Ape
Yes, I know.

The thing you seem unable to explain is why the Universe should use the same system as some bunch of apes.
Title: Re: How do we measure the energy of a photon?
Post by: mxplxxx on 16/07/2019 12:46:12
To those of you who are having difficulty believing the Universe might behave like a computer program, can I say that a computer program is part of the Universe (as are you) ergo the Universe behaves at least in part like a computer program. Another way of saying this is to state that a computer program is a capability of the universe. Given that computer programs are well on the way to becoming super-intelligent it can be seen that this is another capability of the universe. Thanks to us/evolution the Universe may well be on is way to becoming infinitely intelligent.

The more people use computer programs, the more  the Universe behaves like a computer program! It is not beyond the realms of possibility that we will manage to create personal Universes for recreation and learning that are all computer program (which leads to the fascinating possibly that I/you ARE this universe).

All of this is repeating what I have said previously in the topic. Hierarchical Finite State Machines are extraordinarily difficult things to get a handle on (see e.g. https://en.wikipedia.org/wiki/Behavior_tree_(artificial_intelligence,_robotics_and_control)), so It would no surprise me if the penny hasn't dropped yet. Maybe revision of the topic will help your understanding or maybe it is beyond your current skill level.
Title: Re: How do we measure the energy of a photon?
Post by: Bored chemist on 16/07/2019 13:11:36
To those of you who are having difficulty believing the Universe might behave like a computer program
Many, if not most computer programs are built to mimic the universe.
Of course they look the same as eachother.
But it's the height of conceit to imagine that the Universe looks like the software, rather than the other way round.
Maybe revision is the go for your understanding.
Would you like to try writing what you meant there in such a way that I can actually parse it?


Keep in mind though that I have been programming for 55 years and it is sometimes very hard to distill my theories/knowledge into easily-digestible chunks.
That may go some way to explaining why it's so hard to set you back on the right track when you go astray.
It may also explain why you think that your ideas are "right" when, at best, you can show that they are "possible".
Title: Re: How do we measure the energy of a photon?
Post by: mxplxxx on 17/07/2019 07:06:04
There exist very similar bosonic and fermionic systems. A bosonic system has a central object of that can assume a single state only. A fermionic system has a central object that can assume multiple states. A bosonic system is basically related to events whilst a fermionic system is related  to data.
Actually, it makes a lot more sense to have Boson and Fermion System types. So, Boson.System and Fermion.System. These would have the "Must Inherit" attribute set, meaning they could only exist in "partnership" with "real" particles (or that they are particles in their own right but too small too small to detect). Similarly, there would exist Quark and Lepton Fermion types and Guage and Scalar Boson  types.

So:


Object
    Must Inherit

System
    Inherits Object
    Must  Inherit

Fermion.System
    Inherits System
    Must Inherit

Quark.System
    Inherits Fermion.System
    Must Inherit

UpQuark.System
    Inherits Quark.System

These are software constructs, and I suspect that the same or very similar relationships will exist in Reality. Irrespective, they would almost certainly form the basis for a software simulation of reality.


Title: Re: How do we measure the energy of a photon?
Post by: mxplxxx on 25/07/2019 22:36:22
The future is here now and it is light. A light diode that works without using charge. https://scienceblog.com/509143/developing-technologies-that-run-on-light/?utm_source=feedburner&utm_medium=email&utm_campaign=Feed%3A+scienceblogrssfeed+%28ScienceBlog.com%29
Title: Re: How do we measure the energy of a photon?
Post by: Bored chemist on 26/07/2019 10:31:18
The future is here now and it is light. A light diode that works without using charge. https://scienceblog.com/509143/developing-technologies-that-run-on-light/?utm_source=feedburner&utm_medium=email&utm_campaign=Feed%3A+scienceblogrssfeed+%28ScienceBlog.com%29
What they are talking about is-  as the article points out, A Faraday isolator. The novelty is that it's very small- which is interesting.
The Faraday isolator is called that because it was invented by Faraday in the 19th century.

It's hardly "the future".
Title: Re: How do we measure the energy of a photon?
Post by: mxplxxx on 26/07/2019 11:13:24
What they are talking about is-  as the article points out, A Faraday isolator. The novelty is that it's very small- which is interesting.
They are talking about something that is similar to a Faraday Isolator but uses light, rather than magnetism, to produce the diode effects. As the article says:

"In order to produce a strong enough rotation of the light polarization, these kinds of diodes must be relatively large – much too large to fit into consumer computers or smartphones. As an alternative, Dionne and Lawrence came up with a way of creating rotation in crystal using another light beam instead of a magnetic field. This beam is polarized so that its electrical field takes on a spiral motion which, in turn, generates rotating acoustic vibrations in the crystal that give it magnetic-like spinning abilities and enable more light to get out. To make the structure both small and efficient, the Dionne lab relied on its expertise in manipulating and amplifying light with tiny nano-antennas and nano-structured materials called meta-surfaces."

So, a revolutionary way to create a diode from light which will dramatically change the future.
Title: Re: How do we measure the energy of a photon?
Post by: mxplxxx on 06/08/2019 11:14:01
These are software constructs, and I suspect that the same or very similar relationships will exist in Reality.
Big question. Can we program a software system that mimics the universe and include in it a sentient system that experiences the simulated universe exactly the same way we experience our own universe? Absolutely, I suspect:) Meaning it is not beyond the realms of possibility that we exist in a computer. What advantages could this knowledge give us? For example, could we devise a machine that allows us to time travel for example or travel between universes or talk to our creators or reprogram ourselves (we are close to this anyway)?

Note, a universe in software would allow for a single object to be accessed using multiple reference objects, thus allowing for the seemingly impossible phenomenon of quantum entanglement.

Note we already experience our own universe via a set of computer-like programs in our brain. So much so that we cannot be absolutely convinced via our senses that our universe actually exists. A neural net is just a type of HFSM (Hierarchical Finite State Machine) which is a software/computer construct.

How would I go about writing a program that mimics a universe? I could look at our current universe and program all of its objects/functions. But this would take ages. Or I could program a basic set of functions/objects that could evolve via natural selection and sit back (so to speak) and wait for evolution to do its work. Many are trying to do so.
Title: Re: How do we measure the energy of a photon?
Post by: mxplxxx on 09/08/2019 09:41:38
Object
    Must Inherit

System
    Inherits Object
    Must  Inherit

Fermion.System
    Inherits System
    Must Inherit
It would seem therefore that the universe contains a basic particle (object) that, possibly,  cannot exist on its own and has no attributes.i.e. it just "exists". Or, maybe it can exist on its own and is the missing "dark matter"?

All other objects in the universe derive from/inherit this object.
Title: Re: How do we measure the energy of a photon?
Post by: mxplxxx on 20/08/2019 01:20:51
It would seem therefore that the universe contains a basic particle (object) that, possibly,  cannot exist on its own and has no attributes.i.e. it just "exists"
Or maybe it "possibly" exists. i.e. in a qubit state between 0 and 1.  The following recent discovery backs up this possibility (and also provides solid evidence that the universe is a hologram:).

 “The new discovery of topological superconductivity in a two-dimensional platform paves the way for building scalable topological qubits to not only store quantum information, but also to manipulate the quantum states that are free of error,

https://scienceblog.com/509610/scientists-discover-new-state-of-matter/?utm_source=feedburner&utm_medium=email&utm_campaign=Feed%3A+scienceblogrssfeed+%28ScienceBlog.com%29
Title: Re: How do we measure the energy of a photon?
Post by: Hayseed on 20/08/2019 01:32:49
You can give a photon, 1 watt, or 10 watts.  But after emission, the photon starts decaying.

The energy that you are talking about, is a RATE potential stored in a field pattern.

This rate is preserved, no matter how much the Photon energy has decayed with distance

This rate is the velocity of the torque, or twist, that is put on a charge, when absorbed.

Whether it's a 1 watt, or 10 watt photon........the rate of twist is constant on receiving charge.

Even at great distances.
Title: Re: How do we measure the energy of a photon?
Post by: mxplxxx on 21/08/2019 10:59:22
“The new discovery of topological superconductivity in a two-dimensional platform p
In some sense, a photon seems to be a two-dimensional particle. The electro and magnetic components of the particle are slices of reality. Actually, particles are points which makes them 1-dimensional in nature (they either exist
or don't exist). In the case of photons two 2-dimensional opposing slices are created when the particle vibrates and 3-dimensional "possibility" space exists in which the photon "may" be found/observed, finally a 4-dimensional spacetime exists in which the photon is born, lives and dies.
Title: Re: How do we measure the energy of a photon?
Post by: mxplxxx on 21/08/2019 11:12:49
You can give a photon, 1 watt, or 10 watts.  But after emission, the photon starts decaying.

The energy that you are talking about, is a RATE potential stored in a field pattern.

This rate is preserved, no matter how much the Photon energy has decayed with distance

This rate is the velocity of the torque, or twist, that is put on a charge, when absorbed.

Whether it's a 1 watt, or 10 watt photon........the rate of twist is constant on receiving charge.

Even at great distances.
Are you referring to the transverse wave motion of a photon? You need to be much much clearer in your posts and supply references.
Title: Re: How do we measure the energy of a photon?
Post by: Hayseed on 21/08/2019 11:53:40
Please pardon me for posting on this thread.  That's the second time that I have mis-understood a notification.

I thought I was posting to a new thread.

I think the notification link took me to the first post in this thread, instead of the last one.

I try not to join long threads, being not aware of what all has been discussed.

I will try to be more careful.

My understanding of a photon is of the modern classical model.  Not familiar to many.

Not taught as mainstream, but not dead yet.  It satisfies the quantum physics non-believers.
Title: Re: How do we measure the energy of a photon?
Post by: Kryptid on 21/08/2019 21:54:40
But after emission, the photon starts decaying.

What experiment demonstrated this? Last I looked, all existing evidence points to photons being stable particles with a minimum half-life of 1018 years: https://physicsworld.com/a/what-is-the-lifetime-of-a-photon/

Whether it's a 1 watt, or 10 watt photon........

That makes no sense. A single photon contains a certain amount of energy, but it doesn't have a power.
Title: Re: How do we measure the energy of a photon?
Post by: mxplxxx on 22/08/2019 03:46:49
That makes no sense. A single photon contains a certain amount of energy, but it doesn't have a power.
You are talking about a photon as a particle. It also exists as an electromagnetic wave. This means the photon vibrates at a certain frequency. The higher the frequency the more powerful the photon.  A gamma wave is more powerful than a Infra-red wave. Energy without an associated time is pretty meaningless.
Title: Re: How do we measure the energy of a photon?
Post by: Kryptid on 22/08/2019 09:58:22
You are talking about a photon as a particle. It also exists as an electromagnetic wave. This means the photon vibrates at a certain frequency. The higher the frequency the more powerful the photon.  A gamma wave is more powerful than a Infra-red wave. Energy without an associated time is pretty meaningless.

Okay, so what is the time associated with a 1 keV photon? What is the power?
Title: Re: How do we measure the energy of a photon?
Post by: mxplxxx on 22/08/2019 11:22:18
Okay, so what is the time associated with a 1 keV photon? What is the power?
Neither are applicable to a photon as a particle. As I mentioned a photon is also a wave and you should be easily able to calculate your problem values via E=h/t.
Title: Re: How do we measure the energy of a photon?
Post by: Bored chemist on 22/08/2019 18:58:08
As I mentioned a photon is also a wave and you should be easily able to calculate your problem values via E=h/t.
If it is easy, why don't you do it and show us what fools we are?
Title: Re: How do we measure the energy of a photon?
Post by: mxplxxx on 23/08/2019 01:50:39
Neither are applicable to a photon as a particle. As I mentioned a photon is also a wave
Particles are timeless. None of the states associated with a particle in the standard model are time-reliant. For a particle to exist in this time-bound universe it must also exist as a wave. i.e. it can only "acquire" time by comparing itself against another particle (or itself in a past existence) i.e. as per Einstein's relativity. The System object I have described previously in this topic allows an object to exist both as a particle and a wave,

Particles are likely to be the consciousnesses of the systems they belong to. Maybe a ghost is a particle that has lost its system:). Your consciousness is likely to be a system in your brain.

The central object of the central system of the central object for a system is the particle object for the system. e.g. the central object of a solar system is the sun. The sun contains an abstraction hierarchy of systems. The top of this hierarchy  (at the centre of the sun)  is a system object and the central object of this system object is the sun particle. 

The sun particle contains the current state of the sun at its most abstract level. None of this state is time-reliant (as is the case for all particles in all central objects). It is likely this state is a variable (e.g. a combination of quarks) which make a particle very computer-like.

All systems in the universe have this structure/functionality.   

It is possible that we can see/detect particle objects but not the system objects themselves giving rise to dark matter.

In high-gravity systems, the particle objects will likely eventually become black holes.

We exist near the bottom of a sun abstraction hierarchy. This may make our existence pretty volatile. Most of the photons resulting from our actions will not end up affecting the sun particle, but it is possible for some things we do to to end up affecting this particle which then responds with an event that could affect the whole of the sun/evolution.

Photons from diverse sources can accumulate at various higher abstraction levels making the centre of the sun very hot (in fact hot enough to fuse particles).

It is likely the sun particle object can be affected directly via the sun's magnetic field and vice versa. If fact it is possible that magnetic fields are the general way events/photons are distributed in a system.
Title: Re: How do we measure the energy of a photon?
Post by: Kryptid on 23/08/2019 07:27:18
As I mentioned a photon is also a wave and you should be easily able to calculate your problem values via E=h/t.

So what is the value of "t"?
Title: Re: How do we measure the energy of a photon?
Post by: Bored chemist on 23/08/2019 13:24:49
Neither are applicable to a photon as a particle. As I mentioned a photon is also a wave
Particles are timeless. None of the states associated with a particle in the standard model are time-reliant. For a particle to exist in this time-bound universe it must also exist as a wave. i.e. it can only "acquire" time by comparing itself against another particle (or itself in a past existence) i.e. as per Einstein's relativity. The System object I have described previously in this topic allows an object to exist both as a particle and a wave,
You are a likely a timeless particle (your consciousness?) as well as a time-bound human being..

You forgot to answer my (implicit) question and Kryptid's explicit one.
Okay, so what is the time associated with a 1 keV photon? What is the power?

It's a reasonable enough question.
Why don't you answer it?
Title: Re: How do we measure the energy of a photon?
Post by: mxplxxx on 24/08/2019 09:04:33
Neither are applicable to a photon as a particle. As I mentioned a photon is also a wave
Particles are timeless. None of the states associated with a particle in the standard model are time-reliant. For a particle to exist in this time-bound universe it must also exist as a wave. i.e. it can only "acquire" time by comparing itself against another particle (or itself in a past existence) i.e. as per Einstein's relativity. The System object I have described previously in this topic allows an object to exist both as a particle and a wave,

Particles are likely to be the consciousnesses of the systems they belong to. Maybe a ghost is a particle that has lost its system:). Your consciousness is likely to be a system in your brain.

The central object of the central system of the central object for a system is the particle object for the system. e.g. the central object of a solar system is the sun. The sun contains an abstraction hierarchy of systems. The top of this hierarchy  (at the centre of the sun)  is a system object and the central object of this system object is the sun particle. 

The sun particle contains the current state of the sun at its most abstract level. None of this state is time-reliant (as is the case for all particles in all central objects). It is likely this state is a variable (e.g. a combination of quarks) which make a particle very computer-like.

All systems in the universe have this structure/functionality.   

It is possible that we can see/detect particle objects but not the system objects themselves giving rise to dark matter.

In high-gravity systems, the particle objects will likely eventually become black holes.

We exist near the bottom of a sun abstraction hierarchy. This may make our existence pretty volatile. Most of the photons resulting from our actions will not end up affecting the sun particle, but it is possible for some things we do to to end up affecting this particle which then responds with an event that could affect the whole of the sun/evolution.

Photons from diverse sources can accumulate at various higher abstraction levels making the centre of the sun very hot (in fact hot enough to fuse particles).

It is likely the sun particle object can be affected directly via the sun's magnetic field and vice versa. If fact it is possible that magnetic fields are the general way events/photons are distributed in a system.

A theory of Particles.
Title: Re: How do we measure the energy of a photon?
Post by: Bored chemist on 24/08/2019 12:16:00
Neither are applicable to a photon as a particle. As I mentioned a photon is also a wave
Particles are timeless. None of the states associated with a particle in the standard model are time-reliant. For a particle to exist in this time-bound universe it must also exist as a wave. i.e. it can only "acquire" time by comparing itself against another particle (or itself in a past existence) i.e. as per Einstein's relativity. The System object I have described previously in this topic allows an object to exist both as a particle and a wave,
You are a likely a timeless particle (your consciousness?) as well as a time-bound human being..

You forgot to answer my (implicit) question and Kryptid's explicit one.
Okay, so what is the time associated with a 1 keV photon? What is the power?

It's a reasonable enough question.
Why don't you answer it?

Title: Re: How do we measure the energy of a photon?
Post by: mxplxxx on 25/08/2019 11:15:46
In high-gravity systems, the particle objects will likely eventually become black holes.
But, how can a point particle affect a black hole? Answer, the point particle is a reference object in 3d space to the "real" deal particle which exists outside of time. In other words, a particle cannot exist in time, but a reference to it can. Explains how quantum entanglement works.

ps in a VB.NET or C# Microsoft program, all objects are reference objects.
Title: Re: How do we measure the energy of a photon?
Post by: Kryptid on 25/08/2019 14:46:58
How do you expect me to solve the equation if you won't tell me what the value of "t" is?
Title: Re: How do we measure the energy of a photon?
Post by: Bored chemist on 25/08/2019 17:42:32
How do you expect me to solve the equation if you won't tell me what the value of "t" is?
He isn't fooling anyone, is he.
There isn't a defined value of t so he's talking nonsense and he knows it.
That's why he won't (and can't) answer.

I think he is hoping we might forget about it.
Title: Re: How do we measure the energy of a photon?
Post by: mxplxxx on 02/09/2019 13:25:35
It would appear that this group of scientists have been able to modify the frequency of an EM wave so that it can be "trapped" in a hole and later released at the original frequency.  Fascinating but highly unlikely:). https://www.sciencedaily.com/releases/2019/08/190830150746.htm

Actually, the article, like most articles on innovative physics techniques, fails to explain the process involved. In fact these people have been experimenting with mechanical waves and then blithely saying the techniques are applicable to light waves.

"While we ran our proof-of-concept experiment using elastic waves traveling in a solid material, our findings are also applicable to radiowaves and light, offering exciting prospects for efficient energy harvesting, wireless power transfer, low-energy photonics, and generally enhanced control over wave propagation," said Ruzzene.

They seem to think "radiowaves" and light are different kettles of fish:)
Title: Re: How do we measure the energy of a photon?
Post by: Kryptid on 02/09/2019 15:48:12
What is the value of "t"?
Title: Re: How do we measure the energy of a photon?
Post by: mxplxxx on 13/09/2019 01:11:02
IT would seem that the centre of the milky way galaxy contains a gigantic object of unknown purpose (https://scienceblog.com/510438/giant-balloon-like-structures-discovered-at-center-of-milky-way/?utm_source=feedburner&utm_medium=email&utm_campaign=Feed%3A+scienceblogrssfeed+%28ScienceBlog.com%29).

Possibly a hierarchical finite state machine (hfsm) object that I predicted in my theory of universal systems documented in this topic?

p.s. state changes in a hfsm are considered to be instantaneous in a computer implementation of the hfsm object. The black hole at the centre of the central object in a universal system would likely rely on this fact to change state. i.e. the whole of the black hole would change state instantaneously whenever a state change event was received.  A black hole is outside of time and  would likely handle this situation by "containing" all future possible states.
Title: Re: How do we measure the energy of a photon?
Post by: Kryptid on 13/09/2019 05:50:52
IT would seem that the centre of the milky way galaxy contains a gigantic object of unknown purpose (https://scienceblog.com/510438/giant-balloon-like-structures-discovered-at-center-of-milky-way/?utm_source=feedburner&utm_medium=email&utm_campaign=Feed%3A+scienceblogrssfeed+%28ScienceBlog.com%29).

Possibly a hierarchical finite state machine (hfsm) object that I predicted in my theory of universal systems documented in this topic?

p.s. state changes in a hfsm are considered to be instantaneous in a computer implementation of the hfsm object. The black hole at the centre of the central object in a universal system would likely rely on this fact to change state. i.e. the whole of the black hole would change state instantaneously whenever a state change event was received.  A black hole is outside of time and  would likely handle this situation by "containing" all future possible states.

Is there any particular reason you keep ignoring my question?
Title: Re: How do we measure the energy of a photon?
Post by: Bored chemist on 14/09/2019 00:22:31
There isn't a defined value of t so he's talking nonsense and he knows it.
That's why he won't (and can't) answer.
Is there any particular reason you keep ignoring my question?
Yes.
He's only pretending to be competent
Title: Re: How do we measure the energy of a photon?
Post by: mxplxxx on 28/10/2019 10:54:45
https://www.vice.com/en_in/article/j5yngp/the-universe-is-made-of-tiny-bubbles-containing-mini-universes-scientists-say

And what is a bubble if not a spherical Shell as explained previously in this topic (https://www.thenakedscientists.com/forum/index.php?topic=75960.msg576986;topicseen#msg576986)? And what is a mini-universe if not a System as also explained previously in this topic? And Systems "live" in Shells.In fact, s System is probably a type of shell. Which may mean Shells are "must inherit" objects and therefore cannot be detected on their own.
Title: Re: How do we measure the energy of a photon?
Post by: Hayseed on 28/10/2019 16:11:58
A photon only describes one component of a wave.  The rate.

An EM wave is intermittent pulses.  It has direction, E polarity, M polarity, intensity and rate.

The direction is out from emitter.  The E polarity remains constant.  But the M polarity alternates. One pulse has the N pole and the next pulse has the S pole.

The intensity sets the number of absorbents.  The process of absorbing.....is applying a torque rate to the absorbent.

The photon is a measure of the torque applied to the absorbent.  Only a charge can absorb.

And only a charge can emit.    Photon=RATE.

All that is required to measure this is frequency......which is rate.


Edit:  Pardon me.  When I see the word photon, I automatically think of particles.  And particles only have 1 E field polarity.

With dipole emission, the E component, alternates with the pulse also.
Title: Re: How do we measure the energy of a photon?
Post by: mxplxxx on 29/10/2019 00:07:15
A photon only describes one component of a wave.  The rate.

An EM wave is intermittent pulses.  It has direction, E polarity, M polarity, intensity and rate.

The direction is out from emitter.  The E polarity remains constant.  But the M polarity alternates. One pulse has the N pole and the next pulse has the S pole.

The intensity sets the number of absorbents.  The process of absorbing.....is applying a torque rate to the absorbent.

The photon is a measure of the torque applied to the absorbent.  Only a charge can absorb.

And only a charge can emit.    Photon=RATE.

All that is required to measure this is frequency......which is rate.


Edit:  Pardon me.  When I see the word photon, I automatically think of particles.  And particles only have 1 E field polarity.

With dipole emission, the E component, alternates with the pulse also.

You have pretty much given an explanation of standard EM wave/photon theory. This has been replaced by quantum theory. In any case, you are in new theories and if you take the time to peruse the topic you will find that I have proposed a "System' theory that explains wave/particle duality (and a few other things as well).

Title: Re: How do we measure the energy of a photon?
Post by: Hayseed on 29/10/2019 02:35:10
Pardon me.  I thought the word photon came from the study of QT.  We used to call it QM.

And I thought you were asking how to measure one.  One would measure the frequency.  Or perhaps an indirect frequency measurement.

But you are right, I have never needed QT.   But I can still measure frequency.  So, I can measure a photon......with a QT constant.

Didn't mean to disturb your post.   I will retire.
Title: Re: How do we measure the energy of a photon?
Post by: mxplxxx on 05/11/2019 09:29:59
http://blogs.discovermagazine.com/d-brief/2019/11/04/voyager-2s-first-reports-from-interstellar-space-surprise-scientists/#.XcEww0YzZ3g

More bubbles/shells:)
Title: Re: How do we measure the energy of a photon?
Post by: mxplxxx on 23/11/2019 12:33:26
Shells could equally be called time bubbles. A system living in a shell/time bubble experiences time. Systems "contain" their own time bubbles /shells in which "live" subsystems. All systems perceive time passing at the same rate but within a time bubble, time "actually" passes at a rate that is decided by the "level" (more about this and its relationship to the quantum of action later) of the bubble. The recursive network of time bubbles/shells and systems constitute a universal hierarchical abstraction database that "IS" the universe.
Title: Re: How do we measure the energy of a photon?
Post by: mxplxxx on 06/12/2019 02:35:56
Of course Shells are quantum entities. This means that all Shells have the same amount of Action, h (Planck's constant as in E=h/t). What varies between Shells is the rate of flow of time (t in E=h/t). Less time equates to more distance (expansion) and more time equates to less distance (contraction).

A system can modify the time component of the shell that encloses it. Contraction of the shell will occur when the system receives a graviton. Expansion occurs when the system receives a ? (an anti-graviton?). Thus gravity and universal expansion occur. The system will forward the graviton/anti-graviton to each of its subsystems and the process repeats. No change in Action occurs in this process (although E as in E=h/t will vary).

In fact, given that E is a function of time, it is likely that time is the only State variable in this universe.

There ARE variables like Direction and Location that have meaning only where two or more variables are involved. In a universe that works via the Systems described in this topic, this type of variable will be defined relative to the parent system (thus negating the need for the relativity described by Einstein).

Then there are other states like Charge and Spin that are read-only for a particular particle. i.e. not variables (not 100% sure of this).
Title: Re: How do we measure the energy of a photon?
Post by: mxplxxx on 08/12/2019 20:08:39
https://scienceblog.com/512559/a-platform-for-stable-quantum-computing-a-playground-for-exotic-physics/?utm_source=feedburner&utm_medium=email&utm_campaign=Feed%3A+scienceblogrssfeed+%28ScienceBlog.com%29

Topological insulators are materials that can conduct electricity on their surface or edge, but not in the middle. Given that a feature of the Systems described by this post is that the central object is a dead end for the flow of events/photons, it is likely that all Systems will be found to be topological insulators. 
Title: Re: How do we measure the energy of a photon?
Post by: mxplxxx on 19/12/2019 22:05:58
How to use Systems to control brain states.

https://scienceblog.com/512914/helping-to-uncover-the-mechanism-controlling-brain-state

If the basic type of a cell is a system, as described in this post, changing states on a brain-wide basis is simple. The top level system simply sends a change-basic-state message to all its subsystems. The subsystems in turn change state and forward the message to their subsystems. Each system is a hierarchical finite state machine. There will be single a top-level (level 1) state, controlling, and two level 2 sub-states, hunting and exploring as per the scienceblog article. This may be what happens in a startle reflex.
Title: Re: How do we measure the energy of a photon?
Post by: Bored chemist on 20/12/2019 10:44:43
This may be what happens in a startle reflex.
No, it can't be.
You still need to do some learning.
Title: Re: How do we measure the energy of a photon?
Post by: mxplxxx on 25/12/2019 03:16:55
Tabby’s Star: Vanishing stars could lead to alien

Wednesday, December 25, 2019/
9:59 AM

Clipped from: https://www.news.com.au/technology/science/space/vanishing-stars-may-lead-to-alien-civilisations/news-story/4d309aa7025608b75f812bcd26375d57

Or... vanishing stars are just objects that are no longer referenced by any other object. Computer programs delete such objects in a "garbage collection" process.

Or ... vanishing stars are part of the quantum "virtual particle" process whereby objects flash in and out of existence (which may be related to alternative 1).

Or ... Our 3d Universe is just a projection of an underlying 2d Universal System. Something has deleted (or otherwise made unavailable) Tabby's Star from the 3d part of the database (the Display System in my software) or from the 2d part of the database (the Data system in my software) or the 4d part of the database (the Datastore system in my software). The disappearance of Tabby's Star gives much more credence to the Hologram theory regarding the nature of the universe.

Given all objects are part of the Universe System, the only way they can be in a "garbage collect" state is for "Something" to have set them to "Nothing". This makes them vanish as far as the "Something" is concerned but they still exist until all of the "Somethings" that reference them have set them to "Nothing". Soon after this point they stop being physically present (i.e. they are "garbage collected" by the Universal run-time system - which we know nothing of).   

It means that an object like the earth will only reference those objects that are relevant for its existence. Also that Dark matter may "merely" be matter that is not being "observed". 

Actually, vanish is a relative phenomenon. Assuming we live in a digital universe, everything has to vanish when moving from place A to place B. We don't perceive this happening for close-by systems because the time system we are immersed in does not allow us to register it. The same may not be the case for distant systems such as Tabby's Star. This could be a system in which, for reasons unknown,  time moves, relatively, much much slower than our own. We see it as if in slow motion and we see in much more detail the whole process of moving from one place to another. In other words, it has not vanished but instead is in an interim un-observable state in the moving process when we try and observe it.


Title: Re: How do we measure the energy of a photon?
Post by: mxplxxx on 27/12/2019 08:00:46
https://phys.org/news/2019-12-quantum-silicon-bits-long-distance-relationship.html

"Each qubit is composed of a single electron trapped in a tiny chamber called a double quantum dot. Electrons possess a property known as spin, which can point up or down in a manner analogous to a compass needle that points north or south. By zapping the electron with a microwave field, the researchers can flip the spin up or down to assign the qubit a quantum state of 1 or 0."

So, no quantum superposition of states. Just plain vanilla computing 0/1 states. Why then do they call it a quantum state?
Title: Re: How do we measure the energy of a photon?
Post by: Kryptid on 27/12/2019 17:22:05
Or ... vanishing stars are part of the quantum "virtual particle" process whereby objects flash in and out of existence (which may be related to alternative 1).

That would violate conservation of mass and/or the uncertainty principle. Virtual particles appearing and disappearing is different because the timescales involved adhere to the uncertainty principle and keep the average energy budget of the vacuum constant.
Title: Re: How do we measure the energy of a photon?
Post by: mxplxxx on 27/12/2019 22:04:51
Or ... vanishing stars are part of the quantum "virtual particle" process whereby objects flash in and out of existence (which may be related to alternative 1).

That would violate conservation of mass and/or the uncertainty principle. Virtual particles appearing and disappearing is different because the timescales involved adhere to the uncertainty principle and keep the average energy budget of the vacuum constant.
It seems to me that, at the point where the virtual particle is present, the law of the conservation of energy has been violated. It also seems to me that, because of relativity, the times involved in the uncertainty principle will vary from object to object, making it a very shaky principle.
Title: Re: How do we measure the energy of a photon?
Post by: Kryptid on 27/12/2019 22:15:57
It seems to me that, at the point where the virtual particle is present, the law of the conservation of energy has been violated.

Depends on how you look at it. One way that I've seen it described (In Kip Thorne's Black Hole and Time Warps) is that the part of the vacuum that temporarily acquires a positive net energy has taken it from a nearby portion of the vacuum, leaving that portion with negative energy to compensate. Alternatively, you can say that conservation of energy isn't violated because the time period for which the virtual particles were created and then destroyed happens inside of a window of time that is too short to allow the particles to be observed. Thus, any attempt to measure the system will never come up with mass violations.

A star that was once observable but then disappeared would not fall under either of these scenarios, which means a definite violation of conservation of mass must have taken place.
Title: Re: How do we measure the energy of a photon?
Post by: mxplxxx on 13/01/2020 23:23:47
Here is an interesting article on simulations. https://scienceblog.com/513429/supercomputer-simulations-showcase-novel-planet-formation-models/?utm_source=feedburner&utm_medium=email&utm_campaign=Feed%3A+scienceblogrssfeed+%28ScienceBlog.com%29.
The penny seems to have dropped that, to simulate large scale data, you need abstractions, HFSMs (Hierarchical Finite State Machines)  and Systems as described in this post.
Title: Re: How do we measure the energy of a photon?
Post by: Bored chemist on 14/01/2020 07:20:17
The penny seems to have dropped that, to simulate large scale data, you need abstractions, HFSMs (Hierarchical Finite State Machines)  and Systems as described in this post.
That's nice.
Do you have any idea when the penny will drop that this has nothing to do with measuring photon energies?
Title: Re: How do we measure the energy of a photon?
Post by: mxplxxx on 14/01/2020 10:33:45
This post is better titled "The Theory of Universal Systems". "How to measure the energy of a photon" was how it started out in the Physics forum before it was moderated to be a new theory. I have no control over the title.

Tinkle, tinkle (the sound of a penny dropping).
Title: Re: How do we measure the energy of a photon?
Post by: Bored chemist on 14/01/2020 18:49:48
"The Theory of Universal Systems"
No
https://en.wikipedia.org/wiki/Scientific_theory
Title: Re: How do we measure the energy of a photon?
Post by: mxplxxx on 21/01/2020 05:55:57
The Universal System described in this post has outer and inner components. For a solar System, these are the Solar System itself and the Sun System respectively. The Inner component contains the time-independent state of the  system and the Outer component contains the time-dependent state of the System.

The state variables in the Outer component of a System are:

Direction

    the direction the System is travelling relative to its parent System (which will be a galaxy System in the case of a Solar System)

Velocity

    the speed which the System is travelling at relative to its parent System. This is zero for the Inner (i.e. Central) System.

Location

    the current location of a System relative to it's parent System.

There are shades of Einstein's relativity here, but only two objects (the System and its parent) are involved and no need for an absolute speed of light.

Amongst the state variables in the Inner component of a System are:

Energy

    the energy of the  System

Charge

    positive, negative or neutral

Spin

    as per physics
 
The energy of a system is contained as a variable in in the Inner component of a System. Energy and time are not things. They are states of a System, i.e. just concepts. Thus, we need inner and outer components (energy and velocity) interacting to obtain the momentum of a System (which will include the momenta of all its subsystems).

The Inner component is a System in its own right (it inherits a Central System object) but has limited ability to be able to interact with other systems.

So, here is the Visual Basic code for calculating the momentum of a System.

public readonly Property Momentum as double

    Get
        dim momentum as double = (me.Star.Energy / (c * c)) * me.velocity

        for each subsystem as System in me.subsystems
            momentum += subsystem.momentum
       next

    return momentum

    end Get

end Property


where me is the current System, c is SOL and Star is the central object of my central system. Thus, the momentum of the universe is given by universe.Momentum where universe is defined as:

 dim universe = New Universe.System(Star:=new Universe.Star())
 
Title: Re: How do we measure the energy of a photon?
Post by: mxplxxx on 02/02/2020 20:38:02
https://scienceblog.com/513858/how-supercomputers-are-helping-us-link-quantum-entanglement-to-cold-coffee/?utm_source=feedburner&utm_medium=email&utm_campaign=Feed%3A+scienceblogrssfeed+%28ScienceBlog.com%29

Quantum entanglement has been at work since the early days of computing. In fact, it is integral to the way modern computers work. A language like Microsoft's Visual Basic.NET implements an object by creating a "behind the scene" object and making it available to Apps via reference objects. The App cannot access the original object directly. All change in state in the original object (it can change its own state or be changed via reference objects) are immediately available to the reference objects. This is the essence of quantum entanglement.

Quantum Entanglement implies instantaneous access. This is not available to the "simulated" part of the Universe, but would be available to the Universe's run-time system.
Title: Re: How do we measure the energy of a photon?
Post by: mxplxxx on 02/02/2020 23:11:28
IMHO I think the current Physics theory regarding the nature of a photon leaves a lot to be desired.

I would like to, very tentatively, propose that a photon is a type of gyroscope, the purpose of which is to maintain the angular momentum of the photon as it travels through the hell and high water of Reality.

The E and M waves of standard photon theory become the E and M Gimbals of a gyroscope.

So, according to my theory of Universal Systems proposed in this post, a photon would be the central object in a Photon System with the following structure.

Photon System
    Inherits Gyroscope.System

    Photon (central object/system of photon system) with Property Frequency (of Spin)
    Subsystems
        E Gimbal
        M Gimbal

There is no energy here, which means the photon system is both stable (via its Gyroscopic nature) and nonreactive as it travels.

THe photon will interact with and be destroyed by a system that accepts its Frequency. The accepting system will increase in energy by an amount equal to E=hf where h is Planck's constant and f is the frequency of the photon.
Title: Re: How do we measure the energy of a photon?
Post by: Bored chemist on 03/02/2020 07:25:14
IMHO I think ...
What does the H stand for?
Title: Re: How do we measure the energy of a photon?
Post by: mxplxxx on 04/02/2020 17:08:29
THe photon will interact with and be destroyed by a system that accepts its Frequency. The accepting system will increase in energy by an amount equal to E=hf where h is Planck's constant and f is the frequency of the photon.
Probably "forgotten" rather than "destroyed" and collected by the Universal garbage collection function (whatever that is).

Note that time and energy in this proposal are just different forms of each other. The fundamental quantum equation of quantum physics, E=hf (or E=h/t), screams out this relationship. So we will have conservation of time and we each likely have a fixed amount of time to play with in the game of life/Reality. As this time runs down, so does our energy.

It is highly likely that Reality is a school of the future and time/energy are the basic teaching tools of this school. Why have a school of the future? Possibly because the universe/us are evolving entities that need to get better and better organised over lifetimes. Why? Heavens knows! Or maybe Reality is  just a game for us to have fun in:)   
Title: Re: How do we measure the energy of a photon?
Post by: mxplxxx on 04/02/2020 17:25:16
https://scienceblog.com/513921/sand-dunes-can-communicate-with-each-other/?utm_source=feedburner&utm_medium=email&utm_campaign=Feed%3A+scienceblogrssfeed+%28ScienceBlog.com%29

Of course they interact; they will be Universal Systems.
Title: Re: How do we measure the energy of a photon?
Post by: Bored chemist on 04/02/2020 19:09:00
IMHO I think ...
What does the H stand for?
Title: Re: How do we measure the energy of a photon?
Post by: mxplxxx on 05/02/2020 16:38:56
A photon travels at the speed of light. Absolute (versus perceived) time passes at the speed of light. This means that a photon exists only in the present. The photon is still whilst (likely) the universe evolves around it. Nothing else (apart from other bosons) exists in the present. The photon will pass information to the brain about the nature of past events. So the brain will perceive Reality as being current when, if fact, everything except bosons is a past event. 
Title: Re: How do we measure the energy of a photon?
Post by: Bored chemist on 05/02/2020 19:37:32
IMHO I think ...
What does the H stand for?

Title: Re: How do we measure the energy of a photon?
Post by: mxplxxx on 08/02/2020 00:21:35
So we will have conservation of time and we each likely have a fixed amount of time to play with in the game of life/Reality. As this time runs down, so does our energy.
Please ignore this. Nothing to do with E=hf.
Title: Re: How do we measure the energy of a photon?
Post by: Bored chemist on 08/02/2020 11:44:37
IMHO I think ...
What does the H stand for?


Title: Re: How do we measure the energy of a photon?
Post by: mxplxxx on 10/02/2020 22:47:17
Nothing else (apart from other bosons) exists in the present.
Everything else is memories. The big question is "what does the remembering":). DNA-based organisms would appear to be one answer. But these are just systems. The universe itself is just the top-level system of a HFSM tree of systems. Maybe the universal run-time system can "remember" systems, so we should look behind Reality to find answers as to what makes the universe tick and why.

Actually, the is no reason why a HFSM cannot be infinite in nature. A Universal HFSM (Hierarchical Finite State Machine) is a tree of systems. Going up the tree, Systems become more and more abstract Going down the tree, systems become more and concrete. At infinity at the top of the tree everything becomes an abstraction. This System may be what we think of as God. There is no reality at this point. At infinity at the bottom of the tree, there is nothing but a continuous Reality. There is no abstraction at this point. We are somewhere in between and may be capable of accessing the top and bottom of the HFSM that we exist in.
Title: Re: How do we measure the energy of a photon?
Post by: mxplxxx on 11/02/2020 22:11:24
I have mentioned that Universal Systems have an infinite aspect about them. This comes about from the fact that they contain a Parent reference and Subsystem references. A System can reference its Parent which can reference its own Parent and so on ad infinitum. A Subsystem can reference its child (Subsystems) which can then reference its own child and so on ad infinitum.

Subsystems can exist by value as well as by reference. If this is the case, a System "contains" all its Subsystems. At first glance, this seem to be so, mostly, for Reality. Systems that have been created via Quantum Entanglement are an exception. An entangled System may have two or more parents. This is called "multiple inheritance" in computer science. This makes the Universe a type of neural net, with the obvious possibility that the Universe, and everything in it, is intelligent.

In a "by value" System, a parent system will be larger than its children (probably quantized larger). So the top of the parent/child tree will be infinitely large (everything) and the bottom of the parent/child tree will be infinitely small (nothing). But, thanks to the scale up/down feature of Universal Systems (described elsewhere), The top system and the bottom systems will experience themselves as being the same size. 

A simulation is an abstraction of something "real". The more abstract something is the more it is a simulation. The less abstract something is, the closer it is to Reality. We exist as a System that is at a certain level of abstraction. Hence we are a simulation, possibly very close to Reality. But abstractions appear to be relative and our system is a scaled up version of another system. We consider ourselves to be "Real" and, in a sense we are but with limitations because of our place in a Universe of abstractions. This is how we learn.     

A sufficiently detailed simulation is indistinguishable from Reality. An infinitely detailed simulation may be GOD.


Title: Re: How do we measure the energy of a photon?
Post by: mxplxxx on 15/02/2020 01:49:24
Subsystems can exist by value as well as by reference. If this is the case, a System "contains" all its Subsystems. At first glance, this seem to be so, mostly, for Reality. Systems that have been created via Quantum Entanglement are an exception. An entangled System may have two or more parents. This is called "multiple inheritance" in computer science. This makes the Universe a type of neural net, with the obvious possibility that the Universe, and everything in it, is intelligent.
Actually, on closer analysis, it would seem that subSystems in an Outer System are accessed "by value" and subSystems in an Inner System are accessed "by reference". The "by reference" access makes for very flexible access and, as a result, Inner systems are very easy to change.

Thus, the subSystems of a (outer) Solar System (such as the Earth System) are physically part of the system but those of the (Inner or Central) Sun System are references to the physical Systems that are part of the Sun.

As mentioned previously, combined with parent/co-parent access "by reference" and the Quantum nature of Reality, much of the Universe will exist as neural networks. The universe will have an extraordinarily level of intelligence. The quantum nature of Reality ensures that energy adds up to a specific value before an event "fires/activates", thus paralleling (but much, much superior to) an artificial neural network. 
Title: Re: How do we measure the energy of a photon?
Post by: Bored chemist on 15/02/2020 12:48:12
IMHO I think ...
What does the H stand for?



Title: Re: How do we measure the energy of a photon?
Post by: mxplxxx on 17/02/2020 02:35:27
All Systems have State.

States are either intrinsic, i.e. determined only by factors local to the system or extrinsic, i..e. determined only by factors external to the System (think Relativity). Or a mix. Outer Systems (e.g. a Solar System) will tend to have mainly extrinsic state and Inner Systems (e.g a Sun System) will tend to have mainly intrinsic state.

States may have occur in a hierarchy (as per a HFSM).

States change when the system process an event that is deemed to affect the current state. This can be at any level. If at a high level the the state change will likely propagate to lower levels.

The map of a state changing over time is a wave. In a computer system, state changes are achieved by events linked to processes. A rhythmically occurring process or set of processes may be described via a mathematical Wave Function but, in general, most waves are too complex to be mathematically described. 

It is interesting to note that the System proposed in this topic will result in two waves (inner System and outer System) when and if the system states can be measured over time. This may give rise to an EM wave.

In my software, State is represent in an App by fundamental data types (such as integer, string etc.). Anything more complex is represented by a System. The same may be true of reality, i.e. only elementary particles (such a Quarks) and aggregates of same may contain State and more complex states are represented by Systems.

for example:

Race
    inherits InnerSystem

    property Status as integer

Status is the current state of Race (Open = 1, Closed = 2 etc.) and will never be anything more that a fundamental data type.

It is interesting to note that we "know" what State is but have to resort to mathematics/language to describe/manipulate it. i.e. our brains needs to simulate Reality to deal with it.


 
Title: Re: How do we measure the energy of a photon?
Post by: Bored chemist on 17/02/2020 07:22:01
IMHO I think ...
What does the H stand for?




Title: Re: How do we measure the energy of a photon?
Post by: mxplxxx on 21/02/2020 04:56:05
A 3D App is a type of System that can be executed.

It comes with 3 built-in subsystems:

Data

  contains an abstraction hierarchy of Systems . The hierarchy is part Hierarchical Finite State Machine (HFSM) and part Hierarchical Database. It is basically a neural network with database capabilities. It is loaded into memory as required.

    It is 2D in nature

Display

  contains the "visible" part of a System. It is a feedback interface. Space consists of System Displays. THe expansion of space is therefore caused by more Apps appearing in the universe (like us!). 

    It is 3D in nature. i.e. Space

Datastore

    contains the permanent memories of a System, organised by time of happening and Key (All systems have a unique Key).

    It is 4D in nature (i.e. SpaceTime)

And App also contains subApps (or maybe subSystems ... not sure of this).  In a simulation using Apps, the Universe is a single App. It contains subApps that are galactic Cluster Apps. 
Title: Re: How do we measure the energy of a photon?
Post by: Bored chemist on 21/02/2020 13:38:09
IMHO I think ...
What does the H stand for?





Title: Re: How do we measure the energy of a photon?
Post by: mxplxxx on 24/02/2020 10:09:10
https://www.slashgear.com/otago-researchers-hold-individual-atoms-in-place-in-a-new-experiment-2161053

Which makes 3d Systems that much more likely.

in Visual Basic,

  dim OxygenMolecule = new Oxygen.Molecule(Oxygen.Atom1, Oxygen.Atom2)

Here is a VERY condensed structure of an Oxygen (O2) molecule according to 3d.

Stars are Central 3d Systems (Star as in star of the show).

Proton
    Star
    System
        Up Quark-1
        Up Quark-2
        Down Quark

Neutron
    Star
    System
        Down Quark-1
        Down Quark-2
        Up Quark

Electron
    Star (an Electron Ion)
    System (no subsystems - Electron is considered to be elementary)

Oxygen (Atom)
    Star (nucleus)
        Shell-1
            Proton-1-1
            Neutron-1-1
            Proton-1-2
            Neutron-1-2
        Shell-2
            Proton-2-1
            Neutron-2-1
            Proton-2-2
            Neutron-2-2
            Proton-2-3
            Neutron-2-3
            Proton-2-4
            Neutron-2-4
            Proton-2-5
            Neutron-2-5
            Proton-2-6
            Neutron-2-6
    System
        Shell-1
            Electron-1-1
            Electron-1-2
        Shell-2
            Electron-2-1
            Electron-2-2
            Electron-2-3
            Electron-2-4
            Electron-2-5
            Electron-2-6

Oxygen (O2)
    Star
    System
        Shell-1
            Oxygen (Atom)-1
            Oxygen (Atom)-2


Title: Re: How do we measure the energy of a photon?
Post by: Bored chemist on 24/02/2020 10:27:54
IMHO I think ...
What does the H stand for?






Title: Re: How do we measure the energy of a photon?
Post by: mxplxxx on 26/02/2020 06:36:11
Humans and other DNA-based entities are likely 3d Systems.

In our case, the central System is our skeletal System with subSystems respiratory, digestive, lymphatic etc. We have an executive brain System. The central System of our brain System is probably the seat of our awareness with the top of this system being our consciousness.

We evolved from the sea and are now part of the atmosphere system at a fairly concrete level of abstraction. Not surprisingly then, we feel a very strong love of and connection with water. We have evolved to the point where we can leave the earth provided we take an atmosphere System with us.

Being water/oxygen-based, climate change is likely to be more dangerous to us than most people believe.
Title: Re: How do we measure the energy of a photon?
Post by: Bored chemist on 26/02/2020 08:16:25
IMHO I think ...
What does the H stand for?







Title: Re: How do we measure the energy of a photon?
Post by: mxplxxx on 29/02/2020 13:15:55
In our case, the central System is our skeletal System
Actually, this is probably incorrect. The Solar Plexus system is a better choice for the central system given it is truly central.

So in a Human Being system we have top-level 3d systems Nervous, Respiratory, Circulatory, Integumentary (skin) etc.with a central Solar Plexus system. Each system is essentially a hierarchical finite state machine (HFSM) that provides services of a particular type. The systems are intertwined and cooperate in the formation of life. The Nervous System provides message services to the other systems. The Circulatory system delivers blood to other systems - and so on. A complex time-based spatially-oriented process sets systems next to one another so that services can be precisely delivered and so that systems can grow in a scaled manner.

Chances a DNA-based cell is a type of 3d System. It has a central system (the nucleus) and subsystems. If this is the case then the cells in a DNA-based entity will form a HFSM. Subsystems will be accessed "by reference".
Title: Re: How do we measure the energy of a photon?
Post by: Bored chemist on 29/02/2020 14:33:19
IMHO I think ...
What does the H stand for?








Title: Re: How do we measure the energy of a photon?
Post by: mxplxxx on 03/03/2020 08:21:06
https://www.fastcompany.com/90470552/surprising-study-reveals-what-makes-a-good-coder-and-its-not-math

Good software development requires language skills not mathematics as is usually supposed. Software is mostly  about concepts and only partially about maths (Einstein would probably have made a good software developer).  This is possibly the main reason physics has been slow to embrace Software Development as a means of defining physics theories.

I am proficient in 25 Software languages as well as French, Latin and English. I also have HD results in University Maths.

I am also an innovator, an adventurer, a simplifier and have a good sense of humor. My top 10 Realise2 strengths are:

1. Gratitude
2. Innovation
3. Growth
4. Explainer
5. Creativity
6. Improver
7. Adventure
8. Humor
9. Persuasion
10. Relationship Deepener

I also have a psychology/coaching degree which gives me great analytical skills.

Maybe a top-class physicist of the future also needs to be a top-class software developer and a psychologist :) . Mathematics - great for single-object and state change descriptions; software - great for multi-object and time interactions.
Title: Re: How do we measure the energy of a photon?
Post by: Bored chemist on 03/03/2020 19:20:02
I am also an innovator, an adventurer, a simplifier and have a good sense of humor. My top 10 Realise2 strengths are:

1. Gratitude
2. Innovation
3. Growth
4. Explainer
5. Creativity
6. Improver
7. Adventure
8. Humor
9. Persuasion
10. Relationship Deepener
Other barnum statements are also available.
Title: Re: How do we measure the energy of a photon?
Post by: Bored chemist on 03/03/2020 19:22:35
IMHO I think ...
What does the H stand for?










Are you going to answer this?
Title: Re: How do we measure the energy of a photon?
Post by: mxplxxx on 07/03/2020 11:21:36
https://scienceblog.com/514613/solving-a-mystery-in-126-dimensions/?utm_source=feedburner&utm_medium=email&utm_campaign=Feed%3A+scienceblogrssfeed+%28ScienceBlog.com%29

Here is a Benzene molecule (C6h6) built with 3d Abstractions technology. It supplies the basic structure for simulating a Benzene molecule. For clarity, 3d Shells have been omitted.

Carbon.Atom (C)
    inherits Atom (a 3d Outer System)

    Carbon.Ion
         inherits Ion (a 3d Central System)
   
        Star (current state of Carbon.Ion)
      
        Subsystems
            Proton 1 (a Proton is effectively a Hydrogen Ion)
            Proton 2
            Proton 3
            Proton 4

    Subsystems
        Electron 1
        Electron 2
        Electron 3
        Electron 4

Methine.Molecule (CH)
    inherits Molecule (a 3d Outer System)

    Methine.Controller
        inherits Controller (a 3d Central System)
   
       Star (current state of Methine.Controller)
      
       Subsystems
           (as necessary)

    Subsystems
        Carbon.Atom
        Hydrogen.Atom

triMethine.Molecule
    inherits Molecule (a 3d Outer System)

    triMethine.Controller
        inherits Controller (a 3d Central System)
   
        Star (current state of triMethine.Controller)
      
        Subsystems
           (as necessary)

    Subsystems
        Methine.Molecule 1
        Methine.Molecule 2
        Methine.Molecule 3

        (electrons can be either all Up or all Down)

Benzene.Molecule
    inherits Molecule (a 3d Outer System)

    Benzene.Controller
         inherits Controller (a 3d Central System)
   
         Star (current state of Benzene.Controller)
      
        Subsystems
            (as necessary)

    Subsystems
        triMethine.Molecule (Up version)
        triMethine.Molecule (Down version)
Title: Re: How do we measure the energy of a photon?
Post by: Bored chemist on 07/03/2020 11:33:59
https://scienceblog.com/514613/solving-a-mystery-in-126-dimensions/?utm_source=feedburner&utm_medium=email&utm_campaign=Feed%3A+scienceblogrssfeed+%28ScienceBlog.com%29

Here is a Benzene molecule (C6h6) built with 3d Abstractions technology.

Carbon.Atom (C)
    inherits Atom (a 3d Outer System)

    Carbon.Ion
         inherits Ion (a 3d Central System)
   
        Star (current state of Carbon.Ion)
      
        Subsystems
            Proton 1 (a Proton is effectively a Hydrogen Ion)
            Proton 2
            Proton 3
            Proton 4

    Subsystems
        Electron 1
        Electron 2
        Electron 3
        Electron 4

Methine.Molecule (CH)
    inherits Molecule (a 3d Outer System)

    Methine.Controller
        inherits Controller (a 3d Central System)
   
       Star (current state of Methine.Controller)
      
       Subsystems
           (as necessary)

    Subsystems
        Carbon.Atom
        Hydrogen.Atom

triMethine.Molecule
    inherits Molecule (a 3d Outer System)

    triMethine.Controller
        inherits Controller (a 3d Central System)
   
        Star (current state of triMethine.Controller)
      
        Subsystems
           (as necessary)

    Subsystems
        Methine.Molecule 1
        Methine.Molecule 2
        Methine.Molecule 3

        (electrons can be either all Up or all Down)

Benzene.Molecule
    inherits Molecule (a 3d Outer System)

    Benzene.Controller
         inherits Controller (a 3d Central System)
   
         Star (current state of Benzene.Controller)
      
        Subsystems
       (as necessary)

    Subsystems
        triMethine.Molecule  (Up version)
        triMethine.Molecule (down version)

What purpose does that serve?
Can it tell you the melting point of benzene, or what colour it is or...anything at all?
Title: Re: How do we measure the energy of a photon?
Post by: mxplxxx on 09/03/2020 12:49:42
Gravity has long been problematic in physics and theories on it abound but none are completely satisfactory. Below is 3d Abstractions' take on gravity. I am hopeful it will explain the force once and for all.

Gravity in a 3d System is a force that acts from the boundary of System's parent to the boundary of the System itself. It is caused by the increase in density of the space displaced by a 3d System. This creates a push force that  acts in a direction that goes from the boundary of the System's parent to the mid point of the System. The force decreases proportionally to the distance from the System's boundary.

Compressing the space/energy of a system does not change the value of energy but it does change the time to discharge the energy. In other words it changes the Power of the system. As the System expands, so its power decreases and the power of its parent increases. In the same way, as a System contracts so it power increases and the power of its parent decreases.  Maybe P=e/t where e is an Energy constant equivalent to h in E=h/t? If this is the case, then it is also very likely that the energy of all Systems is the same. It also explains how the energy of a photon varies i.e. its is not energy that is varying but power. Makes a lot more sense than current particle theory that has an infinite number of different types of photon existing.

The gravity at a particular point in a 3d System is the sum of all gravitational forces at that point from all 3d subSystems.

The force acting on the 3d System boundary will compress the System inwards. Over time, the System will be compressed to the point where it becomes a black hole. Recreating the System after this time will involve releasing its pent-up spacial energy. This expansion creates gravity in its parent. Thus, creation is a a never-ending process of expansion and contraction of 3d Systems. Expansion creates gravity which creates contraction. Thanks to Systems, the universe is simultaneously expanding and contracting in a balanced state of energy.

The spacial energy of a 3d System is the energy contained in the space the System occupies. Thus the gravity of a system will be related to the spacial energy of the System and the density of the space. In fact, it is likely that time, space and energy are just manifestations of the same underlying object type (or Class in computer science). In computer science terms, they are polymorphic and therefore operate as "black boxes". More on this soon in a separate post.

This theory assumes that 3d Systems "own"  their own space. The System can be moving relative to other Systems, but everything inside it is unaware of this movement. This is the  essence of Relativity.

Support for these propositions may be found in the differing values of Hubble constants:

https://www.vice.com/en_us/article/4agbjn/we-actually-live-inside-a-huge-bubble-in-space-physicist-proposes
Title: Re: How do we measure the energy of a photon?
Post by: mxplxxx on 12/03/2020 01:39:31
https://scienceblog.com/514693/in-the-filaments-of-slime-mold-astronomers-see-the-universe/?utm_source=feedburner&utm_medium=email&utm_campaign=Feed%3A+scienceblogrssfeed+%28ScienceBlog.com%29

The similarity between the slime mould and galaxies can be explained if both have the same underlying base 3d System, as I have explained previously in this post.
Title: Re: How do we measure the energy of a photon?
Post by: Bored chemist on 13/03/2020 16:27:02
The similarity between the slime mould and galaxies can be explained if both have the same underlying base 3d System, as I have explained previously in this post.
You have yet to actually explain anything. You have just posted word salad.

You also seem determined to preach, rather than discuss.
You refuse to answer simple questions.
That puts you in breach of the rules so... once again




https://scienceblog.com/514613/solving-a-mystery-in-126-dimensions/?utm_source=feedburner&utm_medium=email&utm_campaign=Feed%3A+scienceblogrssfeed+%28ScienceBlog.com%29

Here is a Benzene molecule (C6h6) built with 3d Abstractions technology.

Carbon.Atom (C)
    inherits Atom (a 3d Outer System)

    Carbon.Ion
         inherits Ion (a 3d Central System)
   
        Star (current state of Carbon.Ion)
      
        Subsystems
            Proton 1 (a Proton is effectively a Hydrogen Ion)
            Proton 2
            Proton 3
            Proton 4

    Subsystems
        Electron 1
        Electron 2
        Electron 3
        Electron 4

Methine.Molecule (CH)
    inherits Molecule (a 3d Outer System)

    Methine.Controller
        inherits Controller (a 3d Central System)
   
       Star (current state of Methine.Controller)
      
       Subsystems
           (as necessary)

    Subsystems
        Carbon.Atom
        Hydrogen.Atom

triMethine.Molecule
    inherits Molecule (a 3d Outer System)

    triMethine.Controller
        inherits Controller (a 3d Central System)
   
        Star (current state of triMethine.Controller)
      
        Subsystems
           (as necessary)

    Subsystems
        Methine.Molecule 1
        Methine.Molecule 2
        Methine.Molecule 3

        (electrons can be either all Up or all Down)

Benzene.Molecule
    inherits Molecule (a 3d Outer System)

    Benzene.Controller
         inherits Controller (a 3d Central System)
   
         Star (current state of Benzene.Controller)
      
        Subsystems
       (as necessary)

    Subsystems
        triMethine.Molecule  (Up version)
        triMethine.Molecule (down version)

What purpose does that serve?
Can it tell you the melting point of benzene, or what colour it is or...anything at all?
IMHO I think ...
What does the H stand for?










Are you going to answer this?
Title: Re: How do we measure the energy of a photon?
Post by: mxplxxx on 30/03/2020 14:15:16
I have explained how the universe is comprised of Systems and Systems are arranged into types of neural networks. That is, the whole universe is intelligent - at all levels. One of these Systems will be the Earth System. We, and all life on Earth, will be subsystems (or satellite) of the Earth System. We will be intimately connected with the health of the Earth System.

It is totally obvious that Humans are bad news for the Earth and, seemingly, incapable of actually reversing our ways. Given the Earth is a 3d System, it has the intelligence to do something about this situation. A virus like COVID-19 virus will dramatically reduce the population, remove people in poor health (i.e unfit people) and give the whole world a chance to reassess its direction. COVID-19 is a mutated form of SARS that is able to infect humans in a way no other virus before has been capable of - see https://www.theage.com.au/national/the-perfect-virus-two-gene-tweaks-that-turned-covid-19-into-a-killer-20200327-p54elo.html (https://www.theage.com.au/national/the-perfect-virus-two-gene-tweaks-that-turned-covid-19-into-a-killer-20200327-p54elo.html)

It is entirely possible that COVID-19 is EARTH's way of making itself healthier.

COVID-19, like most other entities in Reality, is a 3d System. It has an outer shell, a central system (a nucleus) and subsystems. It is no wonder then that, once it gets entry to a Cell (another type of _3d System), it is easily able to interact with the cell. 

Title: Re: How do we measure the energy of a photon?
Post by: Bored chemist on 30/03/2020 14:34:04
Ihave explained how the universe is comprised of Systems
No.
You have explained how you imagine that the world is so composed.
That's no reason to believe that it actually is.


A virus like COVID-19 virus will dramatically reduce the population,

It depends what you mean by "dramatically".
A few percent- mainly the elderly who are past reproductive age.
I doubt that there will be a single day where there are fewer people at the end of it than there were at the start.

IMHO I think ...
What does the H stand for?










Are you going to answer this?
Title: Re: How do we measure the energy of a photon?
Post by: mxplxxx on 01/04/2020 02:26:47
Here is the skeleton Visual Basic code for a 3dSystem. This allows you to create open-ended systems that are infinite in scope.

Code: [Select]
Namespace Library

    <Serializable()>
    Public MustInherit Class _3dSystem

        ' a 3dSystem is the base building block of the Universe. It consists of an outer Shell containing a Star (central/inner) System and Shells radiating out from the Star.

        Public Property Parent As _3dSystem
        Public Overridable Property DataStore As DataStore.System               ' persistence
        Public Overridable Property Star As _3dStar                            ' central/inner system containing state data (e.g. sun in solar system)
        Public Overridable Property Shells As sortedlist(Of String, _3dShell)  ' shells in order possibly containing (sub) _3dSystems

        ' create a new base system object

        Public Sub New(Parent As _3dSystem, Star As _3dStar, Optional Shells As sortedlist(Of String, _3dShell) = Nothing, Optional DataStore As DataStore.System = Nothing)

            Me.Parent = Parent

            Me.Star = _Star

            Me.Shells = If(Shells Is Nothing, New SortedList(Of String, Shell), Shells)

            If DataStore Is Nothing Then
                Me.DataStore = If(My.App Is Nothing, Nothing, My.App.DataStore)
            Else
                Me.DataStore = DataStore
            End If

        End Sub

    End Class

    Public MustInherit Class _3dShell

        Public Overridable Property Systems As sortedlist(Of String, _3dSystem)

        ' get a group of systems having the same parent

        Public ReadOnly Property Group(Parent As _3dSystem) As List(Of _3dSystem)
            Get
                Dim group_ As New List(Of _3dSystem)

                For Each system_ As _3dSystem In Systems.Values

                    If system_.Parent Is Parent Then
                        group_.Add(system_)
                    End If
                Next

                Return group_
            End Get
        End Property

    End Class

    Public MustInherit Class _3dStar
        Inherits _3dSystem

        Public Overridable Property Version As Integer
        Public Property LastUpdated As DateTime

    End Class

End Namespace
Title: Re: How do we measure the energy of a photon?
Post by: Bored chemist on 01/04/2020 09:23:27
You have explained how you imagine that the world is so composed.
That's no reason to believe that it actually is.
Title: Re: How do we measure the energy of a photon?
Post by: ron123456 on 01/04/2020 22:54:01
Dismissing quantum jump, just think of the electron spiralling down to the next lower energy level by spiralling down around the nucleus. This would produce a diminishing wavetrain from high energy level to low (a photon). For Hydrogen, this could easily be calculated from the Bohr model.
Title: Re: How do we measure the energy of a photon?
Post by: alancalverd on 01/04/2020 23:15:50
Problem is that the Bohr model is so obviously incorrect that any calculation based on it is invalid.
Title: Re: How do we measure the energy of a photon?
Post by: ron123456 on 01/04/2020 23:51:44
Really?....just extrapolate mathematically to 3D for Hydrogen and only Hydrogen for the wave train…...simply can be explained with antenna theory and eliminate the quantum jump!
Title: Re: How do we measure the energy of a photon?
Post by: mxplxxx on 02/04/2020 13:09:12
3dAbstractions is a computer system. You can do whatever you like, including moving an electron between shells/levels on either the receipt of an event (a boson) or the expiry of a timer within a 3dSystem - providing the action obeys any constraints associated with the event (or natural laws in the case of Reality).

Events will be generated at a 3dSystem boundary via receptors (as you would expect in a neural network). I am currently working on this. There is nothing random about events in a 3dSystem. Everything has a purpose (even random events!). Ditto for Reality. Everything is based on things that worked for a System in the past, and random events are probably generated based on the possibility of them creating beneficial mutations.

e.g. in visual basic:

Code: [Select]
   ' Raising system (Photon as Photon.System in this case, Event Happened(Frequency as Double).

   Raisevent Happened(Frequency:=Frequency)

   ' Receiving System (sender = Photon, e = Frequency))

   Private Sub QuantumLeap(sender As Object, e As EventArgs) Handles Photon.Happened

        ' move an electron from one shell to another (just an example - operation would be generalized based on frequency of photon in Real deal

        me.Shells(0).Electrons(1).Move(MoveTo:=me.Shells(0).Electrons(0))
Title: Re: How do we measure the energy of a photon?
Post by: mxplxxx on 04/04/2020 06:08:03
Events in computer science record the change of state of an object at a point in time. This is also the case for 3dAbstractions. 3dEvents are processed by 3dSystems in an identical manner from system to system. That is they are scaled up and down along with the systems themselves. The processing of events in the 3dUniverse system occurs in an identical manner to the processing of events in a 3DAtom system. The recursive nature of this means that events can be designed and programmed (relatively) independently of other systems, with a minimum of effort and executed flawlessly in most cases. It also means that processing of events in a 3dSystem naturally occurs in an abstraction hierarchy without any effort from a developer..

The following is a narrative that explains how events work in a 3dSystem:

1. Events only occur in a 3dStar (e.g. sun system in a solar system) type of system (because a 3dStar system contains the state of the main 3dSystem (i.e. solar system in our example))

2. An event raised in the 3dStar System is handled by an Event Handler (a standard Visual Basic function) method in the Parent 3dSystem (the solar system in our example)

3. This method will typically:

   a. raise the event further, thus making it available in the parent system of the outer/main system (a galaxy system in the case of the a solar system), and/or
   b. do some other processing with the main system (solar system) or in another referenced system.

Case 3a is the means by which a subsystem (or referenced system e.g. the earth system in the case of the solar system) communicates with its parent system. In this case, the parent (the solar system in our example) will typically change the state of the 3dStar system (the sun in our example) based on the nature of the event from the subsystem. The same may be true of reality. There is a lot of evidence that a photon does not contain energy (e.g. it is not affected by gravity). Changing the state of a system does not not necessarily involve energy. A system can change state based on the frequency of a photon only. 

Events in 3dSystems are types of 3dSystems but with no capabilities to change state. They correspond to bosons in Reality. A boson is basically the amount a system has changed state by at some time in the past. Photons in a 3dSystem will be raised from system to system until one system accepts and processes the photon. Other bosons will be reabsorbed by the emitting 3dStar object (I am guessing). I.e bosons/events can only travel upwards in an abstraction hierarchy (travelling down is done via method calls and I am not sure how Reality achieves this).

Code for these actions was supplied in my previous post.

 
Title: Re: How do we measure the energy of a photon?
Post by: Bored chemist on 04/04/2020 10:48:51
IMHO I think ...
What does the H stand for?










Are you going to answer this?
Title: Re: How do we measure the energy of a photon?
Post by: mxplxxx on 08/04/2020 02:45:47
Here is the latest on a photon from a 3dAbstractions perspective

3dPhotons provide information to 3dSystems about the energy involved in collisions.

3dPhoton System
   
    3dPhoton Ion (a _3dStar/central System)
   
Code: [Select]
        WhenHappened as DateTime
WhereHappened as Double
Spin as integer
        Frequency as Double ' frequency of photon's Spin

    Subsystems

        E Gymbal
        M Gymbal
   
    Gymbals provide stability to the photon as per a gyroscope.

For example, two fermion systems A and B of energy EA and EB respectively collide. The resulting two systems A and B have energy EX and EY respectively where EX + EY = EA + EB. Two 3dPhoton systems, PA and PB, will result from this collision. PA will have (from E = hf) frequency (EX - EA)/h. Neither 3dPhoton system contains any energy. A system that processes photon PA will recognize (via E = hf) that system A has lost/gained EX - EA amount of energy in the collision.

That is, a 3dPhoton contains information about the result of a collision, but no energy from the collision (thus answering the question posed by the topic). As per the conservation of energy, all of the energy involved in the collision stays with the two fermion systems involved. Energy and Time are closely related. No Time: No Energy. Because it travels at SOL, a photon does not experience time. It exists in an eternal present (ditto for all bosons). It is likely a Photon is a boson that is adapted for long distance travel with maximum stability (although as "shifts" (i.e. changes in frequency) show,  perfect stability/integrity of information is not attainable). The universe loses information (a boson function ) over time but not energy (a fermion function).

Energy changes momentum, Frequency changes State. Energy is not required to change State. Frequency is an attribute of Spin, an entity in Physics that appears to be conceptual (versus "real") in nature only.     
Title: Re: How do we measure the energy of a photon?
Post by: Bored chemist on 08/04/2020 12:02:49
That is, a 3dPhoton contains information about the result of a collision, but no energy from the collision.
A thing with no energy is not a photon.
Title: Re: How do we measure the energy of a photon?
Post by: mxplxxx on 14/04/2020 02:48:02
State in the central (_3dStar) system of a computer App is mostly implemented by straight-forward computer programming constructs; properties, method (function) calls and event declarations/handlers. For example, in the case of a Door object, a Closed Boolean plus an Open event. In more complex cases, Hierarchical Finite State Machines (HFSM) are used. The EFTPOS pinpad superstate diagram with this post is an example of a HFSM.

The _3dState system is a type of _3dSystem that is a means of implementing a HFSM. It is basically a _3dSystem with SubStates substituted for SubSystems. SubStates are concurrent states. Below is the basics the pinpad HFSM.

Code: [Select]
Pinpad.System
    inherits _3dSystem

    PinPad.Star
        inherits _3dStar

        Pinpad.HFSM
            Inherits _3dState
 
            Property PinpadState as string (e.g. "Existing")

            Property PinpadSubStates as sortedlist(of string, string) (e.g. Test/Production, Active/Inactive)
A pinpad actually does not have a central system. This is similar to reality where the centre of a system is a black hole. I am assuming the overall state of a system with  a black hole resides in the black hole itself.

A state in a computer system is actually a Name ; i.e. a string; i.e. a series of elementary 0/1's (digits). The same seems true in Reality where a state is likely to be a series of elementary fermions (.e.g. quarks). So, events (bosons) interact with state (fermions) to give new states.

State is a necessary part of Reality which is infinite in nature (think Zeno's paradox). Systems in Reality are timestamped with the time of the next state change which is an infinite number. In this way, State changes  proceed infinitely quickly (SOL is a limitation of Reality), a state change at a time and state changes are timeless i.e. occur immediately. A digital computer works similarly. State turns an infinite number of points into something "real" (and manageable). In fact, a major part of computer software design is managing possibilities via assigning groups of them to a state.

Of course, State and Time are intimately related. It is impossible to perceive time as passing unless state changes are taking place. A _3dState System can progress a State through Time starting at the top superstate level. More on this later; it is intimately related to how Realty works.

PS. State is really, really hard to get clued up on. It took me many years in computer science to gain clarity around it.   

(https://www.thenakedscientists.com/forum/proxy.php?request=http%3A%2F%2F&hash=3cd4f4119996b42d10f5ed9eb0e8d712)
Title: Re: How do we measure the energy of a photon?
Post by: mxplxxx on 19/04/2020 00:32:15
A _3dSystem implements what is called in computer science a Model View Controller Persistence (MVCP) pattern. A pattern is a way of computing that is best practice.

In a _3dSystem, a MVCP is implemented as follows:

1. Model

   This is the database that a system implements. It consists of the a _3dStar (Central/Inner system) and Subsystems.

2. View.

   This is the visible part of the system. It consists of a a partial Class that inherits a UserControl. A UserControl is a canvas on which other controls (such as a Button) can be displayed.

3. Controller.

    This is the part of a system that controls the flow of Data between the Model, View and Persistence parts. In a _3dSystem it is the the system itself that is the Controller.

4. Persistence

    This is the part of a system that records the past history of the system (i.e. its memory). A special system, _3dDataStore, handles this aspect of the system. 

In a _3dApp, top level systems Display, Data and Datastore are specialist View, Model and Persistence _3dSystems. The Display system is the top level for screen output. The Data system is the top level for the App's in-memory Database. The Datastore system is the top level for the App's permanent memory. 

Does Reality implement the MVCP pattern? Possibly. It would seem that bosons interacting with a system provide a means to make the past "visible", thus implementing a View. The Model is the hierarchy of systems and central systems that make up the universe. The controller is the outer level of these systems (e.g. a solar system). What we don't know is how/where persistence happens. It may be that the mysterious dark matter is where persistence happens or it may be that persistence happens only in very intelligent systems, like our brain.

Missing from this picture is the future. Maybe we could add a Possibilities part to the MVCP pattern for Quantum Computing?  Hierarchical Finite State Machines (HFSMs) contain all possible states of a System. The Substates in a  _3dState HFSM are a mix of possible and real (concurrent) states with the 3dState system programmed to choose between them when an appropriate event is received/occurs. This is the way 3d Abstractions implements Quantum Probabilities. The vast amount of possibilities present in a system are handled/reduced in computer programs by the judicious choice of abstractions when designing the program. Note that this use of abstractions will introduce a degree of UNCERTAINTY into the computation. Something that would have made Heisenberg smile, I am sure

In fact, the only way to live in a universe of a near-infinite number of points is via abstraction and living with the uncertainty thus caused. We, humans, live at a certain level of abstraction in Reality and thus have to deal with a fair amount of uncertainty. Our cells are programmed to do so but mistakes happen and eventually an accumulation of mistakes likely causes our death. 
Title: Re: How do we measure the energy of a photon?
Post by: mxplxxx on 27/04/2020 03:23:44
It would seem that bosons interacting with a system provide a means to make the past "visible"
According to current physics theory, a photon travels at a constant speed of SOL. It may be more logical to think of a photon as stationary (its gyroscopic nature will keep it that way) and the 3dSystem that the photon is part of as travelling at SOL relative to it (this is the rate at which the present passes). A stationary photon is timeless. A 3dSystem ages. A stationary photon's location also identifies where the event happened that caused the photon's current state.

A stationary photon can enable multiple viewers to "see" an event. This is in contrast to the current theory that has the photon destroyed by the viewing process. This aligns more closely with computer science where a single event can have multiple viewers ("handlers").

It is more than possible that the universe is a "sea" of photons that give a picture of how the energy distribution of the universe has changed over time. This way, photons do not need to be created (much like the digits in a digital computer). In fact, because bosons can coexist (as per Pauli Exclusion principle), it may be possible, that photons can be stacked, one on top of the other. This fascinating possibility may allow the past to be recreated. 3d Abstractions software minimizes dependencies amongst 3dSystems by creating data from the past states of the systems as necessary, rather than maintaining the new state of a system into the future. Could Reality be operating this way? Very likely in my opinion, maybe even from the start of time given Reality never appears to "crash". If so, it is very likely that fermions are conceptual only in nature and created temporarily as necessary - from "nothing".

This mechanism is consistent with my postulate that bosons are about State and fermions are about Motion/Time.

If a 3dSystem moves at SOL relative to a photon, then it will possibly do so in "leaps" that are SOL/t in length where t is time as per E=h/t and E is the energy of the system. Relativity may come into this picture but I am not too sure how at present. See also Zeno's paradox that makes continuous motion unlikely.
Title: Re: How do we measure the energy of a photon?
Post by: mxplxxx on 06/05/2020 16:07:30
It is possible a photon is a "shadow" of the 3dSystem it represents. That  makes a photon generated by a 3dSystem the same size as the system which means it can easily interact with the 3dSystem's parent and siblings. It is consistent with my theory that 3dSystems are scaled-down versions of their parents. 

Actually, a "shadow" system would be a 3dSystem in its own right but contain a 3dCentralSystem that is the "shadow" of a real 3dSystem. e.g. a Photon.System would be a normal 3dSystem with 3dGymbal subsystems as previously discussed but would contain a "shadow" 3dStar system that represents the system that created the photon.

e.g a photon created by the sun would have the following structure:

3dShadow
    inherits 3dStar

Photon
    System
        inherits 3dSystem
       
       property Star as Star (created via Sun.Star)

       property Subsystems as sorted list of 3dSystem
            E_Gymbal
            M_Gymbal

    Star
         inherits 3dShadow
               
A photon going from one 3dSystem to another would have to be scaled-up/down is some way, probably at the system boundary. Actually, the 3dSystem goes (moves) rather than the photon and chances are the photon will be too small or too large to be even noticed by the 3dSystem, or the photon will not be a type that the 3dSystem is interested in.

This also gives rise to an interesting possible that a fermion is just a "real" photon.

w and z bosons will also have "shadows" . Note that a Higgs boson fits neatly into this scheme and will occupy the same space as its parent system.

If, as suggested in a previous post (https://www.thenakedscientists.com/forum/index.php?topic=75960.msg601611#msg601611) photons are absolute/stationary and can be stacked one on top of the other, the "shadow" aspect of a photon can allow space/time to exist and even allow time travel if photons can exist as possibilities.

Title: Re: How do we measure the energy of a photon?
Post by: Bobolink on 06/05/2020 18:08:01
I was wondering how in the world is this thread still active?  Now I see it is that it is just mxplxxx talking to himself.  Mystery solved.
Title: Re: How do we measure the energy of a photon?
Post by: mxplxxx on 08/05/2020 11:43:56
As discussed recently, Photons and other bosons "live" in the present. Photons contain information about the past, but live in the present. What else "lives" in the present? Nothing I suspect except space itself (which is timeless). Fermions "travel" at a constant SOL meaning they are never stationary - a precursor for being present it would seem. A fermion encountering a photon that will cause it to change state will do so instantaneously - this feature of state in reality.

So, it would seem, all things being equal, we never really "exist". An interesting thought! Maybe quantum possibilities are necessary for Reality to come into existence.           
Title: Re: How do we measure the energy of a photon?
Post by: Bobolink on 08/05/2020 11:58:13
As discussed recently, Photons and other bosons "live" in the present. Photons contain information about the past, but live in the present. What else "lives" in the present? Nothing I suspect except space itself (which is timeless). Fermions "travel" at a constant SOL meaning they are never stationary - a precursor for being present it would seem. A fermion encountering a photon that will cause it to change state will do so instantaneously - this feature of state in reality.

So, it would seem we never really "exist". An interesting thought! 
Have you been eating wild mushrooms?  I think you might have accidentally gotten a hold of psilocybin.
I live in the present, heck I even have information from the past.
Title: Re: How do we measure the energy of a photon?
Post by: mxplxxx on 08/05/2020 12:10:33
I live in the present
Yet everything you experience is a past event. If you do live in the present, what and how long is the present?  We don't have an answer to that, so how can you be so sure you live in the present? Chances are you experiencing something initially in your short-term memory, in which case you are experiencing the past. Maybe your awareness is in the present but, given we don't know what awareness is, this statement is pretty meaningless anyway.
Title: Re: How do we measure the energy of a photon?
Post by: Bobolink on 08/05/2020 12:50:42
I live in the present
Yet everything you experience is a past event. If you do live in the present, what and how long is the present?  We don't have an answer to that, so how can you be so sure you live in the present? Chances are you experiencing something initially in your short-term memory, in which case you are experiencing the past. Maybe your awareness is in the present but, given we don't know what awareness is, this statement is pretty meaningless anyway.
So in other words quantum fluctuations harmonizing in resonance with Hilbert space result in a Alexander Hamiltonian.  Hey, I can write gibberish like you!
Bye-bye...
Title: Re: How do we measure the energy of a photon?
Post by: mxplxxx on 08/05/2020 14:41:06
quantum fluctuations harmonizing in resonance with Hilbert space
Rigged Hilbert Space Resonances and Time Asymmetric Quantum Mechanics https://arxiv.org/abs/quant-ph/9909081
If you don't have the domain knowledge, you will likely regard discussions about the domain as gibberish. Or, as Crocodile Dundee would say, jibber-jabber.   
Title: Re: How do we measure the energy of a photon?
Post by: mxplxxx on 11/05/2020 01:59:44
All systems in Reality and  all components of systems likely have purposes. The purpose of a photon is to provide current systems with information about the past. One of the ways it may do this is via the magnetic field which seems to surround most systems. A photon intersecting with a magnetic field may induce a vibration in a "line" in the magnetic field. The vibration will have the same frequency and wavelength as the photon. The vibration then travels via the "line" to the "north pole" (i.e top) of the system and thence to its centre (which is often a black hole) via child/parent system relationships. At this point the original system becomes aware of a past event and may react in some way if the event is of consequence to it.

The reaction will occur firstly via the subsystems of the inner/central system. Asynchronously i.e. multiple systems will react at once. The reaction may stop at any point. It may also "bounce" at any stage in the process meaning an event may start travelling up a system's HFSM rather than down, giving rise to a process that can occur indefinitely (like life!). Bounced events are likely to travel to the south pole of the system (i.e bottom) to the "react" events meaning magnetic field lines operate in two directions.

If at any point in this process the top level of the inner system changes state, the reaction may continue (asynchronously) via the subsystems of the outer system (i.e. via parent/child relationships). Photons and w/z bosons will likely be involved in this process with Higgs bosons being exchanged where energy changes are involved.

Eventually the reaction will stop, possibly at the outer limits of the system with, if a change of state has occurred, a system photon being "created" for processing by the original system's parent.

Thus, it would seem, the purpose of magnetic fields is to transport event notifications between systems.

It is likely that changes of momentum will occur in the systems involved in the reaction process. In the end though, as per the conservation of momentum law, no overall change in the momentum of the original system can occur.

Chances are the process is geared so that the photon will react with only one "line" of the magnetic field and not be destroyed by the process. Thus, a photon may be eternal in nature. 

This process is very similar to the computer science process whereby events are handled in computer Apps. A react event in reality is equivalent to a function Call in an App, and a bounce event in Reality is equivalent to  "raised" event in an App.
Title: Re: How do we measure the energy of a photon?
Post by: hamdani yusuf on 11/05/2020 04:49:40
The vibration then travels via the "line" to the "pole" of the system and thence to its centre (which is often a black hole) via child/parent system relationships  at which point the original system becomes aware of a past event and may react in some way if the event is of consequence to it.
How often do you find black holes in the center of a system?
Title: Re: How do we measure the energy of a photon?
Post by: mxplxxx on 11/05/2020 05:46:25
The vibration then travels via the "line" to the "pole" of the system and thence to its centre (which is often a black hole) via child/parent system relationships  at which point the original system becomes aware of a past event and may react in some way if the event is of consequence to it.
How often do you find black holes in the center of a system?
https://science.nasa.gov/astrophysics/focus-areas/black-holes
Title: Re: How do we measure the energy of a photon?
Post by: Bored chemist on 16/05/2020 12:55:06
Also, the number of cases  COVID-19 seems to parallel the number of 5G installations in major cities.
It probably parallels the number of tins of dog food purchased too- for the same reason.

.
Nowhere is it mentioned that 5G signals are transmitted by Ultra-Short Lasers (lots of coherent photons packed together) of 30 Watts power.
Nowhere is it mentioned that 5G signals are transmitted by carrier pigeon.

That's because they don't use pigeons (or lasers) to broadcast the signal.
Lasers are used, but, of course, they are confined in fibre optic cables.

It's not helping anyone when you spread silly scare stories like this.
Don't do it again
Title: Re: How do we measure the energy of a photon?
Post by: mxplxxx on 17/05/2020 11:15:43
Recently I claimed that 5G was powered by lasers. This was dues to a headline by https://world-of-photonics.com/en/newsroom/photonics-industry-portal/technologies/ultrashort-pulse-lasers-5g/ that claimed that "Ultrashort pulse lasers pave the way for 5G". As far as  I can work out this is just a work in progress for the company and the headline is just plain wrong. 5G is powered by radio waves and relatively low frequency ones at that. Chances are it is a benign technology but the lack of testing by Telecoms is disturbing.
Title: Re: How do we measure the energy of a photon?
Post by: Bored chemist on 17/05/2020 11:45:51
the lack of testing by Telecoms is disturbing.

Just how is that going to work?
Do you think that the telecoms companies have cell biology labs or animal testing facilities?
Why would they have them?

And, if they did, would you be sure that the results were not influenced by the fact that the telecoms companies have a very clear interest in saying the tech is safe?
Wouldn't it be better if the testing was done independently?
(Perhaps paid for by a government which would pick up the bill for any health problems if the technology is not safe)

So, maybe the last thing we want is

testing by Telecoms
Title: Re: How do we measure the energy of a photon?
Post by: mxplxxx on 17/05/2020 13:12:36
the lack of testing by Telecoms is disturbing.

Just how is that going to work?
Do you think that the telecoms companies have cell biology labs or animal testing facilities?
Why would they have them?

And, if they did, would you be sure that the results were not influenced by the fact that the telecoms companies have a very clear interest in saying the tech is safe?
Wouldn't it be better if the testing was done independently?
(Perhaps paid for by a government which would pick up the bill for any health problems if the technology is not safe)

So, maybe the last thing we want is

testing by Telecoms

Fair enough.
Title: Re: How do we measure the energy of a photon?
Post by: mxplxxx on 21/05/2020 07:44:41
It is no wonder Photons cannot be directly measured. Like all bosons except possible Higgs particles they are not "observable". They are the equivalent of Events in computer science which also are not "observable". In computer science, we can raise an event and we can handle an event but we cannot otherwise access it.

It is likely that bosons are not observable because they do not "contain" energy (with the exception of HIggs particles). Energy seems to be required for distance, space and time to exist and may in fact be just different forms of these variables. In computer science, energy would be a polymorphic object.       

Higgs particles seem to straddle the space between a boson and a fermion, probably winking in and out of visibility depending on whether the associated fermion is being "observed" or not. No wonder they are hard to find:). Or maybe they are really fermions (see below).

In simulating a System, the Energy Property will be a Double. The Energy Property of a boson will equal Nothing.The Energy Property of a fermion will be a positive value > zero. It seems impossible for a boson to have a positive Energy Property since bosons would repel each other which is not allowed according to the Fermi Exclusion Principle. There are a number of other reasons why a boson cannot a positive Energy Property not the least being that a Photon travels at SOL and M=E/C2 and M would therefore be infinite for a Photon.

A boson is timeless,  "lives" in the present and is is the "shadow" of a past system at a particular time in that system's evolution. It has a frequency that represents amount of energy (E=hf) that the system has lost/gained in a past event. This frequency may trigger a distribution change in the energy of a current fermion system but the total amount of energy in the system must remain the same (as per conservation of energy). Energy changes in a system occur via collisions of fermions, not bosons.

On this basis, a Higgs particle will be a fermion, which makes a LOT more sense than the current theory.

In other words Spin defines a subset of a Boson or a Fermion. The primary differentiation between a boson and a fermion is that a boson does not "contain" energy and a fermion does "contain" energy.

So:

Class System
      Property Energy as Double = Nothing
      Property Frequency as Long Integer = Nothing
      Property Spin as Single = Nothing

Class Fermion
      Inherits System

      Property Overrides Spin as Single = s

 Class Higgs
     Inherits Fermion

    Property Overrides Energy as Double = E
    Property Overrides Frequency as Long Integer = Energy/h

Class Boson
    Inherits System     

    Property Overrides Spin as Single = 1
    Property Overrides Frequency as Long Integer = f

The big difference between Photons and other Bosons is that Photons can "travel" anywhere. All other Bosons "belong" to the 3dSystem that "created" them. Gluons will have subsystems that are the 3dSystems (2) that they "glue". W and Z bosons supply energy (via their frequency) to the 3dSystem that created them.
Title: Re: How do we measure the energy of a photon?
Post by: Bored chemist on 21/05/2020 11:13:10
It is no wonder Photons cannot be directly measured.
They can.
https://en.wikipedia.org/wiki/Gamma_spectroscopy
https://en.wikipedia.org/wiki/Photon_counting

Like all bosons except possible Higgs particles they are not "observable"
Carbon atoms are bosons.
They have been observed.

They are the equivalent of Events in computer science which also are not "observable". In computer science, we can raise an event and we can handle an event but we cannot otherwise access it.
That can not be reelevant, ince we can, in fact, observe bosons.
It is likely that bosons are not observable because they do not "contain" energy
It isn't likely that they are unobservable.
Because, they can, of course, be observed.
Higgs particles seem to straddle the space between a boson and a fermion,
Higgs bosons aer, obviously bosons.
Particles either are, or are not bosons.
It's not possible to be both.
Title: Re: How do we measure the energy of a photon?
Post by: mxplxxx on 26/05/2020 22:59:04
According to current physics theory, a photon travels at a constant speed of SOL. It may be more logical to think of a photon as stationary (its gyroscopic nature will keep it that way) and the 3dSystem that the photon is part of as travelling at SOL relative to it. A stationary photon is timeless.
It is also possible that, in the same way a photon is stationary, a neutrino may also be stationary. This gives rise to the possibility that a photon and a neutrino go hand-in-hand. The two are created together and together provide information about an "event" in Reality.

It may also be that the tiny amount of mass that is often attributed by physics to a photon may come from an accompanying neutrino.

Or, it may be that a neutrino is a reference to a "real" or "shadow"  particle. i.e., in the case of a photon, a stationary "shadow" particle/photon is created whenever a state change occurs in a particle and a neutrino is created to reference the photon. The mass of the neutrino in this case may hold the "universal id" of the photon (paralleling the "object id." of a reference object in a computer program).

In fact, it may be that all particles have a "Universal Id" meaning all particles have at least a tiny amount of mass.

Title: Re: How do we measure the energy of a photon?
Post by: Kryptid on 26/05/2020 23:31:00
The idea of a photon sitting still while the rest of the Universe moves around it at the speed of light falls apart as soon as you add a second photon moving in a different direction to the first.
Title: Re: How do we measure the energy of a photon?
Post by: mxplxxx on 27/05/2020 01:38:07
The idea of a photon sitting still while the rest of the Universe moves around it at the speed of light falls apart as soon as you add a second photon moving in a different direction to the first.
Thanks for your observation. My point is that Photons (and neutrinos) generally do not move. i.e. a second photon is also stationary. In any case, direction is relative. In the case of 3dAbstractions, a 3DSystem will be moving in a particular direction relative to a particular photon. The only direction of consequence for a photon is the direction of its intrinsic angular momentum. This is an absolute direction.

 From this perspective, a photon (and a neutrino) naturally stays in the present whilst the rest of the universe has to speed to catch up with the present:)
Title: Re: How do we measure the energy of a photon?
Post by: Kryptid on 27/05/2020 05:47:19
My point is that Photons (and neutrinos) generally do not move.

A point at odds with observation. There is no reference frame from which we can observe a photon not moving (at least in a vacuum).

The universe will have changed direction when the second photon is created.

And what if those two photons were created at the same time (for instance, when an electron and positron annihilate)? You realize that the Universe is currently filled with countless photons moving in different directions, don't you?

From this perspective, a photon (and a neutrino) naturally stays in the present whilst the rest of the universe has to speed to catch up with the present:)

This statement makes no sense.
Title: Re: How do we measure the energy of a photon?
Post by: mxplxxx on 27/05/2020 07:23:37
My point is that Photons (and neutrinos) generally do not move.

A point at odds with observation. There is no reference frame from which we can observe a photon not moving (at least in a vacuum).

The universe will have changed direction when the second photon is created.

And what if those two photons were created at the same time (for instance, when an electron and positron annihilate)? You realize that the Universe is currently filled with countless photons moving in different directions, don't you?

From this perspective, a photon (and a neutrino) naturally stays in the present whilst the rest of the universe has to speed to catch up with the present:)

This statement makes no sense.
Sorry, I should have been talking about 3dPhotons and 3dNeutrinos. Reference frames have been replaced by 3dSystems in my theory. 3dSystems is a simulation of Reality, as is Quantum theory. In any case, I make the following points about the photons of Quantum theory.

1. Would a photon not see itself as stationary (as per a Lorenz transformation)? Are you not stationary?
2. Photons cannot be observed.
3. If photons cannot be observed  who is is to say photons can be created simultaneously?
4. For an "observer", the present "happens" at the speed of light.
5. The spin angular momentum of a photon can supply the direction of travel of the photon (i.e. no need for a frame of reference). Not sure of this.
Title: Re: How do we measure the energy of a photon?
Post by: Bored chemist on 27/05/2020 10:23:43
1. Would a photon not see itself as stationary. Are you not stationary ?
Ignoring the philosophical question of what a photon "sees"...
If it sees anything then what it will see is that the journey it takes is compressed by fitzgerald contraction to zero distance.
And it covers that zero distance in zero time.
From the PoV of a photon (if it has one) the rest of the Universe does not exist.
Title: Re: How do we measure the energy of a photon?
Post by: Bored chemist on 27/05/2020 10:28:58
Photons cannot be observed.
Yes they can.
In particular, if a gamma ray photon hits your eye you might actually see the scintillation.
If photons cannot be observed
They can be.
who is is to say photons can be created simultaneously?
Logic.
They are brought into being by the same event. It follows that they happen at the same time.
There's also experimental observation of this - used every day throughout the world in medical imaging.
https://en.wikipedia.org/wiki/Positron_emission_tomography#Emission

Why are you pretending that reality doesn't happen?

Title: Re: How do we measure the energy of a photon?
Post by: Kryptid on 27/05/2020 15:20:21
4. For an "observer", the present "happens" at the speed of light.

That statement makes no sense. Time is not a velocity.

5. The spin angular momentum of a photon can supply the direction of travel of the photon (i.e. no need for a frame of reference). Not sure of this.

I can't make sense of this either.
Title: Re: How do we measure the energy of a photon?
Post by: mxplxxx on 27/05/2020 15:43:26
That statement makes no sense. Time is not a velocity.
A light year is related to velocity. A photon that is currently a light year away will create the present if it interacts with you in a year's time. i.e. the present is proceeding at the speed of light.     

 
Quote from: mxplxxx on Today at 07:23:37
5. The spin angular momentum of a photon can supply the direction of travel of the photon (i.e. no need for a frame of reference). Not sure of this.

I can't make sense of this either.
Not to worry, I am not sure about it myself. Maybe ResearchGate can help: https://www.researchgate.net/publication/327365074_Spin_and_orbital_angular_momentum_of_photons
Title: Re: How do we measure the energy of a photon?
Post by: Kryptid on 27/05/2020 20:14:41
A light year is related to velocity. A photon that is currently a light year away will create the present if it interacts with you in a year's time. i.e. the present is proceeding at the speed of light.     

That still doesn't make sense. Time is not a velocity. Being related to velocity is not sufficient.

Not to worry, I am not sure about it myself. Maybe ResearchGate can help: https://www.researchgate.net/publication/327365074_Spin_and_orbital_angular_momentum_of_photons

What I can't make sense of is why you think the photon's intrinsic angular momentum can supply its direction.
Title: Re: How do we measure the energy of a photon?
Post by: Bored chemist on 27/05/2020 21:29:35
A light year is related to velocity. A photon that is currently a light year away will create the present if it interacts with you in a year's time. i.e. the present is proceeding at the speed of light.
No, because
A light year "Bullet second" is related to velocity. A photon "Bullet " that is currently a second year away will create the present if it interacts with you in a year's second's time. i.e. the present is proceeding at the speed of light. a bullet.
Or a train, if it's important that you don't miss it.
Or an asbestos fibre, if that's what kills you.




What I can't make sense of is why you think the photon's intrinsic angular momentum can supply its direction.
I presume he thinks it travels like a corkscrew.
Title: Re: How do we measure the energy of a photon?
Post by: mxplxxx on 27/05/2020 21:35:14
Quote from: mxplxxx on Today at 15:43:26
A light year is related to velocity. A photon that is currently a light year away will create the present if it interacts with you in a year's time. i.e. the present is proceeding at the speed of light.     

That still doesn't make sense. Time is not a velocity. Being related to velocity is not sufficient.

Quote from: mxplxxx on Today at 15:43:26
Not to worry, I am not sure about it myself. Maybe ResearchGate can help: https://www.researchgate.net/publication/327365074_Spin_and_orbital_angular_momentum_of_photons

What I can't make sense of is why you think the photon's intrinsic angular momentum can supply its direction.
You seem to be telling me you can't make sense of some of my posts. Noted, best I can do sorry.

Title: Re: How do we measure the energy of a photon?
Post by: mxplxxx on 27/05/2020 21:44:08
Quote from: Kryptid on Today at 20:14:41
What I can't make sense of is why you think the photon's intrinsic angular momentum can supply its direction.
I presume he thinks it travels like a corkscrew.
YES - with good reason! See https://phys.org/news/2019-08-corkscrew-photons-spontaneous.html
Title: Re: How do we measure the energy of a photon?
Post by: Bored chemist on 27/05/2020 21:51:29
You seem to be telling me you can't make sense of things. Noted.
No.
He is telling you the things you say do not make sense.
You also forgot to address these problems
Photons cannot be observed.
Yes they can.
In particular, if a gamma ray photon hits your eye you might actually see the scintillation.
If photons cannot be observed
They can be.
who is is to say photons can be created simultaneously?
Logic.
They are brought into being by the same event. It follows that they happen at the same time.
There's also experimental observation of this - used every day throughout the world in medical imaging.
https://en.wikipedia.org/wiki/Positron_emission_tomography#Emission

Why are you pretending that reality doesn't happen?


Title: Re: How do we measure the energy of a photon?
Post by: mxplxxx on 27/05/2020 22:02:23
Quote from: mxplxxx on Today at 15:43:26
A light year is related to velocity. A photon that is currently a light year away will create the present if it interacts with you in a year's time. i.e. the present is proceeding at the speed of light.
No, because
A light year "Bullet second" is related to velocity. A photon "Bullet " that is currently a second year away will create the present if it interacts with you in a year's second's time. i.e. the present is proceeding at the speed of light. a bullet.
Or a train, if it's important that you don't miss it.
Or an asbestos fibre, if that's what kills you.

Cute, but It won't be a bullet or a train or a fiber of asbestos that is interacting with you (you can't interact with an abstraction). It will most likely be a photon interacting. If you still disagree, tell me what is involved in a bullet/you interaction according to particle physics.
Title: Re: How do we measure the energy of a photon?
Post by: Kryptid on 27/05/2020 22:23:24
fermions cannot interact.

Do you even know what a fermion is? This statement suggests that you don't.
Title: Re: How do we measure the energy of a photon?
Post by: Bored chemist on 27/05/2020 22:46:22
fermions cannot interact.
How do you think it looks when you try to tell the grown ups that electrons don't interact?
do you think it is
Cute
?
Title: Re: How do we measure the energy of a photon?
Post by: Bored chemist on 27/05/2020 22:48:07
You also forgot to address these problems
Quote from: Bored chemist on Today at 10:28:58
Quote from: mxplxxx on Today at 07:23:37
Photons cannot be observed.
Yes they can.
In particular, if a gamma ray photon hits your eye you might actually see the scintillation.
Quote from: mxplxxx on Today at 07:23:37
If photons cannot be observed
They can be.
Quote from: mxplxxx on Today at 07:23:37
who is is to say photons can be created simultaneously?
Logic.
They are brought into being by the same event. It follows that they happen at the same time.
There's also experimental observation of this - used every day throughout the world in medical imaging.
https://en.wikipedia.org/wiki/Positron_emission_tomography#Emission

Why are you pretending that reality doesn't happen?

Title: Re: How do we measure the energy of a photon?
Post by: mxplxxx on 27/05/2020 22:49:40
fermions cannot interact.

Do you even know what a fermion is? This statement suggests that you don't.
Dumb post. Do you really think I could attract 44000+ views without knowing basic physics? Can you please elucidate me as to how fermions interact, especially as all fermions obey the Pauli Exclusion principle.

PS A search on google "How do fermions interact" yields no usable results.
Title: Re: How do we measure the energy of a photon?
Post by: Kryptid on 27/05/2020 23:34:50
Do you really think I could attract 44000+ views without knowing basic physics?

The number of views is not directly proportional to the knowledge of the poster.

Can you please elucidate me as to how fermions interact, especially as all fermions obey the Pauli Exclusion principle.

If they didn't interact, then the Pauli exclusion principle wouldn't even exist. One fermion obviously has to be able to interact with the other one, otherwise you could put as many fermions in one place and state as you wanted.
Title: Re: How do we measure the energy of a photon?
Post by: Bored chemist on 28/05/2020 13:12:19
Do you really think I could attract 44000+ views without knowing basic physics?
Yes, I do think that.
That's what the evidence says.

How many of the 44000 are there to laugh at you?
Title: Re: How do we measure the energy of a photon?
Post by: mxplxxx on 28/05/2020 15:15:27
From Catcher In The Rye "All morons hate it when you call them a moron.".
Title: Re: How do we measure the energy of a photon?
Post by: alancalverd on 28/05/2020 16:01:42
Do you really think I could attract 44000+ views without knowing basic physics?
No problem. You can get a lot more, and even become President of the USA, without knowing anything.
Title: Re: How do we measure the energy of a photon?
Post by: mxplxxx on 28/05/2020 17:44:40
Do you really think I could attract 44000+ views without knowing basic physics?
No problem. You can get a lot more, and even become President of the USA, without knowing anything.
IYHO and what does this say about people who follow such a person :)?
Title: Re: How do we measure the energy of a photon?
Post by: Bored chemist on 28/05/2020 19:23:21
Do you really think I could attract 44000+ views without knowing basic physics?
No problem. You can get a lot more, and even become President of the USA, without knowing anything.
IYHO and what does this say about people who follow such a person :)?

Who cares?
I'm,m here to laugh at you and set the record straight for anyone else who comes by.

So. Once again...
You also forgot to address these problems
Quote from: Bored chemist on Today at 10:28:58
Quote from: mxplxxx on Today at 07:23:37
Photons cannot be observed.
Yes they can.
In particular, if a gamma ray photon hits your eye you might actually see the scintillation.
Quote from: mxplxxx on Today at 07:23:37
If photons cannot be observed
They can be.
Quote from: mxplxxx on Today at 07:23:37
who is is to say photons can be created simultaneously?
Logic.
They are brought into being by the same event. It follows that they happen at the same time.
There's also experimental observation of this - used every day throughout the world in medical imaging.
https://en.wikipedia.org/wiki/Positron_emission_tomography#Emission

Why are you pretending that reality doesn't happen?


Title: Re: How do we measure the energy of a photon?
Post by: Kryptid on 28/05/2020 21:15:10
You obviously care about very little in life. You are a tiny, tiny person intellectually. You are an internet troll. You are a talentless mediocrity at best. You have not got a creative bone in your body. You get your kicks from trying to belittle people who care about making a difference in this world. In the process you are only belittling yourself. You constantly embarrass yourself with your stupidity and seem blithely unaware you are doing so. GET A LIFE.

Trying to correct the misconceptions of others is not being a troll.
Title: Re: How do we measure the energy of a photon?
Post by: mxplxxx on 28/05/2020 21:21:59
Trying to correct the misconceptions of others is not being a troll.
Why are you replying to a BoredChemist post? BoredChemist  has admitted in a previous post that he gets pleasure from belittling people.
Title: Re: How do we measure the energy of a photon?
Post by: Bored chemist on 28/05/2020 21:28:24
Who cares?
I'm,m here to laugh at you and set the record straight for anyone else who comes by.
You obviously care about very little in life. You are a tiny, tiny person intellectually. You are an internet troll. You are a talentless mediocrity at best. You have not got a creative bone in your body. You get your kicks from trying to belittle people who care about making a difference in this world. In the process you are only belittling yourself. You constantly embarrass yourself with your stupidity and seem blithely unaware you are doing so. GET A LIFE.

You will not get any reaction from me in the future.
No.
You are the one who deliberately posts nonsense on the web to get a reaction.
That makes you the troll.
BoredChemist  has admitted in a previous post that he gets pleasure from belittling people.
Where?
Title: Re: How do we measure the energy of a photon?
Post by: Bored chemist on 28/05/2020 21:30:14
You will not get any reaction from me in the future.
That's pretty clearly a childish excuse.
The real reason you don't reply is that you know I'm right.
You can't reply to my comments without admitting you were hopelessly wrong.
You also forgot to address these problems
Quote from: Bored chemist on Today at 10:28:58
Quote from: mxplxxx on Today at 07:23:37
Photons cannot be observed.
Yes they can.
In particular, if a gamma ray photon hits your eye you might actually see the scintillation.
Quote from: mxplxxx on Today at 07:23:37
If photons cannot be observed
They can be.
Quote from: mxplxxx on Today at 07:23:37
who is is to say photons can be created simultaneously?
Logic.
They are brought into being by the same event. It follows that they happen at the same time.
There's also experimental observation of this - used every day throughout the world in medical imaging.
https://en.wikipedia.org/wiki/Positron_emission_tomography#Emission

Why are you pretending that reality doesn't happen?


Title: Re: How do we measure the energy of a photon?
Post by: Kryptid on 28/05/2020 21:33:29
Why are you replying to a BoredChemist post?

Because I see this same thing happen all too often here. When people with fringe/alternative views are called out by another member, they often resort to calling that member a troll.

BoredChemist  has admitted in a previous post that he gets pleasure from belittling people.

I would like to see where he posted that.
Title: Re: How do we measure the energy of a photon?
Post by: mxplxxx on 28/05/2020 21:35:22
I would like to see where he posted that.
https://www.thenakedscientists.com/forum/index.php?topic=75960.msg571703#msg571703
This is also relevant https://www.thenakedscientists.com/forum/index.php?topic=75960.msg578292;topicseen#msg578292
Title: Re: How do we measure the energy of a photon?
Post by: Kryptid on 28/05/2020 21:38:14
I would like to see where he posted that.
https://www.thenakedscientists.com/forum/index.php?topic=75960.msg571703#msg571703

The only one in that post claiming that Bored Chemist gets pleasure from belittling people is you.
Title: Re: How do we measure the energy of a photon?
Post by: Bored chemist on 28/05/2020 21:41:55
Why are you replying to a BoredChemist post? BoredChemist  has admitted in a previous post that he gets pleasure from belittling people.
So, now we have had a proper look, we see that, in addition to being a troll, you are a liar.

Why do you do it?
Title: Re: How do we measure the energy of a photon?
Post by: mxplxxx on 28/05/2020 21:43:02
Because I see this same thing happen all too often here. When people with fringe/alternative views are called out by another member, they often resort to calling that member a troll.
So you are biased against members with alternative/fringe views? No wonder I upset you! Suggest you let accused internet trolls fight their own battles. Otherwise you may be seen to siding with internet trolls.
Title: Re: How do we measure the energy of a photon?
Post by: Kryptid on 28/05/2020 21:46:15
So you are biased against members with alternative/fringe views?

Not against the members, but rather against statements that they make that are either unsupported by evidence or contradicted by known evidence.

No wonder I upset you!

Whoever said you upset me?

Suggest you accused internet trolls fight their own battles. Otherwise you may be seen to siding with internet trolls.

Are you sure you know what a troll is? Someone isn't a troll just because they disagree with you.
Title: Re: How do we measure the energy of a photon?
Post by: mxplxxx on 28/05/2020 21:59:49
Whoever said you upset me?
An educated guess. I have a psychology degree. You seem to be a conservative and averse to change or innovation.
Title: Re: How do we measure the energy of a photon?
Post by: mxplxxx on 28/05/2020 22:01:46
Are you sure you know what a troll is? Someone isn't a troll just because they disagree with you.
I have a psychology degree (Australian College of Applied Psychology 2013).
Title: Re: How do we measure the energy of a photon?
Post by: Kryptid on 28/05/2020 22:06:43
An educated guess. I have a psychology degree. You seem to be a conservative and averse to change or innovation.

(1) Your comments don't upset me. They are more likely to just make me roll my eyes.
(2) Whether or not I'm conservative strongly depends upon how you define it. My personal values are in many ways conservative, but my political values lean towards liberal. I'm pretty sure that upholding science as the current state of evidence supports is neither conservative nor liberal. Politics shouldn't have any place in science.
Title: Re: How do we measure the energy of a photon?
Post by: Bored chemist on 28/05/2020 22:07:46
So you are biased against members with alternative/fringe views?
The alternative to being right is being wrong.
There's nothing "fringe" about it.
You make absurd statements then lie about me when you get  called out.

Otherwise you may be seen to siding with internet trolls.
Don't worry.
We are not on your side.

I have a psychology degree
You forgot to answer the question.
Title: Re: How do we measure the energy of a photon?
Post by: Bored chemist on 28/05/2020 22:08:39
You seem to be a conservative and averse to change or innovation.
Being wrong about, for example, fermions, is not "change or innovation", is it?
Title: Re: How do we measure the energy of a photon?
Post by: mxplxxx on 29/05/2020 15:46:18
Quote from: mxplxxx on 27/05/2020 22:49:40
Can you please elucidate me as to how fermions interact, especially as all fermions obey the Pauli Exclusion principle.

If they didn't interact, then the Pauli exclusion principle wouldn't even exist. One fermion obviously has to be able to interact with the other one, otherwise you could put as many fermions in one place and state as you wanted.
Doesn't answer my question though, which suggests to me that you don't know how fermions interact. Better find out otherwise people may start laughing at you:)
Title: Re: How do we measure the energy of a photon?
Post by: mxplxxx on 29/05/2020 15:58:30
Do you really think I could attract 44000+ views without knowing basic physics?
No problem. You can get a lot more, and even become President of the USA, without knowing anything.
IYHO and what does this say about people who follow such a person :)?

No answer to this so. No probs; I would not be in any hurry to if I were you. You do realize that you have probably insulted many of the 45000 plus people who seem to find my posts interesting. Tell me what do you think the energy E in E=hf means. Basic physics but I am betting you will struggle to answer my question.
Title: Re: How do we measure the energy of a photon?
Post by: Kryptid on 29/05/2020 17:35:15
Doesn't answer my question though, which suggests to me that you don't know how fermions interact. Better find out otherwise people may start laughing at you:)

They interact using one of the four fundamental forces (electromagnetism, gravitation, strong nuclear and weak nuclear). That's pretty common knowledge.
Title: Re: How do we measure the energy of a photon?
Post by: Bored chemist on 29/05/2020 17:45:29
Can you please elucidate me as to how fermions interact
In addition to not knowing about fermions, you also don't know what elucidate means.
We can't make you clear.

Title: Re: How do we measure the energy of a photon?
Post by: Bored chemist on 29/05/2020 17:46:05
Doesn't answer my question though, which suggests to me that you don't know how fermions interact
It suggests that nobody bothered to try to answer a question which didn't make sense.
Title: Re: How do we measure the energy of a photon?
Post by: mxplxxx on 29/05/2020 17:49:15
Doesn't answer my question though, which suggests to me that you don't know how fermions interact. Better find out otherwise people may start laughing at you:)

They interact using one of the four fundamental forces (electromagnetism, gravitation, strong nuclear and weak nuclear). That's pretty common knowledge.
yes they do, but it is not fermions interacting. What interacts are a fermion and a boson. Fermions can't interact. Two electrons e.g. cannot interact. Maybe you can tell me then how a fermion and a boson interact?

PS do you and BC have some kind of a mutual admiration club going?
Title: Re: How do we measure the energy of a photon?
Post by: Bored chemist on 29/05/2020 17:49:54
Basic physics but I am betting you will struggle to answer my question.
You forgot to ask a meaningful question.

However, if you think the that fermions interact is basic physics (and, yes, it s) then why did you say they don't interact?

Are you telling us that you don't know basic physics?

Well, I guess that answers this.
Do you really think I could attract 44000+ views without knowing basic physics?

You did get those posts and you, by your own admission, don't know basic physics.
Title: Re: How do we measure the energy of a photon?
Post by: Bored chemist on 29/05/2020 17:52:38
Doesn't answer my question though, which suggests to me that you don't know how fermions interact. Better find out otherwise people may start laughing at you:)

They interact using one of the four fundamental forces (electromagnetism, gravitation, strong nuclear and weak nuclear). That's pretty common knowledge.
yes they do, but it is not fermions interacting. What interacts are a fermion and a boson. Fermions can't interact. Two electrons e.g. cannot interact.
What you seem to have misunderstood is that the interaction of two fermions- for example two electrons- is mediated by bosons- virtual photons in this case.

But the fermions still interact.If one electron wasn't there, the other electron would behave differently.
Title: Re: How do we measure the energy of a photon?
Post by: Kryptid on 29/05/2020 17:59:40
yes they do, but it is not fermions interacting. What interacts are a fermion and a boson. Fermions can't interact. Two electrons e.g. cannot interact.

"Officer, I didn't kill him, the bullet did!"
Title: Re: How do we measure the energy of a photon?
Post by: mxplxxx on 29/05/2020 18:25:57
yes they do, but it is not fermions interacting. What interacts are a fermion and a boson. Fermions can't interact. Two electrons e.g. cannot interact.

"Officer, I didn't kill him, the bullet did!"
Good for a laugh:) "Mxplxxx, I wasn't wrong, my logic was"
Title: Re: How do we measure the energy of a photon?
Post by: Kryptid on 29/05/2020 20:59:39
Good for a laugh:) "Mxplxxx, I wasn't wrong, my logic was"

Then by your reasoning, you, Bored Chemist and I are not interacting right now. So if you read anything from Bored Chemist or me that you don't like, you shouldn't complain about us. Instead, you should complain about the light coming from your computer screen.
Title: Re: How do we measure the energy of a photon?
Post by: Bored chemist on 29/05/2020 21:05:33
Good for a laugh:) "Mxplxxx, I wasn't wrong, my logic was"

Then by your reasoning, you, Bored Chemist and I are not interacting right now. So if you read anything from Bored Chemist or me that you don't like, you shouldn't complain about us. Instead, you should complain about the light coming from your computer screen.
Not to mention complaining to himself for understanding it (Or misunderstanding as the case may be.)

I'm trying to work out if the OP is clever enough to be a troll.
Title: Re: How do we measure the energy of a photon?
Post by: mxplxxx on 30/05/2020 01:27:06
Internet troll - Wikipedia
 
In internet slang, a troll is a person who starts flame wars or upsets people on the Internet by posting inflammatory and digressive, extraneous, or off-topic messages in an online community (such as a newsgroup, forum, chat room, or blog) with the intent of provoking readers into displaying emotional responses and normalizing tangential discussion, either for the troll's amusement or a specific gain.

It seems to me one of the functions of a moderator is to protect the general membership of a forum from a person such as this. It is not happening on Naked Scientist Forums and I will not put up any more with the abuse that I have received in that last 3 days. Sorry people no more laughs.
Title: Re: How do we measure the energy of a photon?
Post by: Kryptid on 30/05/2020 05:24:23
Internet troll - Wikipedia
 
In internet slang, a troll is a person who starts flame wars or upsets people on the Internet by posting inflammatory and digressive, extraneous, or off-topic messages in an online community (such as a newsgroup, forum, chat room, or blog) with the intent of provoking readers into displaying emotional responses and normalizing tangential discussion, either for the troll's amusement or a specific gain.

It seems to me one of the functions of a moderator is to protect the general membership of a forum from a person such as this. It is not happening on Naked Scientist Forums and I will not put up any more with the abuse that I have received in that last 3 days. Sorry people no more laughs.

In what way has anyone here been a troll? Bored Chemist and I don't correct people for personal amusement. We correct people in order to make the truth clear. If people are upset or provoked by that, then they need to do some thorough self-examination and ask themselves if they are being rational.
Title: Re: How do we measure the energy of a photon?
Post by: mxplxxx on 30/05/2020 08:37:02
In what way has anyone here been a troll?
Can't answer that. You need to speak for yourself

Bored Chemist and I don't correct people for personal amusement.
Again,  you need to speak for yourself

We correct people in order to make the truth clear.
Again  you need to speak for yourself. You will have a deeper aim than arriving at the truth. You will be doing so to make yourself feel good.

Beware that the truth is just that, i.e., scientific truth (so hard to come by in Physics) and not a personal opinion. This is New Theories and the truth is near impossible to come by. Ask yourself, why am I in New Theories where the purity of your mission is senseless. No chance, being conservative, you are trying to stop innovation in physics? Conservative physicists have been very successful at this for the last 50 years.

If people are upset or provoked by that, then they need to do some thorough self-examination and ask themselves if they are being rational.
I guarantee you MOST people will be provoked by what you have been posting. It takes a huge amount of skill to criticize and not antagonize people ( even more to have them thank you for it). You don't have the required skill, and, worse, your criticisms are mostly baseless.

At the very least, never criticize the person, always the idea.

I would suggest you ditch your partner who is a light year more provocative than you (he likes to disrespect people "because they deserve it").

Finally Naked Scientists will be risking patronage if this incredibly inflammatory partnership is allowed to continue. I have no intention of returning.
Title: Re: How do we measure the energy of a photon?
Post by: Bored chemist on 30/05/2020 11:26:01
I think the Coronavirus is making us all a lot more touchy than normal.
Not especially; I never did like it when people said derogatory untruths about me.
Why did you do that?
Title: Re: How do we measure the energy of a photon?
Post by: mxplxxx on 30/05/2020 22:05:24
In what way has anyone here been a troll?
Can't answer that. You need to speak for yourself

Bored Chemist and I don't correct people for personal amusement.
Again,  you need to speak for yourself

We correct people in order to make the truth clear.
Again  you need to speak for yourself. You will have a deeper aim than arriving at the truth. You will be doing so to make yourself feel good.

Beware that the truth is just that, i.e., scientific truth (so hard to come by in Physics) and not a personal opinion. This is New Theories and the truth is near impossible to come by. Ask yourself, why am I in New Theories where the purity of your mission is senseless. No chance, being conservative, you are trying to stop innovation in physics? Conservative physicists have been very successful at this for the last 50 years.

If people are upset or provoked by that, then they need to do some thorough self-examination and ask themselves if they are being rational.
I guarantee you MOST people will be provoked by what you have been posting. It takes a huge amount of skill to criticize and not antagonize people ( even more to have them thank you for it). You don't have the required skill, and, worse, your criticisms are mostly baseless.

At the very least, never criticize the person, always the idea.

I would suggest you ditch your partner who is a light year more provocative than you (he likes to disrespect people "because they deserve it").

Finally Naked Scientists will be risking patronage if this incredibly inflammatory partnership is allowed to continue. I have no intention of returning.

Actually, I think I will stick around and see just much laughter I can provoke.:)
Title: Re: How do we measure the energy of a photon?
Post by: mxplxxx on 30/05/2020 23:40:45
Looks like we have a team of intrepid physicists in Naked Scientists New Theories on a mission to to purge the world of physics untruths. How heroic.
Title: Re: How do we measure the energy of a photon?
Post by: Kryptid on 30/05/2020 23:53:24
Can't answer that.

If you are accusing people of being trolls, then you absolutely can answer it. Either that or you just admitted that you called people trolls without a reason. Which is it?

You need to speak for yourself

About what?

You will have a deeper aim than arriving at the truth.

What do you mean by "arriving at the truth"? I'm simply stating it as it is currently known. I'm not "arriving at it", I'm repeating it.

You will be doing so to make yourself feel good.

Yes, it feels good to let people know the difference between actual scientific evidence and speculation.

Beware that the truth is just that, i.e., scientific truth (so hard to come by in Physics) and not a personal opinion.

I'm well aware of that. Opinions are subjective. Facts are objective.

This is New Theories and the truth is near impossible to come by.

You can say that again.

Ask yourself, why am I in New Theories where the purity of your mission is senseless.

I'm here to (1) correct misconceptions, and (2) to point out nonsense for what it is when I see it. If that is senseless, then we might as well make this a fantasy section devoid of science or reason.



No chance, being conservative, you are trying to stop innovation in physics?

Nope. Innovation is good so long as it follows the evidence.

I guarantee you MOST people will be provoked by what you have been posting. It takes a huge amount of skill to criticize and not antagonize people ( even more to have them thank you for it).

And what have I said that is provocative? Can you quote me on it?

Quote
You don't have the required skill, and, worse, your criticisms are mostly baseless.

Yes, it's baseless for me to correct you when you claim that fermions don't interact. You might as well be arguing that no one has ever heard anyone else. They heard sound instead. No one has ever seen anyone else. They saw light instead. It's a technicality that people who speak English would find redundant and unhelpful. It's clear what people mean when they say that they are interacting with someone else. It's the same thing when someone says that one fermion is interacting with another fermion.

At the very least, never criticize the person, always the idea.

Very well. I will only address your ideas from now on without mentioning you.

Looks like we have a team of intrepid physicists in Naked Scientists New Theories on a mission to to purge the world of physics untruths. How heroic.

And that one is the straw-man fallacy. Bored Chemist is a chemist and I'm not even a scientist. I'm well aware that I cannot "purge the world of physics untruths", but I can try to correct people's misunderstandings on this tiny corner of the Internet.
Title: Re: How do we measure the energy of a photon?
Post by: mxplxxx on 31/05/2020 00:18:06
I'm not even a scientist.
It shows. Chances are you are just wasting my time and the time of others on Naked Scientists.
Title: Re: How do we measure the energy of a photon?
Post by: mxplxxx on 31/05/2020 00:22:04
Yes, it feels good to let people know the difference between actual scientific evidence and speculation.
I can assure you it don't feel good to them. Why do you feel this need to put people right. Do you have anxiety issues around control?
Title: Re: How do we measure the energy of a photon?
Post by: Kryptid on 31/05/2020 00:29:54
It shows. Chances are you are just wasting my time and the time of others on Naked Scientists.

I suppose that depends upon how one defines "wasting time". I do agree that trying to correct people in New Theories often is a waste of time when it comes to convincing them that they are mistaken. At least a few lurkers can look at my posts and get a handle on where and why a new idea is likely wrong.

I can assure you it don't feel good to them.

How would you know how anyone other than yourself feels about it?

Why do you feel this need to put people right.

I see no good reason not to. If I know that someone has the wrong answer and I know the right answer, I might as well speak up.

Do you have anxiety issues around control?

I don't think so.
Title: Re: How do we measure the energy of a photon?
Post by: mxplxxx on 31/05/2020 00:30:47
Quote from: mxplxxx on Yesterday at 23:40:45
Looks like we have a team of intrepid physicists in Naked Scientists New Theories on a mission to to purge the world of physics untruths. How heroic.

And that one is the straw-man fallacy. Bored Chemist is a chemist and I'm not even a scientist. I'm well aware that I cannot "purge the world of physics untruths", but I can try to correct people's misunderstandings on this tiny corner of the Internet.
Sorry, I should have said "Looks like we have a team of intrepid NON-physicists"
Title: Re: How do we measure the energy of a photon?
Post by: mxplxxx on 31/05/2020 00:31:53
How would you know how anyone other than yourself feels about it?
An educated guess.
Title: Re: How do we measure the energy of a photon?
Post by: mxplxxx on 31/05/2020 00:33:05
I don't think so.
So you are not a control freak? You sound like one.
Title: Re: How do we measure the energy of a photon?
Post by: Kryptid on 31/05/2020 00:39:03
So you are not a control freak? You sound like one.

How so?
Title: Re: How do we measure the energy of a photon?
Post by: mxplxxx on 31/05/2020 00:57:32
So you are not a control freak? You sound like one.

How so?
In you reason for your hobby i.e. to make the world "right".  You also don't like being told you are wrong and will twist and turn ad nauseam to prove the accuser wrong.
Title: Re: How do we measure the energy of a photon?
Post by: mxplxxx on 31/05/2020 01:05:29
Quote from: Kryptid on Yesterday at 05:24:23
In what way has anyone here been a troll?
Can't answer that. You need to speak for yourself

Quote from: Kryptid on Yesterday at 05:24:23
Bored Chemist and I don't correct people for personal amusement.
Again,  you need to speak for yourself

Quote from: Kryptid on Yesterday at 05:24:23
We correct people in order to make the truth clear.
Again  you need to speak for yourself. You will have a deeper aim than arriving at the truth. You will be doing so to make yourself feel good.
"Can't" should be "Won't". I won't answer any query that involves your partner. Always speak on you own behalf.
Title: Re: How do we measure the energy of a photon?
Post by: Kryptid on 31/05/2020 02:50:25
In you reason for your hobby i.e. to make the world "right".  You also don't like being told you are wrong and will twist and turn ad nauseam to prove the accuser wrong.

I've been corrected many times in other threads. The most recent example to memory is that light travels more slowly in a gravitational field (in the relevant thread in the Physics section of the forum). I had originally thought that its velocity was invariant in a vacuum. If I have been proven wrong in this thread, please let me know what I've gotten wrong.

If you want to psycho-analyze me, feel free to continue doing so via personal messages. This thread has been derailed for long enough. We should be talking about photons.
Title: Re: How do we measure the energy of a photon?
Post by: mxplxxx on 31/05/2020 03:15:05
I suppose that depends upon how one defines "wasting time".
Like between you and your partner, 3 days of my time wasted on series of wild goose chases. Like about 2000 views of time wasted because the viewer likely expected some of my pearls of wisdom or at least a good laugh. BTW, a perfect result if you were an  Internet Troll.
Title: Re: How do we measure the energy of a photon?
Post by: Bored chemist on 31/05/2020 10:51:13
Actually, I think I will stick around and see just much laughter I can provoke.
Why do you think it is funny to tell lies about people?
Title: Re: How do we measure the energy of a photon?
Post by: Bored chemist on 31/05/2020 10:52:08
You also don't like being told you are wrong and will twist and turn ad nauseam to prove the accuser wrong.
Get a mirror.
Title: Re: How do we measure the energy of a photon?
Post by: Bored chemist on 31/05/2020 10:55:34
It shows.
Look who's talking.

Title: Re: How do we measure the energy of a photon?
Post by: mxplxxx on 31/05/2020 12:58:15
Looks like we have a team of intrepid physicists in Naked Scientists New Theories on a mission to to purge the world of physics untruths. How heroic.
And unlikely. Especially as they appear to have little in common. What is more likely? That they are being financed to disrupt any new physics theory showing promise would seem to be a strong possibility. There will be any number of corporations that will stand to lose from the introduction of new technology based on successful new physics theories. You have to ask the question "why has physics advanced so little in the last 50 years".
Title: Re: How do we measure the energy of a photon?
Post by: Bored chemist on 31/05/2020 14:07:09
That they are being financed to disrupt any new physics theory showing promise would seem to be a strong possibility.
You just jumped the shark.
There will be any number of corporations that will stand to lose from the introduction of new technology based on successful new physics theories.
Seriously?
You think progress harms  the economy?
Who do you think finances most research?

The most likely explanation is the real one. I put the record straight to avoid having people misled into believing tosh like the idea that fermions don't interact.

You, on the other hand, post drivel, and then refuse to even talk to those who point out the errors of your ways.
You are the antithesis of science, so I wonder what you are doing here.
BTW...
you forgot to answer this
You also forgot to address these problems
Quote from: Bored chemist on Today at 10:28:58
Quote from: mxplxxx on Today at 07:23:37
Photons cannot be observed.
Yes they can.
In particular, if a gamma ray photon hits your eye you might actually see the scintillation.
Quote from: mxplxxx on Today at 07:23:37
If photons cannot be observed
They can be.
Quote from: mxplxxx on Today at 07:23:37
who is is to say photons can be created simultaneously?
Logic.
They are brought into being by the same event. It follows that they happen at the same time.
There's also experimental observation of this - used every day throughout the world in medical imaging.
https://en.wikipedia.org/wiki/Positron_emission_tomography#Emission

Why are you pretending that reality doesn't happen?



And this
IMHO I think ...
What does the H stand for?










Are you going to answer this?
Title: Re: How do we measure the energy of a photon?
Post by: Kryptid on 31/05/2020 14:58:26
That they are being financed to disrupt any new physics theory showing promise would seem to be a strong possibility. There will be any number of corporations that will stand to lose from the introduction of new technology based on successful new physics theories.

You have got to be kidding me. Why would UPS (my employer) care about anything in this thread?
Title: Re: How do we measure the energy of a photon?
Post by: mxplxxx on 31/05/2020 21:41:37
Looks like we have a team of intrepid physicists in Naked Scientists New Theories on a mission to to purge the world of physics untruths. How heroic.
And unlikely. Especially as they appear to have little in common. What is more likely? That they are being financed to disrupt any new physics theory showing promise would seem to be a strong possibility. There will be any number of corporations that will stand to lose from the introduction of new technology based on successful new physics theories. You have to ask the question "why has physics advanced so little in the last 50 years".
The answer may be that talented people like myself just get sick of the incessant attacks by talentless people whose agenda is driven by an unrelenting pathological need to belittle others, or money is involved. Money is usually involved when the status quo is threatened.

Their favorite troll-like tactic seems to repeat ad nauseam a post that they have not got a reply to. Why? Because they can. The Naked Scientist's moderator seems to be in collusion with the practice. It is certainly not because they feel they can increase their chances of getting a reply. I never  reply to such awful behavior. This topic, unfortunately, is riddled with this sort of thing. If I were a follower, I would be thinking twice before visiting the topic. Eventually, if the behavior continues to be tolerated, Naked Scientists and Physics are going to be the losers here.
Title: Re: How do we measure the energy of a photon?
Post by: Bored chemist on 31/05/2020 22:19:39
Their favorite troll-like tactic seems to repeat ad nauseam a post that they have not got a reply to. Why?
Because you don't reply.

The mods allow it because you signed up to the rules and the rules say you should address a reasonable point- otherwise you are just soapboxing, and that's not allowed.
Specifically the rulse sauy "The site is not for evangelising your own pet theory.  It is perfectly acceptable that you should post your own theory up for discussion, but if all you want to do is promote your own idea and are not inviting critical debate about it, then that will not be acceptable.".

But, if you don't like it, it's easy enough to put a stop to it.
Just reply; answer the question.

Why didn't you think of that?
Title: Re: How do we measure the energy of a photon?
Post by: Bored chemist on 31/05/2020 22:21:01
The answer may be that talented people like myself
If you have a talent in physics,
(1) why do you make so many  mistakes?
(2) why did you study something else?
Title: Re: How do we measure the energy of a photon?
Post by: Bored chemist on 31/05/2020 22:23:54
I never  reply to such awful behavior.
If you think that asking a question is "such awful behavior", why did you join a discussion forum?
There are blog sites where you can be as wrong as you like, and nobody will object.
Title: Re: How do we measure the energy of a photon?
Post by: Kryptid on 31/05/2020 23:11:42
The answer may be that talented people like myself just get sick of the incessant attacks by talentless people whose agenda is driven by an unrelenting pathological need to belittle others, or money is involved. Money is usually involved when the status quo is threatened.

Neither of those things are true of me. I don't feel the need to belittle others (challenging a claim is not the same as belittling a person) nor is anyone paying me to post on this forum.

Enough of this off-topic banter. If you want to continue to complain about us, do it via PMs or talk to another moderator/administrator. Keep this discussion about the topic title. If you keep derailing this topic by calling us trolls or the like, you may be given a temporary suspension (don't blame me for it. It was a different moderator that brought up the possibility of suspension).
Title: Re: How do we measure the energy of a photon?
Post by: mxplxxx on 02/06/2020 14:50:38
Looks like we have a team of intrepid physicists in Naked Scientists New Theories on a mission to to purge the world of physics untruths. How heroic.
And unlikely. Especially as they appear to have little in common. What is more likely? That they are being financed to disrupt any new physics theory showing promise would seem to be a strong possibility. There will be any number of corporations that will stand to lose from the introduction of new technology based on successful new physics theories. You have to ask the question "why has physics advanced so little in the last 50 years".
The answer may be that talented people like myself just get sick of the incessant attacks by talentless people whose agenda is driven by an unrelenting pathological need to belittle others, or money is involved. Money is usually involved when the status quo is threatened.

Their favorite troll-like tactic seems to repeat ad nauseam a post that they have not got a reply to. Why? Because they can. The Naked Scientist's moderator seems to be in collusion with the practice. It is certainly not because they feel they can increase their chances of getting a reply. I never  reply to such awful behavior. This topic, unfortunately, is riddled with this sort of thing. If I were a follower, I would be thinking twice before visiting the topic. Eventually, if the behavior continues to be tolerated, Naked Scientists and Physics are going to be the losers here.
One of the questions being posed by the team leader is "What does the H stand for" (in one of my replies which started with IMHO)" It has been repeated ad nauseam 15 times.  Many others are abusive. I refuse to reply to such a person at all, in this case, their partner.

The team is good guy, bad guy. One is meek and mild and the other is arrogant and egotistic. They are as alike as chalk and cheese. Their chances of becoming bosom buddies are pretty much nil. Why therefore the partnership? Maybe they think good cop, bad cop interrogation is going to achieve better results.  And maybe there is not a partnership. Maybe it is the one guy with different User Ids. Whatever, it is likely to be bad news for Naked Scientists.

Bad Cop has 2 Topics on New Theories out of 18000 posts. He is obviously bereft of ideas/creativity. Why then is he spending 33% of his time in New Theories? Because, being new he can criticize the theory any way he wants without the need for scientific proof. Never mind that he is mostly laughably wrong, he gets his 4 hits of belittling satisfaction a day. He is a bully. If you criticize him, he almost always comes back at you with more drivel or aggressively attacks you.

These guys don't like not having the last word. They will post and post and post until we give up and stop posting. An internet troll is best handled by not replying to their posts.

Why a moderator would put up with all this is totally beyond me. I have contacted the moderator and others in Naked Scientists about the situation but to no avail.

Bad cop BC, C2B, B2C?

In future in New Theories, I will reply only to posts that have references, address the idea and not the person, are neutral or pleasant in tone, are helpful and address the theory. I will not reply to nor read posts from people I know to be trouble-makers.
Title: Re: How do we measure the energy of a photon?
Post by: Bored chemist on 02/06/2020 14:53:08
The mods probably think you should follow the rules and reply to reasonable questions.
Why don't you do that?
Are you unable to look up the answer?
Are you concerned that, if you say what it means that will show that you were factually incorrect?
Why not just answer the question?
Title: Re: How do we measure the energy of a photon?
Post by: Kryptid on 02/06/2020 20:47:28
Why therefore the partnership?

I didn't team up with Bored Chemist. We both came here of our own volition. None of this was pre-arranged.

As I said before, stop cluttering up this thread with complaints. Keep all of the complaining in PMs to the relevant parties. I won't warn you again.
Title: Re: How do we measure the energy of a photon?
Post by: Bored chemist on 02/06/2020 22:24:37
The team is good guy, bad guy. One is meek and mild and the other is arrogant and egotistic. They are as alike as chalk and cheese. Their chances of becoming bosom buddies are pretty much nil. Why therefore the partnership?

It's not a partnership.
It's just that we both, independently, think it would be better if you didn't post tripe, and answered criticism.

So, let's make a start.
It takes one word to answer this

Quote from: Bored chemist on 03/03/2020 19:22:35
Quote from: Bored chemist on 29/02/2020 14:33:19
Quote from: Bored chemist on 26/02/2020 08:16:25
Quote from: Bored chemist on 24/02/2020 10:27:54
Quote from: Bored chemist on 21/02/2020 13:38:09
Quote from: Bored chemist on 17/02/2020 07:22:01
Quote from: Bored chemist on 15/02/2020 12:48:12
Quote from: Bored chemist on 08/02/2020 11:44:37
Quote from: Bored chemist on 05/02/2020 19:37:32
Quote from: Bored chemist on 04/02/2020 19:09:00
Quote from: Bored chemist on 03/02/2020 07:25:14
Quote from: mxplxxx on 02/02/2020 23:11:28
IMHO I think ...
What does the H stand for?



Why are you so scared of that word?
Title: Re: How do we measure the energy of a photon?
Post by: mxplxxx on 04/06/2020 02:33:14
Evidence that the universe is rotating. https://futurism.com/the-byte/astronomers-entire-early-universe-rotating

The Universe is the top level 3dSystem of Reality. It will naturally rotate around its centre of gravity.  As will all of its subsystems and their subsystems. You are a 3dSystem that seemingly does not rotate. But, this is because gravity is stopping you from doing so.

A 3dSystem is initially created as a rotating sphere with no components. The rotating sphere has an Intrinsic Angular Momentum. There are distinct types of this momentum which is often called Spin and is identified by a Spin number.

The energy of a system is related to its Intrinsic Angular Momentum and is spread out initially evenly across the space occupied by the system when it is first created. Being Intrinsic means the energy is absolute (not relational) This energy is given by the basic equation of quantum physics E = h/t, where:

E = the energy in units of joules that is present in a single rotation of the system
h = Planck's constant of Action (6.62607015×10⁻³⁴ joules times seconds)
t = the time taken for a single rotation of the system.

This means, amazingly, that all systems "contain" a constant amount of Action, h. Action: walk for 1k, run for 1k: same action but first way is low energy long time, second is high energy short time.

It also so happens that f = 1/t where f = the frequency of the frequency of the rotation in Hertz (cycles/rotations per second) units, so:

E = hf is an alternative way of calculating energy.

This energy will change only if the frequency of the system changes. For example, if the system expands, the frequency will decrease and so will the energy. If the system contracts, the frequency will increase and so will the energy. Much like a spinning ice-skater.

PS all 3dSystems are universes in their own rights. They are infinite in nature. They also each contain two centres of awareness. One processes all communication with the parent 3dSystem and the other processes all communication with the 3dShells/3dSubsystems. In the case of a personal 3dSystem, these centres would likely be the third eye and the solar plexus.
Title: Re: How do we measure the energy of a photon?
Post by: Bored chemist on 04/06/2020 08:41:41
The Universe is the top level 3dSystem of Reality. It will naturally rotate around its centre of gravity.  As will all of its subsystems and their subsystems. You are a 3dSystem that seemingly does not rotate. But, this is because gravity is stopping you from doing so.
If gravity stops me spinning, how come the much bigger gravity of the universe does not stop it spinning?

It takes one word to answer this
Quote from: Bored chemist on 31/05/2020 14:07:09

Quote from: Bored chemist on 03/03/2020 19:22:35
Quote from: Bored chemist on 29/02/2020 14:33:19
Quote from: Bored chemist on 26/02/2020 08:16:25
Quote from: Bored chemist on 24/02/2020 10:27:54
Quote from: Bored chemist on 21/02/2020 13:38:09
Quote from: Bored chemist on 17/02/2020 07:22:01
Quote from: Bored chemist on 15/02/2020 12:48:12
Quote from: Bored chemist on 08/02/2020 11:44:37
Quote from: Bored chemist on 05/02/2020 19:37:32
Quote from: Bored chemist on 04/02/2020 19:09:00
Quote from: Bored chemist on 03/02/2020 07:25:14
Quote from: mxplxxx on 02/02/2020 23:11:28
IMHO I think ...
What does the H stand for?



Why are you so scared of that word?
Title: Re: How do we measure the energy of a photon?
Post by: Bored chemist on 04/06/2020 08:45:14

This means, amazingly, that all systems "contain" a constant amount of Action,
When you say action, do you mean action, or are you just misusing a "sciencey" word?

https://en.wikipedia.org/wiki/Action_(physics)
Title: Re: How do we measure the energy of a photon?
Post by: mxplxxx on 09/06/2020 06:34:56
A flat universe? https://futurism.com/the-byte/universe-actually-flat. Chances are they are looking at the 2d (Data) part of a Universe 3dApp (the other parts being 3d Display - Space and 4d Datastore - Spacetime).
Title: Re: How do we measure the energy of a photon?
Post by: Bored chemist on 09/06/2020 08:45:52
A flat universe? https://futurism.com/the-byte/universe-actually-flat. Chances are they are looking at the 2d (Data) part of a Universe 3dApp (the other parts being 3d Display - Space and 4d Datastore - Spacetime).
You don't understand what flat means, do you?
https://en.wikipedia.org/wiki/Shape_of_the_universe
Title: Re: How do we measure the energy of a photon?
Post by: Colin2B on 09/06/2020 11:44:04
Chances are they are looking at the 2d (Data) part of a Universe 3dApp (the other parts being 3d Display - Space and 4d Datastore - Spacetime).
@Bored chemist is right, they are not talking about topology.
Title: Re: How do we measure the energy of a photon?
Post by: mxplxxx on 09/06/2020 12:11:39
Chances are they are looking at the 2d (Data) part of a Universe 3dApp (the other parts being 3d Display - Space and 4d Datastore - Spacetime).
@Bored chemist is right, they are not talking about topology.
They are talking about large scale topology.

From https://blogs.scientificamerican.com/degrees-of-freedom/httpblogsscientificamericancomdegrees-of-freedom20110725what-do-you-mean-the-universe-is-flat-part-i/

"What I do want to talk about here is what it is that is supposed to be flat.

When cosmologists say that the universe is flat they are referring to space—the nowverse and its parallel siblings of time past. Spacetime is not flat. It can’t be: Einstein’s general theory of relativity says that matter and energy curve spacetime, and there are enough matter and energy lying around to provide for curvature. Besides, if spacetime were flat I wouldn’t be sitting here because there would be no gravity to keep me on the chair. To put it succinctly: space can be flat even if spacetime isn't.

Moreover, when they talk about the flatness of space cosmologists are referring to the large-scale appearance of the universe. When you “zoom in” and look at something of less-than-cosmic scale, such as the solar system, space—not just spacetime—is definitely not flat. Remarkable fresh evidence for this fact was obtained recently by the longest-running experiment in NASA history, Gravity Probe B, which took a direct measurement of the curvature of space around Earth. (And the most extreme case of non-flatness of space is thought to occur inside the event horizon of a black hole, but that’s another story.)
"

Maybe the Universe 3dSystem contains a map/database/class library to the rest of the systems in the universe.

PS I am still to find a lucid explanation in physics of what curved spacetime actually means and how it causes gravity.

Also, a computer program is a 2d (flat) object that can describe/manipulate 3d objects. i.e. a flat universe COULD conceivably exist and be translated into a 3d object when "observed".
Title: Re: How do we measure the energy of a photon?
Post by: Bored chemist on 09/06/2020 12:49:29
They are talking about "flat" in the sense where a 3D torus is flat.
The universe is flat, but 4D
Also, a computer program is a 2d (flat) object
A computer program is a set of instructions in order.
In "the good old days"  they had line numbers or memory locations against every instruction or datum in them.
If you can allocate a running number against each item from which an object is composed, it is a 1D item.
So, computer programs are not 2D.
Please stop posting about stuff you don't understand.

Title: Re: How do we measure the energy of a photon?
Post by: mxplxxx on 10/06/2020 01:23:59
A flat universe? https://futurism.com/the-byte/universe-actually-flat. Chances are they are looking at the 2d (Data) part of a Universe 3dApp (the other parts being 3d Display - Space and 4d Datastore - Spacetime).
Flat as in 2d flat. Computer programs are 2d in nature. What appears on your computer screen is usually a table of pixels in the program (raster graphics). The pixels may be generated via 2d polygons (vector graphics). A 3dApp is part of the 3dAbstractions framework that can be used to simulate reality. It is a Windows 10 App written in the VB.Net programming language. It is a type of the 3dSystem that has been described in this post as being a basic Universal pattern. A 3dApp has a basic structure as follows:

App as 3dApp.System
    Display as 3dDisplay.System
    Data as 3dData.System
    Datastore as 3dDatastore.System

Display, Data, Datastore - hence 3d.

Display contains human interface data.
Data contains a database in HFSM format.
Datastore contains persistent data that is usually stored on disk.

All of these systems contain data in 2d/flat format. What you see displayed is usually a 2d image which is turned into 3d by your brain. The database is in 2d format. Persistent data is 4d (think spacetime) in nature but exists in the App as a 2d/flat table.     

There is no reason why our Universe cannot exist as a 2d/flat structure, possibly a hologram. See https://www.wired.co.uk/article/our-universe-is-a-hologram. Given Occam's razor, the likelihood that the Universe is a hologram is high. I have discussed this before in this topic.
Title: Re: How do we measure the energy of a photon?
Post by: Bored chemist on 10/06/2020 09:03:12
Flat as in 2d flat. Computer programs are 2d in nature. What appears on your computer screen is usually a table of pixels in the program (raster graphics)
And the thing about a raster scan is that it is a single line. so it is one dimensional. (The dimension is time)
The same goes for screens with "memory mapped IO"- though I accept that's rather old tech. (In that instance, the dimension is memory address)

More importantly, the screen and the output are not the programme.
So, even if they were 2D (and they are not) the programme itself would still be 1D. The dimension being line number or memory address

You might be able to make an argument for a multi threaded  program being 2D.
Title: Re: How do we measure the energy of a photon?
Post by: mxplxxx on 10/06/2020 12:16:48
Looks like we have a team of intrepid physicists in Naked Scientists New Theories on a mission to to purge the world of physics untruths. How heroic.
And unlikely. Especially as they appear to have little in common. What is more likely? That they are being financed to disrupt any new physics theory showing promise would seem to be a strong possibility. There will be any number of corporations that will stand to lose from the introduction of new technology based on successful new physics theories. You have to ask the question "why has physics advanced so little in the last 50 years".
The answer may be that talented people like myself just get sick of the incessant attacks by talentless people whose agenda is driven by an unrelenting pathological need to belittle others, or money is involved. Money is usually involved when the status quo is threatened.

Their favorite troll-like tactic seems to repeat ad nauseam a post that they have not got a reply to. Why? Because they can. The Naked Scientist's moderator seems to be in collusion with the practice. It is certainly not because they feel they can increase their chances of getting a reply. I never  reply to such awful behavior. This topic, unfortunately, is riddled with this sort of thing. If I were a follower, I would be thinking twice before visiting the topic. Eventually, if the behavior continues to be tolerated, Naked Scientists and Physics are going to be the losers here.
One of the questions being posed by the team leader is "What does the H stand for" (in one of my replies which started with IMHO)" It has been repeated ad nauseam 15 times.  Many others are abusive. I refuse to reply to such a person at all, in this case, their partner.

The team is good guy, bad guy. One is meek and mild and the other is arrogant and egotistic. They are as alike as chalk and cheese. Their chances of becoming bosom buddies are pretty much nil. Why therefore the partnership? Maybe they think good cop, bad cop interrogation is going to achieve better results.  And maybe there is not a partnership. Maybe it is the one guy with different User Ids. Whatever, it is likely to be bad news for Naked Scientists.

Why a moderator would put up with all this is totally beyond me. I have contacted the moderator and others in Naked Scientists about the situation but to no avail.

In future in New Theories, I will reply only to posts that have references, address the idea and not the person, are neutral or pleasant in tone, are helpful and address the theory. I will not reply to nor read posts from people I know to be trouble-makers.

It now seems that the moderator has joined the good cop/bad cop team, presumably as a good cop. It doesn't matter whether this team has knowledge or not about one of my posts they will still post their (mostly) drivel. A recent one on software development is just laughable. I repeat, I will not reply or even read those posts or posts relating to these posts.
Title: Re: How do we measure the energy of a photon?
Post by: Bored chemist on 10/06/2020 12:50:11
It doesn't matter whether this team has knowledge or not about one of my posts
The problem is that you do not have knowledge about your posts.
Now, let's see if you can post the right word...

Quote from: Bored chemist on 02/06/2020 22:24:37
It takes one word to answer this
Quote from: Bored chemist on 31/05/2020 14:07:09

Quote from: Bored chemist on 03/03/2020 19:22:35
Quote from: Bored chemist on 29/02/2020 14:33:19
Quote from: Bored chemist on 26/02/2020 08:16:25
Quote from: Bored chemist on 24/02/2020 10:27:54
Quote from: Bored chemist on 21/02/2020 13:38:09
Quote from: Bored chemist on 17/02/2020 07:22:01
Quote from: Bored chemist on 15/02/2020 12:48:12
Quote from: Bored chemist on 08/02/2020 11:44:37
Quote from: Bored chemist on 05/02/2020 19:37:32
Quote from: Bored chemist on 04/02/2020 19:09:00
Quote from: Bored chemist on 03/02/2020 07:25:14
Quote from: mxplxxx on 02/02/2020 23:11:28
IMHO I think ...
What does the H stand for?



Why are you so scared of that word?
Title: Re: How do we measure the energy of a photon?
Post by: mxplxxx on 15/06/2020 03:32:29
So, given what we now know of 3dSystems, what will be the next big thing in 3d Science (Physics "married" to Computer Science)? Possibly the ability to travel up and down a system's "tree" (HFSM) and "view" each system in the tree from a "local" perspective. In the process, find out the "purpose" of each system in the tree. Maybe a special type ("inherits 3dSystem") of 3dSystem will be created to allow such a function to occur. Eventually we may be able to "marry" our brains to 3dSystems so that the experience of "visiting" systems will be indistinguishable from Reality.
Title: Re: How do we measure the energy of a photon?
Post by: Bored chemist on 15/06/2020 10:23:29
Now, let's see if you can post the right word...

Quote from: Bored chemist on 04/06/2020 08:41:41
Quote from: Bored chemist on 02/06/2020 22:24:37
It takes one word to answer this
Quote from: Bored chemist on 31/05/2020 14:07:09

Quote from: Bored chemist on 03/03/2020 19:22:35
Quote from: Bored chemist on 29/02/2020 14:33:19
Quote from: Bored chemist on 26/02/2020 08:16:25
Quote from: Bored chemist on 24/02/2020 10:27:54
Quote from: Bored chemist on 21/02/2020 13:38:09
Quote from: Bored chemist on 17/02/2020 07:22:01
Quote from: Bored chemist on 15/02/2020 12:48:12
Quote from: Bored chemist on 08/02/2020 11:44:37
Quote from: Bored chemist on 05/02/2020 19:37:32
Quote from: Bored chemist on 04/02/2020 19:09:00
Quote from: Bored chemist on 03/02/2020 07:25:14
Quote from: mxplxxx on 02/02/2020 23:11:28
IMHO I think ...
What does the H stand for?



Why are you so scared of that word?
Title: Re: How do we measure the energy of a photon?
Post by: alancalverd on 16/06/2020 14:44:01
There is no reason why our Universe cannot exist as a 2d/flat structure, possibly a hologram
A hologram of what?
Title: Re: How do we measure the energy of a photon?
Post by: mxplxxx on 16/06/2020 15:11:22
There is no reason why our Universe cannot exist as a 2d/flat structure, possibly a hologram
A hologram of what?
Of itself. See https://futurism.com/the-holographc-universe-principle-what-is-what-should-never-be. A Hologram is basically a 2d representation of a 3d reality.
Title: Re: How do we measure the energy of a photon?
Post by: Bored chemist on 16/06/2020 16:36:44
It makes just as little sense to consider that the 4D Universe might be a projection of a 5D  "something".
Yes, it might, but we can't tell.
Title: Re: How do we measure the energy of a photon?
Post by: Bored chemist on 16/06/2020 16:44:59
Now, let's see if you can post the right word...

Quote from: Bored chemist on 04/06/2020 08:41:41
Quote from: Bored chemist on 02/06/2020 22:24:37
It takes one word to answer this
Quote from: Bored chemist on 31/05/2020 14:07:09

Quote from: Bored chemist on 03/03/2020 19:22:35
Quote from: Bored chemist on 29/02/2020 14:33:19
Quote from: Bored chemist on 26/02/2020 08:16:25
Quote from: Bored chemist on 24/02/2020 10:27:54
Quote from: Bored chemist on 21/02/2020 13:38:09
Quote from: Bored chemist on 17/02/2020 07:22:01
Quote from: Bored chemist on 15/02/2020 12:48:12
Quote from: Bored chemist on 08/02/2020 11:44:37
Quote from: Bored chemist on 05/02/2020 19:37:32
Quote from: Bored chemist on 04/02/2020 19:09:00
Quote from: Bored chemist on 03/02/2020 07:25:14
Quote from: mxplxxx on 02/02/2020 23:11:28
IMHO I think ...
What does the H stand for?



Why are you so scared of that word?

Title: Re: How do we measure the energy of a photon?
Post by: mxplxxx on 17/06/2020 00:04:22
How do elementary particles fit into the 3dAbstractions framework? They are types of 3dSystems as is everything else in the framework. The big difference between this sort of elementary system and composite systems is that an elementary system has no subsystems.

Having no subsystems means that an elementary system is totally concrete (versus abstract) in nature.

These systems comprise the top-level properties of a 3dStar central system and contain the overall state of the system.

A system can have subsystems and the subsystems can have subsystems infinitely (same sort of thing for the child/parent tree). Any system without subsystems therefore is not as "real"/concrete as it could possibly be. A corollary of this is that the more levels a system has, the closer it comes to representing "reality". Absolute "reality" is therefore infinite in nature.

This makes each 3dSystem a universe in its own right. In fact, each 3dSystem can be considered as a stationary system surrounded by in-motion 3dsystems,

Finally it can be noted that an event cannot escape a parent-child tree of 3dSystems except by proceeding to the top of the tree (centre of the system) and then being sent downwards via a parent/child 3dSystems tree to finally execute a function call. A 3dSystem is a control freak. Nothing happens as a result of an event in a system unless it is approved by the parent. i.e. all sibling to sibling contact is done via the parent. The same may apply to a photon in Reality i.e. a photon created in a galaxy light years away must have had to "travel" via parent/child systems to the centre of the Universe system and then be "sent" based on probabilities from this system via parent/child systems to a local telescope to be "observed". This has many possible consequences which I will explore in later posts.     
Title: Re: How do we measure the energy of a photon?
Post by: mxplxxx on 23/06/2020 06:38:11
In a 3dBstractions framework, a 3dShadow is a type of 3dSystem but without many of the states/capabilities of the 3dSystem it shadows. Its equivalent in Reality is a boson.

Four types of 3shadow systems exist:

3dEvent - a record of the past state of a 3dSystem

    Can travel up, down and across 3dSystems in a 3d Abstraction hierarchy. Actually it doesn't really "travel". The originating 3dSystem "raises" an event and the "handling" function (which may be in another 3dSystem) handles it. The runtime system handles this process. Something similar likely happens in Reality, so photons do not really "travel"/"move" distances at all) 
   
    Equivalent to a photon in reality. A 3dPhoton is a type of 3dEvent that will scale up/down as it crosses 3dSystem
    boundaries. It does so by increasing or decreasing its volume and thus decreasing or increasing its frequency (it
    is a rotating sphere).

    PS theoretically any system can travel in this way. So you could travel and observe an Atom in action if only you
    knew how to scale yourself down/up:)

3dTop - A event processor (awareness) near the top of a 3dSystem

    Mediates the processing of 3dEvents between a 3dStar System and the parent of the 3dSystem it belongs to.
    Equivalent to a W boson in Reality.

3dBottom - An event processor (awareness) near the bottom of a 3dSystem

    Mediates the processing of 3dEvents between a 3dStar System and its associated 3dShells/3dSubsystems.
    Equivalent to a Z boson in Reality.

3dController- Links two 3dSystems sideways.

    Mediates the processing of 3dEvents between the 3dSystem the 3dController belongs to and another 3dSystem
    e.g. System B raises a 3dEvent. The 3dContrioller in System A processes the event and (possibly) calls a function in System B as a result.
    Equivalent to a Gluon in Reality.

In 3dAbstractions, a 3dEvent is optionally "forgotten" after it is processed but prior to this may optionally be added to the history for the 3dSystem it shadows in a 3dDatastore system. In Reality, this would mean a photon is added to space/time continuum.




Title: Re: How do we measure the energy of a photon?
Post by: Bored chemist on 23/06/2020 08:56:24
Now, let's see if you can post the right word...

Quote from: Bored chemist on 04/06/2020 08:41:41
Quote from: Bored chemist on 02/06/2020 22:24:37
It takes one word to answer this
Quote from: Bored chemist on 31/05/2020 14:07:09

Quote from: Bored chemist on 03/03/2020 19:22:35
Quote from: Bored chemist on 29/02/2020 14:33:19
Quote from: Bored chemist on 26/02/2020 08:16:25
Quote from: Bored chemist on 24/02/2020 10:27:54
Quote from: Bored chemist on 21/02/2020 13:38:09
Quote from: Bored chemist on 17/02/2020 07:22:01
Quote from: Bored chemist on 15/02/2020 12:48:12
Quote from: Bored chemist on 08/02/2020 11:44:37
Quote from: Bored chemist on 05/02/2020 19:37:32
Quote from: Bored chemist on 04/02/2020 19:09:00
Quote from: Bored chemist on 03/02/2020 07:25:14
Quote from: mxplxxx on 02/02/2020 23:11:28
IMHO I think ...
What does the H stand for?



Why are you so scared of that word?


Title: Re: How do we measure the energy of a photon?
Post by: mxplxxx on 01/07/2020 15:09:57
Particles in Reality have anti-particles which are identical in all respects except for charge and magnetic moment.

3dAntiSystems are the 3d Abstractions equivalent of anti-particles. A 3dAntiSystem inherits a 3dSystem.

Whereas the execution direction for a 3dSystem is down, that of a 3dAntiSystem is up. In other words a function call in a 3dSystem will start at a particular abstraction level and continue down at lower and lower levels. This is Top-Down programming. A function call to 3dAntiSystem on the other hand will start at a particular abstraction level and continue up at higher and higher levels. In other words Bottom-Up programming.

The main purpose of a 3dAntiSystem is to create an abstraction hierarchy from the bottom up. In the process, all 3dAntiSystems are converted into "ordinary" 3dSystems.

Just possibly, this is also the main purpose of an anti-particle in Reality. i.e. it is involved in the creation of the universe from the bottom up and then discarded. This would explain why so few anti-particles seem to exist. Photons are supposed to be their own anti-particle. This is exactly what a 3dAntiSystem is. The anti part will be involved in the creation of the photon and then, possibly, discarded.
Title: Re: How do we measure the energy of a photon?
Post by: Bored chemist on 01/07/2020 17:28:07
How do you cope with photons?
(Remember them? they were meant to be the topic).

A photon is its own antiparticle.
Title: Re: How do we measure the energy of a photon?
Post by: mxplxxx on 04/07/2020 05:16:24
https://scienceblog.com/517166/quantum-fluctuations-can-jiggle-objects-on-the-human-scale/

From the article:

"Now for the first time, a team led by researchers at MIT LIGO Laboratory has measured the effects of quantum fluctuations on objects at the human scale. In a paper published today in Nature, the researchers report observing that quantum fluctuations, tiny as they may be, can nonetheless “kick” an object as large as the 40-kilogram mirrors of the U.S. National Science Foundation’s Laser Interferometer Gravitational-wave Observatory (LIGO), causing them to move by a tiny degree, which the team was able to measure."

From Wiki, "In quantum physics, a quantum fluctuation (or vacuum state fluctuation or vacuum fluctuation) is the temporary change in the amount of energy in a point in space".

    Equivalent to a photon in reality. A 3dPhoton is a type of 3dEvent that will scale up/down as it crosses 3dSystem
    boundaries. It does so by increasing or decreasing its volume and thus decreasing or increasing its frequency (it
    is a rotating sphere).

From a 3dSystems point of view, if it were possible, measuring the energy of a photon at the level (quantum) of the originating system and at the level of the current (classical) system will give a difference in frequency. The authors of the above article seem to have been able to measure the movement of a mirror as a result of it being "kicked" by a laser photon. Presumably then, because a laser has a fixed frequency, the difference in frequencies can be inferred.
Title: Re: How do we measure the energy of a photon?
Post by: Bored chemist on 04/07/2020 13:48:42
The authors of the above article seem to have been able to measure the movement of a mirror as a result of it being "kicked" by a laser photon.
You have misunderstood.
For decades, with a good analytical balance and an industrial laser we have been able to measure the kick from a laser beam.
If I got the arithmetic right, a 1 Watt laser produces a force of about a third of a microgram if it's absorbed and twice that if it's reflected. That's perfectly measurable.

The report shows something more interesting.
They have shown that, even without any external influence, the position of the mirrors varies due to the uncertainty principle.

As it happens, they use lasers to make the measurements- but that's not the point.
It's also a truly wonderfully impressive experiment.

You also seem not to know that measuring shifts of wavelength caused by reflection has been done for about a hundred years.
https://en.wikipedia.org/wiki/Compton_scattering

Now, please grow up a bit and answer the question I asked.




How do you cope with photons?
(Remember them? they were meant to be the topic).

A photon is its own antiparticle.
Title: Re: How do we measure the energy of a photon?
Post by: mxplxxx on 07/07/2020 08:33:10
Congrats to me on 50000 views :) The next 50000 promises to be even more entertaining.
Title: Re: How do we measure the energy of a photon?
Post by: Bored chemist on 07/07/2020 08:55:29
Now, please grow up a bit and answer the question I asked.




Quote from: Bored chemist on 01/07/2020 17:28:07
How do you cope with photons?
(Remember them? they were meant to be the topic).

A photon is its own antiparticle.
Title: Re: How do we measure the energy of a photon?
Post by: mxplxxx on 20/07/2020 18:23:23
In a 3dBstractions framework, a 3dShadow is a type of 3dSystem but without many of the states/capabilities of the 3dSystem it shadows. Its equivalent in Reality is a boson.

Four types of 3shadow systems exist:

3dEvent - a record of the past state of a 3dSystem

    Can travel up, down and across 3dSystems in a 3d Abstraction hierarchy. Actually it doesn't really "travel". The originating 3dSystem "raises" an event and the "handling" function (which may be in another 3dSystem) handles it. The runtime system handles this process. Something similar likely happens in Reality, so photons do not really "travel"/"move" distances at all) 
   
    Equivalent to a photon in reality. A 3dPhoton is a type of 3dEvent that will scale up/down as it crosses 3dSystem
    boundaries. It does so by increasing or decreasing its volume and thus decreasing or increasing its frequency (it
    is a rotating sphere).

    PS theoretically any system can travel in this way. So you could travel and observe an Atom in action if only you
    knew how to scale yourself down/up:)

3dTop - A event processor (awareness) near the top of a 3dSystem

    Mediates the processing of 3dEvents between a 3dStar System and the parent of the 3dSystem it belongs to.
    Equivalent to a W boson in Reality.

3dBottom - An event processor (awareness) near the bottom of a 3dSystem

    Mediates the processing of 3dEvents between a 3dStar System and its associated 3dShells/3dSubsystems.
    Equivalent to a Z boson in Reality.

3dController- Links two 3dSystems sideways.

    Mediates the processing of 3dEvents between the 3dSystem the 3dController belongs to and another 3dSystem
    e.g. System B raises a 3dEvent. The 3dContrioller in System A processes the event and (possibly) calls a function in System B as a result.
    Equivalent to a Gluon in Reality.

In 3dAbstractions, a 3dEvent is optionally "forgotten" after it is processed but prior to this may optionally be added to the history for the 3dSystem it shadows in a 3dDatastore system. In Reality, this would mean a photon is added to space/time continuum.





https://futurism.com/nasa-hubble-observed-flapping-shadow-distant-space

Possibly evidence of the shadow nature of photons and/or Top/bottom Event Processors (3dTop/3dBottom systems).
Title: Re: How do we measure the energy of a photon?
Post by: Bored chemist on 20/07/2020 18:47:27
Now, please grow up a bit and answer the question I asked.




Quote from: Bored chemist on 01/07/2020 17:28:07
How do you cope with photons?
(Remember them? they were meant to be the topic).

A photon is its own antiparticle.

Title: Re: How do we measure the energy of a photon?
Post by: mxplxxx on 24/07/2020 06:43:39
From the world leaders in quantum computing. https://www.zdnet.com/article/unsw-use-flat-electron-shells-from-artificial-atoms-as-qubits/

Thus creating a simple 3dSystem?
Title: Re: How do we measure the energy of a photon?
Post by: Bored chemist on 24/07/2020 12:48:25
Now, please grow up a bit and answer the question I asked.




Quote from: Bored chemist on 01/07/2020 17:28:07
How do you cope with photons?
(Remember them? they were meant to be the topic).

A photon is its own antiparticle.


Title: Re: How do we measure the energy of a photon?
Post by: mxplxxx on 25/07/2020 20:31:29
There is a great battle going on in Computer Science at present between the proponents of Object Oriented Programming (OOP) and those of Functional Programming (FP).

See https://www.educba.com/functional-programming-vs-oop/ for a discussion of the two methods.

My first programming language (50 years ago) was Autocoder running on an IBM 1401 computer with 16K of memory. This was very much a functional programming language. So was its successor, COBOL which still runs on many, many mainframes in the world today. Then, in 1967, along came Simula which is now considered to be the first genuine OOP programming language. Today, Java, which is considered to be an OOP is the world's most popular programming language. Along the way, FP has to been pretty much neglected because it was thought that OOP was so much better. But this was a bit like throwing out the baby with the bath water.

Microfocus COBOL combined the functional aspects of COBOL with OOP feature to become a dominant mainframe programming language in the early part of this century. I personally thought it was the best programming language ever written. A 3dSystem combines the best of OOP and FP methods. Because 3dSystem is based on the way Reality works, this means that the Universe also uses this paradigm to function.

The Central system of a 3dSystem is functional in nature. It expands outwards from its centre via a hierarchy (HFSM) of 3dState objects (e.g. the Sun System). The outer system of a 3dSystem is the System itself (e.g. a Solar System). It is time-based and collapses inwards (e.g. Solar System, Earth System, Moon System). It incorporates all the relationships of Systems in the universe. This relationship gives rise to the often-discussed wave-particle nature of reality. The universe seems to have combined the wave-particle duality ability into one type of entity, a 3dSystem.

The 3dSubSystems of a Central System are actually references to Systems in lower level (of abstraction) shells. The 3dSubSystems of an Outer System are the subsystems themselves.

Thus, Reality has combined the best features of OOP and FP programming to produce the miracle we call the Universe.

BTW, it is debatable whether you are are subsystem in the earth system (i.e. a satellite of the earth) or a subsystem (substate) of the Earth (i.e. the Earth Central System) or something else entirely.
Title: Re: How do we measure the energy of a photon?
Post by: Bored chemist on 25/07/2020 21:11:43
Thus, Reality has combined the best features of OOP and FP programming to produce the miracle we call the Universe.
There really is no evidence supporting that assertion.
Title: Re: How do we measure the energy of a photon?
Post by: Bored chemist on 25/07/2020 21:12:50
Now, please grow up a bit and answer the question I asked.




Quote from: Bored chemist on 01/07/2020 17:28:07
How do you cope with photons?
(Remember them? they were meant to be the topic).

A photon is its own antiparticle.



Title: Re: How do we measure the energy of a photon?
Post by: mxplxxx on 28/07/2020 21:01:39
From https://scienceblog.com/517615/looking-into-the-black-box-of-deep-learning/?utm_source=feedburner&utm_medium=email&utm_campaign=Feed%3A+scienceblogrssfeed+%28ScienceBlog.com%29 (https://scienceblog.com/517615/looking-into-the-black-box-of-deep-learning/?utm_source=feedburner&utm_medium=email&utm_campaign=Feed%3A+scienceblogrssfeed+%28ScienceBlog.com%29)

"The intuition is that a hierarchical neural network should be better at approximating a compositional function than a single “layer” of neurons, even if the total number of neurons is the same. "

3dAbstractions uses this "intuition" to create that create the 3dSystems that can be used to simulate a universe. It is clear to me that the universe is a type of neural network. Far from being "intuition" the knowledge involved is likely to be only available to someone who has a deep understanding of both Reality/physics and Software Development.

In my case, that knowledge has taken over 50 years to develop. The complexity involved requires an older/mature brain to process. In turn, this requires good health. In my case, I have planned my health and, as a result, I have near-perfect health even though I am approaching 80.

Intuition IS involved in being able to "intuit"/feel whether a particular line of reasoning is right or not. I never publish my work unless I have a strong "eureka" feeling that it is right. 
Title: Re: How do we measure the energy of a photon?
Post by: mxplxxx on 07/08/2020 12:30:56
Quantum theory alerted us to the fact that the universe interacts via packets of energy called Quanta. A single packet is called a Quantum. Each Quantum contains a constant amount of Action (where E=h/t where E is the energy of the Quantum, h the Action (Planck's constant) and t is the time required to complete one cycle (revolution) of the Quantum). Once started, a Quantum interaction must complete. i.e. all interactions in the universe must occur in packets each packet containing h amount of Action.

This is standard physics. Personally, I cannot see why the frequency of a quantum cannot be modified, meaning that a Quantum can assume an infinite values of energy (but only one value, h, of Action)
 
This in effect tells us that Reality is all about Power.

This is very much like the 3dSystems of 3dAbstractions. A top-level function (i.e. method) cannot be interrupted once it has started. In other words, once a state change has started, it cannot be interrupted prior to completion. This is a great way of managing complexity. The entities that designed Reality (us from the future ? - see the fascinating Interstellar movie) were obviously very much aware of this.
Title: Re: How do we measure the energy of a photon?
Post by: mxplxxx on 22/08/2020 01:18:50
You may recall a discussion of the 3dApp System. This is a 3dSystem that runs an App. It contains standard top-level systems, 3dDisplay, 3dData and 3dDatastore. 3dDisplay is effectively your display interface.

I have recently introduced a 3dController system (a type of 3dSystem). Its purpose is to control another 3dSystem. It can take the place of a person when running a 3dApp. It is an "expert" on the System it is running. I.E. it "knows" how to run a 3dApp and often (but not always) does so via the 3dDisplay interface. This is 3dAbstraction's version of AI Automation. It would enable e.g. a 3dApp to run itself - as 3dRacing does via the SetAndForget function.

A 3dController can "see" and manipulate all of a system at once. It has a "sideways" view of the system (as well as the database/top-down black-box view of normal systems) meaning it has full access to the system's subsystems and the subsystems' subsystems, all the way down a system hierarchy. Such a System's functions can exist as whole units  making it easier to understand and locate them and having many parallels to an indivisible quantum.

Is there a similar setup in Reality? Sure, a person can be considered to be a 3dController system in Reality. Actually, a person is probably two controllers working together. A Brain controller that controls the body and a Body controller (the body itself) itself which controls its environment. Which likely makes us a symbiosis - similar to a mitochondria/cell. That is, a 3dSymbiosis is a 3dSystem controlling another 3dSystem.

It is interesting to note that 3dControllers are temporary systems and that so are people!
Title: Re: How do we measure the energy of a photon?
Post by: mxplxxx on 31/08/2020 21:25:10
In 3dRacing, the abstraction hierarchy for a Raceday is Raceday/Meeting/Race/Runner. Of these systems, only Runner exists in Reality. Is this an issue with 3dAbstractions as it pertains to a simulation of Reality? Not at all. Raceday, Meeting and Race exist as systems in a Brain system:) which is itself a Reality system. i.e. life has extended the capabilities of the original Reality system in ways that are only limited by our imaginations.

So, it would appear that systems in the universe are either "Real"/permanent or imaginary/temporary. An imaginary system "references" a "Real" system via an abstraction hierarchy. In a living organism, this type of system often occurs in a brain. A reference often happens via a vision system. The brain system also has the ability to create permanent systems via the body it inhabits.

So, the brain is a form of DNA-based artificial intelligence created via evolution. And the brain is now creating an advanced form of computer-based Artificial Intelligence. These intelligences complement each other for the purpose of evolving the universe.

In a 3dApp system (a type of 3dSystem), the Datastore top-level system is a permanent system. The Data top-level system of a 3dApp is a temporary system that references the Datastore system. It is created each time the 3dApp is run. The Data system can create Datastore systems. In this way, a 3dApp is based on Reality.

Actually, all systems in Reality are "Real", but some are "Real"er than others. Here are the different levels of "Real" a system may be in.

1. Lifespan

    May be Temporary or Permanent
   
2. Abstraction Level

    Depends on the level the system occupies in the abstraction hierarchy it belongs to. The lower the level, the more concrete a system is. The higher the level, the more abstract a system is.

3. Normality

    May have the following values:

    Artificial-DNA
    Artificial-IT
    Natural

4.  Originality

    Original
    Reference (points to the Original system)

    An Original System is one that can stand alone. Reference systems reference Original systems or other Reference systems. Events (IT) and Bosons (Reality) communicate state changes (happenings/past) and Change requests (functions/future) between systems. This is how time arises. For example, a reference system that references the Sun may exist. It will react to a photon from the Sun. That way reference systems can exists at various time scales and at various places in the universe that reference a single original system and, as far as the reference system is concerned IS the original system. Thus we "See" the Sun in our brains as a reference system.     

A reference allows a system to participate in multiple other systems simultaneously. In combination with the capabilities of an abstraction hierarchy, it turns Reality into a neural network. Your brain, possibly, will contain one system that represents the sun. This system will exist at a certain level of an abstraction hierarchy that includes the sun at the bottom (concrete part) of the hierarchy. The abstracted sun system may have references to it throughout the brain such that changes to it are propagated to all references and, conversely, changes to references are propagated to the abstracted sun system. This is the essence of neural network. 

As mentioned previously, references may explain the "spooky action at a distance" of  quantum entanglement. Reality seems to have the ability to timelessly propagate changes to entangled systems. This is probably necessary to maintain the integrity of Reality and mandates that state changes are timeless (i.e. instantaneous).
Title: Re: How do we measure the energy of a photon?
Post by: Bored chemist on 31/08/2020 21:28:37
Intuition IS involved in being able to "intuit"/feel whether a particular line of reasoning is right or not. I never publish my work unless I have a strong "eureka" feeling that it is right. 
It's a well known system.
Aristotle was known to use it.
https://www.wired.com/story/aristotle-was-wrong-very-wrong-but-people-still-love-him/

Title: Re: How do we measure the energy of a photon?
Post by: mxplxxx on 16/09/2020 22:55:33
I have added 3dCurrent to the widening range of 3dSystem types in 3dAbstractions. This is a temporary system that comes between a 3dController and a 3dData system. Its purpose is to enable the composite manipulation of a branch of the 3dData system.

In 3dRacing, the Raceday system is a top-level 3dData system that has a branch containing Pool, Race and Strategy systems. Tote system is a 3dContraller system that manipulates this branch via the RaceDay.Current system. e.g. one of Raceday. Current's functions is RunSetAndForget.

It equivalent in Reality may be a Gluon (or it may be a virtual particle, given its temporary nature).

It enables multiple instances of the same branch in a neural network to communicate simultaneously with original (permanent) systems.
Title: Re: How do we measure the energy of a photon?
Post by: mxplxxx on 05/10/2020 12:33:51
The following is a water molecule built with 3dSystems.

A water molecule built with 3dSystems has a very nice result - the outer shell is full (8 electrons):).

Two views are presented merely for clarity. The first is minus shells and the second includes shells.

Water.System (water molecule)
    Water.Star
      Property: State (liquid, gas, solid (ice))
    Subsystems
        H2.System (Hydrogen molecule)
            H2.Star
            Subsystems
                Hydrogen.System 1 (Hydrogen atom)
                    Hydrogen.Star (Hydrogen ion)
                        Subsystems
                            Proton.System
                    Subsystems
                        Electron.System
                            Electron.Star
                Hydrogen.System 2 (Hydrogen atom)
                    Hydrogen.Star (Hydrogen ion)
                        Subsystems
                            Proton.system
                    Subsystems
                        Electron.System
                            Electron.Star
        Oxygen.System (Oxygen atom)
            Oxygen.Star (Oxygen ion)
                Subsystems
                    Proton.system 1
                    Proton.system 2
                    Proton.system 3
                    Proton.system 4
                    Proton.system 5
                    Proton.system 6
                    Proton.system 7
                    Proton.system 8
            Subsystems
                Electron.System 1
                    Electron.Star
                    Subsystems
                        Electron.System 1
                            Electron.Star
                        Electron.System 2
                            Electron.Star
                        Electron.System 3
                            Electron.Star
                Electron.System 2
                    Electron.Star
                    Subsystems
                        Electron.System 1
                            Electron.Star
                        Electron.System 2
                            Electron.Star
                        Electron.System 3
                            Electron.Star


A view of this system including Shells is as follows:

Shell 0
    Water.System
Shell 1
    H2.System Oxygen.System
Shell 2
    Hydrogen.Systems (2) Electron.Systems (2 from Oxygen.System)
Shell 3
    Electron.Systems (8 - 2 from the 2 Hydrogen.Systems and 6 from Oxygen.System)

This configuration of 3dSystems is obtained in 3dAbstractions when using references (VB Byref) to 3dSystems in place of of 3dSystem themselves (VB Byval). The more familiar, H+/OH- arrangement of ions is obtained when we use 3dSystems themselves rather than references to them. It is possible that this nice result is an indication that Reality also makes use of references.

Note 1: Protons are shown serially. They are likely to occur in an identical configuration to electrons.
Note 2: With this system, Hydrogen molecules can flow from water molecule to water molecule, forming the waves in water.

An Electron is a "Byval" 3dSystem. Chances are it contains subsystems (possibly neutrinos) which, in turn, themselves contain subsystems possibly ad infinitum. A Proton is a "Byref" 3dSubsystem. It references an Electron. A change in state of an Electron is detected immediately via the "Byref" nature of by its associated Proton which then calls a 3dFfunction. 3dFunctions are possibly Neutron 3dSystems in Reality. Neutrons likely reside in the Star (or central or Ion) 3dSystem of a 3dSystem.

Thus, a Proton is a reference to an Electron and is thus effectively the Electron itself. This is likely be the case for all situations involving particles with opposite charges..

Coming up. How 3dStars (central systems)  makes 3dSystems intelligent and aware (its all about Reference 3dSystems - quantum entanglement - and how this system type enables photons to leap across the universe in no time at all)
[/quote]
Title: Re: How do we measure the energy of a photon?
Post by: Bored chemist on 05/10/2020 12:57:33
The following is a water molecule built with 3dSystems.

A water molecule built with 3dSystems has a very nice result - the outer shell is full (8 electrons):).

Two views are presented merely for clarity. The first is minus shells and the second includes shells.

Water.System (water molecule)
    Water.Star
      Property: State (liquid, gas, solid (ice))
    Subsystems
        H2.System (Hydrogen molecule)
            H2.Star
            Subsystems
                Hydrogen.System 1
                    Hydrogen.Star (Hydrogen ion)
                        Subsystems
                            Proton.System
                    Subsystems
                        Electron.System
                            Electron.Star
                Hydrogen.System 2
                    Hydrogen.Star (Hydrogen ion)
                        Subsystems
                            Proton.system
                    Subsystems
                        Electron.System
                            Electron.Star
        Oxygen.System (Oxygen atom)
            Oxygen.Star (Oxygen ion)
                Subsystems
                    Proton.system 1
                    Proton.system 2
                    Proton.system 3
                    Proton.system 4
                    Proton.system 5
                    Proton.system 6
                    Proton.system 7
                    Proton.system 8
            Subsystems
                Electron.System 1
                    Electron.Star
                    Subsystems
                        Electron.System 1
                            Electron.Star
                        Electron.System 2
                            Electron.Star
                        Electron.System 3
                            Electron.Star
                Electron.System 2
                    Electron.Star
                    Subsystems
                        Electron.System 1
                            Electron.Star
                        Electron.System 2
                            Electron.Star
                        Electron.System 3
                            Electron.Star


A view of this system including Shells is as follows:

Shell 0
    Water.System
Shell 1
    H2.System Oxygen.System
Shell 2
    Hydrogen.Systems (2) Electron.Systems (2 from Oxygen.System)
Shell 3
    Electron.Systems (8 - 2 from the 2 Hydrogen.Systems and 6 from Oxygen.System)

This configuration of 3dSystems is obtained in 3dAbstractions when using references (VB Byref) to 3dSystems in place of of 3dSystem themselves (VB Byval). The more familiar, H+/OH- arrangement of ions is obtained when we use 3dSystems themselves rather than references to them. It is possible that this nice result is an indication that Reality also makes use of references.

Note 1: Protons are shown serially. They are likely to occur in an identical configuration to electrons.
Note 2: With this system, Hydrogen molecules can flow from water molecule to water molecule, forming the waves in water.
When you can calculate the melting point, let us know.
Title: Re: How do we measure the energy of a photon?
Post by: Bored chemist on 05/10/2020 12:58:32
With this system, Hydrogen molecules can flow from water molecule to water molecule,
... which is physically wrong.
Title: Re: How do we measure the energy of a photon?
Post by: mxplxxx on 15/10/2020 08:09:47
A 3dStar system is the central system of a 3dSystem. It is a type of 3dSystem that has one main difference to a normal 3dSysytem - it accesses subsystems via references (VB Byref key word). This gives it the ability to expand and contract - something not available to a normal 3dSystem. It can call functions in its subsystems via references and react to events in its subsystems via references. This means that a single 3dSystem can participate in multiple systems via references. Everything that happens via references does so immediately, meaning 3dStar systems are time-independent (but note that it still takes time for a reference to get from point a to point b).

References can be associated with event handlers meaning events can be raised by a "byval" system and handled immediately by a reference system. Could Reality handle a Photon in a similar manner, meaning Photons don't travel at all? I suspect so, given that physics cannot prove that Photons actually travel. 

A brain is probably a type of 3dStar (in a Person System), as is a Sun (in a Solar System). Protons in the atoms of Reality are probably references (to Electrons). References in Reality are probably behind the spooky "action at a distance" quantum phenomenon.

A brain contains the highest levels of abstraction in a "Person" system. This makes it highly intelligent. The subsystems in this system are respiratory system, digestive system etc. In a "person" simulation these subsystems would be declared Byval. Because it is a 3dStar type system, a brain can expand and contract. A bigger skull, would therefore make a person more abstract/intelligent, as evolution has noted! The brain will contain a top-level system where events of the highest level of abstraction from its subsystems are processed. This is where awareness resides.

What this all points to is that the systems of Reality, from the universe itself to the smallest particle, are both intelligent and aware. Mind boggling.

When the blazing sun is gone,
When he nothing shines upon,
Then you show your little light,
Twinkle, twinkle, all the night.
Twinkle, twinkle, little star,
How I wonder what you are!


Coming soon. How a 3dSystem manages time and how this might be applicable to Reality.
Title: Re: How do we measure the energy of a photon?
Post by: Bored chemist on 15/10/2020 08:42:54
Do you write other poetry?
Title: Re: How do we measure the energy of a photon?
Post by: mxplxxx on 20/10/2020 13:50:33
Most of the complexities, and therefore problems, with computer programming are due to time-based dependencies. 3dAbstractions avoids this by using a form of programming I call POP; Present Oriented Programming. For example, AverageOdds is a Readonly Property of the TransactionHistory  System. In this form of programming, the past is preserved as values of the present, i.e. memories. In Visual Basic:

Code: [Select]
Public ReadOnly Property AverageOdds() As Decimal

        Get
            Dim totalodds As Decimal = 0

            For Each trn As Transaction.Star In Me.Transactions
                totalodds += trn.Odds
            Next

            Return math.round(totalodds / me.Transactions.Count, 2)
        End Get

End Property

This is the way Reality works. If we need to recall the past we preserve it as aspects of the present. A photo is a perfect example of this. As is a Photon. It is highly unlikely, therefore, that spacetime exists except as some aspect of the present. Reality doesn't need to worry about the past. You could say Reality is mindful:). It also probably means that time is just an emotion that results from experiencing memories.

In fact, given that everything that exists in the present once existed in the past, it could be surmised that the Universe is just some entity's memory. It is highly likely that Reality is language coded in memories and coupled to run-time systems - i.e. Reality is a computer:)

Given, as proposed above, that the past is an aspect of the present and the future can be extrapolated from the current state of the present, it would seem that all there is to Reality is an eternal present i.e. an expanded big-bang singularity. In software, the future state of an app is determined by the app's code/data in combination with the run-time system. It is likely a similar situation exists for the Universe and physics will be enhanced when we know what aspects are code/data and what aspects are run-time. In the process, we will probably discover a universal language that is "executed" by the universal run-time system.     

A 3dSystem is created from the top down. i.e. its most abstract level (the topmost) is created first followed by the next most abstract level and so on. In Reality, the same process is also likely. A human embryo is likely formed similarly. The topmost level in Reality will likely be created as a singularity. As lower levels are created, so the upper levels expand. This is probably the cause of the expanding universe, i.e. lower levels being created all the time. It is possible the universe is in the process of "growing up".

A 3dSystem can both execute itself and create its own subsystems. Because of their recursive nature you only need one system and you can have as many as you need, limited only by the amount of space available. This type of processing in computer science is called SOA/Microservices (see https://www.ibm.com/cloud/blog/soa-vs-microservices (https://www.ibm.com/cloud/blog/soa-vs-microservices). Reality works similarly. A particle is effectively a microservice. i.e. once you have one system in Realty, you have a Universe (or, in fact, a Cosmos/Multiverse)
Title: Re: How do we measure the energy of a photon?
Post by: Bored chemist on 20/10/2020 14:21:59
This is the way Reality works.
LOL
Title: Re: How do we measure the energy of a photon?
Post by: mxplxxx on 31/12/2020 08:25:43
You may remember when I postulated that black holes are at the centre of all systems. The following research may back up this proposition https://www.slashgear.com/researchers-believe-dark-matter-could-be-composed-of-primordial-black-holes-30652903/ (https://www.slashgear.com/researchers-believe-dark-matter-could-be-composed-of-primordial-black-holes-30652903/).

In 3dAbstractions, black holes correspond to 3dStar objects. A 3dStar is a central 3dSystem. It is timeless and contains the state of the system the 3dStar is central to. It can also be a Hierarchical Finite State Machine that changes when an event is received from the outer system. A black hole is likely to behave similarly. An event for a black hole would likely be a system falling into it.

I think I also postulated that the total mass of a 3dSystem is the sum of the masses of itself (the 3dStar) plus the masses of its 3dSubsystems. Given that a 3dSystem describes any system in the the universe, then it follows that 3dStars and therefore black holes are included in the mass of the universe.
Title: Re: How do we measure the energy of a photon?
Post by: Bored chemist on 31/12/2020 10:24:10
You may remember when I postulated that black holes are at the centre of all systems.
Yes.
You offered no evidence, so we laughed at you.

The following research may back up this proposition
Or not...
" Nevertheless, tight limits on their abundance have been set up from various astrophysical and cosmological observations, so that it is now excluded that they contribute significantly to dark matter over most of the plausible mass range."
From
https://en.wikipedia.org/wiki/Primordial_black_hole

Title: Re: How do we measure the energy of a photon?
Post by: mxplxxx on 18/01/2021 06:47:08
In a 3dSystem , an outer/enclosing system, in conjunction with subsystems, supplies structure to the system and a central system provides function (normally via program functions but also via HFSM's in more complicated cases). The equivalent in Reality is e.g. an atom where the atom itself in conjunction with electron subsystems supplies structure to the system and a central ion supplies function in the form of protons.

This is not a hard and fast distinction as all systems will have a mixture of structure and function.

So, basically the Universe consists of two types of HFSM's/Systems. One relating to structure (what has state) and the other relating to function (how state is modified). e.g. in the solar system, the Sun is related mainly to function/state (as evidenced by light which is related to the fusion of the atoms that make up the sun) while the planets are related mainly to structure.

In 3dRacing, 3dSystems Meeting.System, Race.System and Runner.System specify a HFSM that defines the structure of a Raceday whilst a 3dStar (central system) Race.3dStar defines the states of a Race (such as InProgress, Completed etc.) . A 3dStar is sometimes implemented by a UML State Machine but, for simplicity and readability, mostly via functions (see https://1drv.ms/u/s!AkkAzGDByUeBmJIdrDw6iIInoiS_PQ?e=OPQC4B (https://1drv.ms/u/s!AkkAzGDByUeBmJIdrDw6iIInoiS_PQ?e=OPQC4B)
Title: Re: How do we measure the energy of a photon?
Post by: Bored chemist on 18/01/2021 08:47:57
The equivalent in Reality
You have offered no evidence that there is any meaningful equivalence.
Title: Re: How do we measure the energy of a photon?
Post by: mxplxxx on 05/03/2021 07:46:36
https://www.popularmechanics.com/science/a35718463/scientists-say-physical-warp-drive-is-possible/

Faster than light - by using "bubbles" of spacetime. Sounds suspiciously like the Systems of spacetime I proposed some time back. The author of the article states "The physical model uses almost none of the negative energy and capitalizes on the idea that spacetime bubbles can behave almost however they like." - just like 3dSystems.

Actually the warp drive proposed by the above scientists works via the drive boundary. I have previously proposed system boundaries where photons are scaled up/down. See https://www.thenakedscientists.com/forum/index.php?topic=75960.msg602604;topicseen#msg602604.

Not too sure whether a fermion (i.e. matter) could cross a system boundary.  Actually the 3dSubSystems of a 3dSystem cannot cross a system boundary. But references to it can. It is possible that the system boundaries of Reality can convert subsystems into references (photons) and that the photons/references, being massless, can travel faster than the speed of light and that the referenced system will be able to experience  whatever the photon/reference experiences.  This is basically the way a 3dApp would work.
Title: Re: How do we measure the energy of a photon?
Post by: Bored chemist on 05/03/2021 08:32:00
ounds suspiciously like the Systems of spacetime I proposed some time back.
It only sounds that way to you, but this does illustrate the problem.
If your idea was any use then you would have been able to predict FTL travel.
But  you didn't.
Because your model is useless.
Title: Re: How do we measure the energy of a photon?
Post by: mxplxxx on 02/04/2021 13:38:48
A system in 3dAbstractions can be duplicated, but it will then be two physically separate systems. The following code for a PoolRace.System creates two physically separate systems that are, in effect, the same system.

Code: [Select]
    Namespace PoolRace

        Public Class System  '  PoolRace.System

              Inherits _3dLibrary._3dSystem
 
             ' create a normal PoolRace System
 
             Public Sub New(PoolRace As PoolRace, Optional DataStore As _3dLibrary.DataStore.System = Nothing)

                MyBase.New(Type:=eSystemTypes.POOLRACE, SubType:=eSystemSubTypes.DATA, PoolRace:=PoolRace, Changeable:=False, Addable:=False, Deleteable:=False, DataStore:=DataStore)

             End Sub

             ' create a reference where all updates/queries are made to original
             ' SubSystems and Star of new PoolRace.System are set to those of original PoolRace.System - effctely making the two systems identical
 
             Public Sub New(PoolRace As System, Optional DataStore As _3dLibrary.DataStore.System = Nothing)

                 MyBase.New(Type:=eSystemTypes.POOLRACE, SubType:=eSystemSubTypes.DATA, Star:=PoolRace.PoolRace, SubSystems:=PoolRace.SubSystems, Changeable:=False, Addable:=False, Deleteable:=False, DataStore:=DataStore)

              End Sub

            Function Demo() as Boolean

        Dim originalrace as PoolRace.System = New PoolRace.System(PoolRace:=New PoolRace.PoolRace(Raceno::=1, Type;=:"R", StartTime = "01/04/2021"))
                Dim race1  as PoolRace.System = New PoolRace.System(PoolRace:=originalrace)
                Dim race2  as PoolRace.System = New PoolRace.System(PoolRace:=originalrace)

        race1.Type = "H"

                Print("originalrace.Type is " & originalrace.Type) '  (will display "originalraceType is H")
                Print("race1.Type is " & race1.Type)               '  (will display "race1.Type is H")
                Print("race2.Type is " & race2.Type)               '  (will display "race2.Type is H")

           End Function

        End Class

    End Namespace

The same code allows two systems in a 3dAbstrations simulation of Reality to be entangled. It is likely that a similar construct exists in Reality to allow quantum entanglement. In other words two entangled particles of light are different physical entities but are, in effect, identical,  when the code above is applied to the entangled (versus the original) particle.

The reason for entanglement is to allow multiple physically separate systems to represent a single original system. It likely allows separate areas of a human brain to access or react to changes in a single central system (and maybe forms the basis for a neural network and imagination/dreams). Maybe ditto for Reality.

In a 3dSystem, the Subsystems field handles links to other systems. A link can be a another System or it can be a reference to another System. In the first case, the system is contracting as it gets more concrete and in the second case the system is expanding as it gets more concrete. This may also be the case in Reality, in which case references will be used to create intelligent systems (like proteins).

It is likely that proteins are the universe's equivalent of computer program references.   
 
Title: Re: How do we measure the energy of a photon?
Post by: mxplxxx on 06/04/2021 21:26:54
New research shows how the brain is organized and functions in a way that is pretty much the same as I postulated in 3dSystems some time ago.

https://scienceblog.com/522140/new-blueprint-of-brain-connections-reveals-extensive-reach-of-central-regulator/?utm_source=feedburner&utm_medium=email&utm_campaign=Feed%3A+scienceblogrssfeed+%28ScienceBlog.com%29 (https://scienceblog.com/522140/new-blueprint-of-brain-connections-reveals-extensive-reach-of-central-regulator/?utm_source=feedburner&utm_medium=email&utm_campaign=Feed%3A+scienceblogrssfeed+%28ScienceBlog.com%29)

Along the same lines, the following article postulates that a brain contains black holes which is something I postulated was the case for all systems. It also postulates that consciousness is timeless, something which is likely if systems contain a central system that is pure state - which is something that I have also previously postulated.

https://www.sydney.edu.au/news-opinion/news/2019/09/12/new-insights-into-brains-and-black-holes.html (https://www.sydney.edu.au/news-opinion/news/2019/09/12/new-insights-into-brains-and-black-holes.html)



Title: Re: How do we measure the energy of a photon?
Post by: hamdani yusuf on 06/04/2021 23:25:52
New research shows how the brain is organized and functions that is pretty much the same as I postulated in 3dSystems some time ago.

https://scienceblog.com/522140/new-blueprint-of-brain-connections-reveals-extensive-reach-of-central-regulator/?utm_source=feedburner&utm_medium=email&utm_campaign=Feed%3A+scienceblogrssfeed+%28ScienceBlog.com%29 (https://scienceblog.com/522140/new-blueprint-of-brain-connections-reveals-extensive-reach-of-central-regulator/?utm_source=feedburner&utm_medium=email&utm_campaign=Feed%3A+scienceblogrssfeed+%28ScienceBlog.com%29)

What are the similarities?  What are the differences?
Title: Re: How do we measure the energy of a photon?
Post by: mxplxxx on 06/04/2021 23:40:55
New research shows how the brain is organized and functions that is pretty much the same as I postulated in 3dSystems some time ago.

https://scienceblog.com/522140/new-blueprint-of-brain-connections-reveals-extensive-reach-of-central-regulator/?utm_source=feedburner&utm_medium=email&utm_campaign=Feed%3A+scienceblogrssfeed+%28ScienceBlog.com%29 (https://scienceblog.com/522140/new-blueprint-of-brain-connections-reveals-extensive-reach-of-central-regulator/?utm_source=feedburner&utm_medium=email&utm_campaign=Feed%3A+scienceblogrssfeed+%28ScienceBlog.com%29)

What are the similarities?  What are the differences?
According to the research, the brain has a central system, that directs commands to downstream centres and forwards the results to upstream  processors. This is pretty much the way a _3dSystem works with a Central Star, Downstream Subsystems and a link to the Parent (which can also connect with other upstream systems).

From the article:

“The new findings led by Dr. McElvain offer an important lesson in motor control,” said Kleinfeld, a professor in the Division of Biological Sciences (Section of Neurobiology) and Division of Physical Sciences (Department of Physics). “The brain does not control movement though a hierarchy of commands, like the ‘neural networks’ of self-driving cars, but through a scheme of middle management that directs motor output while informing the executive planners.”
Title: Re: How do we measure the energy of a photon?
Post by: hamdani yusuf on 07/04/2021 03:23:17
According to the research, the brain has a central system, that directs commands to downstream centres and forwards the results to upstream  processors. This is pretty much the way a _3dSystem works with a Central Star, Downstream Subsystems and a link to the Parent (which can also connect with other upstream systems).
How is this related to the title of this thread?
It seems like you are trying to build an accurate model of reality, just like my own thread about building a virtual universe. What's your motivation to do it? What's the terminal goal? What are the instrumental goals to achieve that?
Title: Re: How do we measure the energy of a photon?
Post by: mxplxxx on 07/04/2021 03:50:44
According to the research, the brain has a central system, that directs commands to downstream centres and forwards the results to upstream  processors. This is pretty much the way a _3dSystem works with a Central Star, Downstream Subsystems and a link to the Parent (which can also connect with other upstream systems).
How is this related to the title of this thread?
It seems like you are trying to build an accurate model of reality, just like my own thread about building a virtual universe. What's your motivation to do it? What's the terminal goal? What are the instrumental goals to achieve that?

I cannot change the title. If I could it would be "A TOE based on hierarchical abstractions". I am not willing to discuss my goals in posting the theory.
Title: Re: How do we measure the energy of a photon?
Post by: hamdani yusuf on 08/04/2021 05:56:02
I cannot change the title. If I could it would be "A TOE based on hierarchical abstractions".
You could start a new thread instead. If you think that some posts here are critical for discussing your new topic, you can just quote them there.
Title: Re: How do we measure the energy of a photon?
Post by: mxplxxx on 08/04/2021 06:09:34
I cannot change the title. If I could it would be "A TOE based on hierarchical abstractions".
You could start a new thread instead. If you think that some posts here are critical for discussing your new topic, you can just quote them there.
Good advice. Thanks:)
Title: Re: How do we measure the energy of a photon?
Post by: mxplxxx on 14/05/2021 17:04:37
Everything in the universe is a 3dSystem. A Virus is a 3dSystem. A Cell is a 3dSystem. So a Cell is a type of Virus. Is it possible that Viruses are an integral part of evolution? I think so. Chances are Cells are collections of Viruses that can reproduce themselves. I.E. Cells are an evolution of Viruses and Viruses continue to exist to carry out or assist in cellular functions, one of which is to do cellular updates - i.e. transfer beneficial mutations between living entities.

More on this later in the post when I will supply the 3dAbstractions structure of a Cell and also a Virus and demonstrate how alike they are.
Title: Re: How do we measure the energy of a photon?
Post by: Bored chemist on 14/05/2021 17:11:42
Everything in the universe is a 3dSystem.
No.
Spacetime is a 4 D system.
Plenty of my time at work used to involve 256D systems.
Title: Re: How do we measure the energy of a photon?
Post by: mxplxxx on 22/05/2021 21:28:27
https://1drv.ms/u/s!AkkAzGDByUeBm7gPkFEg2-6nqO_YqA?e=EF4M6K (https://1drv.ms/u/s!AkkAzGDByUeBm7gPkFEg2-6nqO_YqA?e=EF4M6K)

The image is that of a Sars-Cov-2 (COVID-19) virus as simulated in my 3dVirus (not for sale) program. As far as Bases go, I have included examples only (the virus has thousands of Bases). Each of the nodes displayed is a 3dSystem. It illustrates what I contend, that everything in Reality can be simulated via a 3dSystem.

The following is the high-level code I used to create the virus
Code: [Select]
Namespace SarsCov2

    Public Enum eVersion

        CURRENT_VERSION = 1

    End Enum


    ' SarsCov2.System

    Public Class system
        Inherits _3dLibrary._3dSystem

        ' create a blank Sars-Cov-2 virus system

        Public Sub New(SarsCov2 As SarsCov2.Star, Optional Changeable As Boolean = True, Optional Addable As Boolean = False, Optional Deleteable As Boolean = False, Optional DataStore As _3dLibrary.DataStore.System = Nothing)

            MyBase.New(Type:=eSystemTypes.SARSCOV2, SubType:=eSystemSubTypes.DATA, Star:=SarsCov2, SubSystems:=New SubSystems, StarList:=New List(Of SarsCov2.Star), Changeable:=Changeable, Addable:=Addable, Deleteable:=Deleteable, DataStore:=DataStore)


        End Sub


        Public Sub PopulateMe()

            Dim evirus As eVirus.System = Me.AddSubSystem(New eVirus.System(New eVirus.Star()))

            With evirus.AddSubSystem(New Projections.System(New Projections.Star()))
                CType(.AddSubSystem(New Projection.System(New Projection.Star(ProjectionNo:=1))), Projection.System).PopulateMe(eProtein:=New eProtein.Star(), Spike:=New Spike.Star(), mProtein:=New mProtein.Star)
                CType(.AddSubSystem(New Projection.System(New Projection.Star(ProjectionNo:=2))), Projection.System).PopulateMe(eProtein:=New eProtein.Star(), Spike:=New Spike.Star(), mProtein:=New mProtein.Star)
                CType(.AddSubSystem(New Projection.System(New Projection.Star(ProjectionNo:=3))), Projection.System).PopulateMe(eProtein:=New eProtein.Star(), Spike:=New Spike.Star(), mProtein:=New mProtein.Star)
            End With

            Dim cvirus As cVirus.System = evirus.AddSubSystem(New cVirus.System(New cVirus.Star(Capsid:=_3dVirus.cVirus.eCapsid.nProtein.ToString)))

            Dim nvirus As nVirus.System = cvirus.AddSubSystem(New nVirus.System(New nVirus.Star()))

            With nvirus
                With .AddSubSystem(New Genome.System(New Genome.Star(Rna:=True, SingleStranded:=True, Circular:=False)))
                    With .AddSubSystem(New Segments.System(New Segments.Star()))
                        With .AddSubSystem(New Segment.System(New Segment.Star(SegmentNo:=1)))
                            With .AddSubSystem(New Bases.System(New Bases.Star()))
                                With .AddSubSystem(New Base.System(New Base.Star(BaseNo:=1)))
                                    With .AddSubSystem(New Nucleotide.System(New Nucleotide.Star(NucleotideNo:=1)))
                                        .AddSubSystem(New Phosphate.System(New Phosphate.Star()))
                                        .AddSubSystem(New Sugar.System(New Sugar.Star()))
                                        .AddSubSystem(New Cytosine.System(New Cytosine.Star()))
                                    End With
                                    With .AddSubSystem(New Nucleotide.System(New Nucleotide.Star(NucleotideNo:=2)))
                                    End With
                                End With

                                With .AddSubSystem(New Base.System(New Base.Star(BaseNo:=2)))
                                    With .AddSubSystem(New Nucleotide.System(New Nucleotide.Star(NucleotideNo:=1)))
                                        .AddSubSystem(New Phosphate.System(New Phosphate.Star()))
                                        .AddSubSystem(New Sugar.System(New Sugar.Star()))
                                        .AddSubSystem(New Uracil.System(New Uracil.Star()))
                                    End With
                                    With .AddSubSystem(New Nucleotide.System(New Nucleotide.Star(NucleotideNo:=2)))
                                    End With
                                End With
                            End With
                        End With

                        With .AddSubSystem(New Segment.System(New Segment.Star(SegmentNo:=2)))
                            With .AddSubSystem(New Bases.System(New Bases.Star()))
                                With .AddSubSystem(New Base.System(New Base.Star(BaseNo:=1)))
                                    With .AddSubSystem(New Nucleotide.System(New Nucleotide.Star(NucleotideNo:=1)))
                                        .AddSubSystem(New Phosphate.System(New Phosphate.Star()))
                                        .AddSubSystem(New Sugar.System(New Sugar.Star()))
                                        .AddSubSystem(New Adeine.System(New Adeine.Star()))
                                    End With
                                    With .AddSubSystem(New Nucleotide.System(New Nucleotide.Star(NucleotideNo:=2)))
                                    End With
                                End With

                                With .AddSubSystem(New Base.System(New Base.Star(BaseNo:=2)))
                                    With .AddSubSystem(New Nucleotide.System(New Nucleotide.Star(NucleotideNo:=1)))
                                        .AddSubSystem(New Phosphate.System(New Phosphate.Star()))
                                        .AddSubSystem(New Sugar.System(New Sugar.Star()))
                                        .AddSubSystem(New Guanine.System(New Guanine.Star()))
                                    End With
                                    With .AddSubSystem(New Nucleotide.System(New Nucleotide.Star(NucleotideNo:=2)))
                                    End With
                                End With
                            End With
                        End With
                    End With
                End With
            End With

        End Sub
        Public Overloads Function getHeader() As String
            Return Parent.Id & " " & Id
        End Function

    End Class

    <Serializable()>
    Public Class Star
        Inherits _3dLibrary._3dStar

        Public Sub New()

            MyBase.New()

        End Sub

        Public Sub New(Properties As Properties)

            MyBase.New(Properties:=Properties)

        End Sub

        ' clone

        Public Sub New(SarsCov2 As Star, Optional Decrypt As Boolean = False, Optional BypassParent As Boolean = True, Optional BypassReadonly As Boolean = False)

            MyBase.New(Properties:=Properties.GetProperties(pObject:=SarsCov2, pEncrypt:=Decrypt, BypassParent:=BypassParent, BypassReadonly:=BypassReadonly))

        End Sub


        Public Overloads Sub Sync(SarsCov2 As SarsCov2.Star)

            With SarsCov2
            End With

        End Sub

        Public Overrides Function getId() As String
            Return Me.getKey
        End Function

        Public Overrides Function getKey() As String
            Return system.getSystemName(eSystemTypes.SarsCov2) & Me.KeyQualifier
        End Function

        Public Overrides Function Valid() As String

            'If Trim(Name) = "" Then
            '    Return "Error: Name cannot be blank"
            'Else
            Return ""
            ' End If

        End Function

        Public Overrides Sub ApplyFormatting(DetailsList As DetailsList)

        End Sub

        Public Overrides Sub ApplyFormatting(PageList As PageList)

        End Sub

    End Class


    ' subsystems

    Public Class SubSystems
            Inherits _3dLibrary._3dSubSystems

            Public Sub New()
                MyBase.New()

                ' for e.g. Race.System this would be:
                ' Add(System.getSystemName(eSystemTypes.RUNNERS), New Runners.System(New Runners.Star))

            End Sub

        End Class

    End Namespace

Title: Re: How do we measure the energy of a photon?
Post by: CrazyScientist on 22/05/2021 21:48:53
https://1drv.ms/u/s!AkkAzGDByUeBm7gPkFEg2-6nqO_YqA?e=EF4M6K (https://1drv.ms/u/s!AkkAzGDByUeBm7gPkFEg2-6nqO_YqA?e=EF4M6K)

The image is that of a Sars-Cov-2 (COVID-19) virus as simulated in my 3dVirus (not for sale) program. As far as Bases go, I have included examples only (the virus has thousands of Bases). Each of the nodes displayed is a 3dSystem. It illustrates what I contend, that everything in Reality can be simulated via a 3dSystem.
All physical processes are taking place in time. Without time it's not possible to express changes in the natural environment. Sure, we can discuss what is the nature of time - is it a dimension or just an abstract concept? But we have to use time to describe the physical reality as it is...
Title: Re: How do we measure the energy of a photon?
Post by: mxplxxx on 22/05/2021 22:17:33
https://1drv.ms/u/s!AkkAzGDByUeBm7gPkFEg2-6nqO_YqA?e=EF4M6K (https://1drv.ms/u/s!AkkAzGDByUeBm7gPkFEg2-6nqO_YqA?e=EF4M6K)

The image is that of a Sars-Cov-2 (COVID-19) virus as simulated in my 3dVirus (not for sale) program. As far as Bases go, I have included examples only (the virus has thousands of Bases). Each of the nodes displayed is a 3dSystem. It illustrates what I contend, that everything in Reality can be simulated via a 3dSystem.
All physical processes are taking place in time. Without time it's not possible to express changes in the natural environment. Sure, we can discuss what is the nature of time - is it a dimension or just an abstract concept? But we have to use time to describe the physical reality as it is...
Time is not a requirement in most computer programs. If is is required, it is done via a Timer event  or a Wait function. Time in Reality is merely perception (of which we know little) which, in turn, depends on the level of abstraction that the perceiving system exists at.
Title: Re: How do we measure the energy of a photon?
Post by: CrazyScientist on 22/05/2021 22:26:44
https://1drv.ms/u/s!AkkAzGDByUeBm7gPkFEg2-6nqO_YqA?e=EF4M6K (https://1drv.ms/u/s!AkkAzGDByUeBm7gPkFEg2-6nqO_YqA?e=EF4M6K)

The image is that of a Sars-Cov-2 (COVID-19) virus as simulated in my 3dVirus (not for sale) program. As far as Bases go, I have included examples only (the virus has thousands of Bases). Each of the nodes displayed is a 3dSystem. It illustrates what I contend, that everything in Reality can be simulated via a 3dSystem.
All physical processes are taking place in time. Without time it's not possible to express changes in the natural environment. Sure, we can discuss what is the nature of time - is it a dimension or just an abstract concept? But we have to use time to describe the physical reality as it is...
Time is not a requirement in most computer programs. If is is required, it is done via a Timer event  or a Wait function. Time in Reality is merely perception (of which we know little) which, in turn, depends on the level of abstraction that the perceiving system exists at.
When you execute any algorythm it runs in time. All lines in a code are being executed in a specific order - and order of events is what specifies a timeline. Commends like "repeat until" or "if/then" need some kind of timeline to work. Without time everything you can get is a static image, but not a working simulation
Title: Re: How do we measure the energy of a photon?
Post by: CrazyScientist on 22/05/2021 22:36:49
Quote
Time in Reality is merely perception (of which we know little) which, in turn, depends on the level of abstraction that the perceiving system exists at

As I said, we can discuss what is the nature of time, but the physical reality IS changing in time. Without time you can't define any measurable change. Do we age in time? Yes we do. Aging is a continuous process, which take place in time. Try to describe the aging process without using the concept of time...
Title: Re: How do we measure the energy of a photon?
Post by: mxplxxx on 22/05/2021 22:59:47
Try to describe the aging process without using the concept of time...
AS you say, time is a concept, i.e. it is not "Real" (a good reason to suspect Reality is a computer program).  A virus goes through various stages during Replication (e.g. Attachment, Transcription etc.). These are im[plemented via a series of functions in the 3dVirus program. There is no time involved.

ps aging can be described via a series of events.   
Title: Re: How do we measure the energy of a photon?
Post by: CrazyScientist on 22/05/2021 23:12:32
Try to describe the aging process without using the concept of time...
AS you say, time is a concept, i.e. it is not "Real" (a good reason to suspect Reality is a computer program).  A virus goes through various stages during Replication (e.g. Attachment, Transcription etc.). These are im[plemented via a series of functions in the 3dVirus program. There is no time involved.

ps aging can be described via a series of events.   
"AS you say, time is a concept, i.e. it is not "Real"" - I didn't say it is or isn't. I said it's a matter of discussion (which has probably more to do with philosophy than actual physics).

"A virus goes through various stages during Replication" - yes and when you describe those stages you define a specific timeline of events: something happening before or after a given stage

"ps aging can be described via a series of events." - yes and series of events is what makes a timeline.

Here's another example - velocity of motion, which is expressed as a distance which is being passed in time - e.g. km/h
Title: Re: How do we measure the energy of a photon?
Post by: mxplxxx on 04/06/2021 01:41:39
I have added the Punter system to the 3dRacing App. Its purpose is to simulate a "real" punter, thus giving 3dRacing the ability to run in SetAndForget mode. Punter is the first of four top level systems in 3dRacing. It is followed in sequence by Display, Data and Datastore (the 3D's of 3dAbstractions). Punter runs functions in Display and processes events from Display. Display has no knowledge of Punter. Display runs functions in Data and processes events from Data and so on. Thus, as well as a vertical abstraction hierarchy in 3d Apps, there is a horizontal functional hierarchy. Lower levels have no knowledge of higher levels.

Punter inherits 3dController which inherits 3dPerson which inherits 3dBeing which inherits 3dSystem. Punter gives 3dRacing the ability to be intelligent. All 3d Apps now have this ability. In a "3dPerson" App, the equivalent of Punter is Brain.

The 3dRacing intelligent app looks like this.

3dRacing.App
    inherits 3dIntelligentApp.System
    3dRacing.Star
        inherits 3dStar
    3dRacing.Subsystems
        inherits 3dSubsystems
        Punter.System
            inherits 3dController.System
        Racing.App
            inherits 3dApp.System
            Racing.Display
                inherits 3dDisplay.System
            Racing.Data
                inherits 3dData.System
            Racing.Datastore
                inherits 3dDatastore.System

3dController.System
    inherits 3dPerson.System

3dPerson.System
    inherits 3dBeing.System
    3Person.Star
    3dPerson.Subsystems
        3dBrain.System
        3dBody.System
   
3dPerson.Star controls the overall state of the 3dPerson App. It likely includes  the solar plexus in Reality  (which would be why so many feelings are concentrated in that area).

Brain/body are likely symbiotic in Reality. 3dBrain knows about 3dBody but not vice versa. At startup time, 3dbPerson adds 3dBrain event handlers to 3dBody.

e.g.
Code: [Select]
AddHandler Brain.Touch Body.ProcessTouch
adds an event handler for the Brain Touch event that will cause the Body ProcessTouch function to execute when a Brain Touch event is raised (via
Code: [Select]
RaiseEvent(Brain.Touch).

Punter, Display, Data and Datastore are dimensional in nature. Punter is 4 dimensional (Spacetime) in nature, 3dDisplay is 3 dimensional in nature, 3dData is 2 dimensional in nature and 3dDatastore is 1 dimensional in nature. 3dDatastore is  3dApp memory. It a static class meaning it doesn't need instantiating to operate. Maybe just like our own memory?

Punter/3dBeing contains all time-based/cyclic functions for 3dRacing - which makes it likely that time requires life (3dBeing) to exist. All other functions can be considered to run instantaneously.
Title: Re: How do we measure the energy of a photon?
Post by: mxplxxx on 11/08/2021 03:20:13
Everything in 3dAbstractions is  a type of 3dSystem.

A 3dSystem is made up of a 3dOuter system containing a 3dInner system and 3dSubsystems. A Solar system, for example, is a type of 3dOuter system and contains a Sun system which is a type of 3dInner system and planets which are types of 3dSubsystems.

3dOuter systems are infinite in nature. A 3dOuter system has a parent and the parent has a parent and so on potentially to infinity. A 3dOuter system also has subsystems and each subsystem has subsystems and so on potentiality to infinity. Systems are 3 dimensional or 2 dimensional in nature. Reality is a 3 dimensional system. In this case a parent is larger in size than its children and a chain of systems will grow exponentially larger. 3dApps are 2 dimensional but can still simulate 3 dimensional systems. In this case the result ill be type of hologram. 

3dInner systems are fully self-contained. The events in top level system have nowhere to go and in Reality may end up as e.g. a black hole or a sun's magnetic field.   

The further up the chain of systems we go, the more abstract the systems become. Conversely the further down the chain of systems we go the more concrete the systems become. There is a relativity of systems here that parallels the relative nature of Reality. A system that does not have a parent is perfectly abstract. A system that does not have subsystems is perfectly concrete.

One of the consequences of the abstract nature of systems is that a parent controls its children via function calls and the children communicate with the parent by raising events. A child has no knowledge of its parent. A function in the parent has been previously nominated as a handler of the event (Addhandler in VB.Net). If this scheme of doing things is present in Reality, it introduces a measure of uncertainty into systems which certainly corresponds with the quantum nature of reality.

If the Earth is our parent in Reality, we may have a much closer relationship with it than we realise.
Title: Re: How do we measure the energy of a photon?
Post by: mxplxxx on 25/08/2021 23:01:50
The Earth System looks a bit like this:

Earth.System
    Earth.Star
    Earth.Subsystems
        Moon.System
        People.System
        etc.

where: Earth.System is the outer system and Earth.Star is an inner system (i.e. the planet earth). People.System is the 9 or so billion people who inhabit the earth. An individual person interacts with the earth by causing (Rasing) an event which is linked to one or more of the Earth.Star's functions

This is obviously an extraordinarily complex process that would need controlling in practice. How is this control achieved? Via the Brain system of a Person. Instead of interacting directly with the earth, a Person does so via their Brain. i.e. a Brain is a system that simulates a subset of Reality, thus allowing us to interact with Reality in a way that is manageable and scaled to the Person.System's size. More on this later when I will explain how a Brain.System operates in 3dAbstractions.
Title: Re: How do we measure the energy of a photon?
Post by: Origin on 26/08/2021 01:58:41
The Earth System looks a bit like this:

Earth.System
    Earth.Star
    Earth.Subsystems
        Moon.System
        People.System
        etc.

where: Earth.System is the outer system and Earth.Star is an inner system (i.e. the planet earth). People.System is the 9 or so billion people who inhabit the earth. An individual person interacts with the earth by causing (Rasing) an event which is linked to one or more of the Earth.Star's functions

This is obviously an extraordinarily complex process that would need controlling in practice. How is this control achieved? Via the Brain system of a Persom. Instead of interacting directly with the earth, a Person does so via their Brain. i.e. a Brain allows us to interact with a manageable subset of Reality. More on this later when I will explain how a Brain.System operates in 3dAbstractions.
What does this have to do with how we measure the energy of a photon?
Title: Re: How do we measure the energy of a photon?
Post by: mxplxxx on 23/11/2021 13:11:13
Happy 90000th view to me. Coming soon: How Reality handles Inheritance. Mindaboggaling😊

Actually, coming now.

A Ferrari, a sports car, a car, a vehicle is an inheritance hierarchy.

Does the sun e.g. have an inheritance hierarchy? Very Likely.

The sun is the central (or inner) system in a solar system, The sun will have its own central system (maybe a core system) which will also have its own central system and so on and so on.

This hierarchy of central systems is an inheritance hierarchy: i.e. the sun is a type of core.

This type of hierarchy is applicable to all the systems of Reality.

Actually, it seems to me a central system can be a black hole in which case inheritance hierarchy may reveal the structure of a black hole. And the component systems of the black hole may not necessarily be black holes. This is really speculative on my part:)

All systems in an inheritance hierarchy can interact with each other. It may be possible therefore that a system outside a black hole can interact with one inside. How? - given nothing can escape from a black hole - at least intentionally. Via references maybe (Visual Basic mostly operates on references)  i.e.,. the system inside the black will contain a reference to the system outside the black hole and the system outside the black hole will contain a reference to the system inside the black hole. The two references will interact and, in the process, the original systems are updated. A reference is likely to be some kind of exotic system, yet to be discovered.

This is the mechanism by which inter-system communication/updating is achieved in 3dAbstractions.

It is also likely to be how our brains interact with with Reality (i.e. possibly our brains are full of inheritance hierarchies e.g. the TV concept in the brain may be a type of TV in Reality). Viruses are also likely to interact with cells in this way (makes much more sense than a virus actually entering the cell via a narrow receptor).
Title: Re: How do we measure the energy of a photon?
Post by: Bored chemist on 23/11/2021 19:52:33
What does this have to do with how we measure the energy of a photon?
Title: Re: How do we measure the energy of a photon?
Post by: mxplxxx on 23/03/2022 08:42:40
A 3dSystem is infinite in nature. It can be infinitely large or infinitely small. An infinitely large 3dSystem is often referred to as the Cosmos. This may be the equivalent of the one God of certain religions. An infinitely small 3dSystem is nothing. The space between subsystems is nothing, but is also an infinite number of infinitely small 3dSystems.

The infinite nature of a 3dSystem means the cosmos is simultaneously continuous (or everlasting, timeless, unending etc.) and discontinuous (or cyclic, periodic, timeful, or REAL). This is probably why quantum theory is so weird and why fluid dynamics is applicable to the universe.

A 3dSystem can contain an infinite number of subsystems and, given that 3dSystems are not aware of their parent, as far as the 3dSystem is concerned, it IS a Cosmos. Thus, you appear to be Everything to yourself.
Title: Re: How do we measure the energy of a photon?
Post by: Bored chemist on 23/03/2022 12:51:35
What does this have to do with how we measure the energy of a photon?

Title: Re: How do we measure the energy of a photon?
Post by: alancalverd on 26/03/2022 23:44:55
A Ferrari, a sports car, a car, a vehicle is an inheritance hierarchy.
Big fleas have little fleas
upon their backs to bite 'em
and little fleas have smaller fleas
ut sic ad infinitum
Title: Re: How do we measure the energy of a photon?
Post by: mxplxxx on 27/03/2022 00:27:00
A Ferrari, a sports car, a car, a vehicle is an inheritance hierarchy.
Big fleas have little fleas
upon their backs to bite 'em
and little fleas have smaller fleas
ut sic ad infinitum

And each flea thinks its own level of existence is the "flea all and end all":)
Title: Re: How do we measure the energy of a photon?
Post by: mxplxxx on 30/05/2022 07:54:51
Albert Sneppen is an astrophysistist who has proposed a theory whereby global and local weather on the earth are connected.

See https://scienceblog.com/530840/astrophysics-student-finds-link-between-global-warming-and-locally-unstable-weather/?utm_campaign=ScienceBlog.com&utm_medium=email&utm_source=Revue%20newsletter

“For decades, the heat radiation of the early Universe has been studied in the night sky. Researchers use the so-called »angular power spectrum« which tells you how much all parts of the night sky — both locally and globally — are connected. And that is exactly what you want in climate research; a method of examining all scales of climate change at the same time,” Sneppen explains.

This is a consequence of the universe being composed of a single system recursively enclosing an unlimited number of identically constructed subsystems as proposed in 3dAbstractions. You and I are a type of universe. We are abstraction hierarchies whose top level is the universe itself. In other words, global systems, like the earth, control and react to local weather.   

Title: Re: How do we measure the energy of a photon?
Post by: mxplxxx on 30/05/2022 09:30:48
3dAbstractions is to change to 4dAbstractions in near future, in order to add 4dDriver systems to a 4dApp. So the secondd level abstractions in a 4dApp are Driver, Display, Data and Datastore (hence the 4 D's). BTW they are horizontally abstracted systems, so Driver can conrol Display but not vice versa and so on.  n the same manner, the Sun can control the earth but not vice versa.

A 4dDriver allows any 4dApp to be “driven” by a person system or by an intelligent 3dSystem. So the Punter system in 4dRacing can by driven be a person or by a 4dDriver system, the latter allowing for automatic (Set and Forget) betting.

Similarly, a Car system can be driven by a person or by a 4dDriver system (i.e. self-driven). Allows for simulation of a punter or a car or can be used to actually run a car. VERY modern😊😊😊(bet Boeing wished they had something similar).

BTW a Driver system is set up to control time which makes 4D a very appropriate change. So we have Driver 4D, Display 3D, Data 2D and Datastore 1D. Given that all systems derive from Datastore and Datastore is 1D, it is likely that the Big Bang is the Universes equivalent of Datastore.

So, the physical Universe may be a 4dApp, where

Drivers are time-based 4D life or intelligent 4dApps
Display is the 3D part of the universe
Data is the 2D data structure of the univere
Datastore is the singularity from which the universe sprang (may alo be where universal memories are held)


Title: Re: How do we measure the energy of a photon?
Post by: mxplxxx on 30/05/2022 10:15:20
All systems in 4Dabstractions inherit a base system; _4dSystem. i.e all systems are a type of _4Dsystem

The same setup is likely to occur for physical systems. i.e the Universe would exist as a top level system and inherit a base system that also inherited by all universal systems.
Title: Re: How do we measure the energy of a photon?
Post by: Bored chemist on 30/05/2022 10:51:03
All systems in 4Dabstractions inherit a base system; _4dSystem. i.e all systems are a type of _4Dsystem

The same setup is likely to occur for physical systems. i.e the Universe would exist as a top level system and inherit a base system that also inherited by all universal systems.
Why do you post this stuff?
Title: Re: How do we measure the energy of a photon?
Post by: alancalverd on 30/05/2022 11:57:24
inherited by all universal systems.
If it's universal, it can't inherit because it includes all subsystems and therefore already owns whatever they do.
And if it's universal it must be unique, so you can't use the plural.
Not a good idea to use meaningless phrases in a swamp populated by pedantic alligators.
Title: Re: How do we measure the energy of a photon?
Post by: mxplxxx on 30/05/2022 12:47:54
inherited by all universal systems.
If it's universal, it can't inherit because it includes all subsystems and therefore already owns whatever they do.
And if it's universal it must be unique, so you can't use the plural.
Not a good idea to use meaningless phrases in a swamp populated by pedantic alligators.
How about a billabong populated by casuistic crocodiles. What I meant was all systems belonging to the universe.

A simulation of the Uniiverse in 4dAbstractions starts off with:

Code: [Select]
Namespace Universe

Public Class System
    Inherits _4dSystem

So it is likely that in Reality, a system the equivalent of _4dSystem exists prior to the Universe coming into being.
Title: Re: How do we measure the energy of a photon?
Post by: alancalverd on 30/05/2022 16:12:50
Unlikely since "system" is a human construct.
Title: Re: How do we measure the energy of a photon?
Post by: mxplxxx on 30/05/2022 17:43:06
Unlikely since "system" is a human construct.
Like the Solar System?
Title: Re: How do we measure the energy of a photon?
Post by: alancalverd on 30/05/2022 21:07:13
There are various objects orbiting the sun. We have devised equations that tell us where they will be at any given moment, but they weren't put there for any particular reason so the notion of "system" is human - it is the name we give to those bits of  nature for which we can make reasonable predictions.
Title: Re: How do we measure the energy of a photon?
Post by: mxplxxx on 31/05/2022 02:39:11
There are various objects orbiting the sun. We have devised equations that tell us where they will be at any given moment, but they weren't put there for any particular reason so the notion of "system" is human - it is the name we give to those bits of  nature for which we can make reasonable predictions.
And we spend lifetimes trying to make reasonable predictions about those bits of nature that seem to defy our predictive powers.

There are also "bit of nature" that are not bits at all but, instead, references to bits. or concepts rather than things. A human system, the brain, specialises in them. In a 4dSystems App, this type of "byref" system is far more numerous than the "byval" (or original) sytem referenced. In fact, Visual Basic, and hence 4dAbstractions, works pretty much exclusively with "byref" systems.

Thus, our brains will likely contain a description of a hamburger (a class) and will generate an instance of the hamburger class when experiencing a particular hamburger. There are systems involved in this process even though they involve references rather than real things. As per the Matrix, what we experience is not "real".

In fact "real" is relative. The universe is an abstraction hierarchy of systems  and the further we go down the hierarchy, the "realer" (or more concrete, or less abstract) a system becomes. At the centre of the Universe is the top of the hierarchy. Things are very abstract here. Nothing much changes. This is the essence of a black hole. At the edge of the Universe is the bottom of the hierarchy. Things are very concrete here. Change is constant with very little uncertainty. All systems, from the Universe to the smallest particle are identically constructed systems.

We, ie, human systems, occupy a particular level in the Universe's abstraction hierarchy and, as a result, are not fully real. In any case, we are, internally, own own abstraction hierarchies. So externally we belong to the Universe abstraction hierarchy and internally we contain our own abstraction hierarchy. Which makes for a lot of fun and games where mistakes will abound, but evolution will be heavy. . (Which may also explain a lot of quantum weirdness and even relativity, as discussed previously)..

It is interesting to note that the top-level Universe System is a common super-system for all of us. Be good to remember this in our dealings with our neighbours.

Learning about abstractions and how they interact does not come easy. It was 40 years as a software developer before I was able to call myself an expert. IMHO, systems, as described in 4dAbstractions, are the key to being able to describe and manage abstractions and are also the future of physics..


Title: Re: How do we measure the energy of a photon?
Post by: mxplxxx on 02/06/2022 15:14:33
Unlikely since "system" is a human construct.
Hardly. A good definition, according to Wiki is:

"A system is a group of interacting or interrelated elements that act according to a set of rules to form a unified whole.[1] A system, surrounded and influenced by its environment, is described by its boundaries, structure and purpose and expressed in its functioning."
Title: Re: How do we measure the energy of a photon?
Post by: Origin on 02/06/2022 15:36:58
A good definition, according to Wiki is:

"A system is a group of interacting or interrelated elements that act according to a set of rules to form a unified whole.[1] A system, surrounded and influenced by its environment, is described by its boundaries, structure and purpose and expressed in its functioning."
In other words a human construct, we set the rules and set the boundaries.
Title: Re: How do we measure the energy of a photon?
Post by: Origin on 02/06/2022 15:39:49
And we spend lifetimes trying to make reasonable predictions about those bits of nature that seem to defy our predictive powers.
This has nothing to do with measuring the energy of a photon.  Why don't you start a new thread if you want to discuss another topic?  Better yet why don't you start your own blog.
Title: Re: How do we measure the energy of a photon?
Post by: mxplxxx on 02/06/2022 16:10:37
And we spend lifetimes trying to make reasonable predictions about those bits of nature that seem to defy our predictive powers.
This has nothing to do with measuring the energy of a photon.  Why don't you start a new thread if you want to discuss another topic?  Better yet why don't you start your own blog.
Asked and answered previously.
Title: Re: How do we measure the energy of a photon?
Post by: alancalverd on 02/06/2022 22:25:26
And we spend lifetimes trying to make reasonable predictions about those bits of nature that seem to defy our predictive powers.

Up to a point. Most scientists are happy with the concept of essential randomness and unpredictability, without which the hydrogen atom would collapse and we wouldn't exist. Which is why science is better than philosophy.
Title: Re: How do we measure the energy of a photon?
Post by: mxplxxx on 03/06/2022 02:12:10
And we spend lifetimes trying to make reasonable predictions about those bits of nature that seem to defy our predictive powers.

Up to a point. Most scientists are happy with the concept of essential randomness and unpredictability, without which the hydrogen atom would collapse and we wouldn't exist. Which is why science is better than philosophy.

Maybe the randomness is an illusion because of the abstract nature of the universe. In a abstraction hierarchy like 4dAbstractions, a system is unaware of its higher level parent. The parent controls its children via function calls. As far as the children are concerned, what happens during one of these function calls may appear to be random, but in fact is purposeful at the parent's level. Given that the universe is an abstraction hierarchy, something similar may be what is happening with the Hydrogen atom.

A simulation of a Hydrogen atom in 4dAbstractions (below) will have an outer Hydrogen system and an inner abstraction hierarchy containing a central Proton in level 1 and an Electron in level 2. The outer Hydrogen System controls the Proton via function calls and the Proton in turn controls the Electron via function calls. The Electron communicates with the Proton via Photons (a type of event) and, in turn, the Proton communicates with the Hydrogen system also with Photons. What happens to the Proton can only be explained if we know how the Hydrogen atom works and, I am guessing, we don't.


Code: [Select]
namespace Hydrogen

    System (Atom)

        Inherits Atom.System

        Star (Proton)
            Inherits Ion.System

        Subsystems (1 Electron)
End Namespace
Title: Re: How do we measure the energy of a photon?
Post by: mxplxxx on 27/01/2023 05:07:22
A 4dSystem is the base object in 4dAbstractions. Once you have one 4dSystem you can create an infinite number of 4dSubSystems. The  (top level)  4dSystem in this system could be considered to be the equivalent of "God" in religious terms.

A 4dSystem contains a single central 4dStar and an unlimited number of 4dSubSystems. This is the equivalent in physics of a solar system, a sun/star and the planetary systems.

A 4dStar inherits a 4dSystem. i.e. a 4dStar is a type of 4dSystem. It has its own central 4dStar and a set of its own 4dSubSystems. In 4dAnsstractions this often implements a 4dStateMachine especially if the parent 4dSystem involved is a reactive system.

So, if the Sun is a type of 4DStar, it will itself contain a central 4dStar which likely implements a 4dStateMachine. If the central system of the Sun is a black hole, then a black hole cannot create new systems but can change state. Which makes a lot of sense. Maybe dark matter is a state machine component.

Since State is effectively a record of something that happened in the past, it may also be that a black hole is a record of the past.

In 4dAbstractions, 4dEvents travel UP an abstraction hierarchy until they reach the top-level 4dStar system, at which point they simply vanish. The top level 4dStar system in a Sun simulation is likely to be a Black Hole in Reality, so events that reach it go no further. Events in a simulation of Reality are light particles in Reality. This may be why a black hole is black.
   
How is the future handled in 4dAbstractions? By randomly modifying the variables involved in a calculation and rerunning the calculation and evaluating the result of the rerun against the original result. Do this n times for the current calculation and use the variables that produced the best result in the future. 4dRacing, a personal gambling package does this extensively. Sounds a bit quantum in nature? :). Possibly. Maybe black holes are involved in this way (somehow) in quantum calculations
Title: Re: How do we measure the energy of a photon?
Post by: mxplxxx on 10/04/2023 08:28:47
A 4dStar is a a type of 4dSystem that deals mainly in state/functionality. It sits at the centre of all 4dSystems (ie. it is the STAR attraction :) ).

Because it is a type of 4dSystem, it will, of itself, contain a 4dStar at its centre. So we can have a, possibly infinite, hierarchy of 4dStars contained within a 4dSystem.

This is how a solar system is simulated in 4dAbstractions and it is likely how Reality implements Inheritance hierarchies.

Code: [Select]
Namespace Solar

    ' Solar.System - the enclosing shell

    Class System
        inherits 4dSystem

    ' Solar.Star (i.e the Sun)

    Class Star
        inherits 4dStar

        ' note: 4dStar is a type of (i.e. inherits) 4dSystem

    ' Solar.Subsystems (Shells)

    Class Subsystems
        inherits 4dSubsystems

        Subsystems(0) Sun.Shell
        Subsystems(1) Mercury.Shell
        Subsystems(2) Venus.Shell
        Subsystems(3) Earth.Shell
        Subsystems(4) Mars.Shell
        Subsystems(5) Jupiter.Shell
        Subsystems(6) Saturn.Shell
        Subsystems(7) Neptune.Shell
        Subsystems(8) Uranus.Shell
        Subsystems(9) Pluto.Shell

    ' Solar.Shell - inherted by individal Solar.Subsystems (Shells)
        inherits 4dShell

        ' an Earth.Shell e.g. is a type of 4dShell, consisting of an Earth.System and miscellaneous subsystems (usually asteroids).

  ' the Earth.Shell, is (likely) controlled by the Venus.Shell (via charged particles) and the Venus.Shell processes events (light particles) from the Earth.Shell. 

End Namespace


There are lots and lots of implications from this. e.g. all systems in Reality may inherit a Hydrogen ion at their centre i.e. a solar system (and a universe) may be a type of Hydrogen Atom.

Note 1:

The above are 2D structures. They will be changed to 3D when a Solar.System is displayed (via the 4dAbstractions class 4dDisplay). 2d structures are invariably simpler than their 3d counterparts, which leads to the strong possibility that the Universe is a Hologram.

Note 2:

The process of changing a System from 2D to 3 state will expand the area the System occupies in Reality and may account for the expanding universe. Actually 1D to 2D to 3D to 4D (time) to 5D (multiverse) all result in increasing areas.

Note 3:

The central system of a 4dSolar.System is a 4dSolar.Star (or Sun in colloquial terms). It participates in two abstraction hierarchies.

1: An INTERNAL hierarchy that maintains the internal state of the 4dSolar.Star system. The top of this hierarchy will be 4dCore.Star system (or 4dBlack.Star system for a dense sun)

2: An EXTERNAL hierarchy that interacts with the 4d planetary systems to maintain related states.

This is a common feature of all 4dStar systems and likely to be a common feature of all central systems in Reality.

Note : 4

A 4dSolar.Star (or Sun) is a controller for the various planetary subsystems. A controller in 4dAbstractions works via function calls to the subsystems involved, which in turn return event notifications to the controller.  The equivalent in Reality may (very possibly) be charged particles as function calls and light as event notifications, This would allow light to operate massless https://www.thenakedscientists.com/forum/Smileys/default/smiley.gif


Title: Re: How do we measure the energy of a photon?
Post by: Origin on 10/04/2023 14:25:12
Thanks for stopping by to dump another load pseudoscience. ::)
Title: Re: How do we measure the energy of a photon?
Post by: paul cotter on 10/04/2023 18:22:31
Good grief!, what a load of cobblers. Being a holiday with the weather too inclement for gardening and suffering a lack of motivation for general maintenance I read through this whole thread-now my brain has melted and is oozing out my ear canals.
Title: Re: How do we measure the energy of a photon?
Post by: mxplxxx on 11/04/2023 01:14:05
Good grief!, what a load of cobblers. Being a holiday with the weather too inclement for gardening and suffering a lack of motivation for general maintenance I read through this whole thread-now my brain has melted and is oozing out my ear canals.

Abstraction hierarchies will do that to you😎 IT took me 40 years to become comfortable with them. Half of IT still work with functional (versus Object Oriented i.e. Abstract) code and most will never get any further.
Title: Re: How do we measure the energy of a photon?
Post by: mxplxxx on 11/04/2023 01:18:53
Thanks for stopping by to dump another load pseudoscience. ::)
Pretty sure Einstein would have got some of the same sort of replies😎I do have a whole operating system based on the pseudoscience and it works pretty well.
Title: Re: How do we measure the energy of a photon?
Post by: alancalverd on 11/04/2023 09:33:51
a solar system (and a universe) may be a type of Hydrogen Atom.
If that is your conclusion, then your life's work has been in vain.
Title: Re: How do we measure the energy of a photon?
Post by: Origin on 11/04/2023 13:11:20
I do have a whole operating system based on the pseudoscience and it works pretty well.
It works well at generating more pseudoscience...
Title: Re: How do we measure the energy of a photon?
Post by: hamdani yusuf on 11/04/2023 14:50:15
There are lots and lots of implications from this. e.g. all systems in Reality may inherit a Hydrogen ion at their centre i.e. a solar system (and a universe) may be a type of Hydrogen Atom.
How does your system define other objects in solar system, such as planets' moons and rings, asteroids, comets, corona, solar flares?
Title: Re: How do we measure the energy of a photon?
Post by: Bored chemist on 11/04/2023 16:23:38
Pretty sure Einstein would have got some of the same sort of replies
No.
He had logic and evidence.
Title: Re: How do we measure the energy of a photon?
Post by: mxplxxx on 11/04/2023 16:43:48
Pretty sure Einstein would have got some of the same sort of replies
No.
He had logic and evidence.
But no proof! Hence the THEORY of Relativity.
Title: Re: How do we measure the energy of a photon?
Post by: mxplxxx on 11/04/2023 16:53:46
a solar system (and a universe) may be a type of Hydrogen Atom.
If that is your conclusion, then your life's work has been in vain.
Never in vain. It has been and still is "FUN". In any case, "May" is not a conclusion.

 In computer science all objects inherit a base object. So in my system-based computer universe, I can make the base object any type of object I please. So I figure, why do the entities that created our universe not think likewise. 
Title: Re: How do we measure the energy of a photon?
Post by: Bored chemist on 11/04/2023 16:55:18
Pretty sure Einstein would have got some of the same sort of replies
No.
He had logic and evidence.
But no proof! Hence the THEORY of Relativity.
[ Invalid Attachment ]

https://en.wikipedia.org/wiki/Scientific_theory
Title: Re: How do we measure the energy of a photon?
Post by: mxplxxx on 11/04/2023 17:02:12
There are lots and lots of implications from this. e.g. all systems in Reality may inherit a Hydrogen ion at their centre i.e. a solar system (and a universe) may be a type of Hydrogen Atom.
How does your system define other objects in solar system, such as planets' moons and rings, asteroids, comets, corona, solar flares?
Everything in 4dAbstractions is a type of 4dSystem. A 4dSystem is a spherical container that contains a type of 4dStar system at its centre and an unlimited number of 4dSubsystems.

With this scheme and a reasonably powerful Object Oriented Language (like Visual Basic),  I can create any type of system, and so can a Universal Language and run-time system like Reality.

The following is the bare bones code I would need to create the 4dEarth System which contains the planet Earth at its centre.

Code: [Select]
Namespace 4dEarth

      ' Earth.System - Earth the system (includes Earth and Moon and Asteroids)

Class System
    inherits  4dSystem


      ' Earth.Star - Earth the planet (is the centre of a Earth.System)
 
Class Star
    inherits  4dStar

          Subsystems(0) Earth.Cryosphere
          Subsystems(1) Earth.Hydrosphere
          Subsystems(2) Earth.Atmosphere
          Subsystems(3) Earth.Biosphere
          Subsystems(4) Earth.Lithosphere
          Subsystems(5) Earth.Magnetosphere
          Subsystems(6) Earth.Technosphere

    ' Earth.Subsystems - Moon and Asteroids

    Class Subsystems
         inherits 4dSubsystems

         Subsystems(0) Moon.System
         Subsystems(1) Asteroids.System

End Namespace

' create an Earth.System which includes planet Earth at its centre and Moon and Asteroids

Dim Earth as Earth.System = new Earth.System(Type:=EARTH, SubType:=COLLECTION, Star:=New Earth.Star(), SubSystems:=New Earth.Subsystems())

Title: Re: How do we measure the energy of a photon?
Post by: Bored chemist on 11/04/2023 17:30:36
I can create anything with this scheme (and so can a Universal run-time system).
But that is no reason to suppose that the universe does, in reality, act that way, is it?
Title: Re: How do we measure the energy of a photon?
Post by: mxplxxx on 11/04/2023 17:41:30
I can create anything with this scheme (and so can a Universal run-time system).
But that is no reason to suppose that the universe does, in reality, act that way, is it?
Just an educated guess. There seems to be no purpose to a system that seems designed to work on the principle of survival of the fittest except to add to the store of knowledge of a civilisation that has run out of ideas:)
Title: Re: How do we measure the energy of a photon?
Post by: Bored chemist on 11/04/2023 17:56:13
Just an educated guess.
And do you now understand why that's different from a theory?
Title: Re: How do we measure the energy of a photon?
Post by: mxplxxx on 16/04/2023 11:07:54
Pretty sure Einstein would have got some of the same sort of replies
No.
He had logic and evidence.
But no proof! Hence the THEORY of Relativity.
[ Invalid Attachment ]

https://en.wikipedia.org/wiki/Scientific_theory

The word "theory" seems to have many nuances, often dependent on the context in which it is used.

From https://plato.stanford.edu/entries/einstein-philscience/

"But what strikes the “systematic epistemologist” as mere opportunism might appear otherwise when viewed from the perspective of a physicist engaged, as Einstein himself put it, in “the critical contemplation of the theoretical foundations.” The overarching goal of that critical contemplation was, for Einstein, the creation of a unified foundation for physics after the model of a field theory like general relativity (see Sauer 2014 for non-technical overview on Einstein’s approach to the unified field theory program). Einstein failed in his quest, but there was a consistency and constancy in the striving that informed as well the philosophy of science developing hand in hand with the scientific project."
Title: Re: How do we measure the energy of a photon?
Post by: Bored chemist on 16/04/2023 11:31:07
When you have finished typing twaddle, theories still depend on evidence.
You have a conjecture.

Do you realise that the high level features of computer architecture which you describe only work because there's a lot of underlying code and that is explicit, not implicit.

What you describe as "The following is the bare bones code" only works because there's a hell of a lot of real code in the background.

Are you saying that underlying code for the universe exists somewhere?
Where?
Title: Re: How do we measure the energy of a photon?
Post by: mxplxxx on 16/04/2023 12:06:28
Are you saying that underlying code for the universe exists somewhere?
Where?

Thank you for those useful questions.

I rarely "say".

It seems to me that, if the universe is a type of computer, then somewhere there likely exists a run-time system that is separate from the observable universe.

My theories are mainly based on eureka moments which come from the intuition that arises from 50 years of software development. Only about 1 in 20 eureka moments turn out to be pragmatically worthwhile.

Title: Re: How do we measure the energy of a photon?
Post by: mxplxxx on 16/04/2023 12:10:47
What you describe as "The following is the bare bones code" only works because there's a hell of a lot of real code in the background.
Of course. So what?
Title: Re: How do we measure the energy of a photon?
Post by: mxplxxx on 16/04/2023 12:14:14
Do you realise that the high level features of computer architecture which you describe only work because there's a lot of underlying code and that is explicit, not implicit.
As a software developer, I deal mainly in high-level languages. Visual Basic is my favourite because of its readability and flexibility. How the languages are implemented rarely comes into it.
Title: Re: How do we measure the energy of a photon?
Post by: Bored chemist on 16/04/2023 16:21:10
. How the languages are implemented rarely comes into it.
It does if you are talking about the structure of the universe.
Of course. So what?
So you have to answer this question.
Are you saying that underlying code for the universe exists somewhere?
Where?
My theories are mainly based on eureka moments which come from the intuition that arises from 50 years of software development. Only about 1 in 20 eureka moments turn out to be pragmatically worthwhile.
You don't have a theory; you have a conjecture.
It's not even internally consistent because, if the "universe" is a high level language then that requires a low level language in which it is implemented.

But at least we can be grateful that you accept that your idea is based on something with a 95% failure rate.
Title: Re: How do we measure the energy of a photon?
Post by: alancalverd on 16/04/2023 17:13:35
if the universe is a type of computer,
It clearly isn't, so the rest of your conjecture is void.
Title: Re: How do we measure the energy of a photon?
Post by: mxplxxx on 16/04/2023 19:53:34
if the universe is a type of computer,
It clearly isn't, so the rest of your conjecture is void.
IYHO. "Clearly". Such a definitive opinion deserves a reason!

I would have thought the fact that the brain is a biological computer (think Neural Networks, DNA etc.) would make at least part of the Universe a computer.

 A computer is part of the universe and uses the physics of the universe to compute. I would have thought this screams out that the Universe could and does do the same.

The essence of a computer is that it processes abstractions. The universe is an abstraction hierarchy so this makes it likely that it is, in essence, a computer.

I have no trouble simulating a Universe in 4dAbstractions. I estimate sometime in the next 50 years we will be able to hook our brain up to a simulation of a universe and believe we are living in that universe.

Many other eminent physicists think the Universe is a computer.

https://thenextweb.com/news/physicists-working-with-microsoft-think-the-universe-is-a-self-learning-computer

http://cmsw.mit.edu/angles/2015/is-the-universe-actually-a-giant-quantum-computer/#:~:text=According%20to%20MIT%20professor%20Seth,and%20not%20even%20know%20it

https://www.scientificamerican.com/article/confirmed-we-live-in-a-simulation/

https://studyfinds.org/universe-vr-how-to-test-if-were-living-in-computer-simulation/

etc. etc.

Title: Re: How do we measure the energy of a photon?
Post by: Bored chemist on 16/04/2023 20:11:44
Sorry, but you don't have a level of Computer Science knowledge needed for me to debate these issues with you.
Fair enough.
I usually go out drinking on Friday nights and come home... let's say it is just as well I'm not driving.

Perhaps, if we agree to debate it then, I will be down to a low enough level that you can understand me.
I see you have clarified your standing as a cutting edge expert in the matter.
Visual Basic is my favourite because of its readability and flexibility


In the mean time, unless you can prove that the universe actually IS a computer, your posts are empty guesswork.
Title: Re: How do we measure the energy of a photon?
Post by: mxplxxx on 04/05/2023 00:05:10
Thanks for stopping by to dump another load pseudoscience. ::)
Unfortunately, most physics seems to be based on Mathematics. Which leaves us with very little idea HOW things work. There is not a lot of mathematics in 4dAbstactions but there is a lot of software which leaves us with a good idea of HOW things work in a simulation of Reality. Pseudoscience? Who cares, if it works?:)
Title: Re: How do we measure the energy of a photon?
Post by: Origin on 04/05/2023 02:43:39
Pseudoscience? Who cares, if it works?
It doesn't work, it's gibberish.
Title: Re: How do we measure the energy of a photon?
Post by: Bored chemist on 04/05/2023 06:03:43
Pseudoscience? Who cares, if it works?
It doesn't work, it's gibberish.
Then why do you bother spending your time on it?
We should be asking you that.
Why do you waste your time posting pseudoscience here?
Title: Re: How do we measure the energy of a photon?
Post by: alancalverd on 04/05/2023 07:23:59
No, there are plenty of philosophy, politics, economics and religion forums for pseudoscience. This one is supposed to be about novel theories that have some relation to reality.
Title: Re: How do we measure the energy of a photon?
Post by: hamdani yusuf on 09/05/2023 13:27:54
E = hf is Planck's definition of h. How can that be controversial?
Planck's quantization of energy has a mathematical implication in relationship with Planck's law.
The minimum non-zero energy implies Emin/hf=1.
2f.f/{c.c.e^[hf/(kB.T)] -1} = 1
In the equation above, the only variables are frequency and temperature. Which means for every value of temperature, there is a corresponding frequency.

I can no longer use ASCII symbols in my post. Does anyone experience the same problem?
Title: Re: How do we measure the energy of a photon?
Post by: Bored chemist on 09/05/2023 13:41:10
E = hf is Planck's definition of h. How can that be controversial?
Ask a flat earther.
Title: Re: How do we measure the energy of a photon?
Post by: mxplxxx on 11/05/2023 00:48:47
E = hf is Planck's definition of h. How can that be controversial?

Planck's quantization of energy has a mathematical implication in relationship with Planck's law.
The minimum non-zero energy implies Emin/hf=1.
2f.f/{c.c.e^[hf/(kB.T)] -1} = 1
In the equation above, the only variables are frequency and temperature. Which means for every value of temperature, there is a corresponding frequency.
You are asking me a question about a post on 24/1/2019 by alancalverd. You then start by e=hf and end up with 2f.f/{c.c.e^[hf/(kB.T)] -1} = 1

Do I have to be a maths genius to understand this? The post is unintelligible (at least to me).

Actually, Naked Scientists, I am pretty certain you are being taken for a ride by the author of the above post.  He seems to be a pseudo-intellectual posting garbage.
Title: Re: How do we measure the energy of a photon?
Post by: Bored chemist on 11/05/2023 08:39:00
Do I have to be a maths genius to understand this?
No.
Title: Re: How do we measure the energy of a photon?
Post by: hamdani yusuf on 12/05/2023 10:07:34
The post is unintelligible (at least to me).
You can start by Planck's formula which you can find in Wikipedia. Divide both sides by hf.
Title: Re: How do we measure the energy of a photon?
Post by: mxplxxx on 12/05/2023 11:43:29
The post is unintelligible (at least to me).
You can start by Planck's formula which you can find in Wikipedia. Divide both sides by hf.
I suggest you attend a course in clear expression. Your posts are much too complex. The ability to simplify complex logic is top of the list of desirable attributes for software developers. Maybe should be the same for physicists.
Title: Re: How do we measure the energy of a photon?
Post by: Bored chemist on 12/05/2023 13:36:11
The post is unintelligible (at least to me).
You can start by Planck's formula which you can find in Wikipedia. Divide both sides by hf.
I suggest you attend a course in clear expression; if you have it to think logically and rationally. The few posts of yours that I have read to date lead me to believe you are one sandwich short of a picnic.
You are the only one who is complaining that you don't understand what he said.
This suggests that the problem does not lie in his writing...
Title: Re: How do we measure the energy of a photon?
Post by: hamdani yusuf on 12/05/2023 14:57:36
The post is unintelligible (at least to me).
You can start by Planck's formula which you can find in Wikipedia. Divide both sides by hf.
B(f,T) = 2hf3/(c2.(e(hf/kT)-1))
Minimum radiation energy = Emin = hf
Substitute B(f,T) with Emin and combine both equations.
1 = 2f2/(c2.(e(hf/kT)-1))
Title: Re: How do we measure the energy of a photon?
Post by: Eternal Student on 12/05/2023 18:35:56
Hi.

I can no longer use ASCII symbols in my post. Does anyone experience the same problem?
   See this thread:    https://www.thenakedscientists.com/forum/index.php?topic=86297.msg704606#msg704606

I'm getting lots of problems with the forum,  especially getting mathematical notation.

Best Wishes.
Title: Re: How do we measure the energy of a photon?
Post by: Bored chemist on 12/05/2023 18:54:45
What happens when I type alt 123?
{
Alt 64 gives me
@

ASCII seems to be working for me.
What happens when I post it?
Title: Re: How do we measure the energy of a photon?
Post by: Bored chemist on 12/05/2023 23:03:22
So, what does this mean in plain English?
Is this the bit you don't understand?
You can start by Planck's formula which you can find in Wikipedia.
Title: Re: How do we measure the energy of a photon?
Post by: Bored chemist on 14/05/2023 15:12:05
Please be so kind as to elucidate me.
No.
You are beyond comprehension.
But I might be able to elucidate the problem.

How far have you got?
Do you understand what Wikipedia is?
Have you looked for Planck's formula(e) in it?
Did you find this?
https://en.wikipedia.org/wiki/Planck%27s_law

Title: Re: How do we measure the energy of a photon?
Post by: Bored chemist on 14/05/2023 17:14:46
I am supposedly talking to experts. Who tell me to look at Wikipedia rather than telling me about e=hf. Give me the benefit of your expert knowledge or stop wasting my time.

BTW E=hf is discussed extensively by me in this topic. If you give me a lucid explanation of it, I will be truly amazed.
If you can't be bothered to read an explanation on Wiki, there's no reason to suppose that you will read it if we write an explanation here.


What difference does it make if you read it here or on wiki?


stop wasting my time.
If you can't understand what we or wiki write, then I'm sorry but you are wasting your time.
You just aren't cut out for science.
Title: Re: How do we measure the energy of a photon?
Post by: Bored chemist on 15/05/2023 08:37:48
I have read everything I can get my hands on on social media re e=hf.
Try reading the wiki article instead.

As far as I can see, there is no physicist, living or dead, who has a lucid explanation of what it means. This is a big chance for you to demonstrate your expertise. Promise I will read your explanation if you are game to post it
You say that you can't find an explanation that makes sense to you.
I know I'm not a miraculously good teacher.
So there's no chance of my being able to explain it to you when everyone else has failed.

If everyone else can't explain it to you, I might as well try to explain it to next door's cat.
Title: Re: How do we measure the energy of a photon?
Post by: Bored chemist on 15/05/2023 08:46:20
PS I did mention that the current topic has a lot to do with e=hf. Did you miss this?
No. But you kept on missing the point.
As Alan summed it up
Your handling of mathematical symbols is impeccable, but has nothing to do with photon physics.
And I did try to explain why you were so gravely wrong.

E = hf
P = E/t
t = 1/f
P = Ef

QED?
No
Because you are using  "t" for two different things.
One is the time period over which energy is transferred, and the other is the time between successive peaks in the em field of the photon.

It's like muddling up the reciprocal of the frequency of a car radio  and time take to get from one town to the next.
Title: Re: How do we measure the energy of a photon?
Post by: Bored chemist on 15/05/2023 11:43:37
How would you know what the point is if you can't explain e=hf?
You have leapt to a false conclusion.
It's not that I can't explain it.
The problem seems to be that you can't understand it.
Plenty of people have put forward explanations.
You say you have read them.
One more guy adding an explanation isn't going to fix that.

You seem not to recognise that the problem isn't with me, wiki, social media or physics.
The problem is you can't understand e=hf.

Is it that you don't understand energy or frequency or do you struggle with the idea that there's a  constant of proportionality?

Title: Re: How do we measure the energy of a photon?
Post by: Bored chemist on 15/05/2023 12:48:08
But you are a self-confessed bad explainer.
I "confessed" to not being a miracle worker.
You then lied about what I had said.

Are you saying that everyone on the internet who has sought to explain it is a "bad explainer", or do you realise that maybe you just aren't good at understanding explanations?
Title: Re: How do we measure the energy of a photon?
Post by: Bored chemist on 15/05/2023 18:08:39
How could I have lied when I just repeated what you said
You did not repeat what I said.
You said "you are a self-confessed bad explainer.". And that is obviously different from what I said.
You lied.
(Twice now).

Analysing something like e=hf comes easy.
Problem solved then.
You already understand it.
Title: Re: How do we measure the energy of a photon?
Post by: Bored chemist on 15/05/2023 18:12:06
(ps beware of defamation)
Was that meant to be irony?
Title: Re: How do we measure the energy of a photon?
Post by: mxplxxx on 16/05/2023 01:04:36
You say that you can't find an explanation that makes sense to you.
I know I'm not a miraculously good teacher.
So you can't explain it to me, or maybe you just don't know.
Title: Re: How do we measure the energy of a photon?
Post by: Bored chemist on 16/05/2023 08:49:21
Pity you can't offer your understanding of e=hf.
Again; you say I can't.
But in fact, I can. Maybe it's because because I am (rather than was)  one of the analysts at a ?1.1 trillion pa outfit.
I have already pointed this out to you.

Why do you keep saying things that are not true?
Title: Re: How do we measure the energy of a photon?
Post by: Bored chemist on 16/05/2023 08:53:46
So you can't explain it to me, or maybe you just don't know.
And, once again for the "hard of thinking"...
I could explain it.
But my explanation would look a lot like the one in WIKI and you already said you are not able to to follow that. (I assume it's because you simply are not clever enough.)
The point is not that I don't know- because, obviously,I do.
The point is that even if I explained it to you it seems very likely that you still wouldn't understand it.

So why would I waste my time?

In particular, why would I waste my time for the benefit of someone who lies about me?
Title: Re: How do we measure the energy of a photon?
Post by: Bored chemist on 16/05/2023 10:23:20
Because it may increase your credibility from where it currently sits close to zero.
LOL.
Seriously, it's liars who have no credibility; you know people who say things like "
You confessed to not being "a miraculously good teacher"
when they said nothing of the sort.
Another way to blow your own credibility is to try to "borrow" someone else's.
I was a senior analyst/programmer for a $50 billion pa company.
The important word there is "was".
:-)
Title: Re: How do we measure the energy of a photon?
Post by: mxplxxx on 09/08/2023 10:35:49
4dAbstractions is a computer system that allows us to simulate Reality.

A 4dSystem is the base object of 4dAbstractions.

All objects in 4dAbstractions are types of 4dSystems.

An App.System allows us to simulate Reality.

It consists primarily of a central App.Star system plus the following App.Subsystems:

1. Datastore.System - 1D Data storage
2. Data.System - 2d abstraction hierarchy
3. Display.System - 3d representation of  2d Data.System. (see See https://1drv.ms/i/s!AkkAzGDByUeBpv9KsnyPS2utrwYhdg?e=WTNURu)
4. Driver.System - 4d App Driver (or e.g. Punter in 4dRacing)

See https://www.thenakedscientists.com/forum/index.php?topic=75960.msg640534;topicseen#msg640534
This is a simulation of a SARS-COV-2 Virus using 4dAbstractions.

This App structure allows us to simulate Gravity. 4dGravity is the set of calculations that allow a 2d Data.System to be transformed into a 3d Display.System.

Given that Reality is likely a type of computer system, this transformation is likely how Reality creates gravity.

The structure also allows us to simulate the Big Bang where data is transformed from a 1D Datastore.System to a  2d Data.System to a 3d Reality.

A Person.App in 4dAbstractions is a type of Driver.System that is time-enabled.
 
Title: Re: How do we measure the energy of a photon?
Post by: paul cotter on 09/08/2023 11:06:09
And what has post #806 got to do with photon energy? Looks to me that this thread is turning into "my bodily appendage is bigger than yours".
Title: Re: How do we measure the energy of a photon?
Post by: Bored chemist on 09/08/2023 12:19:23
I am (rather than was)  one of the analysts at a ?1.1 trillion pa outfit.

Can't locate a company on Google that EARNS more than $1 trillion p.a.. Microsoft, and a few others are WORTH more than $1 trillion p.a.. No chance you are fibbing here is there:)



I didn't say it was a company, did I?
I remain an analyst at an outfit with a 1.1 trillion dollar turnover.

And calling me a liar doesn't help anyone, does it?
Title: Re: How do we measure the energy of a photon?
Post by: Origin on 09/08/2023 15:25:14
4dAbstractions is a computer system that allows us to simulate Reality.
What are you talking about?
I put "4dAbstractions" into google and the only hit is this thread.  So this is something you just made up?
Title: Re: How do we measure the energy of a photon?
Post by: Bored chemist on 09/08/2023 16:22:18
So this is something you just made up?
It is.
That point has already been made.

And what has post #806 got to do with photon energy?
Nothing, but, this approach means the OP only has one thread full of rambling junk.
That's probably a good thing.
Title: Re: How do we measure the energy of a photon?
Post by: Kryptid on 09/08/2023 17:18:48
4dAbstractions is a computer system that allows us to simulate Reality.

A 4dSystem is the base object of 4dAbstractions.

All objects in 4dAbstractions are types of 4dSystems.

An App.System allows us to simulate Reality.

It consists primarily of a central App.Star system plus the following App.Subsystems:

1. Datastore.System - 1D Data storage
2. Data.System - 2d abstraction hierarchy
3. Display.System - 3d representation of  2d Data.System.
4. Driver.System - 4d App Driver (or e.g. Punter in 4dRacing)

This structure allows us to simulate Gravity. 4dGravity is the set of calculations that allow a 2d Data.System to be transformed into a 3d Display.System.

Given that Reality is likely a type of computer system, this transformation is likely how Reality creates gravity.

The structure also allows us to simulate the Big Bang where data is transformed from a 1D Datastore.System to a  2d Data.System to a 3d Reality.

A Person.App in 4dAbstractions is a type of Driver.System that is time-enabled.
 

What kind of experiment(s) can we perform to test these assertions?
Title: Re: How do we measure the energy of a photon?
Post by: Kryptid on 10/08/2023 06:30:05
None. The software is for personal use only.

So it's not a new theory?
Title: Re: How do we measure the energy of a photon?
Post by: Bored chemist on 10/08/2023 08:29:53
None. The software is for personal use only.

So it's not a new theory?
What makes you think that?
Well, it's not new, and it's not a theory.
Title: Re: How do we measure the energy of a photon?
Post by: mxplxxx on 23/09/2023 06:43:39
2d Systems exist, both in Reality and 4dAbstractions, because they are so much smaller than the corresponding 3d systems. e.g. a 2d Universe would not contain variables for Space. A hologram, for example, is a 2d System that can be expanded to a 3d System.

3d Systems in Visual Basic are very well catered for by the language via Partial classes. A 4dAbstractions system contains a Partial class for 2d Code and a Partial class for 3d Code. THe 2d Code handles 2d Abstraction hierarches. The 3d Code handles 3d objects such as buttons, menus etc. The 2d Partial class reacts to events in the 3d Class and calls functions in the 3d class.

4dAbstractions uses this scheme to convert 2d abstraction hierarchies to single 3d objects. The top of a hierarchy contains a Merge function that converts the 2d part of an abstraction hierarchy into a single 3d Object. Merge function code is attached.

Is this type of system present in Reality? Don't know, but the existence of positive and negative particles would suggest something similar exists.

Code: [Select]
            Public Sub Merge()

                Dim MergedForm As Global.System.Windows.Forms.Form = New Global.System.Windows.Forms.Form

                Merge1(CurrentSystem:=Me, MergedForm:=MergedForm)

                MergedForm.Show()

            End Sub

            Public Sub Merge1(CurrentSystem As Object, MergedForm As Object)

                Try
                    For Each currentcontrol As Object In CurrentSystem.usercontrol.controls
                        MergedForm.Controls.Add(currentcontrol)
                    Next
                Catch
                End Try

                For Each displaysystem As Object In CurrentSystem.Subsystems.Values
                    Merge1(CurrentSystem:=displaysystem, MergedForm:=MergedForm)
                Next

            End Sub
Title: Re: How do we measure the energy of a photon?
Post by: mxplxxx on 13/10/2023 12:47:07
149000 views and the best is yet to come. 8)
Title: Re: How do we measure the energy of a photon?
Post by: mxplxxx on 15/11/2023 22:37:52
From the Age newspaper, Melbourne, Australia.

"Last year, global astronomers seized on telescope data that picked up evidence of an explosion called a luminous fast blue optical transient, or LFBOT. Only a handful of these blasts, which explode with the energy of 100 billion stars, have been discovered since the first in 2018."

As described in the current post, the universe is composed of systems. Each system is composed of subsystems. The centre of each system is a special type of system, very often a black hole. In a solar system, the central system is a sun. The central system is what is being observed exploding as a LFBOT. So a sun becomes a super nova, and the central system of e.g. a galactic cluster becomes a LFBOT (or "Tassie Devil" as it is fittingly otherwise known).
Title: Re: How do we measure the energy of a photon?
Post by: Bored chemist on 16/11/2023 11:14:18
. In a solar system, the central system is a sun. The central system is what is being observed exploding as a LFBOT
The LFBOT was not the explosion of a sun-like star.

So a sun becomes a super nova
The sun will not become a supernova.
Title: Re: How do we measure the energy of a photon?
Post by: mxplxxx on 16/11/2023 13:26:27
. In a solar system, the central system is a sun. The central system is what is being observed exploding as a LFBOT
The LFBOT was not the explosion of a sun-like star.

So a sun becomes a super nova
The sun will not become a supernova.
1. Not "The" sun - "A" sun. One that is massive enough.

2. I never said an LFBOT was a type of sun.
Title: Re: How do we measure the energy of a photon?
Post by: Bored chemist on 16/11/2023 17:14:52
I never said an LFBOT was a type of sun.
Who wrote this
. In a solar system, the central system is a sun. The central system is what is being observed exploding as a LFBOT.
Title: Re: How do we measure the energy of a photon?
Post by: mxplxxx on 16/11/2023 23:28:27
I never said an LFBOT was a type of sun.
Who wrote this
. In a solar system, the central system is a sun. The central system is what is being observed exploding as a LFBOT.

You need a course in logic.
Title: Re: How do we measure the energy of a photon?
Post by: Bored chemist on 16/11/2023 23:41:47
You need a course in logic.
OK, You specified two things that can be central systems.
A black hole or a sun.

And you said that the central system explodes as a LFBOT.

Black holes don't explode; nor does the sun.


At best all you have done is say "someting went bang" which is true, but uninformative.
Title: Re: How do we measure the energy of a photon?
Post by: mxplxxx on 01/12/2023 12:20:01
4dAbstractions works on Abstraction Hierarchies.e.g.in 4dRacing, a betting Pool is an abstraction hierarchy of Meetings, Races and Runners.

The New function in Visual Basic will create an abstraction hierarchy.

e.g. New (Meetings as list(of Meeting.System), Races as list(of Race.System), Runners as list (of Runner.System))

It takes relatively primitive lists and converts them into the relatively complex and much more useful Pool.System abstraction hierarchy.

This parallels the process in Reality that converts primitive DNA strands into complex Proteins via a Ribosome.

Is a Protein an abstraction hierarchy? Don't know, but suspect this is highly likely.
Title: Re: How do we measure the energy of a photon?
Post by: mxplxxx on 03/01/2024 20:05:52
Time is not a requirement in most computer programs. If is is required, it is done via a Timer event  or a Wait function. Time in Reality is merely perception (of which we know little) which, in turn, depends on the level of abstraction that the perceiving system exists at.

Which may explain the effect perceived time has on physical healing. See https://scienceblog.com/541370/perceived-time-has-an-actual-effect-on-physical-healing/?utm_source=substack&utm_medium=email

It is highly likely that time and consciousness are equivalent or, at least, that time is an aspect of consciousness. i.e. Time is a concept that is biological in nature.
Title: Re: How do we measure the energy of a photon?
Post by: mxplxxx on 15/01/2024 18:10:22
4dAbstractions works on Abstraction Hierarchies.e.g.in 4dRacing, a betting Pool is an abstraction hierarchy of Meetings, Races and Runners.

The New function in Visual Basic will create an abstraction hierarchy.

e.g. New (Meetings as list(of Meeting.System), Races as list(of Race.System), Runners as list (of Runner.System))

It takes relatively primitive lists and converts them into the relatively complex and much more useful Pool.System abstraction hierarchy.

This parallels the process in Reality that converts primitive DNA strands into complex Proteins via a Ribosome.

Is a Protein an abstraction hierarchy? Don't know, but suspect this is highly likely.

I have been trying, unsuccessfully, to use my 4dAbstractions software to simulate a protein. After much Googling, I have come to the conclusion that we don't know how a protein functions, so simulating it is out of the question.

As far as I can work out, a protein performs a similar function in Reality to a function call in a computer program where an object is passed. e.g.  Datastore. Save(me) in 4dRacing.

I will be trying to design a simulation using this insight and the possibility that a protein is driven by a State Machine that functions only in 2D. In this situation, the Subsystems in the Protein System are Polypeptides and each Polypeptide is a collection of Amino Acid residues, where the Amino Acid residues contains transitions. The folding process will convert the 2D Polypeptides to a 3D folded protein.

BTW Reality's version of an Abstraction Hierarchy is linear. This is also true for 3D versions of 4dAbstractions. Thus, a 3D version of a Polypeptides chain is linear and contains headers and footers that differentiate items in the chain. Which gives rise to the possibility that Reality is basically 1D in nature (something that is predicted by the Big Bang Theory).
Title: Re: How do we measure the energy of a photon?
Post by: Origin on 15/01/2024 19:30:52
I will be trying to design a simulation using this insight and the possibility that a protein is driven by a State Machine where the Amino Acid sequence contains transitions
That doesn't seem like it would be any help at all in measuring the energy of a photon.
Title: Re: How do we measure the energy of a photon?
Post by: hamdani yusuf on 16/01/2024 01:28:12
I have been trying, unsuccessfully, to use my 4dAbstractions software to simulate a protein. After much Googling, I have come to the conclusion that we don't know how a protein functions, so simulating it is out of the question.
Have you looked into Alpha Fold?
Perhaps it will give you a glimpse of the complexity of the problem.
Title: Re: How do we measure the energy of a photon?
Post by: mxplxxx on 16/01/2024 02:02:41
I have been trying, unsuccessfully, to use my 4dAbstractions software to simulate a protein. After much Googling, I have come to the conclusion that we don't know how a protein functions, so simulating it is out of the question.
Have you looked into Alpha Fold?
Perhaps it will give you a glimpse of the complexity of the problem.
Thanks for that. I am interested "how" a protein works rather than its structure (although I can see the two are related).

I suspect Reality is a 2d System. The 3d aspect of reality is likely to be a temporary thing (hence the present!) that happens mainly to validate 2d configurations. The fact the we see in 3D is because we interpret a 2D view as 3D.

So, I will be working with proteins mainly in 2d and, most likely, they will form parts of abstraction hierarchies as a specialised form of a 4d System that also has State Machine capabilities..
Title: Re: How do we measure the energy of a photon?
Post by: mxplxxx on 20/01/2024 07:08:46
4dAbstractions software works on abstraction hierarchies; in a top-down fashion for function calls (.I.e.. imperative) and in a bottom-up fashion for events (i.e. reactive).

The following article re brain architecture supports the possibility that this type of architecture is ubiquitous in Reality.

https://scienceblog.com/541768/mit-study-reveals-universal-brain-patterns/?utm_source=substack&utm_medium=email

I am currently trying to simulate a protein in 4dAbstractions. I am experimenting with a design where an entry in a  protein polypeptide chain links/bonds with the bottom system of a 4dAbstraction hierarchy and delivers events to the hierarchy (i.e. in a bottom-up fashion) and processes function calls from the hierarchy (i.e. in a top-down fashion). Similar to what happens with a protein in Reality.
Title: Re: How do we measure the energy of a photon?
Post by: mxplxxx on 30/01/2024 02:26:46
I am currently trying to simulate a protein in 4dAbstractions. I am experimenting with a design where an entry in a  protein polypeptide chain links/bonds with the bottom system of a 4dAbstraction hierarchy and delivers events to the hierarchy (i.e. in a bottom-up fashion) and processes function calls from the hierarchy (i.e. in a top-down fashion). Similar to what happens with a protein in Reality.
It seems to me that proteins supply Reality with the equivalent of the  computer science Inheritance functionality.

 e.g. a Cyclic Adenosine Monophosphate (cAMP).protein would inherit a cAMP.molecule. (which in turn inherits a cAMP.System, which in turn inherits a 4dSystem) I.e.. a cAMP.protein is a type of cAMP.molecule.

The Amino Acid  residue chain in the cAMP.protein likely supplies data to the functions of the cAMP.molecule, probably in much the same way as DNA  does in the creation of a protein via a Ribosome.

Actually, a cAMP.Protein is probably better off inheriting a 4dProtein system. And a cAMP.Protein.Star (the centre system of a cAMP.Protein) will contain a reference to a  cAMP.System. More work needed on this.  8)
Title: Re: How do we measure the energy of a photon?
Post by: hamdani yusuf on 30/01/2024 07:41:23
I suspect Reality is a 2d System. The 3d aspect of reality is likely to be a temporary thing (hence the present!) that happens mainly to validate 2d configurations. The fact the we see in 3D is because we interpret a 2D view as 3D.
I don't think that 3D space is just a matter of perspective without physical significance.
Let's start with 1 kg of polymer clay, and form it into a ball. Measure its diameter.
Then break it down into 8 pieces equally, 125 gram each. Form them into balls. Measure their diameter.
You'll get their diameter to be a half of the original ball.
Since the density of the clay hasn't changed, the volume of one small ball is 1/8 the volume of original large ball, while its diameter is 1/2 the diameter of original large ball. The mass is proportional to diameter cubed, which is a power of 3.
Title: Re: How do we measure the energy of a photon?
Post by: mxplxxx on 30/01/2024 08:35:47
I don't think that 3D space is just a matter of perspective without physical significance.
I didn't say that. What I said was "The 3d aspect of Reality happens mainly to validate 2d configurations.".

Most of the functionality of 4dAbstractions happens at a 2d level. After a change to a 2d System, a "Render" function will convert it to 3d, then to Raster or Vector graphics, and display the result. Any problems in this process and the change to the 2d System will be invalidated. Something similar may be happening in Reality.

A 4dAbstractions App contains top-level systems Driver.System, Display.System, Data.System and Datastore.System.

The 4dCar.App (a self-driving car) will contain a Driver.System controlling the car, a Display.System displaying the car's Controls and current state of the car, a Data.System containing the car database, and a Datastore.System containing a hard copy of the car database.

The App works by the Driver.System virtually pressing the controls managed by the Display.System to drive the car. The Display.System responds by changing the car database (managed by the Data.System). The Data.System, in turn, updates the Display.System which, in turn, updates the display to reflect the change(s) and passes on the changes to the Driver.System.

Note 1: 4dAbstractions needs no change to run the 4dCar.App, which strengthens my assertion that 4dAbstractions can simulate anything in the universe.

Note 2: almost all of the universe is empty space. Thus it makes more sense to compute in 2d and display in 3d.
Title: Re: How do we measure the energy of a photon?
Post by: hamdani yusuf on 30/01/2024 14:24:18
Note that almost all of the universe is empty space. Thus it makes more sense to compute in 2d and display in 3d.
Afaik, self driving Cars like Tesla see the world through several 2D cameras, which is then converted into 3D virtual world. It's just like how human drivers work.
Title: Re: How do we measure the energy of a photon?
Post by: mxplxxx on 30/01/2024 17:44:20
Note that almost all of the universe is empty space. Thus it makes more sense to compute in 2d and display in 3d.
Afaik, self driving Cars like Tesla see the world through several 2D cameras, which is then converted into 3D virtual world. It's just like how human drivers work.
Interesting.

A 4dApp would likely have no trouble driving a Tesla car. The 4d's of 4dabstractions are:

Driver (4d/time)
Display(3d/space)
Data(2d/thought)
 and
 Datastore(1d/memory)

Via function calls (imperative, top-down),  a Driver (4d) System interacts with a Display (3d) System, a Display (3d) System interacts with a Data (2d) System, and a Data (2d) System interacts with a Datastore (1d) System.

Via raising events (reactive, bottom-up), a Datastore (1d) System interacts with a Data (2d) System, a Data (2d) System interacts with a Display (3d) System, and a Display (3d) System interacts with a  Driver (4d) System.

In this scheme of things, a Driver System only interacts with 3d Systems. Which makes for easy programming when designing a Person simulation (which inherits a Driver System) where the Person is not aware of the workings of  Data (thought) or Datastore (memory) Systems.

The Punter System in 4dRacing is a type of (inherits) Driver System.


Title: Re: How do we measure the energy of a photon?
Post by: mxplxxx on 02/02/2024 05:16:31
I suspect Reality is a 2d System. The 3d aspect of reality is likely to be a temporary thing (hence the present!) that happens mainly to validate 2d configurations. The fact the we see in 3D is because we interpret a 2D view as 3D.
I don't think that 3D space is just a matter of perspective without physical significance.
Let's start with 1 kg of polymer clay, and form it into a ball. Measure its diameter.
Then break it down into 8 pieces equally, 125 gram each. Form them into balls. Measure their diameter.
You'll get their diameter to be a half of the original ball.
Since the density of the clay hasn't changed, the volume of one small ball is 1/8 the volume of original large ball, while its diameter is 1/2 the diameter of original large ball. The mass is proportional to diameter cubed, which is a power of 3.

i.e. the volume of a sphere = 4/3 * pi * radius cubed

So, what is your point.
Title: Re: How do we measure the energy of a photon?
Post by: mxplxxx on 05/02/2024 15:35:41
4dControls is a set of standard functions for use by all 4dAbstraction Systems. The functions involved are Shared, which means they can be called from any 4dSystem. In a shared function, any Systems involved need to be passed into the function.

4dControls is inheritable. So, e.g. Pool.Controls in 4dRacing inherits 4dControls and has its own set of Shared Functions as well. e.g. The Winnings. function.

Pool.System in 4dRacing contains an abstraction hierarchy of 4dSystems, and Pool.Controls allows control of all of them from a single system.

Pool.Controls also processes events from PoolRaces.System and PoolMeetings.System which are the two top-level systems in the Pool,System abstractions hierarchy. As a result of receiving an event from either of these systems, Pool.Controls.can then run a Function that depends on the type of event received.

Proteins in Reality have a structure that allow for a similar process to occur, but whether or not it does is not known. e.g. the cAMP protein contains an amino acid sidechain (R) that is bonded to the Phosphate system in the cAMP molecule.. This can allow for an electron to travel from the the cAMP molecule to the cAMP protein which can then change state as a result of receiving the electron.

Is this functionality present in Reality? Very likely. A protein works in a similar fashion. It is what makes us intelligent. I hope to show you how -  next time
.
Title: Re: How do we measure the energy of a photon?
Post by: Bored chemist on 05/02/2024 15:39:05
Is there corresponding functionality in Reality? Very likely.
Why is it likely?
What evidence do you have for any of this speculation?
Title: Re: How do we measure the energy of a photon?
Post by: mxplxxx on 05/02/2024 15:46:02
Is there corresponding functionality in Reality? Very likely.
Why is it likely?
What evidence do you have for any of this speculation?

It is likely because we don't appear to know how a protein works.

There is no evidence. It just so happens that 4dControls and Proteins appear to  do similar things in a similar manner.
Title: Re: How do we measure the energy of a photon?
Post by: Bored chemist on 05/02/2024 16:36:00
It is likely because we don't appear to know how a protein works.

Does that idea make sense to you?

We don't understand something so it's likely to be caused by the first thing some unqualified bloke on the internet dreams up.

I don't understand Swahili, but that doesn't mean it's likely to be a computer simulation.
Title: Re: How do we measure the energy of a photon?
Post by: mxplxxx on 05/02/2024 16:50:09
Does that idea make sense to you?
It is likely because we don't appear to know how a protein works.
If we knew HOW a protein worked, I would not use "likely", I would KNOW for sure.  I
Title: Re: How do we measure the energy of a photon?
Post by: mxplxxx on 17/02/2024 01:18:44
https://www.slashgear.com/otago-researchers-hold-individual-atoms-in-place-in-a-new-experiment-2161053

Which makes 3d Systems that much more likely.

in Visual Basic,

  dim OxygenMolecule = new Oxygen.Molecule(Oxygen.Atom1, Oxygen.Atom2)

Here is a VERY condensed structure of an Oxygen (O2) molecule according to 3d.

Stars are Central 3d Systems (Star as in star of the show).

Proton
    Star
    System
        Up Quark-1
        Up Quark-2
        Down Quark

Neutron
    Star
    System
        Down Quark-1
        Down Quark-2
        Up Quark

Electron
    Star (an Electron Ion)
    System (no subsystems - Electron is considered to be elementary)

Oxygen (Atom)
    Star (nucleus)
        Shell-1
            Proton-1-1
            Neutron-1-1
            Proton-1-2
            Neutron-1-2
        Shell-2
            Proton-2-1
            Neutron-2-1
            Proton-2-2
            Neutron-2-2
            Proton-2-3
            Neutron-2-3
            Proton-2-4
            Neutron-2-4
            Proton-2-5
            Neutron-2-5
            Proton-2-6
            Neutron-2-6
    System
        Shell-1
            Electron-1-1
            Electron-1-2
        Shell-2
            Electron-2-1
            Electron-2-2
            Electron-2-3
            Electron-2-4
            Electron-2-5
            Electron-2-6

Oxygen (O2)
    Star
    System
        Shell-1
            Oxygen (Atom)-1
            Oxygen (Atom)-2



https://scienceblog.com/542469/scientists-locate-the-missing-mass-inside-the-proton/?utm_source=substack&utm_medium=email

Seems I have predicted this result a long time ago. 8)
Title: Re: How do we measure the energy of a photon?
Post by: mxplxxx on 21/02/2024 19:33:41
In Reality, the central system in an Amino Acid is a Carbon Ion. In 4dAbstractions, this makes an Amino Acid a type of Carbon Atom and a Carbon electron in this system a reference (Visual Basic Byref) to a Carboxyl, Hydrogen or Amino group or a Sidechain (R). A seriously interesting possibility for Biology.

Below is the demonstration Visual Basic code for how 4Abstractions would implement CarbonLEU.; a simulation of the Leucine Amino Acid, of Reality. Note that CarbonLEU inherits Carbon.

Re Electrons, 4Abstracations treats them as a type of 4d Super System. i.e. they can operate in Shells. They can also operate as Abstraction Hierarchies where Electrons in the the bottom shell are more concrete than those in the top shells, Events in Electron shells will flow from bottom to top and function calls will flow from top to bottom. One of Electron's purposes is to function as placeholders for References to "Real" systems. So, in this case, the Reference becomes a type of Electron in order to function.

Electricity  in Reality may be a flow of References as types of Electrons and Refences will contain energy, separate form the System it references) References may also be implementations of Quantum Entanglement (changing a Reference in Visual Basic will also change the System it references and all other Reference of that System)

Code: [Select]
Namespace CarbonLEU

    ' contains classes relating to a CarbonLEU (Leucine).System

    ' CarbonLEU.System

    Public Class System
        Inherits Carbon.System ' I am a type of Carbon.System

        ' create a basic CarbonLEU.System using a local Side Chain

        Public Sub New()

            ' create Carbon.System base

            MyBase.New(Type:=eSystemTypes.CARBONLEU, SubType:=eSystemSubTypes.DATA, Star:=New CarbonLEU.Star(Id:=1, Name:="Leucine"))

            ' create Leucine amino acid systems and reference in CarbonLEU.Subsystems and CarbonLEU.Star (central system)

            With CType(Me.Star0, Star)
                .AminoRef = New AminoGroup.System(New AminoGroup.Star())
                .HydrogenRef = New Hydrogen.System(New Hydrogen.Star())
                .CarboxylRef = New CarboxylGroup.System(New CarboxylGroup.Star())
                .SideChainRef = New SideChain.System()

                ' replace electrons 1-2 of shell 2 with aminoacid references

                Me.SubSystems(0)(0) = .AminoRef
                Me.SubSystems(0)(1) = .HydrogenRef

                ' replace electrons 3-4 of shell 2 with aminoacid references

                Me.SubSystems(1)(0) = .CarboxylRef
                Me.SubSystems(1)(1) = .SideChainRef
            End With

        End Sub

    End Class

    ' CarbonLEU.Star

    <Serializable()>
    Public Class Star
        Inherits Carbon.Star

        Property Id As Integer
        Property Name As String

        Public Sub New(Id As Integer, Name As String)

            ' create inherited Carbon.Star (neutrons defaults to 7, Protons defaults = 6)

            MyBase.New()

            Me.Id = Id
            Me.Name = Name

        End Sub

        ' handle events from aminoacid subsystems (SideChain.System local, rest global)

        Public WithEvents AminoRef As AminoGroup.System
        Public WithEvents HydrogenRef As Hydrogen.System
        Public WithEvents CarboxylRef As CarboxylGroup.System
        Public WithEvents SideChainRef As SideChain.System

    End Class

    ' SideChain (different for each amino Acid)

    Namespace SideChain

        Public Class System
            Inherits _4dLibrary.System

            Public Sub New()

                ' create SideChain.System and accompanying abstraction hierarchy

                MyBase.New(Type:=eSystemTypes.SIDECHAIN, SubType:=eSystemSubTypes.DATA, Star:=New Star())

                Dim ch2 As CH2.System = Me.AddSubSystem(New CH2.System(New CH2.Star()))
                Dim ch As CH.System = ch2.AddSubSystem(New CH.System(New CH.Star()))
                Dim ch31 As CH3.System = ch.AddSubSystem(New CH3.System(New CH3.Star()))
                Dim ch32 As CH3.System = ch.AddSubSystem(New CH3.System(New CH3.Star()))

                ' handle events from top of abstraction hierarchy

                CType(Me.Star0, Star).CH2Ref = ch2

            End Sub

        End Class

        Public Class Star
            Inherits _4dLibrary.Star

            ' create SisdChain.Star

            Public Sub New()

                MyBase.New()

            End Sub


            ' handle events from CH2 subsystem (SideChain.System local, rest global)

            Public WithEvents CH2Ref As CH2.System


            ' handle a DemoEvent from CH2.System

            Private Function DemoFunction() As MsgBoxResult Handles CH2Ref.DemoEvent

                Return MsgBox("Displaying Ch2Ref.Pupose after DemoEvent received " & CH2Ref.Purpose)

            End Function


            Public Overrides Function GetKey() As String

                Return String.Empty

            End Function

        End Class

    End Namespace
End Namespace
Title: Re: How do we measure the energy of a photon?
Post by: mxplxxx on 11/04/2024 05:09:45
In 4dAbstractions everything is a system, and all systems are a type of 4dSystem. The central system of a 4dSystem is a 4dStar. Which means that a 4dStar array can occur in a 4dSystem to infinity with each 4dStar tending to be smaller than the previous.

i.e. A 4dSystem is temporal in nature and a 4dStar is infinite in nature.

This type of structure gives rise to systems that are shell-based.

So, a Solar.System contains as its central system a Pluto.Star and a Pluto.star contains as its central system a Neptune.Star and so on and so on. The Stars in this structure are the equivalent of Shells.

Can this type of structure describe Reality? I think so.
Title: Re: How do we measure the energy of a photon?
Post by: Bored chemist on 13/04/2024 14:29:11
Can this type of structure describe Reality? I think so.
Does anyone else?
Title: Re: How do we measure the energy of a photon?
Post by: alancalverd on 13/04/2024 21:58:13
In Reality, the central system in an Amino Acid is a Carbon Ion.
Not the reality that the rest of us live in.
Title: Re: How do we measure the energy of a photon?
Post by: Bored chemist on 14/04/2024 01:16:15
 mxplxxx
Just for the record.
You have falsely attributed your nonsense to me

* misquote.JPG (8.18 kB . 519x64 - viewed 22 times)

I didn't say that.
In reality we have a pretty good idea how proteins work.
Title: Re: How do we measure the energy of a photon?
Post by: mxplxxx on 15/04/2024 07:32:36
In Reality, the central system in an Amino Acid is a Carbon Ion.
Not the reality that the rest of us live in.
AC/BC/PC/C2B/C .Five peas in a pod. Or maybe one?. Whatever, you five do a lot of Criticising without, imho, often persuading the reader of your righteosness.

btw K(C)riptid (see Wiki and translate from Indonesian)

In cryptozoology and cryptobotany, cryptids (from Greek: κρύπτω, translit. Krypto, meaning "hide") is a creature or plant that supposedly exists but is unknown to the scientific consensus and is often very hard to believe. [1] Famous examples of cryptids include the Yeti in the Himalayas and the Loch Ness monster in Scotland. Sometimes cryptids are also called "UMA", an acronym for Unidentified Mysterious Animals. The affix "crypt-" comes from Greek meaning "hidden" or "secret".
Title: Re: How do we measure the energy of a photon?
Post by: paul cotter on 15/04/2024 09:18:17
Disagree strongly. Alan and Bored Chemist have both made a valiant effort to restore truth to this thread, alas in vain.
Title: Re: How do we measure the energy of a photon?
Post by: alancalverd on 15/04/2024 12:20:43
My favorite T-shirt slogan:

Every disaster movie begins with someone ignoring a scientist.

Returning to the question, for the last 60 years I have been well paid to measure the energy of photons, thousands of clients think I'm right, and their patients have for the most part survived and prospered because sensible people believe me.
Title: Re: How do we measure the energy of a photon?
Post by: paul cotter on 15/04/2024 13:34:11
I see you edited your post, when I looked at it there were just "two peas in the pod". There are plenty more peas available to fill the pod to bursting and nobody to back up your false ideas. Give it up and learn some real science.
Title: Re: How do we measure the energy of a photon?
Post by: mxplxxx on 15/04/2024 14:28:19
Give it up and learn some real science
This is New Theories. You, I am guessing, would have advised Einstein simarly when he proposed his Theory of RElativity.
Title: Re: How do we measure the energy of a photon?
Post by: alancalverd on 15/04/2024 14:54:16
Real science begins (or in the case of relativity and quantum mechanics, ends) with what we already know, not by asserting that we don't know it.
Title: Re: How do we measure the energy of a photon?
Post by: paul cotter on 15/04/2024 18:59:34
True, this is the "new theories" subforum. Unfortunately I have yet to see any new theory here, only repetitive gibberish born of a lack of comprehension and puerile egotism.
Title: Re: How do we measure the energy of a photon?
Post by: mxplxxx on 16/04/2024 08:02:33
True, this is the "new theories" subforum. Unfortunately I have yet to see any new theory here, only repetitive gibberish born of a lack of comprehension and puerile egotism.

Yet you keep on reading my posts. This is the definition of insanity.
Title: Re: How do we measure the energy of a photon?
Post by: mxplxxx on 16/04/2024 10:10:45
Real science begins (or in the case of relativity and quantum mechanics, ends) with what we already know, not by asserting that we don't know it.
I would have thought real science begins where mathemtics ends and computer science starts.The mathematics of quantum mechancs for example  has limitations that are transcended by quantum computing.
Title: Re: How do we measure the energy of a photon?
Post by: Bored chemist on 16/04/2024 11:10:58
I would have thought real science begins where mathemtics ends and computer science starts.
The sane people think science was around centuries before computers.
This is the definition of insanity.
You said it...
Your post is the definition of insanity.
Title: Re: How do we measure the energy of a photon?
Post by: Bored chemist on 16/04/2024 11:14:18
This is New Theories. You, I am guessing, would have advised Einstein simarly when he proposed his Theory of RElativity.
Alas, to wear the mantle of Galileo it is not enough that you be persecuted by an unkind establishment, you must also be right.
Title: Re: How do we measure the energy of a photon?
Post by: Bored chemist on 16/04/2024 11:17:35
Returning to the question, for the last 60 years I have been well paid to measure the energy of photons, thousands of clients think I'm right, and their patients have for the most part survived and prospered because sensible people believe me.

While I'm a bit younger than Alan, and the photons I measured were typically about a thousand times less energetic, I too made a career from measuring them.